You are on page 1of 592

CRACK

THE CORE EXAM

7th (2020) EDITION

VOLUME 2
W R IT T E N & IL L U S T R A T E D BY:

PRO M ETHEUS LIONHART, M.D.


1
CR.XCK THE COR.E EX X M - V O L 2

SEVENTH ED. -V E R S IO N 1.0

DISCLX1MEP.:

REXDER.S XR.E XDVISED - THIS BOOK IS X I01 TO BE USED FOB. C L IN IC X L


DECISION M X K IN C . H U M X N ER.R.OP. DOES OCCUR., X N D IT IS YOU P.
B.ESPONSIBILITY TO DOUBLE CHECK X L L FXCTS PP.0V1DED. TO THE
FULLEST EXTENT OF THE LXW, THE AUTHOR. XSSUMES NO
RESPONSIBILITY FOR. X N Y INJUP.Y XND/OP. D X M X C E TO PER.SONS OR.
PR.OPEB.TY XR.ISINC OUT OF OR. R.ELXTED TO X N Y USE OF THE M X T E R JX L
CONTXINED IN THIS BOOK.

ISBNI: 9781673777888

IN DEPEN D EN TLY PUBLISHED — PB.OMETHEUS L IO N H A B T (EA RTH DIM ENSION C-137)

c o p y r j c h t © 2019 By p r .o m e t h e u s l i o n h a r t

A L L P.ICHTS R.ESER.VED - V N D t I L IN T E R N A T IO N A L X N D PA N -A M E R .IC A N CO PYR JCH T CONVENTIONS.


THIS B O O K , OR. PARTS THER.EOF, M X y NOT BE P.EPR.ODUCED IN X N y FOR.M W ITHO UT PER.MISSION
FP.OM THE A U T H O R .

COVER. DESIGN, TEXTS, X N D ILLUSTR.ATIONS: COPYRJCHT © 2019 By PR.OMETHEUS L IO N H X R T

A D D IT IO N A L A R T PP.OVIDED By.- H A A J A P . DESIGN/SHUTTER.STOCK.COM

X10 SLAVE LABOR. (RESIDENT OR. FELLOW ) W AS USED IN THE CR.EATION OF THIS TEXT

2
V O LU M E 2
W RITTEN &. ILLU STRA TED B Y
P R O M E T H E V S L IO K IH A R T , M . D

10 - NIEUR.0 9-182

11 - M U S C U L O S K E L E T A L — 135-302

12 - V A S C U L A R . — 305 - 351

13 - I N T E R V E N T I O N A L — 333-454

14 - M A M M O 457-527

15 - s i K X i e q y — 529-588

V O L U M E 1:

T O P IC S : PEDS, Cl, CM, R6PKODUCTIVE, 6NDOCRJN6, T H O K X C IC , CXfKD IXC, XIUKES

W A R M A C H 1KIE:

T O P IC S : P H Y S IC S , K I O N 1N I L I^P f^LTI V L S K I L L S , B IO S T X T S

3
LECAL STUFF

RGADGR.S A P E A D V IS G D - T H IS BOOK IS XIOT TO Be USED FOB. C L I N I C X L D GCISIO N M A K I N G -


H U M X N GR.R.OR. DOGS O C C U R ., X N D IT I S Y O U P . R E S P O N S IB I L I T Y TO D O U B L e C H e C K X L L
F X C T S P R O V ID E D . TO THG F U L L G S T GXTGNT OF T H 6 L A W , THG A U T H O R . A S S U M G S NO
B.6SPONSI BI L I T Y FOB. X N Y I N J U R Y A N D / O R . D A M A C G TO PGB.SONS OB. PP.OPGR.TY A R .I S I N C
OUT OF OR. B E L X T G D TO X N Y USG OF T H e M A T G R .I A L C O N T X I N G D IN T H IS BOOK.

AR. E R E C A L L S I N T H I S B O O K ? A B S o L w i G L V X i o i .

THG AUTHOR. HXS MXDe X CONSIDGR.ABLG GFFORT (IT'S THe OUTRJCHT PUB.POSG OF THe
TGXT), TO SPGCULXTe HOW QJjeSTIONS MIGHT BG XSKeD. A PHD IN BIOCHGMISTRY CXN FX IL
X MGD SCHOOL BIOCHGMISTB.Y TGST OB. BIOCHGM SeCTION ON THe WSMLG, IN SPITG OF
CLGAB.LY KNOWING MORE BIOCHCM TH XN X MGDICXL STUDGNT. THIS IS BeCXUSe THGY
A P E NOT USGD TO MGDICING STYLG QUGSTIONS. THG X IM OF THIS TGXT IS TO GXPLOBE TH 6
LIKGLY STYLe OF BOAB.D Q.UGSTIONS XN D INCLUDG MXTGB.IXL LIKeLY TO Be COVeR.eD,
INFOB.MGD BY THG ABR'S STUDY GUIDG.

T H B .O U G H O U T T H e TGXT THG A U T H O R . W I L L X T T G M P T TO F A T H O M THG M X N N G B . OF


Q JJG S T IO N IN G X N D IN C L U D G THG C O R .R E S P O N D IN G H I G H Y IG L D M X T G B . I X L . A COB.B.GCT
G S T IM X T IO N W I L L BG W H O L L Y C O I N C I D G N T X L .

H U M O R . / PK.OFANITY W A R .N I N C

I USG P B . O F X N I T Y IN T H IS BOOK. I M X K G "G B .O W N UP" JOKGS. P B .O B X B L Y N O T X


GOOD ID G X TO R E A D THG BOOK OUT LO U D TO S M A L L C H I L D R E N OB. GLDGR.LY
M GM BGP.S OF Y O U R . C H U P .C H / TGM PLG / M OSQUG . X I O W IS N O T THG O P T I M A L
T IM G TO BG R . G C P . e X T I O N X L L y O U TR .X G G D - I'M JU S T T R Y I N G TO M X K G THG BOOK
READ ABLG X N D FUN.

I A L S O T A L K X MGSS OF S H IT A B O U T D I F F G P E N T M G D I C X L S P G C IXLTIG S. I DO T H IS BGCAUS6


R A D IO L O G IS T S A B E T P .IB X L , X N D S TB .O K IN C T H A T UP.GG TGNDS TO C A L M PGOPLG. P P .O B X B L Y
N O T X GOOD ID G X TO R E A D THG BOOK OUT L O U D TO M GM BGP.S OF Y O U R . F A M I L Y T H A T A B E
IN SPG C IXLTIG S OTHGR. T H A N R A D IO L O G Y . THG TB.U TH IS I RESPGCT A L L THG 0 T H 6 R . SUB-
SPG C IALTIG S OF M G D IC IN G (GVGN F A M I L Y M G D IC IN G ) - T H 0 S 6 AR.G D I R T Y JOBS (POOP, PUS, 8^
N 0 T 6 W R J T I N C ) BUT S 0 M 6 0 N G NGGDS TO DO T H G M .

IF Y O U S T IL L FGGL THG DGSIR.G TO G M A I L MG A B O U T H O W I H U R T Y O U P. FGG LING S A N D


P.UINGD Y O U R . LIFG, C O N S ID G P . T H A T " M A Y B G PGOPLG T H A T C B E A T G T H I N G S A R .G N 'T
CONCGR.NGD W I T H Y O U R . D G LIC A T G SGNSIBI LITIGS."

4
W HAT M A K E S THIS BOOK MNIQUE?
SHOK.T ANSW EP r — F U C K IN G E V E R Y T H IN G

HOW ? — W E L L FOP. S T A P IE P r S, THESE BOOKS W E W N 'T W P JT T E N BY 30 PRESIDENTS,


20 FELLOWS, A N D 10 TUIKBO NEP rD A T T E N D IN C S .

INSTEAD THE BOOKS WEP rE W R IT T E N BY A R E LE N T LE S SLY SELF IMPP rO V IN C M A N I A C ,


DRJVEN TO PUP rCE THE HEP rETICS (OP rC SCU M ), L A U N C H V E N C E A N C E UPON THIS
P LA N ET , A N D D ELIVEP r M E P X ILE S S JU STICE UPON THE ENEM IES OF FR EE D O M A N D
LIBEP rTY.

THE IMPETUS FOPr THIS BOOK W A S NOT TO WP rITE A PrEFEP rENCE TEXT OPr
ST A N D A P rD PrEVIEW BOOK, BUT INSTEAD , A STPrA T E C Y M A N U A L FOP r SO LV IN C
M U LT IP LE CHOICE QJJ ESTIONS FOP r R A D IO LO C Y. THE A U T H 0 Pr W ISHES TO C O N V EY
T H A T THE M U L T IP L E CH OICE TEST IS DIFFEP rENT T H A N OPrA L BOAP rDS IN T H A T YOU
C A N 'T A S K THE SAM E KINDS OF OPEN-ENDED ESSAY-TYPE QUESTIONS. "W H A T 'S YOUP r
DIFFEP rE N T IA L? "

Q UESTIONING THE CONTENTS OF ONE'S DIFFEP rE N T IA L W A S THE O N L Y PrE A L


QUESTION ON OPrA L BOAP rDS. M O W T H A T SIM PLE QJJESTION BECOMES N EA P rLY
IMPOSSIBLE TO FOP rM A T INTO A M U L T IP L E CH OICE TEST. INSTEAD , THE FOCUS FOP r
TPrA I N I N C FOP r SUCH A TEST SHOULD BE ON T H IN G S T H A T C A N BE A SKED . FOP r
E X A M P LE , A N A T O M Y FA C TS - W H A T IS IT? ... OR... T P J V I A FA C TS - W H A T IS THE
MOST CO M M O N LO CA TIO N , OPr AGE, OPr A SSO C IA T IO N , OPr SYNDP rOME? ... OR...
W H A T 'S THE N EX T STEP IN M A N A G E M E N T ? T H I N K B A C K TO M E D I C A L SCHOOL
V S M L E STYLE, T H A T IS W H A T YOU A P rE D E A L IN G W IT H ONCE A G A IN . IN THIS BOOK,
THE A U T H O P r TPrIED TO COVEP r A L L THE M ATEP rI A L T H A T COULD BE ASKED
(PrE A SO N A B LY), A N D TH EN APPP rO X IM A T E h o w q j j e s t i o n s m i g h t be a s k e d
A B O U T THE V A P JO U S TOPICS. THP rO UGH OUT THE BOOK, THE A U T H O P r W I L L
IN T IM A T E , "THIS COULD BE ASKED LIKE THIS," A N D "THIS F A C T LENDS ITSELF W E L L
TO A QUESTION." IN C LU D ED IN THE SECOND V O L U M E OF THE SET IS A STPrA T E C Y
CH APTEP r FOCUSING o n h i g h Y IE LD "B U Z Z W O P rDS" T H A T LEND W E L L TO CEP rT A IN
QJJESTIONS.

THIS IS NIOT A PrEFEP rENCE BOOK.


THIS BOOK IS NIOT DESIGNED F O P r PA TIE N T C A P r E.
THIS BOOK IS DESIGNED FOP r STU DYIN G S P E C IF IC A L L Y FOP r M U L T IP L E CHOICE TESTS,
CASE CONFEP rENCE, A N D V IE W -B O X P IM P IN G /Q JJIZ IN G -

5
R ad io lo g y M a s te r o f S p o rt - N a tio n a l C h am p io n
A m e ric a n s love to fig h t. A m e ric a n s love th e cham pion.

This right here is about recognizing a com m itm ent to being the best. Being the best, standing on
the top stand - and more importantly the quest to stand on the top stand and wear the yellow
medal is what I want to recognize.

The quest is very lonely. They say it is lonely at the top - that is not true. W hen I started to
become recognized internationally as the top educator in the field 1 suddenly had alot o f people
who wanted to be my friend. I didn't have any friends on the way there - nobody was getting up
at 4am with me to w rite... nobody, not one person. It was just me - alone. It is lonely getting to
the top and that is where most people fall off. It’s Friday and everyone else is going out, or there
is a party or whatever - I'm tired and I want to sleep in. I was on call last night- it is just not
worth it. W hat I'm hoping to share is that for me that is not what life is. Life is about being the
best person you can be in w hatever it is you are doing. That does not have to be Radiology, but
because you are reading this book well that is what it m eans for you now.

You make the quest. If you get the top score on the exam - I'll be putting your name in this book
next year and you can m otivate the next legend o f tomorrow. Plus. I’ll give you some money and
have an enorm ous lion trophy m ade for you.

The most important thing is not actually winning. People think I only care about winning
because I rant and rave about how only the gold medals count - but the quest is what I really
value. I'm not training for silver. I'm training for gold. The quest for gold is m ore im portant than
someone handing you a gold medal. The katana sword o f the black dragon society cannot be
stolen - it can only be earned. Outcomes really don’t matter, because the truth is in this life you
can do everything 100% perfect and still fail and you can do alm ost everything wrong and still
win. If you don’t focus on outcom es and instead the quest itself you will get som ething much
greater — besides a high probability o f passing the test - what you can gain from the quest, the
ability to really dedicate yourself to something 100% - it is a skill you can use for the rest o f your
life. This is the way to always achieve victory. Once you know the way, you can see it in all
things - as the samurai say.

H X L L O F C H X M P t o m

2017 Cham pion 2018 Cham pion 2019 Cham pion 2020 Cham pion

Dr. Gary Dr. Thom as Dr. Nick I


Dellacerra, D.O. Pendergrast, M .D. Broadbent, D.O.

Y
o Y
o
MayJ the M ightiest
O
W arrior Prevail
1

- 1 lofstra (formerly —Wake Forrest - University of Illinois


N orth Shore-L IJ) in Peoria

6
I FIGHT FOR THE USERS

-TRON 1982

7
8
i o
flm l iTml Rm l ITml ITml Ir s i I r^ J ITml ITml IT ^ l n & i I imJ ITml ITml ITml r E ]

N eu r o r a d io lo g y

P r o m e t h e u s L i o n h a r t , M .D .

9
SECTION i:
* A natom y I
There is a ton o f anatom y that can be asked on a m ultiple choice test. M y idea is to break it
dow n into three categories: (1) soft tissue - brain parenchym a (including norm al
developm ent), (2) bony anatom y - w hich is basically foram ina, and (3) vascular anatom y.

Soft Tissue Brain Anatomy:


C entral Sulcus - This anatom ic landm ark separates the frontal lobe from the parietal lobe,
and is useful to find if you h a v e n ’t learned the lazy N e u ro ra d io lg isf s go to descriptor
“fronto-parietal region.” O ld school grey bearded R adiologists (likely the ones w ho are
im portant enough to w rite test questions) love to ask how you find this im portant structure.
There are about 10 w ays to do this, w hich brings m e to the m ain reason this is a great
pim ping question. Even if you can nam e 9 w ays to do it, they can still correct you by nam ing
the 10th way. I noticed during m y tim e as a “ trainee” that A ttendings tend to be excellent at
know ing the answ ers to the questions they are asking.

Practically speaking, this is the strategy I use for finding the central sulcus:

Pretty high up on the brain, m aybe the 3rd or 4th cut, I find the pars m arginalis. T his is
called the “pars bracket sign ” - because the bi-hem ispheric sym m etric pars m arginalis form
an anteriorly open bracket. The bracket is im m ediately behind the central sulcus. This is
present about 95% o f the tim e - it’s actually pretty reliable.

10
C entral Sulcus Trivia - Here are the other less practical w ays to do it.

S uperior frontal sulcus / Pre-central sulcus sign: T he p osterior end o f the superior frontal
sulcus jo in s the pre-central sulcus
• Inverted om ega (sigm oid hook) corresponds to the m otor hand
• Bifid posterior central sulcus: P osterior CS has a bifid appearance about 85%
• Thin p o st-cen tra l g yru s sign - The precentral gyrus is thicker than the post-central gyrus
(ratio 1.5 : 1).
• Intersection - The intraparietal sulcus intersects the post-central sulcus (w orks alm ost
alw ays)
M idline sulcus sign - T he m ost prom inent sulcus that reaches the m idline is the central
sulcus (w orks about 70% ).
H om unculous Trivia:

• The inverted om ega


(posteriorly directed knob)
on the central sulcus /
gyrus designates the m otor M otor Hand
cortex controlling hand
function.

A C A territory gets legs,

M CA territory hits the rest.

N orm al C erebral C ortex: As a point of trivia, the cortex is normally 6 layers thick, and the
hippocampus is normally 3 layers thick. I only mention this because the hippocampus can look
slightly brighter on FLAIR compared to other cortical areas, and this is the reason why (supposedly).

Dilated P erivascular Spaces (V irchow -R obins): These are fluid filled spaces that accompany
perforating vessels. They are a normal variant and very common. They can be enlarged and associated
with multiple pathologies; mucopolysaccharidoses (Hurlers and Hunters) / ‘gelatinous pseudocysts” in
cryptococcal meningitis, and atrophy with advancing age. They don’t contain CSF, but instead have
interstitial fluid. The common locations for these are: around the lenticulostriate arteries in the lower
third o f the basal ganglia, in the centrum semiovale, and in the midbrain.

C avum Variants:

N o rm a l
-100% of preterm infants, -Posterior continuation of -Extension of the
-15% o f adults. the cavum septum quadrigeminal plate
-Rarely, can cause pellucidum (never exists cistern to foramen of
hydrocephalus without a cavum septum Monro.
-Anterior to the foramen pellucidum) -Seen above the 3rd
o f Monroe -Posterior to the foramen of ventricle and below
Monroe the fomices.
-Between frontal horns
-Between bodies of lateral
ventricles
12
Ventricular Anatomy:

Just a quick refresher on this. Arachnoid Granulations: These are regions where
the arachnoid projects into the venous system
You have two lateral ventricles allowing for CSF to be reabsorbed. They are
that comm unicate with the third hypodense on CT (similar to CSF), and usually round
ventricle via the interventricular or oval. This round shape helps distinguish them
foramen (o f M onro), which in turn from clot in a venous sinus (which is going to be
communicates with the fourth linear). On MR they are typically T2 bright (iso to
CSF), but can be bright on FLAIR (although this
ventricle via the cerebral
varies a lot and therefore probably won’t be tested).
aqueduct. These things can scallop the inner table (probably
from CSF pulsation)

Lateral Vents

Cerebral
Aqueduct

Foramen
S 0f
Ambient Cistern ^ Magendie

The fluid in the fourth ventricle escapes via the median aperture (foramen o f Magendie), and the
lateral apertures (foramen o f Luschka). A small amount o f fluid will pass downward into the
spinal subarachnoid spaces, but most will rise through the tentorial notch and over the surface o f
the brain where it is reabsorbed by the arachnoid villi and granulations into the venous sinus
system.

Blockage at any site will cause a noncommunicating hydrocephalus. Blockage o f reabsorption at


the villi / granulation will also cause a noncommunicating hydrocephalus.

13
Basal Cisterns: The basal cisterns are good for (1) evaluating mass effect, and (2) anatomy questions.
People say the suprasellar cisterns look like a star, with the five corners lending themselves nicely
to multiple choice questions. So let us do a quick review; the top o f the star is the interhemispheric
fissure, the anterior points are the sylvian cisterns, and the posterior points are the ambient cisterns.
The quadrigeminal plate looks like a smile, o r ... I guess it looks like a sideways moon, if you
don’t like smiles.
No Time fo r Smiles. Training fo r Domination.
Snarls Not Smiles. Quadrigeminal
Interpeduncular Plate Cistern
Smile later... once you've passed the test.
Cistern

Anterior
^ Interhemispheric
Fissure
4 " Sylvian Cistern
Ambient
Cistern
Ambient
Quadrigem inal
Cistern D /\
Plate Cistern
The Ambient Cistern is a bridge between the
Interpeduncular C. 4 ► Quadrigeminal C.

Midbrain Tectum vs Tegmentum:

A trophy patterns o f the m idbrain w ill becom e im portant w hen we start talking about som e o f
the m ore obscure neurodegenerative disorders. So it’s good to review the vocab w ords.

W hen I was in m edical school m y anatom y teacher told me the m idbrain looks like a m onkey face.
I said "Please d o n ’t confuse me. Last w eek you said the m onkey face was a penis cut in cross section.’’
I alw ays thought the m idbrain looked more like a stoner dog that just got higher than a giraffe’s vagina.

14
Bony Anatomy: Skull Base Foramina I Rotundum, Oval, Spinosum, Hypoglossal)
First let us review the where - then we will do the what. Remember, they don’t have to show you the
hole in the axial plane. They can be sneaky and show it in the coronal or sagittal plane. In fact,
showing Foramen Rotundum (FR) in the coronal and sagittal planes is a very common sneaky trick.

SN^3I On the coronal view,


FR looks like you are
staring into a gun
barrel.

On the sagittal view,


think about FR as
being totally level or
horizontal.

With regard to the relationship between Spinosum and Ovale, I like to think of this as the footprint a
woman’s high heeled shoe might make in the snow, with the oval part being Ovale, and the pointy
heel as Spinosum.

Foramen
Ovale

Foramen
Spinosum

The Hypoglossal Canal is very posterior


and inferior.

This makes it unique as a skull base foramen.

15
Bony Anatomy: Skull Base Foramina ( Jugular Foramen)
The jugular foramen has two parts which are
separated by a bony “jugular spine.”

Pars Nervosa - The nervous guy in the


front. This contains the Glossopharyngeal
nerve (CN 9), along with it’s tympanic
brach - the “Jacobson’s Nerve”

Pars Vascularis - This is the “vascular


part” which actually contains the jugular
bulb, along with the Vagus nerve (CN 10),
Auricular branch “Arnold’s Nerve,” and
the Spinal Accessory Nerve (CN 11)

Bony Anatomy: Orbital Fissures and the PPF


The relationship between the Superior Orbital Fissure (SOF), the Inferior Orbital Fissure (IOF), Foramen
Rotundum (FR), and the Pterygopalatine Fossa (PPF) is an important one, that can really lead to some
sneaky multiple choice questions (mainly what goes through what - see chart on page 18).
I’ve attempted to outline this relationship on both sagittal and coronal views.

16
Anatomy: Cavernous Sinus
III

CN IV

CN VI

CN V2
Carotid

CN VI

The question is most likely, what’s in it (or asked as what is NOT in it).

• CN 3, CN 4, CN V I, CN V2, CN 6, and the carotid - run through it.

• CN 2 and CN V3 - do NOT run through it.

The only other anatomy trivia I can think o f is that CN6 runs next to the carotid, the rest o f the nerves are
along the wall. This is why you can get lateral rectus palsy earlier with cavernous sinus pathologies.

Anatomy: Internal Auditory Canal - “IAC’ Fundus (top) of


Superior the IAC
CN 7 Vestibular

Inferior
Vestibular

The thing to remember is “7UP, and COKE Down” If it is shown, it is always shown in
- with the 7th cranial nerve superior to the 8th this orientation.
cranial nerve (the cochlear nerve component).
The ideal sequence to find it is a
As you might guess, the superior vestibular branch heavily T2 weighted sequence with
is superior to the inferior one. super thin cuts through the IAC.

17
Anatomy-What’s in itP
C onsidering m ultiple choice questions regarding the skull base anatom y.
" W h a tg o e s w h e re ? ’’ questions essentially w rite them selves.

Hole/Compartment Contents

Foram en O vale CN V 3, and A ccessory M eningeal A rtery

Foram en Rotundum CN V2 ("R 2V 2"),

Superior O rbital Fissure C N 3 , CN 4, CN V I , C N 6

Inferior O rbital Fissure CN V2

Foram en Spinosum M iddle M eningeal A rtery

P ars N ervosa: CN 9,
Jugular Foram en
Pars Vascularis: CN 10, CN 11

H ypoglossal Canal CN 12

Optic Canal CN 2 , and O pthalm ic A rtery

C avernous Sinus CN 3, CN 4, CN V I, CN V2, CN 6, and the carotid

CN 7, CN 8 (Cochlear, Inferior Vestibular and Superior


Internal A uditory Canal
Vestibular components). “7 Up - Coke Down”

M eckel Cave Trigeminal Ganglion

D orello's Canal Abducens Nerve (CN 6), Inferior petrosal sinus

18
Vascular Anatomy-
Arterial vascular anatom y can be thought o f in four sections. (1) The branches o f the external
carotid (com m only tested as the order in w hich they arise from the com m on carotid).
(2) Segm ents o f the internal carotid, w ith pathology at each level and variants. (3) Posterior
circulation, (4) Venous anatom y

(11 Branches of the External Carotid

Som e A dm inistrative assistants Love Fucking O v e r P oor M edical Students

Superior Thyroid
A scending Pharyngeal
Lingual P osterior Superficial
Temporal
Facial A uricular
Maxillary
O ccipital
Posterior A uricular
M axillary
Superficial Tem poral

Facial

O ccipital'
A scending L ingual
Pharyngeal

THIS VS THAT: External vs Internal Carotid via Ultrasound


Internal External

Branches Nope Yup

Orientation Posterior Anterior

Resistance Low (continuous diastolic) High

Temporal Tap (poking the head) No Change in Waveform Waveform Reacts

19
(2) Segments of the Internal Carotid

• The bifurcation o f the IAC and ECA usually occurs at C3-C4

• Cervical ICA has no branches in the neck - if you see branches either
(a) they are anomalous or more likely
(b) you are a dumb ass and actually looking at the external carotid.
Internal Carotid *Remcmber finding branches is a way you can tell ICA from ECA on
ultrasound.

• Low resistance waveform with continuous forward flow during diastole

• Flow reversal in the carotid bulb is common

• Atherosclerosis: The origin is a very common location


• Dissection: Can be spontaneous (women), and in Marfans or Ehlers-
Danlos, and result in a partial H om er’s (ptosis and miosis), followed by
Cl (Cervical)
MCA territory stroke.
Can have a retropharyngeal course and get “drained” by ENT accidentally.
Pharyngeal infection may cause pseudoaneurysm at this level.

,. , Aneurysms here can be


C2 (Petrous) Not much goes on at this level. ^,
surprisingly big (thats what she said).

Not much here as far as vascular pathology. The anatomic location is


C3 (Lacerum) important to neurosurgeons for exposing M eckel’s cave via a transfacial
approach.

This segment is affected by


multiple pathologies including Aneurysms here are strongly
C4 (Cavernous)
the development o f cavernous - associated with hypertension,
carotid fistula.

Aneurysm here could compress the


C5 (Clinoid)
optic nerve and cause blindness.

C6 (Ophthalmic Origin at the “dural ring” is a Common site for aneurysm


- Supraclinoid): buzzword for this artery. formation.

C7 Aneurysm here may compress CN


(Communicating) III and present with a palsy.

20
Schematic ICA Angiographic Runs

You d o n ’t need to know every branch, but you should be able to recognize the m ain vessels.

Lateral ICA Runs:


^ Anterior Choroidal
Ophthalmic
— P Com
Dural Ring
C5

Petrolingual
Ligament
Carotid Can
C2

Anterior Choroidal
Posterior Communicating

APICA Run:

This is a pretty good look for an AP


ICA run.

Notice that half the image is relatively


blank. This is because you have 2
ICAs (right and left), only one of
which is getting injected. For the
total neuro angiography novice (most
residents), this is a helpful thing to
notice when deciding AP vs Lateral.

Also notice the MCA is lateral, and


the ACA is medial.

21
ACA

Trivia
MCA
Acute CN3 Palsy (unilateral
pupil dilation) a classic
neurology boards question -
grab a relax hammer STAT!
Posterior
Communicating A The answer is PCOM aneurysm
until proven otherwise (although
it can also be caused by an
aneurysm at the apex o f the
basilar artery or its junction with
the superior cerebellar /
posterior cerebral arteries).

The reason is the relationship


between the CN3 and vessels
(arrows).
Anterior
Inferior
Cerebellar
A.
PICA (15%)

PIC A origin is variable

PICA (70%)

AP Vertebral Run Lateral


Vertebral
Run

PICA
Choroid Point

PICA
Caudal Point

22
Vascular Variants:

Fetal O rigin o f the PCA: Most common vascular variant


(probably) - seen in up to 30% of general population.

Definitions vary on what a fetal PCA is. Just think o f this as


a situation where the PCA is feed primarily as an anterior
circulation artery (occipital lobe is feed by the ICA).

Therefore, the PCOM is large (some people define this


vessel as PCOM larger than P 1).

Another piece of trivia is that anatomy with a feta l PCA has


the PCOM superior / lateral to CN3 (instead o f superior /
medial - in normal anatomy).

Persistent T rigem inal A rtery:

Persistent fetal connection between the cavernous ICA to the basilar.

A characteristic “tau sig n ” on Sagittal MRI has been described.

It increases the risk of aneurysm (anytime you have branch points).


Axial - Connected Basilar and IC

Fetal C onnection

V
C arotid

S a g - C o n n e c te d B a s ila r a n d IC A
-L o o k s lik e a “ T ” au

A berrant C arotid Artery: D iscussed later w ith T-Bone pathology.

23
(4) Venous:
You can ask questions about the venous anatom y in roughly three w ays (1) w hat is it - on a
picture, (2) w hat is a deep vein vs w hat is a superficial vein, (3) trivia.

W hat is it?

Sigmoid Sinus
Cavernous Sinus

Superior Petrosal

Inferior Petrosal

Transverse Sinus
Occipital Sinus

Superior Sagittal Sinus

24
Anastomotic Superficial Veins:

Rolandic Vein (Central Sulcus)


Anastomotic Vein o f Trolard
Superior Sagittal Sinus
- Connects the Superficial
Middle Cerebral Vein and
Superior the Superior Sagittal Sinus
Anastomotic Vein
(Trolard) Anastomotic Vein o f Labbe
- Connects the Superficial
Middle Cerebral Vein and
the Transverse Sinus
Inferior
Anastomotic Vein
(Labbe) Superficial Middle
Cerebral Vein T rolard = T op
*drains to
L abbe = L ow er
Cavernous Sinus

Inferior Petrosal Sinus

Middle Cavernous Sinus ^ \


Cerebral Vein "
, \ Internal Jugular Veins
V '- * *
/iy ' \ < Superior Petrosal Sinus /
A
Superior ________^ Superior Sigmoid Sinus
Cerebral Vein Sa9 ittal Sinus
\ Confluence — ► Transverse Sinus
of Sinuses
Inferior
Sagittal Sinus Straight s
Sinus

Vein of Galen

THIS VS THAT: Superficial vs Deep: Superficial Deep


Basal Vein of
Superior Cerebral Veins
There is a superficial venous system and a deep Rosenthal
venous system . The easiest w ay to test m aterial Superior Anastomotic
Vein o f Galen
Vein o f Trolard
like this is your “w hich o f the fo llo w in g is n o t? ”
or “w hich o f the fo llo w in g is ? ” type question. Inferior Anastomotic Inferior Petrosal
Vein o f Labbe Sinus
The big ones to rem em ber are in the chart. Superficial Middle
Cerebral Veins

25
Venous Trivia:

C ollateral P athw ays: T he dural sinuses have accessory drainage pathw ays (other than the ju g u la r
veins) that allow for connection to extracranial veins. These are good because they can help
regulate tem perature, and equalize pressure. T hese are bad because they allow for passage o f
sinus infection / inflam m ation, w hich can result in venous sinus throm bosis.

Inverse R elationship: T here is a relationship betw een the Vein o f Labbe, and the A nastom otic
Vein o f Trolard. Since these dudes share drainage o f the sam e territory, as one gets large the
other get small.

Sounds Latin o r French: As a general rule, anything that sounds Latin or French has an increased
chance o f being on the test.

Vein o f Labbe: Large draining vein, connecting the superficial m iddle vein and the
transverse sinus

Vein o f Trolard: Sm aller (usually) vein, connecting the superficial m iddle vein and
sagittal sinus

B asal veins o f Rosenthal: D eep veins that passes lateral to the m idbrain through the
am bient cistern and drains into the vein o f G alen. T heir course is sim ilar to the PCA.

Vein o f Galen: Big vein (“great” ) form ed by the union o f the tw o internal cerebral veins.

Superior

26
Venous Gamesmanship

An em bolus o f venous gas is com m on and often not even noticed. The classic location is the
cavernous sinus (w hich is venous), but if the volum e is large enough, air can also be seen in
the orbital veins, superficial tem poral veins, frontal venous sinus, and petrosal sinus.

Why does this happen?

Peripheral (or central IV) had som e air in the tubing. Thats right, you can blam e it on the
nurse (w hich is alw ays satisfying). “N u rse In d u ce d R etrograde Venous A ir E m bolus ”

Significance?

D on’t m ean shit. It pretty m uch alw ays goes aw ay in 48 hours w ith no issues.

Cavernous Sinus
-The most common
spot to see this

27
Sinus Trivial Anatomy Bonus - The Concha Bullosa

This is a com m on variant w here the m iddle concha


is pneum atized. It’s pretty m uch o f no consequence %
clinically unless it’s fucking huge - then (rarely) it / I
can cause obstructive sym ptom s.

In m y private practice, an EN T told m e he w anted


these m entioned in all his C T sinus reports.

That w ay he can ju stify doing F E S S ...

He drives a nice car.


i

^ Gamesmanship for Neuro Anatomy


• F irst O rder Trivia:
"W hat is it? ” Style questions are m ost likely; w ith possibilities including
CTA, M RA , or A ngiogram s. C onsidering w hen the people w riting the
questions trained, angiogram s are probably the m ost likely.
• "W hat g oes through there? ” N euro foram ina
"W hat d o e s n ’t ? ” Style questions - CN 2 and CN V3 d o n ’t go through the
cavernous sinus.

• S eco n d O rder Trivia:


CN 3 Palsy - Think Posterior C om m unicating A rtery A neurysm
CN 6 Palsy - Think increased ICP

Increased ICP ►Brain Stem H erniates Interiorly — ►CN 6 G ets Stretched

28
S E C T I O N 2:
i B r a in D e v e lo p m e n t immpmimmmm

Brain Myelination:

The baby brain has essentially the opposite signal characteristics as the adult brain. The T1 pattern
of a baby is sim ilar to the T2 pattern of an adult. The T2 pattern o f a baby is sim ilar to the T1
pattern o f an adult. This appearance is the result o f m yelination changes.

The process o f m yelination occurs in a predeterm ined order, and


therefore lends itself easily to multiple choice testing. The basic Im m ature M ature
M yelin M yelin
concept to understand first is that immature m yelin has a higher
water content relative to mature myelin and therefore is brighter High Water, Low Water,
on T2 and darker on T 1. During the m aturation process, water Low Fat High Fat
will decrease and fat (brain cholesterol and glycolipids) will
T l dark, T l bright,
increase. Therefore mature white m atter will be brighter on T1
T2 bright T2 dark
and darker on T2.

Testable Trivia: the T1 changes precede the T2 changes (adult T1 pattern seen around age 1, adult T2
pattern seen around age 2). Should be easy to rem em ber (1 f o r Tl, 2 fo r T2).

Take Home Point: T l is most useful for assessing m yelination in the first year (especially 0-6
months), T2 is most useful for assessing m yelination in the second year (especially 6 months to 18
months).

Order o f progression: Just remember,


inferior to superior, posterior to Brain Myelination Pattern
anterior, central to peripheral, and
sensory fibers prior to m otor fibers.
The testable trivia is that the
subcortical white matter is the last
part of the brain to myelinate, with
the occipital white m atter around 12
months, and the frontal regions
finishing around 18 months. The
“terminal zones” o f m yelination occur
In ferio r to Superior, P osterior to A n terio r
in the subcortical
frontotcmporoparietal regions -
finishing around 40 months.

Another high yield piece o f testable trivia is that the brainstem , and posterior limb o f the internal
capsule are norm ally m yelinated at birth.

29
Additional Brain / Skull Development Trivia: Birth A dult

Ant T1 Hyper T1 Iso,


Pituitary: Both the A nterior and Posterior Pituitary are T1 Pituitary T2 Iso
B right at Birth (anterior only T1 bright until 2 m onths).
Posterior T1 Hyper,
T1 Hyper
Pituitary T2 Hypo

Brain Iron: Brain Iron increases w ith age (globus pallidus darkens up).

Bone M arrow Signal: C alvarial B one M arrow w ill be active (T1 hypointense) in young kids
and fatty (T1 hyperintense) in older kids

Sinus D evelopm ent:


Visible
The sinuses form in the Order
on CT
follow ing order:

... . Present at _
1- M axillary, Maxillary 1 g. ^ 5 month
2- Ethm oid,
3- Sphenoid, _ Present at .
Ethmoid 2 Bjrth 1 year
4- Frontal

M ost are fin is h e d c , . . NOT Present „


Frontal 4 . 6 year
fo rm in g by a round 15 at Birth 1
years.

0 , „ NOT Present
Sphenoid 3 4 year

30
Congenital Malformations:
This is a very confusing and com plicated topic, full o f lots o f long Latin and French sounding
w ords. If w e w ant to keep it sim ple and som ew hat high yield you can look at it in 5 basic
categories: (1) Failure to Form , (2) Failure to C leave, (3) Failure to M igrate,
(4) D evelopm ent Failure M im ics, and (5) H erniation Syndrom es.

Failure to Form • Dysgenesis / Agenesis of the Corpus Callosum

A classic point o f trivia is that the corpus


callosum form s front to back
(then rostrum la stl.

T herefore hypoplasia o f the corpus


callosum is usually absence o f the
splenium (w ith the genu intact).

R o s tru m

G A M E S M A N S H IP :

With agenesis o f the corpus callosum, a common


trick is to show colpocephaly (asym m etric dilation
o f the occipital homs).

W hen you see this picture you should think:

(1) Corpus Callosum Agenesis

(2) Pericallosal Lipoma


* D iscussed on next page

Colpocephaly
(asymmetric dilation of the occipital homs).

31
Failure to Form - Dysgenesis / Agenesis of the Corpus Callosum Continued
O ther com m on w ays to show this include:

Why are the lateral


ventricles widely spaced
when you have no corpus
callosum ?

There are these things called


“ Probst bundles” which are
densely packed W M tracts -
destined to cross the CC -
but can’t (because it isn’t
there).

So instead they run parallel


to the interhem ispheric
The “steer horn” “ Vertical V entricles” fissure - m aking the vents
appearance on coronal. w idely spaced look widely spaced.
(racing c a r) on axial.

Failure to Form - Associations - Intracranial Lipoma


Dysgenesis / A genesis o f the C orpus C allosum is associated w ith lots o f other syndrom es/
m alform ations (Lipom a, H eterotopias, Schizencephaly, Lissencephaly, e tc ...). Som e sources
will even say it is the “m ost com m on anom aly seen with other C N S m alform ations. ” —
w hatever the fuck that means.

Intracranial Lipoma:

The m ost classic association with CC Agenesis.


50% are found in the interhem ispheric fissure, as
shown here. The 2nd m ost com m on location is the
quadrigem inal cistern (25% ).

| riv ia . Q yjs Lipom as are congenital m alform ations, not true neoplasm s.

• “ M aldifferenitation o f the M eninx P rim itiva " - is a m eaningless French


sounding explanation for the frequent pericollasal location.

• N on Fat Sat T1 is probably the m ost helpful sequence (m ost non-bleeding


things in the brain are not T1 bright).

• These things d o n ’t cause sym ptom s (usually) are rarely treated.

32
Failure to Form - Open Neural Tube Defects
Anencephaly Iniencephaly

Neuro Tube Defect Neural Tube Defect


(Defect at the top o f head) (Defect at the level o f the cervical spine)

The Top of the Head is Absent Deficient Occipital Bone with Defect in the
(Above the Eyes) Cervical Region. Inion = Back o f Head / Neck

Extreme Retroflexion of the Head.


Reduced /Absent cerebrum and cerebellum. Enlarged foramen magnum.
The hindbrain will be present. Jacked up spines.
Often visceral problems.

Usually, not compatible with life.


Mercifully, not compatible with life.
When they do survive, they tend to have a
Potential to he awful at Jeopardy
natural talent fo r amateur astronomy

Classic Image Appearance:


Classic Image Appearance: “Star Gazing Fetus” - contorted in a way that
makes their face turn upward (hyper-extended
Incredibly creepy “Frog Eye” appearance on cervical spine, short neck, and upturned face).
the coronal plane (due to absent cranial bone /
brain with bulging orbits). It’s every bit as horrible as the Frog Eye thing
(both would make incredible Halloween costumes.)

Secondary Signs / Gamesmanship: AFP will be elevated


(true with all open neural tube defects)
• Antenatal Ultrasound With Polyhydramnios
(hard to swallow without a brain)

• AFP will be elevated


(true with all open neural tube defects)

Failure to Form - Open Neural Tube Defects - Encephalocele ( meningoencephalocele)


N eural tube defect w here brain + m eninges herniate through a defect
in the cranium . T here are lots o f different types and locations — but
m ost are m idline in the occipital region.

There are num erous associations: - m ost classic = C hiari III

33
Failure to Form - Cerebellar Vermis
Rhombencephalosynapsis Joubert Syndrome:

' h J

m, t * 4L
^ ^ \

w
-Note the Vertical Lines Across the Cerebellum-

Vermis is Absent. Vermis is Absent (or Small)

Classic Image Appearance:


Classic Image Appearance:
“Molar Tooth” appearance o f the
Transversely oriented single lobed cerebellum as
superior cerebellar peduncles
shown above (this is an Aunt Minnie).
(elongated like the roots o f a tooth).
Absence o f the vermis results in an Small Cerebellum
abnormal fusion o f the cerebellum.
Absence o f pyramidal decussation
(whatever the fu ck the means)

Small 4th Ventricle Large 4th Ventricle “Batwing Shaped”

Rounded Fastigial Point, Absent Fastigial Point,


Absent Primary Fissure Absent Primary Fissure

Associations: Retinal dysplasia (50%),


Associations: Holoprosencephaly Spectrum Multicystic dysplastic kidneys (30%).
Liver Fibrosis (“COACH” Syndrome)

G am esm anship: This stuff is tricky. Let me suggest the following tactics.

If you are faced with this level o f trivia (on an intermediate level exam), first start by looking for
the two markers of normal vermian development: (1) the primary fissure and (2) fastigial point -
both o f which arc best seen mid sagittal. The ‘fastigial p o in t” is normal angular contour (not
round) along the ventral surface o f the cerebellum. The prim ary cerebellar fissure is a deep
trapezoid shaped cleft along the posterior cerebellum. Absence or abnormal morphology of these
landmarks should trigger a multiple choice brain reflex indicating the vermis is not normal.

A bsent Fissure

N orm al Point should be located A bnorm al V e n tra l C ontour


Point ju st below the m id pons. A bsent V e rm is

34
Failure to Form - Dandy Walker and Friends
Radiologists love to nitpick and obsess over details. The more m eaningless the details the more
intense and emotional the debate. Along those lines, Dandy Walker m alform ations arc typically
described along a spectrum. The arbitrary stops along this spectrum often lead to ferocious beta
male nerd confrontations am ongst Academ ics - filled with ferocious exchanges o f
gossip, innuendo, finger nail scratching / pinching, biting, hair pulling, and empty threats o f
reputation destruction.

Despite the very real threat o f being scratched and pinched by an enraged Academic
- 1 now offer my sim plified strategy for dealing with this complex pathology.

“C la s s ic ” D a n d y W a lk e r :

There are 3 key findings which are consistently present and reliable to m ake the diagnosis.

Comparison

^ Hypoplasia of the Vermis 2 Hypoplastic Vermis is I Dilated Cystic 4th Ventricle


(usually the inferior part) Elevated and Rotated

On axial, there is the nonspecific appearance o f an


enlarged posterior fossa CSF space. It can look like a
retrocerebellar cyst on axial only (although it’s not a cyst
- it’s the expanded 4th ventricle).
Normal fo r
The cerebellar hemispheres will be displaced forward Comparison
and laterally but their overall volume and morphologic Bunch o f Posterior Fossa CSF
characteristics should be preserved. Cerebellum DisplacedAnterolaterally

"TORCULAR-LAMBDOID INVERSION"
This classic buzzword(s) describes the torcula
(confluence o f venous sinuses) above the level o f the
lambdoid suture, secondary to elevation of the tentorium.

It’s worth mentioning that this inversion is often NOT - Norm al - - D andy W alker -
seen in the “variant” version o f Dandy Walker. Lambdoid Torcula Above Lambdoid
Above Torcula
' High-Inserting
Venous Confluence "
Trivia Often identified on OB screening US.
Otherwise, presents with symptoms o f increased intracranial pressure (prior to month 1)
Most Common Manifestation = Macrocephaly (nearly all cases with the first month)
Associations: Hydrocephalus (90%), Additional CNS malformations (~ 40%)
(agenesis o f the corpus callosum, encephaloceles, heterotopia, polymicrogyria, etc...).

35
Failure to Form - Dandy walker and Friends
As I mentioned on the prior page. Dandy Walker m alform ations arc typically described along a
spectrum. We covered the “classic” subtype in depth on the prior page. The stragglers
(presumed to be o f lower yield) and classic type are contrasted on the chart below.

L e a st Severe

Name M ega C isterna M agna Blake Pouch “ Variant” DW M “C lassic” DW M

A lternative
"Hypoplastic
Term Used
Rotated Vermis ”
To Trick You

Mid Sagittal
Doodle

Normal Variant: Sac like cystic Hypoplastic,


protrusion through the Hypoplastic vermis elevated, rotated
O verview Focal enlargement o f foramen o f Magendie with dilation o f the vermis with cystic
the retrocerebellar CSF into the infra / retro 4th ventricle. dilation o f the 4th
space. cerebellar region. ventricle.

Normally Formed But Hypoplastic Hypoplastic


Vermis Normal
Upwardly Displaced (less severe) & Rotated

4th Ventricle Normal Dilated Dilated Markedly Dilated

- Normal In Size
C erebellar
Normal Normal Hypoplastic - Displaced
Hem ispheres
Anterolaterally

Posterior Fossa Normal Normal Normal Expanded

Torcula Normal Normal Normal High Insertion

H ydrocephalus Nope Yes 25 % o f Cases 90% o f Cases

Choroid from the 4th


ventricle swinging
into the pouch is
classic (but not
No supratentorial Diagnosis on antenatal ultrasound must
always present).
abnormalities be done after 18 weeks
Trivia
The pouch only (prior to 18 weeks the vermis hasn i
communicates with finishedform ing).
the 4th ventricle.
NOT the cisternal
CSF.

Now, let us switch gears from fusion to cleaving problems.

36
Failure to Cleave - Holoprosencephaly ( HPE)
This entity also occurs along a spectrum with the common theme being some element o f abnormal
central fusion. Although, it isn’t actually a fusion problem. Instead, it is a failure to perform the
normal midline cleaving. In the normal embryology, the fancy latin word '"P-lon ” starts out like a
peanut butter sandwich, then mom cuts the bread into two perfect halves (separate lateral
hemispheres). The sandwich cutting (cleavage) always occurs back to front (opposite o f the
formation of the corpus callosum), so in milder forms the posterior cortex is normal and the anterior
cortex is fused.
Least Severe
Lobar S em i-L obar A lobar
M ild
Fused
Fusion P ancake / C up
Frontal
Frontal Shape
Lobes
Horns

No rmal
Doodles Comparison

Incom plete Ball


Septum Shaped

Normal R im / M onoventricle
Comparison
Focal areas of Zero midline cleavage.
The back is cleaved (not the front) Cerebral hemispheres are
incomplete fusion
Overview
anteriorly (usually the fused and there is a single
> 50% fusion of the frontal lobes
fomix) midline ventricle
The body of the lateral ventricles
Variable mild fusion of Single Ventricle
are 1 chamber. Occipital and
Ventricles the frontal horns of the (distinct lateral an d third
Temporal horns are partially
lateral ventricles. ventricles are absent)
developed.
Thalam us Normal Fused (partial or complete) Fused
• Septum Pellucidum • Septum Pellucidum
Absent
• Septum Pellucidum • Corpus Callosum (partial) • Corpus Callosum
• Corpus Callosum • Anterior Interhemispheric Fissure • Interhemispheric Fissure
Structures
(partial vs normal) • Anterior Falx Cerebri • Falx Cerebri
Horrible Cleft Lip / Palate Cyclops Monster Face
Things Borat’s Brother Bilo (he is retard) (one eye, one nose hole, etc)
Survive into Adulthood,
Mercifully Bad
O utcom e Survive into Adulthood but terrible at Jeopardy
(stillborn / dead < 1 year)
(average at W heel o f Fortune)

(ff Face predicts Brain, BUT Brain doesn’t predict Face


B o Possible BUZZWORDS for HPE spectrum.
Monster Cyclops Eyes
Cleft lips / Palates
Pyriform Aperture Stenosis (from n a sa l p ro cess overgrow th)
Solitary Median Maxillary Incisor (MEGA-Incisor)

37
Arhinencephaly: Meckel-Gruber Syndrome:
Classic triad:
“Minor” HPE expression. 1. Occipital Encephalocoele m
Midlinc olfactory bulbs / tracts are absent. 2. Multiple Renal Cysts %
“C an’t Sm ell” - is the clinical buzzword. 3. Polydactyly m ■m
m
i v V V
Could be tested as Kallmann Syndrome
(which also has hypogonadism, & mental Also strongly associated with
retardation). Holoprosencephaly

Septo Optic Dysplasia:


This “Minor” HPE expression could be referred to by its French sounding name, for the sole
purpose o f fucking with you — ”de Morsier Syndrome ”

The classic
findings are
inferred by
the name.
Normal for Normal for
comparison comparison

Absent Septum Pellucidum “Septo ” and


Hypoplastic “Optic ” structures such as the Optic Chiasma (circle) and Optic Nerves
Normal
Trivia - Associated with Schizenccphaly Comparison,

Gam esm anship - The other thing they can show is


an azygos anterior cerebral artery - which is basically
a common trunk o f the AC As. This is rare , but
f a associated with SOD and lobar HPE.
Azygos ACA(1 trunk)

Now... switching gears to failures in migration / proliferation.

Failure to Migrate / Proliferate: lissencepiialy-Pachygyria Spectrum and Friends


An understanding of this complex pathology requires a rapid review of embryologic
neuronal cortex formation. Unfortunately, my medical school embryology
professor was Dr. Eleanor Abernathy, M.D. (youtube her if you aren’t familiar).
Anytime I read embryology I can’t help but think of her and wonder "can anyone
who loves animals so much, really be crazy?” The answer to th at... is yes.
Yes they can. They can be a drunken lunatic. So much so, that I was convinced
that anyone who understood embryology risked ending up just like her. J^r' Abe™at*jy as
Then I accepted the truth. You either die a hero or live long enough to see
yourself become the villain. ‘Many more Batman references coming up in the next few pages.

38
TH E FIRE RISES V
-A P P R O M E T H E A N D I A L O G U E O N C O P rT I C A L F O P r M A T I O N - F |

Prologue: The brain is said to form “inside-out, ” as neurons that will eventually make up the cortex
are originally birthed from a thick slurry surrounding the fetal ventricles. Sleep inducing texts will
refer to this as the “proliferative neuroepithelium.” I prefer the term “Lazarus Pit,” or just the “Pit.”
It is from this Periventricular Pit, where cells will make “the climb” to the cortex.

Act 1 - Proliferation: Before making “the climb” to the cortex the neuronal-glial stem cells are
bom into (and molded by) the darkness o f the periventricular Lazarus Pit. It is there that they learn
the truth about despair, first by dividing into additional stem cells in a symmetric fashion (1 stem cell
splits into 2 stem cells). Later this process will change to asymmetric proliferation (1 stem cell splits
into 1 stem cell and 1 differentiated cell - glial cell or neuron). This process continues for several
cycles until the stem cells receive the signal to undergo apoptosis - they expect one o f us in
the wreckage brother.

The number of neurons in the cortex is determined by the frequency and


number o f symmetric / asymmetric divisions by these stem cells.
Disturbance in this process will therefore result in either too many, too © e © s >
Symmetric Asymmetric
few, or improperly differentiated neurons.

Act 2 - Migration (RISE): From the periventricular proliferative pit of despair, cells will make the
climb. As they climb to freedom, they are guided by structural cells, chemical signals, and the chant
“Deshi, Deshi, Basara, Basara.” They make the climb in 6 waves, with the first generation forming
the “pre-plate” and the second generation forming the more permanent "cortical plate.”

In other words, the younger cells always moving past the older ones
becoming more superficial in their final position, (hence the idea -
“inside out” or “outside last"). Disturbance in this mechanism
(guidance, timing o f detachment e tc ...) will result in under­
migration, over-migration, or ectopic neurons.

Act 3 - Organization: At this point you may think the cells have given
everything to the cortex, and they don’t owe them anymore. But, they haven’t
given everything... not yet. There is still the process o f cortical folding
(gyrification).

The process actually occurs simultaneously with and depends heavily on the first two steps. The
differential speed o f cortex expansion (relative to the deeper white matter) is probably the key
mechanism for brain folding. For this expansion to occur properly there needs to be the right
number o f cells (act 1) migrated in the right order (act 2). There is the additional mechanism of
continued differentiation into structural cell types which organize into horizontal / vertical columns
creating an underlying cytoarchitecture need for structure and function. Disturbance in these
mechanisms will result in an absence of or excessive number o f folds.

39
Failure to Migrate / Proliferate: lissencephaly-Pachygyria Spectrum and Friends

Now, we are going to discuss the testable pathologies associated with how this process can fuck up.

I’m going to try and group them according to the stage of disturbance (although they don’t always fit
nicely into a single stage).

Failure to Proliferate: Hemimegalencephaly:

Rare, but unique (Aunt Minnie), malformation characterized by enlargement (from hamartomatous
overgrowth) of all or part/s of one cerebral hemisphere. The presumed cause is a failure in the
nonnal neuronal differentiation in the involved hemisphere - resulting in an “abnormal mixture of
normal tissues” - which defines a hamartoma. This process is often mixed with other errors in
migration resulting in associated polymicrogyria, pachygyria, and heterotopia.

The combo of (1) dilated ventricle and (2) mismatched hemisphere size can be confused with
destructive pathologies. So lets do a quick comparison to negate any potential fuckery.

4 > T H I S ra The trick is to lot)k at which side the dilated ventricle is on.
THAT:

' ”
‘Hamartom ato us
Overgrowth o f all
BIG Side or part o f a cerebral
with BIG Ventricle = hemisphere,
Hemimegalencephaly secondary to
differentiation /
migration failure.

c iu ii 1 Qifip The shrunken half is


with B , e v en,ride - atrophic,
. ex-vacuo dilation of
Atr°Phy the ventricle.
Rasm ussen’s
Encephalitis Zebra viral (or maybe -*ust bke an ° 'd
autoimmune) disease grandpa brain (only
that fucking this is j ust half the
H annihilates half the brailT and the kid is
5rain usually less than 10).

D yke-D avidoff-M asson (C ere b ra l H em i-A trophy):


This is another zebra that can look a lot like Rasmussen encephalitis - but also has

fsSy/J/lfe) weird unilateral skull thickening and expanded sinuses.


The superior sagittal sinus and fissure are moved across the midline.
It is supposedly caused by an in utcro or childhood stroke (supposedly).

Since literally anything is fair game on this exam. I’m including it for completeness
(it’s probably low yield).

40
Failure to Migrate / Proliferate: Lissencephaly-Pachygyria Spectrum and Friends

The pathologies related to abnorm al m igration are best though o f along a spectrum ranging
from agyria (no gyri) to pachygyria (flat gyri) to band heterotopias.

, , Periventricular
Lissencephaly Double Cortex Lissencephalv , , ,
, I ,. -% \ Nodular
“Classic” Type 1 Band Heterotopia “Cobblestone Tvpe 2 „ . . .
*r Heterotopia
Smooth Surface
Thick Cortex

Colpocephaly
Figure 8 Shape

U nderm igration U nderm igration O verm igration Failed M igration


Considered the Neurons in the
Failure to migrate both in mildest form o f periventricular
amount an in order - with a Classic Instead o f failing to migrate (subependymal)
reverse outside-in pattern. Large Lissencephaly an adequate num ber o f region were too
numbers of neurons do not even neurons to the cortical lazy to migrate to
reaching the cortical plate, Disorganized surface (as is the case in the the cortex.
depositing diffusely between the migration results in a classic type o f
second layer of lissencephaly), this The result is
ventricular and pial surfaces.
cortical neurons deep pathology is the result o f an nodular grey
over migration.
The distribution is fucked with 4 to the more matter deposition
superficial cortex. along the ventricle
thick layers formed instead o f 6. This over migration results borders.
oo oo oo This creates the in an additional layer o f
oooooo cortex com posed on gray
classic “double Most common
<3 m atter nodules.
cortex” appearance. location for grey
cL
ST 60 matter heterotopia.
oo
OO
oo s: ^ These nodules come in a
oo oo oo Co ^ variety o f shapes and sizes
Associated with
(unilateral, bilateral, small,
seizure disorders.
large, symmetric or
asymmetric).
As a result o f this disorganized / Associated with
THIS VS THAT:
seizure disorders. Most com m only It is
inadequate migration the process
commonly located adjacent
of cortical folding does not take H eterotopias
Gyral pattern is to the Sylvian fissures
place. follow grey matter
normal
on all sequences
(or mildly
Smooth Surface, Thick Cortex and NOT enhance.
simplified).
Cobblestoned Cortex
Colpocephaly is Common. (variable in size / location)
Subependym al
Subcortical band o f
heterotopic gray tubers o f TS are
“Figure 8” shaped brain on axial usually brighter on
matter Associated with congenital
-due to shallow, vertical Sylvan
muscular dystrophy, and T2 relative to grey
fissures
X-Linked Inheritance retinal detachment - matter and may
(F>M) "muscle- eye-brain disease" also be calcified.
Autosomal Inheritance (M=F)
Associated with CMV (maybe)

41
Failure to Organize: Polymicrogyria “PIKIG”

I ’ve heard people blame this on TORCH infections, toxic exposure, chromosomal issues, G od’s wrath
for “stuff the Democrats do.” There are likely many causes. I wouldn’t expect someone to ask for
“the cause,” other than perhaps the broad category of failed organization.

Having said that. I’ve read some PhD papers saying that layer 5
gets obliterated (by infection, toxins, wrath, etc..) after
completion of normal migration. With layer 5 gone the other
more superficial layers overfold and fuse resulting in an excessive
number o f small folds - the hallmark finding.

Classic Look: Fine undulating / bumpy cortex.


This anomaly come in a variety of shapes and sizes
(unilateral, bilateral, small, large, symmetric or asymmetric).
Most common location is adjacent to the Sylvian fissure Fine Undulations / Bumps
bilaterally.

Trivia: Zika Virus is the most common cause o f PMG in Brazil and South America

Failure to Organize: Schizencephaly— “Split Brain”

Just like polymicrogyria there are likely many causes and I wouldn’t expect someone to ask for “the
cause,” other than perhaps the broad category o f failed organization.
Having said that, one popular theory is the idea o f a vascular insult. What is this vascular insult ?
Well, you could say it’s the cortex’s reckoning (it damages the radial glial fibers). These radial glial
fibers are in charge (or at least they “feel in charge”) of the ropes used by neurons to “make the
climb.” Although, I’ve head it’s best to make the climb as the child did - without the rope. I mention
this because about 30% o f patient’s with schizencephaly also have non-CNS vascular stigmata
(example = gastroschisis - which supposedly occurs from a vascular insult to the abdominal wall).

Classic Look: Schizencephaly literally means “split brain” with the defining feature being a cleft
(lined with grey matter) connecting the CSF spaces with the ventricular system. How wide this cleft is
depends on the flavor; Closed Lip (20%) or (2) Open Lip (80%), although in both cases the cleft
should span the full thickness of the involved hemisphere. The clefts can be unilateral or bilateral.

i Closed Lip (20%) - Less Common, Less Seve Open Lip (80 %) - More Common, More Severe
i In this fonn, the "Lip” will This one is more obvious.
;appear closed without a
ICSF filled cleft. To make To make the call you want
!the call you want to look to see a CSF-filled cleft
Ifor is the grey matter (lined with grey matter)
i running across the normally extending from the
j uniform corona radiata. ventricle to the pial
surface.
j Sometimes you can see a “nipple” o f grey mater
j pouching at the ependymal (ventricular) surface. IThe gray matter lining is often weird looking
j(kinda nodular like a heterotopia).

Associations: Absent Septum Pellucidum (70%), Focally Thinned Corpus Callosum,


Optic Nerve Hypoplasia (30%), Epilepsy (demonic possession)

42
Developmental Failure Mimics— Hydranencephaly and the Porencephalic Cyst

These can be thought o f along a


L e a s t S evere M o re S ev e re
spectrum of severity.

P oren cep h alic C yst H yd ran en cep h aly

These things may look like a severe External Cortical


developmental anomaly but the Mantle
underlying mechanism is different. is Gone

They are “acquired.” Classically by a


vascular insult - but really from
anything that can cause
encephalomalacia (focal necrosis of
both the gray matter and white matter
with eventual cystic degeneration).
This would include a trauma after birth
(this doesn’t have to happen in utero).
Internal
Understanding that the brain develops Herpes is
Brain cleft / hole from Bilateral ICA
normally first - then gets crushed, a prior ischemic event occlusion the most
helps to remember the key findings. causes classic, but
resulting in
In particular, the absence of a gray encephalomalacia. massive in utero
matter lining along the defect. destruction of infection
Cyst/Cleft can both cerebral with toxo or
It’s almost like someone took an ice­ communicated with hemispheres. CMV are
cream scoop to the brain. In the case the Subarachnoid only the also
of Porenchephaly, they just took one Space (“external”) cerebellum. described
scoop. In the case of Hydranencephaly, mimicking an open lip midbrain, and causes.
the glutinous pig took pretty much the Schizcnccphaly or the falx
entire brain - leaving only the communicated only (usually)
cerebellum, midbrain, and the falx. with the ventricular remain.
system (“internal”).

THIS vs THAT: The tric k is to lo o k f o r g ra y m a tte r lin in g the cleft.

Brain cleft / hole from a prior CSF-filled


event (maybe ischemic) cleft
Cleft is
resulting in damage to the extending
Lined with
structural cells needed to from the
Gray
properly organize the cortex. ventricle to
Matter
the pial
Not Normally Formed surface.

Brain cleft / hole from a prior CSF-filled


ischemic / traumatic event cleft
Cleft is
resulting in encephalomalacia. extending
NOT Lined
from the
with Gray
Normally formed - but massive ventricle
Matter
insult make it look and/or the pial
developmental. surface.

43
fa ‘That Brain is Fucked” f a
His only hope for employment is Hospital Administration... or maybe QA Officer.
U nless “B rain is F u c k e d ’’ is a choice on the exam , y o u ’ll need to narrow dow n your
choices. I suggest the follow ing strategic algorithm , sim plified essentially into 2 questions.

(1) Is the C ortical M antle (outside o f brain) present or gone fishing ?


(2) If the M antle is still there (even if it’s very thin), look for the falx cerebri.

That Brain Appears to Be Fucked



Cortical Mantle P
Still There M ^N ope

/ Y

FalxP Hydranencephaly

Severe Hydrocephalus Holoprosencephaly - Alobar


-Still Got a Falx —Anterior falx usually missing in the semi-lobar form
—lobar (mild 1subtype should still have the nax

44
Herniation Syndromes - Testable Vocab and Chiari Malformations
Testable Vocab: Cephaloceles

“Cephalocele” is an umbrella term for a herniation o f the cranial contents through a defect in the
skull. While retaining the suffix “cele” they are then sub-classified based on (1) location, and (2)
what is in the herniation sac.

(1) LOCATION (2) HERNIATION SAC CONTENTS

Nasal
M enin gocele M eningo-
-CSF & E nceph alocele
Meninges -CSF, Meninges,
-NO BRAIN and BRAIN
*For the purpose of fucking with you,
Meningoencephaloceles are sometimes
Trans­
Occipital called Encephaloceles.
sphenoidal
*most classic
A dditional Trivia:
• C ystocele = CSF, M eninges,
B rain, and V entricle
• M yelocele = Spinal C ord
Herniation Syndromes: Chiari Malformations

This topic is incredibly complicated. There are literally entire books written on the individual
subtypes. The following is my best effort to distill the potentially testable trivia down to about 1.5
pages.

There are several numbered sub-types of Chiari malformations, with the shared finding o f a
downward displacement o f the cerebellum. Here is a quick overview o f the subtypes.

Type 1 Type II Type III Type IV

Historically used to describe


Relatively less severe cerebellar hypoplasia
Features o f Chiari 2
tonsillar herniation. without herniation. The term
Herniation o f cerebellar
has fallen out o f favor with the
tonsils (more than 5 Relatively more AND powerful men and women
mm) who control the Chiari
cerebellar vermian
Occipital Encephalocele
displacement nomenclature.
We shall not speak o f it again.

Classic Association Classic Features: Type 1.5


(not always present):
Hybrid term used to describe conditions that have features of
-Low lying torcula
both type 1 and type 2.
• Syrinx (cervical cord) -Tectal beaking
-Hydrocephalus
Not associated with neural tube defects, despite the significant
-Clival hypoplasia
downward movement o f the tonsils and brain stem.

45
C hiari Type I C h iari Type II | C h iari Type^

Thinned C orpus C allosum Tectal Beak

Occipital Encephalocele,
Clival Low Lying (meningoencephalocele)
Classically defined as i or H ypoplasia Torcula containing cerebellum and/
both tonsils > 5mm below Opposite o f or the brainstem, occipital
*Also note (^3
the level o f the Dandy Walker lobe, and sometimes even
the long skinny
B asion Opisthion. 4th ventricle, and the fourth ventricle. PLUS
the “tow ering features o f o f Chiari 2
Note the commonly cerebellum .”
associated Syrinx.
Classic Mechanism: Normal for
Comparison
Congenital underdevelopment
of the posterior fossa, leading
to overcrowding, and Compared to Type 1, there is relatively less
downward displacement. tonsillar herniation, but more cerebellar vermian
displacement
Non-Classic: Post traumatic
defonnity - acquired later in
life.

Clinical Symptoms are


produce in two flavors:
(1) Occipital headache from
pressure o f the cerebellar
tonsils - worse with sneezing
(2) Weakness, spasticity, and Interdigitated
loss o f proprioception from C erebral Gvri
pressure on the cord (most classically M yelom en in gocele-
demonstrated on Lum bar Spine
Classic Association axial CT)
(not always present): Classic Associations:
Classic Mechanism: Neural Tube Defect
• Syrinx o f the cervical cord “sucks” the cerebellum downward prior to full • Syrinx (cervical)
development o f the cerebellar tonsils.
• Tethered cord
Less Classic (but still highly Classic Association:
• Hydrocephalus
testable) association:
-Lumbar myelomeningocele / Spina Bifida
-Klippel-Feil Syndrome • Agenesis o f the corpus
(congenital C-spine fusion). callosum

NOT associated with a Only seen in patients with a


Only seen in patients; neural tube defect (NTD).
neural tube defect
Encephalocele = NTD

46
Special Topic - Mesial Temporal Sclerosis
This is a pattern of findings (hippocampal volume loss + gliosis / scar), which classically result in
intractable seizures. The etiology is not certain, but it is most likely developmental (hence the
inclusion in this section).

Clinical Trivia: This is the most common cause o f partial complex epilepsy.

Clinical Trivia: Surgical removal can “cure” the seizures / demon. Alternatively, perfect
W / i intracranial positioning o f a tooth (from a red haired woman) has been described as
therapeutic in the Kazakhstani literature.

The hippocampal region represents the medial portion o f the temporal lobe (black box).

For the purpose of multiple choice, the primary imaging findings are:

• Reduced Hippocampal Volume (best seen when compared to the opposite site).
*10% o f the time volume loss is bilateral - other findings are necessary to exclude fuckerv
• Increased T2 Signal (from gliosis / scar)
• Loss of Normal Morphology (loss of normal interdigitations)

Note the
compensatory
enlargement o f the
temporal horn o f the
lateral ventricle
*white arrow
Reduced Volume,
Normal Side fo r Comparison
Increased T2 Signal
*black arrow

Additional described findings - less likely to be shown (more likely to be asked)

• Atrophy of the ipsilateral fornix and maxillary body


• Contralateral amygdala enlargement

MR I Epilepsy Protocol Trivia:


• T1 - Superior for Cortical Thickness, Eval of Grey / White
• FLAIR - Superior for Cortical / Subcortical Hight Signal (Gliosis)
• T2* / SWI - Superior for Blood Breakdown Products (for other things that can cause seizures;
calcifications o f tuberous sclerosis, Sturge-Weber, Cavemomas, Gangliogliomas etc..)

47
t
S E C T I O N 3:
V o l u m e 8c E d e m a
I
Monro-Kellie Hypothesis:
The M onro-K ellie H ypothesis is the idea that the head is a closed shell, and that the three
m ajor com ponents: (1) brain, (2) blood - both arterial and venous, and (3) CSF, are in a state
o f dynam ic equilibrium . As the volum e o f one goes up, the volum e o f another m ust go
down.

C S F / Blood Blood
Blood j
\ j
yA
Normal Situation Leaking C S F Intracranial Hypertension
-Venous Blood Expands -Venous Sinuses Compress
to Compensate

Intracranial Hypotension: if you are leaking CSF, this will decrease the overall fixed volum e,
and the volum e o f venous blood w ill increase to m aintain the equilibrium . T he result is
m eningeal engorgem ent (enhancem ent), distention o f the dural venous sinuses, prom inence
o f the intracranial vessels, and engorgem ent o f the pituitary (“pituitary pseudo-m ass”). The
developm ent o f subdural hem atom a and hygrom as is also a classic look (again, com pensating
for lost volum e).

Idiopathic Intracranial Hypertension (Pseudotumor Cerebril: C lassic scenario o f a fat


m iddle-aged w om en w ith a headache. E tiology is not w ell understood (m aking too m uch
CSF, or not absorbing it correctly). It has a lot o f associations (hypothyroid, cushings,
vitam in A toxicity). The findings follow the equilibrium idea. W ith increased C SF the
ventricles becom e slit-like, the pituitary shrinks (partially em pty sella), and the venous
sinuses appear com pressed. You can also have the appearance o f vertical tortuosity o f the
optic nerves and flattening o f the posterior sclera.

C hanges in intracranial pressure can create a dow nw ard displacem ent o f the brainstem
stretching the 6th cranial nerve - it is said that 1/3 o f patients w ith pseudotum or cerebri have
sixth nerve paresis as their only neurologic deficit

48
Hydrocephalus— “Too Much CSF
Questions on this topic are most likely to be centered on the sub-type, location o f obstruction, and
cause. So, let us now review the vocab.
True Obstruction

“Communicating” Blood, Pus, and Cancer - Anything that


plugs up the villi - the three most common
-All Ventricles are Big causes being SAH, Meningitis (TB or
(25% o f the time the fourth Bacterial), and Carcinomatous Meningitis.
ventricle is normal)
Without Obstruction
-Level o f obstruction
between basal cisterns and
Brain Atrophy (ex-vacuo)
arachnoid granulations
Normal Pressure Hydrocephalus
- CSF can exit all the -see discussion below
ventricles
Choroid Plexus Papilloma
-Tumor that secretes CSF.
-Discussed more later in the chapter

Ventricles “NON-Communicating”
Level of Obstruction
are Big
(one or all) -Upstream Ventricles are Foramen of Monro =
Big • Colloid Cyst
Aqueduct =
-Level o f Obstruction is
’ • Aqueduct Stenosis
within the ventricle System
• Tectal Glioma
- CSF can N O T exit all the 4th Ventricle =
ventricles • Posterior Fossa Tumor
• Cerebellar Edema / Bleed

Normal Pressure Hydrocephalus: it s not well


understood - and idiopathic. The step 1 trivia is “wet, Quiz: Is
wackv. and wobbly” - describing the clinical triad of transepcndymal
urinary incontinence, confusion, and ataxia. The key flow seem more
points clinically are the patient is elderly (60s), and with acute
the ataxia comes First and is most pronounced. hydrocephalus
or chronic
The buzz-phrase is “ventricular size out o f proportion
hydrocephalus?
to atrophy.” The frontal and temporal horns o f the
lateral ventricles are the most affected. “Upward Answer: Acute.
bowing of the corpus callosum” is another catch
phrase. On MR1 you may see transependyma 1 flow
and/or a flow void in the aqueduct and 3rd ventricle. Communicating Hydrocephalus +
This is treated with surgical shunting. Elderly + Ataxia - NPH.

Syndrome of Hydrocephalus in the Young and Middle-aged Adult (SHYMA): Similar to NPH but in a
middle aged population - and more headaches less peeing o f the pants (HA+Wacky+Wobbly).
Communicating Hydrocephalus + Middle Aged + Headache = SHYMA.

49
Congenital Hydrocephalus
There are several causes o f hydrocephalus that can be present at birth or be related to fetal
development. These conditions are typically diagnosed prior to birth via routine ultrasound
(discussed partially in the reproductive chapter in volume 1).

The big 4 are: (1) Aqueductal stenosis, (2) Neural tube defect - usually Chiari II, (3) Arachnoid cysts,
and (4) Dandy-Walker. I’ve discussed Chiari II and Dandy-Walker already.

So, let us turn our attention to Aqueductal Stenosis and Arachnoid Cysts.

Aqueductal Stenosis:

This is the most common cause of congenital obstructive hydrocephalus. Classically from a web or
diaphragm at the aqueduct (hence the name). Because o f the location you get a “non-communicating”
pattern with a dilation o f the lateral ventricles and 3rd ventricle with a normal sized 4th ventricle. You
can have a big noggin (macrocephaly) with thinning of the cortical mantle.

Obstruction at the
level of the
aqueduct

Normal for Comparison


Normal
Sized 4th

Treatment is going to be either shunting or


poking a hole in the 3rd ventricle (third ventriculostomy).

Clinical Trivia: Question header may describe “sunset eyes” or an upward gaze paralysis.

Clinical Trivia: A male with "flexed thumbs ” should make you think about the x-linked variant.
(Bickers Adams Edwards syndrome).

Arachnoid Cysts:

As the name implies, these are cysts located in the subarachnoid


space. They are CSF density, without any solid components, or
abnormal restricted diffusion. You wouldn’t even notice them
expect that they can exert mass effect on the adjacent brain, or in
the context of this discussion block a CSF pathway (obstructive
type).

50
CSF Shunt Malfunction
Normal: The most basic shunt consists of a proximal tube (usually placed in
the frontal horn o f the lateral ventricle just anterior to the foramen o f Monro), a
valve to control flow, and a distal tip (usually dumped in the peritoneum, but
can be placed in the pleural space or right atrium).

Shunt Evaluation Options: Your first line options for shunt evaluation are goingto be(a) non-con CT or
(b) rapid single shot T2 sequence - mainly looking at catheter position and ventricle size.*May need to
verify shunt settings with a plain film post magnet. If the ventricles are big (shunt is not working) you
might follow that up with a radiograph series (neck, chest, abd) to make sure the catheter is intact.
Ultrasound or CT can be used to inspect the distal tip for a fluid collection. Alternatively (if you are a
weirdo) you can inject < 0.4ml pertechnetate into the shunt reservoir and take images to look for leakage
or blockage (remember to not aspirate when you inject).

-Proximal > Distal


OX)
s Obstruction -Most common cause = ingrowth o f choroid
(proximal) piexus and particulate debris / blood products
-Can also be from catheter migration
m Prior C om parison
i
SJ
TJ
c -Pseudocyst
3
Obstruction (loculated flu id along the distal tip) L o culated fluid
(distal) -Catheter migration around the tip o f
(more common in children) n the catheter

-"Slit Like Ventricles” - can be meaningless or Slit V entricles


suggest too much shunting.
O ver- - The big fear is that not enough CSF will cause E xtra-A xial Fluid
Shunting subdural hygroma or hematoma formation via C ollections
Monroe Kelly mechanics (less CSF - more (subdural hygrom a)
blood).

-Usually within 6 months o f placement


-Blood cultures are usually negative (fluid from
the shunt should be cultured instead).
-Mild enhancement after catheter placement can
be normal - be on guard for fuckery.
-The best sign is debris within the ventricles,
ideally shown with DWI - this is the weapon of
Infection choice for diagnosis o f ventriculitis. T l+ C : ventricular D W I: D ebris /
ependym al Pus R estricts
enhancem ent D iffusion
-Late stigmata may include ventricular
loculations - which can cause restricted flow /
obstruction and in some case isolate or "trap" the
4th ventricle — as shown in diagrams. ______

Either deliberately or via migration the catheter can end up in the pleural space. A little
H ydrothorax bit o f pleural fluid doesn’t mean shit. But, if the volume gets large enough and the
patient becomes symptomatic - then revision might be needed.
Usually the ascites from a VP shunt isn’t symptomatic, although there are reports o f
A scites inguinal hernias and hydroceles forming secondary to the increased abdominal pressure.

51
Edema:
C ytotoxic: This type o f edem a can be thought about as intracellular sw elling secondary to
m alfunction o f the N a/K pum p. It tends to favor the gray m atter, and looks like loss o f the
gray-w hite differentiation. T his is classically seen w ith stroke (or traum a), and is w hy
EARLY signs o f stroke involve loss o f the G M -W M interface.

Vasogenic: This type o f edem a is extracellular, secondary to disruption o f the blood-brain


barrier. It looks like edem a tracking through the w hite m atter (w hich is less tightly
packed than the gray m atter). This is classically seen with tum or and infection. You can
also see this type o f edem a as a LATE stage o f cerebral ischem ia. A response to steroids is
characteristic o f vasogenic edem a.

i f i THIS vs THAT:
Cytotoxic Edema Vasogenic Edema
Failed Na/K Pump Increased Capillary Permeability
(BBB intact) (BBB NOT intact)
Classic = Ischemia (EARLY) Classic = Tumor, Infection, Ischemia (LATE)
White Matter + Gray Matter - “blurring” White Matter (Spares Gray Matter)

52
Brain Herniation:
Subfalcine ( C in g u la te ) Herniation: This is just a fancy way of saying midline shift (deviation of
ipsilateral ventricle and bowing of the falx). The trivia to know is that the ACA may be compressed,
and can result in infarct.

Descending Transtentorial (U ncal) Herniation: The uncus and hippocampus herniate through the
tentorial incisura. Effacement o f the ipsilateral suprasellar cistern occurs first.

Things to know :

• Perforating basilar artery branches get compressed resulting in “D uret H em o rrh a g es


classically located in the midline at the pontomesencephalic junction (in reality they can also
affect cerebellar peduncles).

CN 3 gets compressed between the PCA and Superior Cerebellar Artery causing ipsilateral
pupil dilation and ptosis

• “Kemohan’s Notch / Phenomenon” - The midbrain on the tentorium forming an indentation


(notch) and the physical exam finding o f ipsilateral hemiparesis - which Neurologists call a
“fa ls e lo ca lizin g sign. " O f course, localization on physical exam is stupid in the age of MRI,
but it gives Neurologists a reason to carry a reflex hammer and how can one fault them for
that.

Ascending Transtentorial Herniation: Think about this in the setting o f a posterior fossa mass. The
vermis will herniate upward through the tentorial incisura, often resulting in severe obstructive
hydrocephalus.

Things to know
The “Smile” of the
quadrigeminal cistern will
be flattened or reversed
• “S p in n in g Top ” is a
buzzword, for the
appearance o f the midbrain
from bilateral compression
along its posterior aspect
Severe hydrocephalus (at
the level o f the aqueduct).
*through
Fracture

Cerebellar Tonsil Herniation:


Ascending Transtentorial
Can be from severe herniation after Transtentorial
downward transtentorial herniation.
Alternatively, if in isolation you arc
thinking more along the lines o f Downward
Chiari (Chiari I = 1 tonsil - 5 mm). Cerebellar
Tonsillar

53
S E C T I O N 4:

t M e t a b o l ic & T o xic

Metabolic Plagues on the Alcoholic Urban Outdoorsman - part 1


O sm otic D em yelin atio n or
W e rn ic k e E n cep h alo p ath y
C e n tra l Pontine M yelinolysis

High T2/FLAIR
High T2/FLAIR Signal in Signal in the
T2 Bright Central Pons (spares the periphery) the Medial Thalamus Periaqueductal Gray
M ost C lassic Scenario: Asshole drunk Hobo shows M ost C lassic Scenario: Very friendly Hobo - known
up to the ER with a low N a. Like most asshole for singing songs from the 70s (mostly Supertramp’s
drunks in the ER, he starts out demanding a goodbye stranger) - starts acting squirrelly. “His
cheeseburger and a Sprite (not a fucking Sierra tempo seems o f f ’ - notes the feminine male nurse.
Mist!), then threatens to leave against medical advice.
.. .after finishing the burger. An above average medical student suggests he is
Family Medicine Resident begs him to stay exhibiting the clinical triad o f ( 1) acute confusion.
(a decision he will soon regret). The Resident (2) ataxia, and (3) ophthalmoplegia, but is dismissed
eventually tires o f his bullshit and decides to correct by the Medicine intern who talks non-stop about
his hyponatremia as rapidly as possible - with the going into Cardiology (“Cards” - he calls it).
goal o f expediting discharge.
Only moments later the same Intern will suggest to
2 days later the guy is still in house, acting like a
his Attending the same triad o f findings before
massive prick - acutely encephalopathic with spastic
stating “my medical student” seems disinterested and
quadriparesis.
may benefit from more call.
Neurology gets consulted and writes "pseudobulbar
palsy” in the chart. Family Medicine Resident Still desperate to honor the clerkship, the student
doesn’t know what the fuck that means, but is humble suggests thiam ine (vitam in B it deficiency as the
enough to ask. A below average 2nd year medical etiology, and says the symptoms could progress to
student explains to him that it is slurred speech, chronic memory loss and confabulation (Korsakoff
sensitive gag reflex, and being an even bigger cry psychosis) or even death.
baby than normal- “labile emotional response”.
The cycle repeats - additional call is assigned, and a
Coma, the above MRI, death, then a lawsuit follow
(in that order). formal letter o f reprimand is issued to the student.

• T2/FLAIR bright classically seen in medial/dorsal


• T2 bright in the central pons (spares periphery)
thalamus (around the 3rd ventricle), periaqueductal
• Earliest change: restricted diffusion in lower pons gray, mamillary bodies, and the tectal plate.
• Trivia: Can also have an extra-pontine presentation • Enhancement is classic in the mamillary bodies
involving the basal ganglia, external capsule, • MR Spect = Lactate
amygdala, and cerebellum. • Treatment = Thiamine replacement.

54
Metabolic Plagues on the Alcoholic Urban Outdoorsman - part 2
M arch iafava-B ig n am i M isc

D ire c t A lc o h o lic Injury:

Most Common / Classic Finding(s):

Brain A trophy. Particularly the cerebellum


and especially the cerebellar vermis

Normal Comparison

Cerebellar Atrophy
-High T2/FLAIR in the Corpus Callosum-

M ost C lassic Scenario: A middle aged (50s) man is


C opper & M a n g a n es e D eposition:
stumbles into the ER. On the pre-assessment forms
he has described him self as a “semi-professional red
wine taster.” He seems to exhibit variable degrees of
mental confusion, and his gait appears altered.
J
His chief complaint is seizure and muscle rigidity.

He interrupts the H&P stating that he needs to urinate


(“drain his hog” - he says). A nurse hands him a
urinal but his muscle rigidity seems to be impair his T l Bright
coordination. It looks like the floor is about to get Basal
very wet, when his the nurse (a shy, but aggressively Ganglia
religious, elderly women) tries to assist him. The
patient barks at the nurse “Not now woman! Leave A
\
the dick alone!” Her face blushes with
embarrassment. The ER Attending can’t help but
laugh. The medical student shadowing him also
laughs (but only after the Attending does so first). -Non-Specific and related To Liver Disease.
-Can be seen without hepatic encephalopathy
Later the Attending will reprimand the medical -Also seen in TPN, Wilson’s Disease,
student for laughing. In a formal letter, the Attending -Also seen in Non-Ketotic Hyperglycemia (HNK) in
says the student’s lack of professionalism is shocking which it’s often unilateral
- additional call is assigned.

• Swelling / T2 bright signal at the corpus callosum


M e th a n o l T o xicity:
(represents an acute demyelination)
• Order is progressive - typically beginning in the “Drinking Windshield Wiper Fluid” as an idiotic
body, then genu, and lastly splenium attempt to get drunk. Can also be seen from
consuming “poorly adulterer moonshine” - or "West
• “Sandw ich sign” on sagittal imaging - describes Virginia Budweiser.”
the pattern o f preference for central fibers with
relative sparing o f the dorsal and ventrals fibers Classic Findings: Optic nerve atrophy, hemorrhagic
• Chronic Phase: Thinned corpus callosum + cystic putam inal and subcortical white matter necrosis
cavities favoring in the genu and splenium

55
T H IS VS THAT: Carbon Monoxide v s Methanol:

Carbon CT Hypodensity / T2 Bright: G lobus Pallidus


M onoxide (carbon monoxide causes “globus ” warming).

T2 Bright: Putam inal - which may be


M ethanol
hem orrhagic, and thus CT Hypcrdense.

PRES (Posterior Reversible Encephalopathy Syndrome]:


Classic Features:

• Asymmetric cortical and subcortical white matter


edema (usually in parietal and occipital regions
*but doesn’t have to be - superior frontal sulcus is
also common).

• Does NOT restrict on diffusion (helps tell you it’s not


a stroke).

Etiology: Poorly understood auto regulation fuck up.


Vasogenic E dem a P attern
Classic History: Acute Hypertension or Chemotherapy. Bilateral, Posterior, Mildly Asymmetric

Post Chemotherapy:

It is fairly common. There are lots o f named offenders. Methotrexate seems to be the one people
write the most papers about (especially in kids with ALL)

There are two main looks:

(1) PRES - As above, chemo is a classic cause. BUT! It tends to have a “non-classic” look relative
to the hypertension type. It will often spare the occipital lobes, and instead target the basal
ganglia, brainstem, and cerebellum.
(2) Leukoencephalopathy (treatment induced): The classic look would be centered in the
periventricular white matter - bilateral, symmetric, confluent, T2/FLAIR bright changes (history
is obviously key to the diagnosis).

Other Misc Trivia:


Can progress to brain atrophy.
“Mineralizing Microangiopathy - the vocab word to use if there are calcifications

“Disseminated necrotizing leukoencephalopathy” - severe white matter changes, which


demonstrate ring enhancem ent, classically seen with leukemia patients undergoing radiation
and chemotherapy. It is bad news and can be fatal (it believes in nothing Lebowski).

56
Post Radiation:
The quick and dirty version is that after radiation therapy to the brain you can see T2 bright areas
and atrophy corresponding to the radiation portal. You can also sec hemosiderin deposition and
mineralizing microangiopathy (calcifications involving the basal ganglia and subcortical white
matter). There is a latent period, so imaging findings don’t typically show up for about two months
post therapy. N ow ... if you want to get crazy, you can discuss changes at different time periods.

Acute
Too rare to give a fuck about (at least for the test)
(Days-Weeks):
Early Delayed The classic look is similar to chcmo - high T2/FLA1R This is reversible
(1-6 months): signal in the periventricular white matter. change (usually).

Described as a “mosaic" pattern with high WM signal


Late Delayed changes again favoring the deep white matter. Can Progressive... but
(6 months): appear “mass-like” and expansile. reversible (mostly)
Classically sparing o f the U-Fibcrs & Corpus Callosum.

Radiation-Induced Vasculopathy: Strokes and Moya-Moya type o f look.


Mineralizing Microangiopathy - 1 mentioned this on the previous page. This is a
delayed finding — like two years following treatment. Think calcifications (basal
ganglia and subcortical white matter) - hence the term “mineralizing.”
Radiation-Induced Vascular Malformations: The most classic types are capillary
Long Term telangiectasias / cavernous malformations. The most classic scenario is a kid
Sequela getting whole brain radiation for ALL. Remember the key finding is blooming on
GRE/SWI sequences.
Radiation-induced Brain Cancer:
• XRT is the “most important risk factor” for primary CNS neoplasm.
• Most common type is a meningiomas (70%) - usually seen ~ 15 years post XRT
• More aggressive types Gliomas, Sarcomas, etc, have a shorter window < 10 years

Chasing the Dragon”- Heroin Inhalational leukoencephalopathy


Most toxic lcukoencephalopathies (either from chemotherapy, immunosuppressives, antibiotics,
or the aristocratic art of paint thinner huffing) all create a similar non-specific pattern o f widespread
high T2/FLAIR signal in the supra and infratentorial white matter. The “Chasing the Dragon”
pattern is also not specific - but it does have a catchy name, so people love collecting cases of it to
show in conference (“catchy name” = high yield for boards).
The most classic look (diagrams are FLAIR sequences):

Sparing of
the dentate
nucleus
High Signal in the Posterior High Signal in the Deep (arrows)
Symmetric “ Butterfly” in
Limb of the Internal Capsule Cerebellar White Matter
the Centrum Semiovale

57
S E C T I O N 5:

t IMP1PJMMHPM ^ N EU R O -D E G E N ER A TI V E [PIMP1MIMMMM

Multiple Sclerosis:
W hite m atter patterns can be confusing as there arc tons o f
overlapping non-specific features. To help understand this (and
avoid being tricked) let me introduce a few concepts. W hite matter
lesions come in a few patterns. MS is the poster child o f the
“perivascular pattern." This pattern favors involvem ent o f the
juxtacortical and periventricular regions with lesions that have
ovoid and/or fusiform morphology.

( /) j Ovoid, Fusiform (2) Periventricular


Size can be helpful. A single
lesion > 15 mm in size suggests the underlying etiology is not
vascular.

Certain locations will also make you think “not Vascular Perivascular
vascular." W hen you think “not vascular Pattern Pattern (MS)
pattern" you should think dcmyelinating. Corpus Callosum RARE COMMON
W hen you think dcm yelinating you should think
MS first (it’s by far the most common). Juxtacortical RARE COMMON
Infratentorial RARE COMMON
Basal Ganglai COMMON RARE
M cD onald D iagnostic Criteria for MS:

This was last revised in 2010, so it’s kind o f an old M cD onald’s Diagnostic Criteria.

• And on his criteria he had a section o f lesions dissem inated in space (periventricular,
juxtacortical, infratcntorial, spinal cord) - more than 1 in at least 2 o f these locations.

• And on his criteria he had a section on dissem ination in tim e: best shown as a T2 bright lesion
that does enhance (active) and a T2 bright lesion that docs not enhance (in-active) — lesions
arc in different phases o f the disease and therefore separated by time.

Epidem iological trivia:

• Usually targets women 20-40 (in children there is no gender difference).


• There are m ultiple sub-types with the relapsing-remitting form being the most comm on (85%).
• Clinical history o f "separated by time and space ” is critical.

58
A dditional M S R ela ted Trivia:

• M ost Classic Finding: T2/FLA1R oval and periventricular perpendicularly oriented lesions.
• Involvem ent o f the calloso-septal interface is 98% specific for MS (and helps differentiate it
from vascular lesions and ADEM ).
• In children the posterior fossa is m ore com m only involved.
• Brain atrophy is accelerated in MS.
• Solitary spinal cord involvem ent can occur but it is typically seen in addition to brain lesions.
• The cervical spine is the m ost comm on location in the spine (65%).
• Spinal cord lesions tend to be peripherally located.
• FLAIR is m ore sensitive than T2 in detection o f juxtacortical and periventricular plaques.
• T2 is m ore sensitive than FLAIR for detecting infratentorial lesions
• MR spectroscopy (discussed later in the chapter) will show reduced N AA peaks within the
plaques.

Active vs N ot A ctive : Acute dem yelinating plaques should enhance and restrict diffusion (on
m ultiple choice tests and occasionally in the real world).

Tumor vs MS: You can som etim es get a big MS plaque that looks like a tumor. It will ring
enhance but classically incom plete (like a horseshoe), with a leading dem yelinating edge.

Tum or = Com plete Ring Dem yelination = Incom plete Ring

Multiple Sclerosis Variants:


ADEM (Acute D issem inated Encephalom yelitis): Typically presents in childhood or
adolescents, after a viral illness or vaccination. Classically has m ultiple LARGE T2 bright
lesions, which enhance in a nodular or ring pattern (open ring). Lesions do N O T involve the
calloso-septal interface.

Acute H em orrhagic L eukoencephalitis (Hurst Disease): This a fulm inant form o f ADEM with
m assive brain swelling and death. The hem orrhagic part is only seen on autopsy (not imaging).

Devics (neurom yelitis optica): Transverse M yelitis + Optic Neuritis.


Lesions in the Cord a n d the Optic Nerve

M arburg Variant: Childhood variant that is fulm inant and terrible leading to rapid death. It
usually has a febrile prodrom e. “M A R B U R G !!! ” = DEATH

59
Subcortical Arteriosclerotic Encephalopathy ISAE1

Also referred to as Binswangcr Disease - for the purpose o f fucking with you.

It’s best thought of as a multi-infarct dementia that ONLY involves the white matter.

Trivia:

• It favors the white matter o f the centrum


semiovale (white matter superior to the
W TF are “U F ib e rs ” ?
lateral ventricles / corpus callosum).
T hey are the
• Classically spares the subcortical U fibers. fibers under the
cortex, that look
• Strong association with Hypertension.
like “ U ”s.
• It’s seen in older people - 55 and up
T hey com e up a
• If they show you a case that looks exactly lot, as being
like SAE but that patient is 40 and has spared or not
migraines they are leading you to the
genetically transmitted form of this disease spared.
called CADASIL.

CADASIL (Cerebral Autosomal Dominant Arteriopathv with Subcortical Infarcts & Leukoencephalopathy)
Basically it is SAE in a slightly younger person (40), with migraines.

Classic Scenario: 40 year old presenting with m igraine headaches, strokes, then eventually
dementia. CADASIL is actually the most common hereditary stroke disorder.

Step 1 Trivia: NOTCH3 mutations on chromosome 19

Classic Imaging Findings: Severe white matter disease (high T2/FLAIR signal) involving multiple
vascular territories, in the frontal and tem poral lobe. The occipital lobes are often spared. Temporal
lobe involvement is classic.

60
Dementia Disorders:
This topic was split up in prior editions on the text, w ith h a lf in neuro and h a lf in nukes. The
reason I did that w as because these disorders are often evaluated w ith FD G PET. To keep
you from having to hop around I decided to consolidate it this tim e around.

FDG PET for dem entia is a w orthless and expensive com ponent o f the w orkup. Like m ost
im aging exam s it is ordered w ith no regard to the im pending collapse o f the health care
system under crippling rising costs (w ith inevitable progression into a M ad M ax style
dystopian future or even better M ega-C ity 1). A s such, it is standard practice in m ost
academ ic centers to obtain the study.

The idea is that “ dem ented brain” will have less perfusion and w ill have less m etabolism
relative to “not dem ented brain.” PET can assess perfusion ( 15O-H20) but typically it uses
l8FD G to assess m etabolism (w hich is analogous to perfusion). Renal clearance o f 18FD G is
excellent, giving good target to background pictures. R esolution o f PET is superior to
SPECT.

HM PAO, and EC D (tracers that are discussed in m ore depth in the nukes chapter) can also be
used for dem entia im aging and the patterns o f pathology are the sam e.

It’s im portant to rem em ber that external factors can affect the results; bright lights
stim ulating the occipital lobes, high glucose (>200) causes m ore com petition for the tracer
and therefore less uptake, e tc .. .e tc ... so on and so forth.

B efore we begin w ith the subtypes, a quick pearl:


On F D G P E T the m o to r strip is alw ays preserved
in a desen era tive type o f dem entia.

Preserved Motor Strip


—Seen in degenerative dementias

61
Dementia - The Primary Tribes
Alzheimer Disease Multi-infarct Dementia Dementia with LewyDodies
Most common cause 2nd most common 3rd most common

Tauopathy, Amyloid Cascade, Also called “Vascular Alpha synuclein and synucle-
and Neurofibrillary Tangles are Dementia” - for the purpose of inopathy are buzzwords people
all buzzwords people use when fucking with you. use when they pretend to
they pretend to understand the understand the
pathophysiology. pathophysiology.

Risk Factor(s): Risk Factor(s): Clinical Scenario: There is a


triad o f classic features.
The biggest one is Age. • McDonalds, Burger King, (1) Visual hallucinations
Taco Bell, Pizza Hut (2) Spontaneous parkinsonism,
A more obscure one (but (3) Fluctuating ability to
certainly testable) is Downs • Hypertension, concentrate / stay alert
Syndrome. Downs patients
nearly always get AD, and they • Smoking (tobacco), and Clinical picture can be similar
get it earlier than normal - that to Parkinson’s dementia - the
extra 21st isn’t doing them any • CADAS1L major difference in DLB, the
favors. dementia comes before the
Parkinsonism

Most Classic Feature(s): Most Classic Feature(s): Most Classic Feature(s):


hippocampal atrophy (which Cortical infarcts and lacunar Mild generalized atrophy
is first and out of proportion to infarcts are seen on MR1. Brain without lobar predominance
the rest of the brain atrophy). atrophy (generalized) is usually (unlike multi-infarct).
They could ask temporal horn advanced for the patients age. Hippocampi will be normal in
atrophy > 3 mm , which is seen size (unlike AD)
in more than 65% o f cases.

FDG Pattern: Low posterior FDG Pattern: Multiple FDG Pattern: decreased FDG
temporoparietal uptake - scattered areas o f decreased uptake in the lateral occipital
"headphones ” or "ear muffs. ” activity. No specific lobar cortex, with sparing o f the mid
predominance. posterior cingulate gyrus
(Cingulate Island Sign).
11C PiB (Pittsburgh Unlike the neurodegenerative
compound B) is an even better dementias - this one could
way to waste money making knock out the motor strip (if the
this diagnosis. It works as an strokes happen to involve that
Amyloid Binding Tracer. region). This is different that
AD and DLB.

Picks: Also be referred to a “frontotemporal dementia ’’ - for the purpose o f fucking with you.
Clinical: Onset is earlier than AD (like 40s-50s). Classic presentation is described as “compulsive
or inappropriate behaviors.” In other words, acting like an asshole (fucking prostitutes, and buying
miracle weight loss potions from Dr. Oz - when you aren’t even going to the gym or trying to cat
right). Just being a real Prick.
Classic Feature(s): Severe symmetric atrophy o f the frontal lobes (milder volume loss in the
temporal lobes).
FDG Pattern: Low uptake in the frontal and temporal lobes.

62
FDGPET-Brain
-Identical to Parkinson
Low posterior
D em entia
A lzheim ers tem poroparietal cortical
-P osterior C ingulate gyrus is
activity
the first area abnorm al
Scattered areas o f decreased
M ulti Infarct
activity
Preservation o f the m id
D em entia with Lew y Bodies Low in lateral occipital cortex posterior cingulate gyrus
(C ingulate Island Sign)

Picks / Frontotem poral Low frontal lobe

Normal Alzheimers Frontotemporal


-Low p o ste rio r tem poroparietal -Low F rontal Lobe

Lew Body Dementia Multi-Infarct


-Low L ateral O ccipital with ■Scattered A reas o f L o w Uptake
sparing o f the cingulate gyrus

63
The Defias Brotherhood of Neurodegeneration - Part 1
Also called “Bilateral
Striatopallidodentate Calcinosis", and Extensive
sometimes "Primary Familial Brain Calcification in the
Fahr D isease Calcification ” for the sole purpose of Basal Ganglia and
fucking with you on the exam. Thaiami.
(syndrom e)
*Globus is typically
Many are asymptomatic. Others go involvedfirst
insane and start stumbling around

Also called PKAN (pantothenate kinase-


associated neuropathy) for the sole
purpose o f fucking with you on the exam.
H a lle rv o rd e n
Etiology: Iron in the Globus Pallidus
S patz
Comparison
T2 Dark G lobus with central bright area
of necrosis “ Eye o f the T iger” . No T2: Dark Medial Basal Ganglia (Globus),
enhancement. No Restricted Diffusion. with centraI high signal dot (necrosis)

A m yotrophic Upper motor neuro loss in the brain and Does NOT show gross volume loss.
L a te ra l spine. Most people die within 5 years T2/FLAIR tends to be Normal (rarely can
S clerosis (unless you are really good at physics). be bright in the posterior internal capsule).

Tauopathy (whatever the F that means).


C o rtic o -b a s al
Asymmetric frontoparietal atrophy.
D e g e n era tio n Awesome clinical manifestations like the
“Alien limb phenomenon ” -50% o f cases.
One o f those AD repeat sequence things. C audate A trophy and reduced FDG uptake.
What Sequence ? 38 CAGs Mother The frontal horns will become enlarged and
Fuckers. Yes, I still remember that outwardly convex (from the atrophy pattern)
worthless factoid from Step 1.

H untington Why? It’s a curse. My mind is like a bear


trap, you gotta chew your leg off to get
D isease out. So, between Step 1 & the CORE
exam. I’ve got tons o f worthless bullshit
up there.
Remember these poor guys turn into huge
assholes - then start flopping around. Normal Comparison Huntingt

Mitochondrial Disorder T2/FLA1R bright lesions in the Brainstem,


Leigh
Basal Ganglia , and Cerebral Peduncles.
D isease Elevated Lactate peak at 1.3 ppm They can restrict, but do NOT enhance.

Mitochondrial Disorder Atypical strokes in the cortical gray matter


with a nonvascular distribution (usually
MELAS
Lactic Acidosis, Seizures, and Strokes occipital and parietal).
Syndrom e
Elevated Lactate “doublet” at 1.3 ppm Underlying WM is normal

Lysosomal Storage Disease / Inferior Anterior Beak


Mucopolysaccharidoses
H u rle r
Syndrom e (1) Macrocephaly with Mctopic “beak”
(2) Enlarged Perivascular Spaces
(3) Beaked Inferior L 1 Vertebral Body “Httrler Holes ”

64
The Defias Brotherhood of Heurodegeneration - Part 2
Classic Clinical Hx: Resting tremor, Rigid /
Slow movements (shuffling gate, etc..).
Impossible to diagnose on CT or MR alone -
Etiology: Reduced dopaminergic input to but supposedly has mild midbrain volume
striatum (whatever the fuck that means). loss with a “butterfly” pattern (this would
P arkinson have to be stated, it is too subtle to show).

D isease DAT Scan - loflupane 123 - This exotic


Nukes study is certainly fair game for an Worth noting in the sparing o f the midbrain
(PD) and superior cerebellar peduncles. This is a
“intermediate level” exam.
ABnormal fairly high yield piece o f trivia as it helps
(Periods) distinction Parkinsons from multi-system
atrophy.
Normal Possible Parkinsonian Syndrome
(PD, MSA or PSP)

This is a monstrously complex entity, that is


“Parkinson- actually 3 separate renamed entities
(“P”, “Q " an(j “A ”).
P lus”
The highest yield pearl is the appearance o f
M ulti- the Cerebellar subtype MSA-CI
System
Trivia: 1-123 M1BG can be used to
A trop hy Cerebellar Hemisphere / Hot Cross Bun
differentiate PD from MSA, by looking at
(MSA) Peduncle Atrophy with a Sign (loss o f the
the cardiac/mediastinal ratio (which is
Shrunken Flat Pons & transverse fibers)
normal in MSA, and abnormal in PD)
an enlarged 4th vent.

• Also called Steele-Richardson-Olszewski >. c


,P 9
“Parkinson- for the purpose o f fucking with you.
Plus” • PSP = Most Common Parkinson Plus
• Unlike PD & MSA, PSP is a Tauopathy
(whatever the fuck that means).
P rogressive
Micky Mouse Sign: Tegmentum Atrophy
S upra­
with Sparing o f the Tectum & Peduncles.
n u c lea r
*If needed anatomy refresher - page 14
Palsy
Hummingbird Sign: Midbrain volume loss
(PSP) S elective M idbrain A trophy, w ith
with a concave upper surface + relative
sparing o f the Pons. Sparing o f P ons (divided by line)

AR copper metabolism malfunction. Once


the liver fills up with copper it starts
spilling over into other organs including the
brain.
Trivia: “Kayser-Fleischer Rings” - seen in
95% o f patients. Prepare the Slit Lamp. T1 and T2 Bright Basal Ganglia
Trivia: Cortical Atrophy is the most T2 Bright Dorsal Medial Thalamus
W ilson common CT finding (although obviously
D isease veiT non-specific).
Trivia: T 1 Bright BG is the most common
initial MR findings (supposedly).
Trivia: Copper has been suggested to be
Metro Man’s only vulnerability (this is Normal Comparison
controversial). Since literally anything is
fair game for the exam, I figured 1 better Panda Sign: T2 Bright Tegmentum with
mention that for completeness. normal dark red nuclei & substantial nigra

65
Deep Brain Stimulators
I want to quickly touch on deep brain stimulators. These things are used in the treatment o f Parkinson
disease, essential tremor, and chronic pain.

It is common to get a CT
immediately after DBS placement to
evaluate for correct positioning of
the electrode or any obvious
complications (bleeding, etc...).
Knowing the “correct” position is the
most useful piece o f trivia.

For Parkinson Disease, the


electrodes are typically positioned in
the sub thalamic nucleus with the
tips o f the electrons located 9 mm
from the midline (just inside the
upper most margin of the cerebral
9 mm 9 mm
peduncle).

66
S E C T IO N 6:
In t r o t o M R S ,
♦ L e u k o d y s t r o p h i e s , 8c F r i e n d s #
Introduction to MRI Spectroscopy
The old joke in Neuroradiology is if you need to use MRS to figure something out, then you need to
go back and read the book again - starting at page 1. Someone told me that’s Yousem’s joke, but I
don’t see how that is possible - because it’s actually a little funny. Regardless o f who made the joke
first, there is near universal acceptance that MRS is “of limited clinical utility” (a worthless turd).
Therefore, it is fair game for an intermediate level exam and we should at least talk about it a little.
I’m not going to get into the physics much here (there will be a write up in the new 3rd Edition o f the
War Machine covering that). For now, I’m going to give a very basic overview and emphasize the
pathology / clinical trivia. Then I’ll be sprinkling more MRS in sporadically throughout the chapter.

Overview: The general idea is that the various metabolites which exist on the cellular level (choline,
lactate, N-acetylaspartate “NAA,” etc... etc..., so on and so forth...) occur in different concentrations
depending on the pathology. For example, “NAA” is a neuronal marker. Things that destroy neurons
(like tumors) will decrease NAA. So, in general the lower the NAA the higher the grade tumor.

You will see a graph like this one, with “PPM” on the X-Axis, and “Intensity” on the Y-Axis.

Intensity is C reatine Norm al


going to tell you
“how much” of a
thing there is. >- Choline
co
It’s not a raw
UJ
number, and E-
better thought of
as a ratio.

4.0 ppm 3.0 ppm 2.0 ppm 1.0 ppm

PPM stands for parts per million. Better understood as a percent of the Larmor Frequency
(1 ppm = 1 millionth of the Larmor frequency). This is important because each metabolite will
have a unique frequency distribution. For example NAA is at 2.0 ppm.

Why are the numbers counting backwards on the scale ? I’m going to answer this with the same
explanation I received as a small child when I asked why I couldn’t just eat my dessert first, and my
vegetables last — “because I’m your mother that’s why!”

H u n te r’s Angle: This is a method to quickly Reversed H u n te r's


decide if the MRS is normal or not. Under Angle in a High G rade
normal conditions Choline, Creatine, and NAA Gliom a (GBM)
should ascending in that order. Using a line to
connect the tips gives you a 45 degree~ish angle.
If it slopes the other way (as shown) then it is
not normal.

67
MRI Spectroscopy High Yield Pearls
Product of brain
destruction - Necrotic Tissue (spilling of membrane lipids).
lipids are present
Lipid 0.9-1.4
in necrotic brain Elevated with high grade tumors, brain infarcts,
tissue (necrosis and brain abscess.
marker).
Classic Trivia: It’s
normal to see lactate
elevated in the first
hours of life
Brain tumor has outgrown
Product of
its blood supply - is
anaerobic Classic Trivia: Lactate
forced into anaerobic
metabolism. and Lipid peaks
Lactate 1.3 pathways for metabolism.
Absent under superimpose - you
normal need to use an
conditions. Also elevated with intermediate TE
cerebral abscess.
(around 140) to causes
an “inversion” of the
lactate peak (so you
can see it)
Alanine 1.48 Amino Acid Found in Meningiomas
Neuronal Marker
Glial tumors have NAA.
(Neuron Classic Trivia: NAA
N -acetvlaspartate The higher the Glial
2.0 Viability). peak is super high
“ NAA” tumor grade, the lower
Usually the tallest with Canavans.
the NAA
peak.

G lutam ine -
2.2-2.4 Neurotransmitter Increased with Hepatic Encephalopathy
“ GLX ”
Energy
C reatine - “ C r” 3.0 Decreased in tumor necrosis.
Metabolism
More turnover more Choline
3.2 Cell Membrane
Choline - “ Co” (thus elevated in high grade tumor, demyelination,
Turnover
inflammation).
- Elevated in low grade
gliomas.
Cell Volume - Elevated in - Reduced in high
Regulator and
Alzheimer’s (decreased grade gliomas
M yoinositol -
3.5 in other dementias)
“ m l” Byproduct of
- Elevated in Progressive - Reduced in Hepatic
Glucose
multifocal Encephalopathy
Metabolism. leukoencephalopathy
(PML)

t h is vs THAT: Demyelinating vs Dysmyelinating


ofscasc'1131' 1^ Example = MS Disease that destroys normal myelin

Dysmyelinating Example = Mctachromatic Disease that disrupts the normal formation


Disease Leukodystrophy and turnover of myelin

68
Leukodystrophies &Friends
On the prior page, I introduced the vocab work "dysm yelinating” disease. Leukodystrophies are the classic
exam ple o f this group o f pathologies. Technically speaking Leukodystrophies can occur from deficiencies
in lysosomal storage, peroxisom al function, or m itochondrial dysfunction. I'm gonna hit on m itochondrial
diseases separately as they tend to be m ore asym m etric and favor the grey matter. W here as the classic
forms target the w hite m atter in a m ore sym m etric and extensive manner.
The distinction betw een the Leukodystrophy subtypes is totally academ ic m ental m asturbation, since they
are all untreatable and fatal. Therefore, distinguishing betw een them is fair gam e on an interm ediate level
exam (and specifically listed on the official study guide).

Leuko dystrophy = Fucked W h ite M a tte r in a Kid

Parieto-occipital Sex-linked recessive


Adreno (peroxisom al
Predominance
Leukodystrophy Normal enzym e deficiency)
(ALD) Head Male Predominant
“Extends across the
Size
Splenium o f the Corpus Can Enhance &
“X-Linked”
Callosum” R estrict
FLAIR

Frontal Predominance
Most common
Normal Periventricular and
Leukodystrophy.
Metachromatic Head Deep White Matter -
Size Tigroid Pattern U-fibers are
(stripes o f milder relatively spared
disease). FLAIR

Also hits the


cerebellum and
Weird Frontal middle cerebellar
Alexander Disease Predominance
Bie Head peduncles
Can Enhance
FLAIR

Diffuse Bilateral
Weird subcortical U fibers. Elevated NAA
Canavan Disease (MRS).
Bin Head “Subcortical
Predom inance”
FLAIR

High density foci on


Centrum semiovale and CT (in the thalamus,
Small periventricular white caudate, and deep
Krabbe white matter).
Head matter with parieto­
occipital predominance Earlv sparine o f the
subcortical U fibers. FLAIR

Typically diffuse “total lack o f normal


myelination” with extension to the subcortical FLAIR
U fibers.
Normal
Pelizaeus- Head Patchy variant is also described as “tigroid” -
Merzbacher Size although that term is more classic for
Metachromatic
No enhancement. No restricted diffusion.

69
Leukodystrophies &Friends

As discussed on the prior page Leukodystrophies can occur from deficiencies in lysosomal storage,
peroxisomal function, or mitochondrial dysfunction. The classic forms tend to target the white matter in a
more symmetric and extensive manner. This is different than mitochondrial diseases which are more
asymmetric and favor the grey matter. Grey Matter needs more oxygen than White Matter (and White
Matter needs more oxygen than trial lawyers). Inability to process oxygen (mitochondrial dysfunction) -
helps me remember the grey matter > white matter thing.

M ELAS - Mitochondrial Enccphalomyopathy, Lactic


Acidosis, and Stroke-like episodes. This is a
mitochondrial disorder with lactic acidosis and stroke
like episodes.

Tends to have a parietooccipital distribution

BUZZW ORD(s): “Migrating Infarcts”


Increased Lactate, Decreased NAA.

Leigh Disease - Also called Subacute Necrotizing Encephalo-Myelopathy - for the purpose o f fucking
with you.

White Matter Distribution: Focal areas o f subcortical white matter.


Gray Matter Distribution: Basal ganglia and Periaquaductal Gray

Trivia: Head size tends to be normal.

70
S E C T IO N 7:

t B rain T u m o r s b ^ ibo^ b m b m

I want to introduce my idea for m ultiple choice brain tum or diagnosis. The strategy is as
follows; (1) decide if it's single or m ultiple, (2) look at the age o f the patient - adults and kids
have different differentials, (3) look at the location - different tum ors occur in different spots,
(4) now use the characteristics to separate them. The strategy centers around narrow ing the
differential based o ff age and location till you are only dealing w ith 3-4 com m on things, then
using the im aging characteristics to separate them. It's so m uch easier to do it that way.

Multifocal Primary from Seeding

Multiple Masses ——— Mets (*5 0 % at solitary)

Syndromes (example NF-2)

Cortical

Intraventricular

Adult -----► C P Angle

Single Mass \ Infratentorial

Before we get rolling, the first thing to do is to ask y o u rself is this a tumor, or is it a m im ic?
M im ics w ould be abscess, infarct, or a big M S plaque. This can be tricky. I f you see an
incom plete ring - you should think giant MS plaque. If they show you diffusion, it is either
lym phom a or a stroke (or an abscess) - y o u ’ll need to use enhancem ent to straighten that out
(rem em ber lym phom a enhances hom ogeneously).

Y es... GBM can restrict, but for m ultiple choice it is w ay m ore likely to be lym phom a.

71
Two m ore high yield topics before w e start crushing the differentials:

intra-Axial” vs “Extra-Axial”
The Brant and Helm s discussion on brain tum ors will have you asking “ intra-axial” vs
“extra-axial” first. T his is not alw ays that sim ple, but it does lend itse lf very well to
m ultiple choice test questions (therefore it’s high yield).

B asically y o u n e e d to m em orize the "signs o f extra-axial location "

• CSF C left
D isplaced Subarachnoid Vessels
C ortical G ray m atter betw een the m ass and w hite m atter
D isplaced and expanded S ubarachnoid spaces
• Broad Dural Base / Tail
• Bony Reaction

Why DoThings EnhanceP


U nderstanding the W H Y is very helpful for problem solving. Let m e first answ er the
question “W hy D O N 'T things en h a n ce ?” They D O N ’T enhance because o f the blood brain
barrier. So, w hen things DO enhance it's because either:
(a) T hey are outside the blood brain barrier (they are extra-axial), or
(b) T hey have m elted the blood brain barrier.

In other w ords, extra axial things (classic exam ple is m eningiom a) will enhance. High
grade tum ors (and infections) enhance. Low grade tum ors ju st aren't nasty enough to
take the blood brain barrier dow n.

A re there exceptions? HA! T here alw ays are. A nd Yes... they are ALWAYS testable.

G angliogliom as and Pilocytic A strocytom as are the exceptions - they are low -grade
tum ors, but they enhance.

72
Multiple Masses
In adults or kids, if you see multiple masses you arc dealing with mets (or infection). Differentiating
between mets and infection is gonna be done with diffusion (infection will restrict). If they want you
to decide between those two they must show you the diffusion otherwise only one or the other will be
listed as a choice.

M ets — H ig h Yield Trivia:

• Most common CNS met in a kid = neuroblastoma (BONES, DURA, ORBIT - not brain)

• Most common location for mets = Supratentorial at the Grey-White Junction (this area has a lot of
blood flow + an abrupt vessel caliber change... so you also see hematogenous infection / septic
emboli go there first too).

• Most common morphology is “round” or “spherical”

• Remember that mets do NOT have to be multiple. In fact, 50% o f mets are solitary. In an adult, a
solitary mass is much more likely to be a met than a primary CNS neoplasm.

• M RC T is the mnemonic for bleeding mets (Melanoma. Renal, Carcinoid / Choriocarcinoma,


Thyroid).

• Usually Mets have more surrounding edema than primary neoplasms o f similar size.

• “Next Step Gamesmanship ” - Because the most common intra-axial mass in an adult is a met, if
they show you a solitary mass (or multiple masses) and want a next step it’s gonna be go hunting
for the primary (think lung, breast, colon... the common stuff).

P rim ary B ra in Tum ors C an A lso B e M u ltip le:

Tum ors that L ike to be M ultifocal T um ors that are M ultifocal from Seeding
Mets — you should still think this first when you
Mcdulloblastoma
see multiple tumors
Lymphoma Ependymoma

Multiccntric GBM GBM

Gliomatosis Cerebri Oligodendroglioma

SY N D R O M E S - Tum ors in Synd rom es are m ore likely to be M ultifocal


NF 1 NF 2 “MSME” Tuberous Sclerosis VHL

Optic Gliomas Multiple Schwannomas Subependymal Tubers Hcmangioblastomas

IV Giant Cell
Astrocytomas Meningiomas
Astrocytomas

Ependymomas

73
Cortically Based (p-dog):
Most intra-axial tumors arc located in the P-DOG:
white matter. So when a tumor spreads to or
is primarily located in the gray matter, you Pleomorphic Xanthoastrocytoma (PXA)
get a shorter DDx. High yield piece o f trivia Dysembryoplastic Neuroepithelial Tumor (DNET)
regarding the cortical tumor / cortical met is Oligodendroglioma,
that they often have very little edema and so Ganglioglioma
a small cortical met can be occult without IV
contrast.
PXA
PXA (P leo m o rp h ic X a n th ro a s tro c y to m a ):
PEDS (10-20)
Superficial tumor that is ALWAYS supratentorial and
usually involves the temporal lobe. They are often in Will Enhance
the cyst with a nodule category (50%). There is usually
Dural Tail***
no pcritumeral T2 signal. The tumor frequently invades
the leptomeninges. Looks just like a Desmoplastic Cyst with Nodule
Infantile Ganglioglioma - but is not in an infant.
Temporal Lobe

D N E T (D y s em b ry o p la stic N e u ro e p ith e lia l Tum or) DNET


Kid with drug resistant seizures. The mass will always be in
PEDS (< 20)
the temporal lobe (on the test - real life 60% temporal).
Focal cortical dysplasia is seen in 80% o f the cases. It is No enhancement
hypodense on CT, and on MRI there will High T2 Signal
be little if any surrounding edema. with Bright
High T2 signal “bubbly lesion.” FLAIR Rim

Bright Rim Sign - “Bubbly”


Persistent rim o f FLAIR signal Temporal Lobe
* Looks Similar to T2-FLAIR
Mismatch o f Astrocytomas O ligodendrogliom a
**discussed later „ .. , , , „ FLAIR
T2 Bright & Bubbly ADULT - (40s-50s)
Bright Rim
O ligodendrogliom a: Can Enhance
Remember this is the guy that calcifies 90% of the time. It’s Calcification Common
most common in the frontal lobe and the buzzword is
“Expands the Cortex”
“expands the cortex”. This takes after its most specific
feature of cortical infiltration and marked thickening. It’s 0 Frontal Lobe
likely you could get asked about this Ip/I9q deletion which I ^
1p /19q
will discuss later when I go into detail about Gliomas (pg 83).
G angliogliom a
G angliogliom a:
Any Age
This guy can occur at any age, anywhere (usually temporal
lobe), and look like anything. However, for the purpose of Can Enhance
multiple choice testing the classic scenario would be a 13
year old with seizures, and a temporal lobe mass that is
cystic and solid with focal calcifications. There may be Mixed Cystic Can look like Anything
& Solid
overlying bony remodeling. Temporal Lobe

74
Intraventricular
Tum ors can arise from the ventricular w all, septum pellucidum , or choroid plexus.

V e n tric u la r W all &


Choroid P lexus M isc
S eptum P ellucidum

C horoid Plexus Papillom a

F.pendvm oma (TED S) (PE D S in T rigone) M ets

(A D U LT in 4th Vent)

C horoid Plexus
M edulloblastom a (TED S) M eningiom a
C arcinom a (TED S)

SEG A (Subependym al
X anthogranulom a
G iant Cell A strocytom a) = C olloid C yst
t “ Found“ in A D U L T S)
PEDS

Subependym om a (A D U LT)

C entral N eurocytom a
(Y O U N G ADULT)

V e n t r ic u la r W all I S e p tu m P e llu c id u m O rig in :

E p e n d y m o m a : B im odal distribution on this one (large peak around 6 years o f age,


tiny peak around 30 years o f age). I w ould basically think o f this as a PE D S tum or.

They com e in two flavors:

(a) 4th Ventricle - w hich is about 70% o f the tim e. T here is frequent extension into the
foram en o f Luschka and M agendie. They are the so-called “ plastic tu m o r” or
"toothpaste ” tum or because they squeeze out o f the base o f the 4 th ventricle.

(b) Parenchym al Supratentorial - w hich is about 30% o f the tim e.


These are usually big (> 4cm at presentation).

75
M e d u llo b la s to m a : Let us ju st assum e we are talking about the “C lassic M edulloblastom a”
which is a type o f PNET. If you w ant to understand the genetic spectrum o f these things, read
O sborn’s Brain — seriously d o n ’t subject y ourself to that.

This is a pediatric tum or - with most occurring before age 10 (technically there is a second
peak at 20-40 but for the purpose o f m ultiple choice tests I ’m going to ignore it). These guys
are cerebellar arising from verm is / RO O F o f the 4th ventricle - project into 4th ventricle. They
are m uch m ore com m on than their ch ief differential consideration the Ependym om a (which
originates from the FLO O R o f the 4th ventricle).

The classic look is a dense m ass on CT, heterogeneous on T1 and T2, and enhances
hom ogeneously. They are hypercellular and m ay restrict. They calcify 20% o f the tim e (less
than Ependym om a).

This is a tum or that loves to m et via CSF pathw ays — they like to “drop m et.” The buzzw ord is
“zuckerguss” which apparently is G erm an for sugar icing, as seen on post contrast im aging o f
the brain and spinal cord (leptom eningeal carcinom atosis). As a point o f absolute trivia, they
are associated with B asal C ell N evus Syndrom e a n d Turcots Syndrome.

G o rlin S y n d ro m e - If you see a M edulloblastom a next look for dural calcs.


If you see thick dural calcs you m ight be dealing with this syndrom e.
»'v i ‘\ They get basal cell skin cancer after radiation, and have odontogenic cysts.

N E X T STE P Trivia: Preoperative im aging o f the entire spinal axis should be done in
any child with a posterior fossa neoplasm , especially if M edulloblastom a or
Ependym om a is suspected. Evidence o f tum or spread is a statistically significant
predictor o f outcom e.

M e d u llo b la s to m a E pendym om a
M ore com m on Less C om m on
O riginate from Vermis / O riginate from the
RO O F o f the 4th Ventricle FLO O R o f the 4th ventricle.

Can extend into basal cisterns like tooth


Can project into 4th ventricle, do N O T usually
paste pushing though foram ina o f
extend into basal cisterns
Luschka and M agendie
E nhance H om ogeneously
E nhance H eterogeneously
(m ore so than E pendym om a anyw ay)

C alcify Less (20% ) C alcify M ore (50% )

L inear “ icing-like” enhancem ent o f the brain


surface is referred to as “Z uckerguss”

76
Subependym al G iant C e ll A stro cyto m a (S E G A ): This is going to be show n
in the setting o f TS. They will m ore than likely show you renal A M L s or tell you the kid has
seizures / developm ental delay.

Because it’s syndrom ic, you see it in kids (average age 11).

It will arise from the lateral wall o f the ventricle (near the foram en o f M onro), often causing
hydrocephalus. It enhances hom ogeneously.

T H IS vs THAT: SEG A vs Subependym al N odule (SEN ) - T he SEN will stay stable in size,
the SEG A will grow. T he SE G A is found in the lateral ventricle near the foram en o f
M onroe, the SEN can occur anyw here along the ventricle. SEN s are w ay m ore com m on.
Both SEN and SEG A can calcify.

jlv Pearl - E nhancing, partially calcified lesion at the foram en o f M onro,


V ^ bigger than 5 m m is a SEG A not a SEN.

— (the next 2 IV tum ors are in A D U LTS) —

Subependym om a: Found in AD ULTS. W ell-circum scribed IV m ass m ost com m only


at the foram en o f M onro and the 4th ventricle. T hey can cause hydrocephalus. They
typically d o n ’t enhance. They are T2 bright (like m ost tum ors).

Central N eurocytom a:
This is the m ost com m on IV
m ass in an A D U L T a g e d
20-40. The buzzw ord is
“sw iss ch eese,” because o f
the num erous cystic spaces
on T2. They calcify a lot
(alm ost like
oligodendrogliom as).

Central Neurocytoma - Two E xam ples - Cystic IV Mass

Swiss Cheese +
Calcification in the
Ventricle

77
C horoid P le x u s O rigin:

C horoid P lexus P apillom a / Carcinom a: Can occur in peds (85% under the age o f 5) or
adults. They m ake up about 15% o f brain tum ors in kids under one. B asically you are
dealing w ith an intraventricular m ass, w hich is often m aking CSF, so it causes
hydrocephalus.

H ere is the trick: Brain tum ors are usually supratentorial in adults and posterior fossa
▼ in kids. This tum or is an exception. R em em ber exceptions to rules are testable.

rw i • •
Tnvia:

• In A dults it’s in the 4th V entricle, in K ids it’s in the lateral ventricle (usually trigone).

• C arcinom a type is ONLY SEEN IN KIDS - and are therefore basically ONLY SEEN IN
LA TERA L V E N T R IC L E / T R IG O N E

• C arcinom a association w ith L i-Fraum eni syndrom e (bad p53)

• A ngiography m ay show enlarged choroidal arteries w hich shunt blood to the tum or,

• C arcinom a type o f this tum or looks very sim ilar (unless it's invading the parenchym a) and
is alm ost exclusively seen in kids.

• The tum or is typically solitary but in rare instances you can have C SF dissem ination

X anthogranulom a -
T his is a benign choroid plexus
m ass. You see it all the tim e (7% )
and d o n ’t even notice it.

The trick is that they


restrict on diffusion, so
they are trying to trick you
U™ into w orking them up.
T hey are b e n ig n ... leave
Xanthogranuloma - Note the Restricted Diffusion
them alone.

78
M isc:

Mets - The m ost com m on location o f intraventricular m etastasis is the trigone o f lateral
ventricles (because o f the vascular supply o f the choroid). T he m ost com m on prim ary is
controversial - and either lung or renal. I f forced to pick I’d go Lung because it’s m ore
com m on overall. I think all things equal renal goes m ore - but there are less renal cancers.
It all depends on how the question is w orded.

Colloid C y st - These are found alm ost


exclusively in the anterior part o f the 3rd
ventricle behind the foram en o f M onro.

They can cause sudden death via acute onset


hydrocephalus.

T heir appearance is som ew hat variable and


depends on w hat they are m ade of. If they have
cholesterol they will be T1 bright, T2 dark. If
they d o n ’t, they can be T2 bright. The trick is a
round w ell circum scribed m ass in the anterior
3rd ventricle. If show n on CT, it w ill be pretty
dense.

C olloid C yst -
- A nterior 3rd Ventricle
- H yperdense on CT

M eningiom a - Can occur in an intraventricular location, m ost com m only (80% ) at the
trigone o f the lateral ventricles (slightly m ore on the left). D etails on m eningiom as are
discussed on the follow ing page.

79
C e re b e lla r P o n tin e A n g le (CPA)

Age is actually less o f an issue here because the DDx isn’t that big. Most o f these are adult
tumors, but in the setting o f NF-2 you could have earlier onset.

Epidemiology: Vestibular Schwannom a is #1 - making up 75% o f the CPA masses, #2 is the


m eningiom a m aking up 10%, and the Epiderm oid is #3 m aking up about 5%. The rest are
uncommon.

(75 % ) S c h w a n n o m a (V e s tib u la r) - These guys Schw annom a M eningioma


account for 75% o f CPA masses. W hen they are bilateral
you should imm ediately think NF-2 (one f o r each side).
Enhances strongly but more heterogeneous than Enhance Less Enhance
Homogeneously Homogeneously
meningomas. May widen the porus acousticus resulting
in a “trum pet shaped” IAC. "Ice Cream Cone IAC. ”

Don't Usually
(10 % ) M e n in g io m a - Second more common CPA Invade IAC
Invade IAC
mass. One o f the few brain tumors that is more common
in women. They can calcify, and if you are lucky they
will have a dural tail (which is pretty close to
IAC can have
pathognom onic - with a few rare exceptions). Because Calcify more
“trumpeted”
they are extradural they will enhance strongly. Radiation often
appearance
o f the head is known to cause meningiomas.

Trivia:

•M ost common location o f a m eningiom a is over the cerebral convexity.

^Meningiomas take up octreotide and Tc-MDP on Nuclear M edicine tests (sneaky).

(5 % ) E p id e rm o id -

Can be congenital or acquired (after trauma - classically after LP


in the spine). Unlike derm oids they are usually o ff midlinc.
They will follow CSF density and intensity on CT and MR1
(the exception is this zebra called a "white epiderm oid" which is
T1 b r ig h t- just forget I ever mentioned it).

The key points are

(1) Unlike an arachnoid cyst they are bright on FLAIR


(som etim es warm - they don't completely null), and

(2)They will restrict diffusion.

E piderm oid - Follow s CSF


Signal - R estricts D iffusion

80
Derm oid C y st - This is about 4x
T h e R u p tu red D erm oid
less com m on than an epiderm oid. It’s
m ore com m on in kids / young adults. It is possible for a derm oid cyst to explode
Usually m idline, and usually are -rare in real life, com m on on m ultiple choice.
found in the 3rd decade. They contain S om etim es this is after a traum a, but usually
lipoid m aterial and are usually it’s spontaneous. T he m ost com m on clinical
scenario is “headache and seizure” - w hich is
hypodense on CT and very bright on
pretty m uch every brain tum or, so that is not
T l. They are associated w ith NF2.
helpful. W hat is helpful is this:
Trivia -
• Buzzw ord: “C hem ical M eningitis ”
•These are usually m idline
• A unt M innie A ppearance: Fat droplets
• M ost com m on location for a (typically show n as low density on CT, or
A derm oid cyst is the suprasellar H igh Signal on T i l floating in the ventricles
4911 cistern (posterior fossa is #2) and/or subarachnoid space.

TH IS vs THAT: D erm o id vs E pid erm o id — T he easy w ay to think o f this is that the


E piderm oid behaves like CSF, and the D erm oid behaves like fat.

IA C Lipom a - It can occur, and is basically the only reason you get a T l w hen you are
w orking up CPA m asses. It w ill fat sat out - because it’s a lipom a. T here is an association
with sensorineural hearing loss, as the vestibulocochlear nerve often courses through it.

A rachnoid C y st - C om m on benign lesion that is


H o w can yo u tell an ep id erm o id
located w ithin the subarachnoid space and contains from an a ra c h n o id cyst?
CSF. They are increased in frequency in
m ucopolysaccharidoses (as are perivascular spaces). T he epiderm oid restricts,
They are dark on FL A IR (like C SF), and w ill N O T the arachnoid cyst does NOT.

restrict diffusion.

81
Infratentorial - Most are PEDS (Hemangioblastoma is the exception).

A typ ic a l T e ra to m a I Rhabdoid Tum or THIS vs THAT:


AT/RT v s M edulloblastom a
(“A T/RT”) - Highly malignant tumors (WHO IV), and
rarely occur in patients older than 6 years. The average Both are WHO Grade 4 destroyers
age is actually 2 years, but they certainly occur in the (AT/RT is worse) that are often seen in the
posterior fossa o f a kid.
first year of life.
Technically they are both subtypes o f
They can occur in supra and infratentorial locations Medulloblastoma - but that’s the kind of
(most common in the cerebellum). These are usually knowledge that causes you to miss
multiple choice questions. For the
large, pissed off looking tumors with necrosis and purpose o f multiple choice:
heterogeneous enhancement. They believe in nothing
• AT/RT is a 2 year old
Lebowski. They fuck you up. They take the money. • Medulloblastoma is a 6 year old

B u zzw ord = • AT/RT has calcifications


• Medulloblastoma does not
JsP “Increased Head Circum ference ”

M edu llo b lasto m a & E pendym om a : Both are discussed with the IV lesions

Ju ven ile P ilo c y tic A stro c y to m a


(JPA): Just think cyst with a nodule in a kid.

They are WHO grade 1, but the nodule will still


enhance. This will be located in the posterior
fossa (or optic chiasm).

Pilocytic Astrocytoma: Cyst + Nodule in Kid

Gamesmanship — if they don’t tell you the age, you can look for enhancement o f the cystic wall which
JPA can have (-50% ) but Hemangioblastomas don’t
I Say Posterior Fossa Cyst
H em an g io b lasto m a: First things first - immediately think with a Nodule - PEDS,
about this when you see a cyst with a nodule in an ADULT. Then
think Von Hippel Lindau, especially if they are multiple. These you say JPA
things are slow growing, indolent vascular tumors, that can cause
hydrocephalus from mass effect. 70% o f the time you will see flow I say Posterior Fossa Cyst
voids along the periphery o f the cyst. About 90% of the time they with a Nodule - ADULT,
are found in the cerebellum. There is an association with
you say Hemangioblastoma

G angliogliom a: Occurs at any age, anywhere, can look like anything - see cortical lesions.

D iffuse Pontine G liom a (DPG): Seen in kids age 3-10. Most common location is the pons,
which is usually a high grade fibrillary glioma. It’s going to be T2 bright with subtle or no
enhancement. 4th ventricle will be flattened. Imaging features arc so classic that no biopsy is needed.

82
Supratentorial - A du lts Tum ors
A strocytom a: Most common primary brain tumor in adults. There is a trend towards “genetically
classifying” tumors - this actually changes the way they arc treated and could be the source of trivia.
I’m going to attempt to simply this - because it can get pretty fucking complicated.

In the simplest terms, you have the neurons and you have the glial Neuron
Neural Progenitor
cells. The glial cells are the “support staff’ — there are lots o f them
and lots o f different kinds. Astrocytes and Oligocytes share a
common origin (both are support staff - “glial cells”) and have a lot
of similarities. In other words, they are both “Gliomas” and are
going to get lumped together in this discussion.
Stem Cell
The new way to think about these things is a spectrum of severity
based on genetic classification - and the treatment and prognosis
follows that. Glial Progenitor

IDH M utation Oligocyte


(earliest genomic event)

IDH M utation
(earliest genomic event)

Yes (10%) No (90% )

Oligodendroglioma Astrocytoma Astrocytoma


/
Astrocytoma Astrocytoma
Low Grade Higher Grade Grade 4 - Grade 4 -
Calcification on -Grade 2 - Diffuse Glioblastoma Glioblastoma
preoperative CT is -Grade 3 - Anaplastic - Younger Patients -Older Patients
associated with -Better prognosis -The Worst
codeletion (Ip/I9q) T 2 - FLAIR -Probably Prognosis
Mismatch Sign “Secondary GBM ” -Probably
from progression of “Primary GBM”
Ribbon pattern of a previous lower-
calcification grade tumor
Classic for
Oligodendroglioma

Bad Prognosis Abandon All Hope

You probably noticed me using this WHO classification (1-4). All brain tumors are bad, but 4 is the
worst - this is your GBM. On the following page, I'll get into a few more details on each type but as a
general rule low grade tumors don’t typically enhance (WHO 2) and higher grades do (mild for grade
3, and intense for grade 4 GBM). The exception to this rule is the pilocytic astrocytoma which often
has an enhancing nodule, and the Subependymal Giant Cell Astrocytomas which enhances because o f
its location (Intraventricular).

GBM is the beast that cannot be stopped. It believes in nothing Lebowski. It grows rapidly, it can
necrose (creating the ring o f enhancement, with a non-enhancing central necrotic c o re ), it can cross
the midline, and it can restrict diffusion. Remember Turcot Syndrom e (that G Ipolyp thing), and NF
1 are associated with GBMs.

83
Supratentorial - A du lts Tum ors - Continued

Astrocytoma Astrocytoma Astrocytoma Astrocytoma


Grade 1 Grade 2 - Diffuse Grade 3 - Anaplastic Grade 4- GBM
Subependymal Giant White Matter is White Matter is White Matter is
Cell Astrocytomas Preferred Preferred Preferred - can cross
-Intraventricular mass the midline.
near the foramen of NO ENHANCEMENT Mild ENHANCEMENT
Monro in a young RING ENHANCEMENT
patient with tuberous T2 Bright - FLAIR Iso T2 Bright-FLAIR Iso (can also be diffuse
sclerosis. (mismatch sign) (mismatch sign) heterogenous
-Can cause obstructive enhancement)
hydrocephalus
T2 & FLAIR Bright

Pilocytic Astrocytoma
Central locations (like
- Cyst with nodule in the thalamus) are
the posterior fossa of a worse than normal.
kid

Remember these NF type 1,


tumors break the rule - Turcot syndrome,
T2 / FLAIR Mismatch: Seen with WHO 2
and enhance despite Li Fraumeni syndrome
(diffuse) and 3 (anaplastic) astrocytoma, not
being low grade.
with WHO 1. T2 tumor has high signal with
surrounding vasogenic edema. On FLAIR the
tumor signal become isointense.

G liom atosis C erebri: A diffuse glioma with extensive infiltration. It involves at least 3 lobes
and is often bilateral. The finding is usually mild blurring o f the gray-white differentiation on CT, with
extensive T2 hyperintensity and little mass effect on MR. It’s low grade, so it doesn’t typically
enhance.

Mets: The most common supratentorial mass. Just like mets favor the lower lobes in the lungs, the
cerebrum is favored over the cerebellum (it is a blood flow thing). They arc usually multiple, but can
be solitary — some sources say 50% o f the time, so don’t be fooled a solitary lesion can totally be a
met. Some other trivia worth knowing — melanoma can be T1 bright even if it doesn’t bleed.
CT-MR is a good way to remember the ones that like to bleed (Choriocarcinoma / Carcinoid, Thyroid,
Melanoma, Renal).

M e ta s ta tic GBM
Irregular
Margin f Spherical

-Multifocal -Solitary
(25-50% solitary) (25%> multifocal)
-Favors -Favors
Grey-White Junction Deep White Matter

84
Supratentorial - A du lts Tum o rs - Continued
P rim ary CNS Lym phom a: Seen in end stage AIDS patients, and those post-transplant. EB
virus plays a role. Most common type is Non-Hodgkin B cell.

Classic picture would be an intensely enhancing homogeneous solid mass in the periventricular
region, with restricted diffusion. However, it can literally look like and do anything.

Classic Multiple choice test question is that it is Thallium Positive on SPECT


(toxo is not).

m ™ I say restricting brain tumor, you say Lymphoma (although GBM can do this also)

T H IS vs THAT: P e r iv e n t r ic u la r / E p e n d y m a l E n h a n c e m e n t P a t t e r n s

«
3
0X5
E pendym itis - £
(C lassic E xam ple " 11V Lym phom a
= CM V) "c JM1 " R im P h o m a ”

2
H

85
Supratentorial - Ped s Tum o rs
DNET & PXA ( P le o m o r p h ic X a n th r o a str o c y to m a ):
Discussed under the cortical tumors .

D e s m o p l a s t i c In fa n tile G a n g lio g lio m a I A s t r o c y t o m a “DIG”:

These guys arc large cystic tum ors that like to involve the superficial
cerebral cortex and leptomeninges. Unlike the Atypical Teratoma /
Rhabdoid, these have an ok prognosis (W HO 1). They ALWAYS arise in
the supratentorial location, usually involve more than one lobe (frontal
and parietal most comm only), and usually present before the first birthday.

o -B u zzw ord is “rapidly increasing head circum ference.”


Big Cystic Tumor
with Hydro
S kull B ase:

C h o r d o m a - This is a locally aggressive tum or that originates from the notochord.


WTF is the “notochord” ? It’s an embryology thing that is related to spine developm ent.

The thing you need to know is that the notochord is a m idlinc structure. Therefore all
Chordomas are midline - either in the clivus, vertebral bodies (especially C2), or Sacrum. You
can NOT get them in the hips, ribs, legs, arms, or any other structure that is not totally midline
along the axis o f the axial skeleton.

*lt is most comm on in the sacrum (#2 is the clivus)


w rl *When it involves the spine, it’s most common at C2 - but typically extends across a disc
space to involve the adjacent vertebral body.
J L W .I t’S T2 Bright
^ *Ifs ALWAYS M idline. — it is never in a leg, arm, e tc ... ONLY M IDLINE structures.

C h o n d r o s a r c o m a —This is the main differential o f the chordom a in the clivus. The thing
to know is that it is nearly always lateral to m idline (chordoma is midline). These are also T2
bright, but will have the classic “arcs and rings” matrix o f a chondrosarcom a. Obviously you’ll
need a CT to describe that matrix.

Dura:

M e n in g io m a - As described above, it is common and enhances homogeneously. The most


common location is over the cerebral convexity and it has been known to cause hyperostosis.

H e m a n g i o p e r i c y t o m a - This is a soft tissue sarcoma that can mimic an aggressive


m eningioma because they both enhance homogeneously. They also can mimic a dural tail, with a
narrow base o f dural attachm ent. They w on’t calcify or cause hyperostosis , but will invade the
skull.

M etS - The most common met to the dura is from breast cancer. 80% will be at the gray-white
junction. They will have more edema than a prim ary tum or o f sim ilar size.

86
Sella I Parasella - Adults

P itu ita ry A d e n o m a - The m ost com m on tum or o f the sella. They are seen 97% o f the
tim e in adults. If they are greater than 1 cm they are “m acroadenom as.” W hen functional,
most are prolactin secreting (especially in w om en). Sym ptom s are easy to pick up in wom en
(menstrual irregularity, galactorrhea). M en tend to present later because their sym ptom s are
more vague (decreased libido). On MR, 80% are T1 dark and T2 bright. They take up contrast
m ore slow ly than normal brain parenchym a. N ext step = Dynam ic contrast enhanced MR.

Things to know (about P ituitary adenom as):

• M icroadenom a under 10 mm,


• M acroadenom a over 10 mm.
m • M icroadenom as typically form in the adenohypophysis (anterior 2/3).
Jyk • Prolactinom a is the m ost com m on functional type.
• Typically they enhance less than norm al pituitary.

P itu ita ry A p o p le x y - H em orrhage or Infarction o f the pituitary, usually into an enlarged


gland (either from pregnancy or a m acroadenom a). Here are the m ultiple choice trivia
association: taking brom ocriptine (or other prolactin drugs), “Sheehan Syndrom e” in
postpartum w om an, Cerebral Angiography. They will be T1 bright (rem em ber adenom a is
usually T1 dark). Supposedly this is an em ergent finding because the lack o f horm ones can
cause hypotension.

R a th k e C left C y s t - U sually an incidental finding. Rarely sym ptom atic. The “ cleft” is
betw een the anterior and posterior pituitary. They are variable on T1 and T2, but are usually
very bright on T2. They do N O T enhance.

E p id erm oid - Discussed on page 80. R em em ber these guys restrict diffusion.

I say "M idline Suprasellar M ass that R estricts D iffusion ”, You sa y Epiderm oid.

C r a n io p h a r y n g io m a - They com e in tw o flavors: (a) Papillary - 10% and (b)


Adam antinom atous - 90% . The Papillary type is the adult type (Papi for Pappi). They are solid
and do not have calcifications. They recur less frequently than the A dam antinom atous form
(because they are encapsulated). They strongly enhance. The relationship to the optic chiasm is
key for surgery. These things occur along the infundibulum . Pediatric type is discussed below
(under on the next page with the peds tumors).

87
Sella I Parasella - Peds

C r a n io p h a r y n g io m a - A s stated above, they


com e in tw o flavors: (a) Papillary and
(b) A dam antinom atous. The kid type is the
A dam antinom atous form . These guys are calcified
(papillary is not). These guys recur m ore (Papillary
does less - because it has a capsule).
B uzzw ord is “m achinery oil. "

• T l Bright
• T2 Bright
Craniopharyngioma
• CT / G R E = C alcifications
- Shown on hone window
• Enhance Strongly (in the solid parts) - Calcifications in the Sella

H y p o t h a l a m i c H a m a r t o m a - A classic A unt ■ ■ ■ m iim iib b m

M innie. This is a ham artom a o f the tuber cinereum (part


o f the hypothalam us located betw een the m am m illary
bodies and the optic chiasm ). The location is the key. Ik

M am m illary Body H iiK , iH jli


M idbrain f W " - ' '
Tuber Cinereum

\ I L
£
<? Pons Hamartoma of the
Tuber Cinereum
Ham artom a

\ Would Be Here

Infundibulum

P ituitary

T l Iso
T2 Iso
Do N O T enhance.

The C lassic H istory is G e la stic S eizu res


(although precocious puberty’ is actually m ore com m on).

88
P in e a l R egion -

There are 3 m ain characters here, all o f w hich can present


w ith “vertical gaze palsy” (dorsal Parinaud syndrom e).

G e r m in o m a : The m ost com m on o f the 3, and seen


alm ost exclusively in boys (G erm inom as in the suprasellar
region are usually in girls). Precocious puberty m ay occur
from secretion o f hCG. C haracteristic findings are a m ass
containing fat and calcification with variable contrast
enhancem ent. It is heterogeneous on T1 and T2 (because
o f its m ixed com ponents). Germinoma:
“E ngulfed" Calcification Pattern

P in e o b la s to m a : Does occur in childhood. U nlike the


pineocytom a, these guys are highly invasive. Som e people
like to think o f these as PNETs in the pineal gland. They
are associated with retinoblastom a (“trilateral” ). T hey
are heterogeneous and enhance vividly.

P in e o c y to m a : Rare in child h o o d . W ell-circum scribed,


and non-invasive. Tend to be m ore solid, and the solid
com ponents do typically enhance.
Pineoblastoma & Pineocytoma:
"Expanded’’ Calcification Pattern

P in e a l C y s t - An incidental findings that is


m ean in g less... although frequently obsessed over. T hey can
have thin enhancem ent. C alcifications occur in 25% .

Pineal Cyst:
Classically— looks like a cyst

89
Special Topics - AFewExtra Tips op Characterization:

“R e s tric tio n ”

If they show a supratentorial case w ith restriction it’s likely to be one o f tw o things
(1) A b scess or (2) L ym phom a. Technically any hypercellular tum or can restrict
(G BM & M edulloblastom a), but lym phom a is the one they classically show restricting.

If it’s a CP angle case, then it’s an E piderm oid.

Lastly, a dirty m ove could be to show H erpes encephalitis restricting in the tem poral horns.

“M id lin e C ro ss in g ”

If they show it crossing the m idline, it’s m ost likely going to be a G BM or Lym phom a.
A lternative sneaky things they could show doing this w ould be radiation necrosis, a big
M S plaque in the coipus callosum , or M eningiom a o f the falx sim ulating a m idline cross.

“C a lc ific a tio n ”

If they show it in the brain it is probably an O ligodendrogliom a. The trick is that


O ligodendrogliom as calcify 90% o f the tim e by C T (and 100% by histopathology),
w hereas astrocytom as only calcify 20% o f the tim e. But astrocytom a is very com m on and
oligodendrogliom a is not. So in other w ords, in real life it’s probably still an astrocytom a.

“T1 B rig h t”

M ost tum ors are T l dark (or interm ediate).


Exceptions m ight include a tum or that has bled
(Pituitary apoplexy or hem orrhagic m ets). T l Bright:
H em orrhagic m ets are classically seen on M R and
F at: D erm oid, Lipom a
C T (M elanom a, R enal, C arcinoid /
M e la n in : M elanom a
C horiocarcinom a, T hyroid). Tum ors w ith fat will
also be T l bright (L ipom a, D erm oid). M elanin is B lood: B leeding M et or T um or

T l bright (M elanom a). Lastly think about C holesterol: C olloid C yst


cholesterol in a colloid cyst.

90
Special Topics - Syndromes
NF-1 O ptic N erve G liom as

NF-2 M SM E: M ultiple S chw annom as, M eningiom as, E pendym om as

VHL H em angioblastom a (brain and retina)

TS Subependym al G iant C ell A strocytom a, C ortical Tubers

N evoid Basal Cell


M edulloblastom a
Syndrom e (G orlin)

Turcot G B M , M edulloblastom a, Intestinal Polyposis

C ow dens- “C O L D ” L herm itte-D uclos (D ysplastic C erebellar G angliocytom a)

MSME
If you see tum ors E V E R Y W H E R E then you are
dealing w ith N F -2 . Ironically there are no
neurofibrom as in neurofibrom atosis type 2 (obviously
that w ould m ake a great distractor).

Just rem em ber M S M E


M ultiple Schw annom as,
M eningiom as.
E pendym om as

L h e r m itte -D u c lo s (D ysplastic C erebellar G angliocytom a)

This thing is very uncommon, but when you see it you need to have the following thoughts:

- Hey! That is Lhermittc D ulcos....

- I guess she has Cowdcns syndrom e....

- I guess she has breast CA

Next Step? - Mammogram

The appearance is classic, with a “tiger stripe” mass, typically


contained in one cerebellar hemisphere (occasionally crosses
the vermis). It’s not a “cancer”, but actually a hamartoma -
which makes sense since Cowdens is a hamartoma syndrome.

91
Brain Tumors - MRSPearls
As cell w alls get broken dow n N A A (a m aker for neuronal viability) w ill go dow n,
C reatine (m arker for cellular m etabolism ) w ill go dow n, and C holine (a m aker for cell
m em brane turnover) will go up. This is w hy the ratios o f N A A /C ho, C ho/C r and N A A /C r
get throw around.

O ther relevant m arker changes:

• L actate m ay go up. You see this in the scenario o f a high grade tum or outgrow ing its
blood supply and changing over to anaerobic pathw ays.

• L ipids m ay go up. You see this in the scenario o f a necrotic tum or. Lipids are associated
w ith necrosis.
• A lanine - is associated w ith m eningiom as.

• N A A - This is a gliom a m aker. N on gliom as tend to have little o r no N A A .

T u m o r G ra d e : R e c u r r e n t T u m o r vs G B M v s M e t:
R a d ia tio n N e c r o s is
H igher G rade Tum ors will Both can look gnarly on
have m ore cellular R ecurrent Tumor: Rising conventional M R (big
destruction, inflam m ation, choline infers that cell w alls enhancing tum or).
and m ore ischem ia / are being turned over The GBM is classically
necrosis. (som ething is grow ing). underestim ated on brain MRI
(if you are ju st looking at the
H igher G rade W ill Have: M o C holine, Mo Problem s
solid enhancing tum or). The
R adiation N ecrosis: W hen surround T2 edem a often
Less N A A you think necrosis you should contains infiltrative m irco-
Less C reatine think elevated lipids (found in tumor. By using a m ultiple
M ore Lactate necrotic tissue) and elevated voxel analysis (looking at the
M ore C holine lactate. You could also reason tum or, and also surrounding
M ore Lipids that N A A , C reatinine, and tissue) M RS supposedly adds
C holine (m akers o f cell value (allegedly).
Relative to a low er grade integrity, m etabolism , and
tumor. For the puipose o f m ultiple
turnover) w ould also be low -
choice, elevated C holine in
if the tissue in that region was
the T2 signal surrounding the
fried like chicken (or bananas
tum or = infiltrating gliom a
- if you enjoy denying your
(rather than a m et)
true nature as the apex
predator).

Voxel Selection: It is im portant to choose an area o f interest w ith enhancing tum or (avoid
cystic parts o f the tum or, calcifications, blood, or frank necrosis).

92
S E C T IO N 8 : J fe

t In f e c t io n nBinB]i[§]iEii[§in§iiiB]HBi

Neonatal Infections:
Wc are talking about TORCH infections. The first critical thing is that they only really m atter in
the first two trimesters (doesn’t cause as much harm in the third trimester). Calcifications and
microcephaly are basically present in all o f them.

Trivia Classic Look Highest Yield Trivia


Most Common
It prefers to target the germinal matrix Most Common
TORCH resulting in periventricular tissue necrosis. TORCH
(3x more The result is the most likely test question =
CMV common than Periventricular calcifications. Periventricular
Toxo - which is Calcifications
the second Of the TORCHs CMV has the highest
most common). association with polymicrogyria. Polymicrogyria

It's seen in the children of women who


clean up cat shit.

The calcification pattern is more random,


This is the and targets the basal ganglia (like most Hydrocephalus,
second most other TORCH infections).
To xoplasm osis common
Basal Ganglia
TORCH. The frequency is increased in the 3rd Calcifications
trimester (but only causes a problem in the
first two).

Associated with Hydrocephalus.

Calcifications are less common than in Vasculopathy /


other TORCHs. Ischemia.
Less common
Rubella because of High T2 signal
Focal high T2 signal might be seen in
vaccines white matter (related to vasculopathy and Fewer
ischemic injury). Calcifications

Unlike adults, the virus does not primarily Hemorrhagic


target the limbic system but instead prefers Infarct, with
It’s HSV-2 in
HSV the endothelial cells resulting in thrombus resulting Bad
90% of cases.
and hemorrhagic infarction with Encephalomalacia
resulting encephalomalacia and atrophy. (Hydranencephaly)

You may have faint basal ganglia


Not a TORCH
enhancement seen on CT and MRI
but does occur
preceding the appearance of basal ganglia Brain Atrophy,
during
H IV calcification. predominantly in
pregnancy, at
the Frontal Lobes
delivery, or via Brain atrophy pattern favors the Frontal
breast feeding.
Lobes

93
Infections of the Immunosuppressed I people with AIDS)
The most common opportunistic infection in patients with AIDS is
toxo. The most common fungal infection (in people with AIDS) is
Gamesmanship:
. Nipple Rings = AIDS
A
Cryptococcus. Two other infections worth talking about are JC . From South Africa = AIDS
Virus, and CMV.

H IV E n cephalitis:

The encephalitis that people


with AIDS get. This is
We are talking about a
actually pretty common and
situation with a CD4 < 200.
affects about 50% of AIDS
patients.

Symmetric increased T2 /
FLAIR signal in the deep
white matter.
These tend to spare the
T1 will be normal. subcortical U-Fibers (PML
The lesions will not enhance. will involve them). HIV Encephalitis
-Symmetric, and Spare Cortical U Fibers
There may be associated brain
atrophy.

P ro g re s s iv e M u ltifo c a l L e u k o e n c e p h a lo p a th y (PM L):

We are talking about a


Caused by the JC virus.
situation with a CD4 < 50.

CT will show single or


multiple scattered
Hypodensitics, with
corresponding T1
hypointensity (remember Will involve
HIV was T1 normal), subcortical U-fibers

T2/FLAIR hyperintensities
PML
out of proportion to mass -Asymmetric, and Involves Cortical U Fibers
effect - buzzword

C M V : Think about brain atrophy, periventricular Ependymal cells are the cells that line
hypodensities (that are T2/FLA IR bright), and thin the ventricles and central portion o f
the spinal cord.
ependym al enhancem ent.

C r y p to c o c c u s : The m ost com m on fungal infection in AIDS. The m ost com m on


presentation is m eningitis that involves the base o f the brain (leptom eningeal enhancem ent).
The m ost likely way this will be shown on a m ultiple choice exam is dilated perivascular
spaces filled with m ucoid gelatinous crap (these will not enhance). The second most likely
w ay this will be show n is lesions in the basal ganglia “cryptococcom as” - these are T1 dark,
T2 bright, and m ay ring enhance.

94
Infections of the Immunosuppressed ( people with AIDSJ- part 2
Toxo: M ost com m on opportunistic infection in A ID S. C lassically w e are talking about T1
dark, T2 bright, ring enhancing (w hen larger than 1 cm ) lesions. T hese guys w ill N O T show
restricted diffusion. Just think “ ring enhan cin g lesion, w ith L O TS o f ed em a.”

High Yield Trivia = Toxo is T hallium Cold, and Lym phom a is T hallium hot.

A t
WTF ?! w ^
i thought abscesses
restrict diffusion?
Typical they do.
However, atypical
infections like Toxo or
fungal don’t always
follow this rule.

T l+ C Ring Enhancing T2 Lots o f Edema DW I: NO Restriction

T H IS vs THAT:
T o xo Lym p h o m a
Ring Enhancing Ring E nhancing

H em orrhage m ore com m on after treatm ent H em orrhage less com m on after treatm ent

T hallium Cold T h alliu m H O T

PET C old (acts like necrosis) PET H ot (acts like a tum or)

M R Perfusion: D ecreased C B V M R Perfusion: Increased (or D ecreased) C B V

Infections of the Immunosuppressed ( people with AIDS) - Summary


A ID S
PM L CM V Toxo C ryptococcu s
E ncephalitis
R ing D ilated
Sym m etric T2 A sym m etric T2 P eriventricular
E nhancem ent + Perivascular
Bright B right T2 Bright
Lots o f Edem a Spaces

N o R estricted B asilar
T1 Dark
T hin E pendym al D iffusion M eningitis
E nhancem ent
Spare U Fibers Involve U Fibers T hallium C old

95
Characteristic Infections:
T B M e n in g itis :

Has a predilection for the basal cisterns


(en hancem ent o f the basilar m eninges C om plications include vasculitis w hich m ay
with m inim al nodularity). result in infarct (m ore com m on in children).
O bstructive hydrocephalus is com m on.
M ay have dystrophic calcifications.

E nhancem ent o f the B asilar M eninges + H ydrocephalus = TB

*Sarcoid can have a nearly identical appearance.


If it looks like TB - but that isn ’t a choice, it’s probably Sarcoid.

H SV - “H e rp e s ” or “T h e D irty H e rp ”

HSV 1 in adults and HSV 2 in neonates.


I mention that because For the purpose o f
a multiple choice
(1) It seems like testable trivia and test think swollen
T2 bright
(2) They actually have different imaging (unilateral or
appearances (as previously bilateral) medial
mentioned, type 1 prefers the limbic temporal lobe.
system).

Earliest Sign = Restricted Diffusion -


related to vasogenic edema.
T H IS vs THAT:
This could be tested by asking
“What sequence is more sensitive? ”, HSV spares the
with the answer being that diffusion is basal ganglia
more sensitive than T2. (distinguishes it
from MCA stroke). Herpes -
Blooming on gradient means it’s bleeding
Edema in the Temporal Lobe
(common in adults, rare in neonate form).

L im b ic E n c e p h a litis :

Not an infection, but a commonly tested mimic.


This could be asked by showing a classic
It is a paraneoplastic syndrome (usually HSV image, but then saying the HSV titer
small cell lung cancer), that looks very is negative. The second order question
similar to HSV. would be to ask for lung cancer screening.

W e s t N ile :

Several viruses characteristically involve the Classic Look: T2 bright basal ganglia and
basal ganglia (Japanese Encephalitis, Murray
thalamus, with corresponding restricted
Valley Fever, West N ile...), the only one
realistically testable is West Nile. diffusion. Hemorrhage is sometimes seen.

96
C J D : C r e u tz fe ld t-J a k o b D is e a s e

Random Factoids:
The imaging features arc variable There are 3 types:
and can be unilateral, bilateral, - Characteristic look on EEG
symmetric, or asymmetric. the “periodic sharp wave ”
- Sporadic (80-90%),
(whatever the fuck that is).
Three most likely testable - Variant “Mad Cow” (rare)
appearances diagramed below. - “ 14-3-3” protein assay is a - Familial (10%).
CSF test neurologists order.

DWI C ortical G yriform Hockey Stick Sign: Pulvinar Sign:


R estricted D iffu sion - - Bilateral FLAIR bright dorsal - Bilateral FLAIR bright
medial thalamus pulvinar thalamic nuclei
Supposedly diffusion is the most
- Described in the variant (posterior thalamus).
sensitive sign. & the cortex is the most
subtype. - Classic in the variant subtype.
common early site o f manifestation.
Basal Ganglia may also be involved.

Another way to show this (which would be more work for the test writer - and is therefore less
likely) would be a series o f MRs or CTs showing rapidly progressive atrophy.

N e u r o c y s tic e r c o s is
M ost com m on locations (in descending order):

Caused by eating pig shit (or undercooked pork). 1- Subarachnoid over the cerebral hem ispheres,
The bug is tinea solium (pork tapeworm).
2- Basal cisterns,
Trivia: Involvement o f the basal cisterns carries the
3- Brain parenchym a,
worst outcome.
4- Ventricles

Stage I: Vesicular Stage 2: Colloidal Stage 3: G ranular


Cyst + Scolex CT: Hyperdense Cyst CT: Early Calcification Stage 4: C alcified /
No Enhancement MR: Edema + Enhancement MR: Smaller Cysts, Involution
Less Edema, CT: Calcification
Less Enhancement MR: Blooming on SWI
(T2* etc..)

97
M e n in g itis a n d C e r e b r a l A b s c e s s

You can think o f meningitis in 4 main categories: V ocab


bacterial (acute pyogenic), viral (lymphocytic), chronic Leptomeningeal: Pial +Arachnoid
(TB or Fungal), and non-infectious (sarcoid). Pachymeningcal: Dural

Complications include:
This pattern
can be seen Venous thrombosis,
Essentially, we Vasospasm (leading to the stroke),
with Bacterial
are talking Empyema,
Meningitis or
about thick Ventriculitis,
Carcinomatous
leptomeningeal Hydrocephalus,
Meningitis
enhancement, in Abscess
the appropriate
clinical setting. Leptomeningeal (Pia-Arachnoid) Fungal and Carcinomatous
Enhancement: Fills the subarachnoid meningitis tend to be “more lumpy”
spaces & extends into the sulci & cisterns. and “thicker”

A bscess F acts (triv ia )


A very testable piece of trivia is that infants will often get - DW1 - Restricts
sterile reactive subdurals (much less common in adults). - MRS - Lactate High
- FDG PET - Increased Metabolic

P a c h y m e n in g e a l (D u r a l) E n h a n c e m e n t

Key Feature: Enhancement does NOT extend into the sulci


Seen this with lots of stuff: Intracranial Hypotension, Dural attachment of
a Meningioma , Sarcoid, TB, Wegener’s , Fugal Infections.

Both Breast and Prostate Cancer can deposit a solitary dural met.

Secondary CNS Lymphoma is often extra-axial and can be dural based or


fill the subarachnoid space ( “Rim Phoma ”)

Em pyem a

Can be subdural or epidural (just like blood).

Follows the same rules as far as crossing dural


attachments (epidurals don’t) and crossing the
falx (subdurals don’t).

Subdurals are more common and have more


complications relative to epidurals.
The vast majority o f subdurals are the sequela of
frontal sinusitis. The same is true o f epidurals
with some sources claiming 2/3 o f epidurals are
secondary to sinusitis. T l+ C
Subdural Em pyem a:
Classic Look: T1 bright and restrict diffusion. Dural Enhancement, Restricts Diffusion

98
Intraaxial Infections: Abscess Continued, with Cerebritis, & Ventriculitis

Abscess: A cerebral abscess is a cavity that contains pus, debris, and necrotic tissue. These can
develop secondary to to bacterial, fungal, or parasitic infection - most commonly via
hematogenous spread. For the purpose of multiple choice, remember to think about right-to-left
shunts and pulmonary AVMs. Direct spread (example = sinus) is possible, but just less common
because of the dura.

CT: Focal area of Tl+C: Smooth Ring T2: Multiple Lesions DWI: Typical Abscess
low density with Enhancement with with Vasogenic Edema (bacteria) will restrict.
surrounding low Multiple Lesions - — this is nonspecific Remember Atypical
density vasogenic Suggests Abscesses (could be mets) (Toxo etc..) doesn’t
edema. always restrict

CT: Flypercellular
Tumor (classic
example would be
Lymphoma) will be
hyper dense instead
on low density like
an abscess

Multiple Rings Mets vs Abscess Smooth Ring = Abscess Irregular Ring = Tumor
Abscess Rings tend to be
The smooth margin suggests Abscess, but "Bumpy” or “Shaggy”
thicker on the “Oxygen
doesn't exclude mets. The difference is that Side” or "Grey Matter inner lip of the ring is
tumor usually starts out as a solid enhancing Side" o f the Brain - and supposed to suggest
mass then becomes ring enhancing with thinner towards the necrosis
ventricle.
necrosis. Also, Abscesses tend to be smaller
(usually less than 10mm). Both Tumor & Abscess will have Vasogenic Edema

Cerebritis is the early form o f intra-axial Ventriculitis: Usually the result of a shunt
infection, which can lead to Abscess if not placement or intrathecal chemo - as discussed
treated. The typical look is the vasogenic on page 51. The ventricle will enhance and you
can sometimes see ventricular fluid-fluid levels
edema without the well defined central
enhancing lesion. There may be spotty If septa start to develop you can end up with
restricted diffusion. obstructive patterns of hydrocephalus.

The intraventricular extension o f abscess is a


very serious / ominous “pre-terminal event” .

99
MRl Gamesmanship - Enhancement Patterns
In general, to solve MR puzzles you will need to be able to work through some M R sequences.
The trick is to have a list o f things that are T1 bright, T2 bright, Restrict diffusion, and Enhance.
Plus you should know the basic enhancem ent patterns (hom ogenous, heterogenous, ring, and
incomplete ring).

STROKE vs TU M O R vs ABSCESS vs MS Plaque

T2: For the most part, T2 is not super helpful for lesion characterization
- as stroke, tumors, abscess, MS, all have edema.

DWI: This is helpful only if they follow the classic rules. Out o f those 4 (stroke, tumors, abscess,
MS) the classical diffusion restrictors are: Abscess, and Stroke. Certain hypercellular tumors
(classically lymphoma) can restrict, and dem yelinating lesions with acute features can restrict.

Enhancement: In this situation this is probably the most helpful.


Out o f those 4 (Tumor, Abscess. MS, and Stroke) each should have a different pattern.

• Tumor usually heterogeneous or hom ogenous if high


grade (or none if low grade). Technically ring How Many Rings ?
enhancem ent can also be seen with Gliomas, and Mets
(though I expect this is less likely to be shown on multiple The number of rings can be a
helpful strategy. A single ring
choice).
(around half of mets and 3/4
• Abscess will classically have RING pattern.
of gliomas are solitary).
• M S will classically have an INCOM PLETE RING pattern.
Abscess and MS Lesions are
• Stroke will have cortical ribbon (GYRIFORM ) type
almost always (like 75-85%)
multiple.
enhancem ent in the sub-acute time period (around 1
week).

Heterogeneous Ring Incomplete Ring Gyriform


-Most likely Tumor -Can be lots of -Classic for -Classic for
(higher grade) stuff: Abscess and demyelinating subacute stroke
Tumor are both lesion (can also be seen
prime suspects with P R E S or
encephalopathy /
encephalitis)

100
___________ S E C T IO N 9 : J fe
B r a in T r a u m a 8c B l e e d in g

P arenchym al Contusion: The rough part o f the skull base ean scrape the brain as it slides
around in a high speed MVA. Typical locations include the anterior temporal lobes and inferior frontal
lobes. The concept of coup (site o f direct injury) and contre-coup (opposite side o f brain along vector
of force). Contusion can look like blood with associated edema in the expected regions.

D iffuse A xonal In ju ry /S h e a r Injury: There are multiple theories on why this happens
(different density o f white and gray matter etc...) they don’t matter for practical purposes or for
multiple choice.

Things Worth Knowing: DAI Grading

• Initial Head CT is often normal Grade 1 = Grey-White Interface

• Favorite sites o f DAI are the posterior corpus callosum, and Grade 2 = Corpus Callosum
GM-WM junction in the frontal and temporal lobes
Grade 3 = Brainstem
• Multiple small T2 bright foci on MRI

S ubarachnoid H em o rrh ag e: Trauma is the most common cause. FLAIR is the most
sensitive sequence. This is discussed in more later in the chapter.

THIS vs THAT: S u b d u ral v s E p id u ra l


Sub dural
E p id u ra l S u b d u ra l - Crescent
Shape
Classic History: Elderly alcoholic with
a shriveled up atrophic brain spent the -No Respect
evening with a bottle of For the
Classic History: “ Rotgut - Hobo Tranquilizer” brand Sutures
Trauma Patient - with a
whiskey, then fell over stretching &
skull fracture
tearing his cortical bridging veins.
A week later he seems to be acting
progressively more confused.
“Bi-convex” or E pidural
“Bi-concave”
Lenticular
- Lentiform
Does not cross the midline, may extend Shape
Can cross the midline into interhemispheric fissure
-Skull
Can NOT cross a suture Can cross a suture
Fracture

Usually arterial Usually venous -Respect for


the Sutures
Can rapidly expand and
More mass effect than expected for size
kill you

101
How Old is that BloodP
C T: This is an extrem ely high yield topic. M aybe the m ost high yield topic in all o f neuro,
w ith regard to m ultiple choice. The question can be asked w ith CT or M RI (M RI m ore
likely). If they do ask the question w ith C T it’s m ost likely to be the subacute subdural that is
isointense to brain, w ith loss o f sulci along the m argins. T hey could also show the “ swirl
sign” - see below.

Blood on CT
Flyperacute A cute (< 1 hour) H ypodense

A cute (1 hour - 3 days) H yperdense

Progressively less dense, eventually becom ing


Subacute (4 days - 3 w eeks) isodense to brain. Peripheral rim enhancem en t
m ay occur with contrast.
C hronic (> 3 w eeks) H ypodense

S w ir l S ig n - This is an om inous sign o f active


bleeding. The central low attenuation blood
represents hyper-acute non-clotted blood, w ith
surrounding acute clotted blood.

S w irl Sign

B lo o d A g e V ia M R :

M RI is m ore difficult to rem em ber. Som e people use the m nem onic “ IB, ID, BD, BB, D D ” or
“ It Be Iddy Biddy, BaBy, D oo-D oo” w hich I find very irritating. I prefer m nem onics that
em ploy know n w ords (just m y opinion). A nother one w ith actual w ords is “ G eorge
W ashington B ridge” For T1 (Gray, W hite, Black), and O reo C ookie for T2 (B lack, W hite,
Black).

102
B lo o d A g e V ia M R (c o n tin u e d ): Instead o f m em orizing baby babbling noises, 1 use
this graph show ing a clockw ise m ovem ent. This thing m ay seem tricky and too m uch to bear,
at first, but it does actually w ork and once you draw it tw ice, y o u ’ll have it m em orized.
You’ll also notice a few things: (1) you w o n ’t feel like a dipshit for m aking baby noises,
(2) y o u ’ll have a renew ed sense o f self-esteem , and (3) you are likely to notice m arked
im provem ent in your golf-sw ing.

Intracellular MetHb Extracellular


“Early-Subacute” “Late-Subacute”

T1 Bright
3 -

I
T1 Iso

Oxy-Hb
DeOxy-Hb “Hyperacute
“Acute”

Hemosiderin - Chronic

O
T1 Dark I

T2 Dark o o
T2 Iso T2 Bright

A nother strategy (w hich is som ew hat unconventional) is to actually try and understand the
MRI changes (I strongly discourage this). If you insist on trying to understand this I have a 40
min lecture on T itanR adiology.com explaining it (this lecture is also free on m y YouTube
Channel — google “ Prom etheus L ionhart Blood A ge” ).

Hyperacute < 24 hours Oxyhemoglobin, Intracellular T l- Iso, T2 Bright

Acute 1-3 days Deoxyhemoglobin, Intracellular T1 - Iso, T2 Dark

Early Subacute > 3 days Methemoglobin, Intracellular Tl Bright, T2 Dark

Late Subacute >7 days Methemoglobin, Extracellular T l Bright, T2 Bright

T 1/T 2 Dark Peripherally,


Chronic > 14 days Ferritin and Hemosiderin, Extracellular
Center may be T2 bright

103
Hemorrhage INon-Traumatic)
S u b a ra c h n o id H e m o rrh a g e :

Yes, the m ost com m on cause is traum a. A com m on point S e q u e la o f SAH


o f trivia is that the m ost sensitive sequence on M RI for
(1) H ydrocephalus - Early
acute SAH is FL A IR (because it w o n ’t suppress out -
(2) V asospasm - 7-10 days
m aking it hyperintense). Be aw are that supp lem ental
(3) Superficial Siderosis - Late
oxygen (usually 50-100% ) can give you a fake out that
looks like SA H on FLAIR.

W hen the blood is real, in the absence o f traum a, there are a few other things to think about.

A neurysm - D iscussed later in the chapter.

Benign N on-A neurysm Perim esencephalic hem orrhage:


T his is a well described entity (although not well
understood). This is N O T associated w ith aneurysm
(usually - 95% ), and m ay be associated w ith a venous
bleed. *You have to prove that - you need a negative CTA.
The location o f the blood - around the m idbrain and pons
w ithout extension into the lateral Sylvian cisterns or
interhem ispheric fissures is classic. Just think anterior to
the brainstem . R e-bleeding and ischem ia are rare- and
they do extrem ely well.

S uperficial Siderosis: This


is a si^e effect o f repeated
episodes o f SAH. I like to
think about this as “staining
the surface o f the brain with
hem osiderin. ” The classic
look is curvilinear low
signal on gradient coating
the surface o f the brain.
The classic history is
S uperficial Siderosis - H em osiderin Staining
sensorineural hearing loss
and ataxia.

104
P s e u d o -S u b a r a c h n o id H e m o r r h a g e
This is a described mimic of SAH that is seen in
the setting of diffuse cerebral edema (most
commonly anoxic brain injury). Near
drowning, or suicide attempt by hanging would
be classic clinical vignettes.
What you are seeing is actually two things at
once. (1) You are seeing diffuse edema which
lowers the attenuation of the brain (makes it
darker). (2) You are seeing compression and
collapse of the sub arachnoid spaces which
gives them a hyper dense appearance. The
combination of these factors gives the
suggestions of hyper density in the cerebral
sulci, fissures, and cisterns which can mimic
SAH (hence the name).
TH IS vs THAT: Pseudo SAH vs Real SAH: If they give you history that should help (anoxic brain
injury vs headache / trauma). The absence of any intraventricular bleeding can suggest pseudo SAH.
Lastly density of the Pseudo SAH will be less than 40. Acute blood tends to be around 60-70 HU.

In tra p a re n c h y m a l H e m o rrh a g e :

Hypertensive Hemorrhage: Common locations arc the basal ganglia, pons, and cerebellum.
For the purpose o f multiple choice tests, the basal ganglia is the most common location
(specifically the putamen). You typically have intraventricular extension o f blood.

Amyloid Angiopathy: History o f an old dialysis patient (or some other history to think
Amyloid). The classic look is multiple lobes at different ages with scattered microbleeds on
gradient.

• Septic Emboli: These are seen in certain clinical scenarios (IV drug user, organ transplant,
cyanotic heart disease, AIDS patients, people with lung AVMs). The classic look is
numerous small foci of restricted diffusion. Septic emboli to the brain result in abscess
and mycotic aneurysms (most commonly in the distal MCAs), The location favors the
gray-white interface and the basal ganglia. There will be surrounding edema around the tiny
abscesses. The classic scenario should be parenchymal bleed in a patient with infection.

Other Random Causes: These would include AVMs, vasculitis, brain tumors (primary and
mets) - these are discussed in greater detail in various sections o f the text.

In tra v e n tric u la r H e m o rrh a g e :


Not as exciting. Just think about trauma, tumor, hypertension, AVMs, and aneurysms - all
the usual players.

Ep id ural I S u b d u ral H e m o rrh a g e :

Obviously these are usually post-traumatic.

Dural AVFs and High Flow AVMs can bleed causing subdurals / subarachnoid hemorrhage.
These are discussed further later in the chapter.

105
S E C T I O N lO :

t V a sc u la r ^

Stroke
Stroke is a high yield topic. You can broadly categorize stroke into ischem ic (80% ) and
hem orrhagic (20% ). It’s critical to rem em ber that stroke is a clinical diagnosis and that imaging
findings com plim ent the diagnosis (and help exclude clinical m im ic o f stroke - tum or etc..).

Vascular Territories: Below is a diagram show ing the various vascular territories. The junction
betw een these zones is som etim es referred to as a “w a te rsh e d ’. These areas are prone to ischem ic
injury, especially in the setting o f hypotension or low oxygen states (near drow ning or Roger
G racie’s m ounted cross choke or a M arcello G arcia high elbow guillotine).

Lenticulostriate

W atershed Ischemia favors the border


Border Zone
zones o f different vascular territories (just
Between AC A
like the bowel).
and MCA

The classic clinical scenario for watershed Border Zone Between


infarcts would be severe hypotension Deep White Matter
(shock / CPR / E tc ..), severe carotid Branches o f the MCA
stenosis, or a 2009 IBFFJ worlds match and the ACA.
up with Roger Gracic.

Gamesmanship: W atershed Infarcts


in a Kid = M oyam oya (Idiopathic Border Zone
supraclinoid ICA vasculo-occlusive Between MCA
A disease) and PCA

106
S u b ac u te In fa rc t:
Im aging S ig n s on C T: A Unique that it Enhances but creates NO Mass Effect

Dense MCA Sign Intraluminal thrombus is dense, usually in the M 1 and/or M2 segments

Insular Ribbon Sign Loss o f normal high density insular cortex from cytotoxic edema

Loss of GM-WM
Basal Ganglia / Internal Capsular Region and Subcortical regions
differentiation

Mass Effect Peaks at 3-5 days

Enhancement Rule o f 3s: Starts in 3 days, peaks in 3 weeks, gone by 3 months.

Fogging:

“Fogging” is classically
described with non-contrast CT,
1 1 but T2 MRI sequences have a
similar effect (typically
occurring around day 10). In the
Day 1 Day 14
real world, you could give IV
This is a phase in the evolution o f stroke when the infarcted contrast to demarcate the area of
brain looks like normal tissue. This is seen around 2-3 weeks infarct or just understand that
post infarct, as the edema improves. fogging occurs.

A rte ry of P erch eron S troke:


Classic V Shaped bilateral infarct o f the paramedian thalami.
This can only occur in the setting o f the Artery o f Percheron

¥
vascular v a ria n t. This variant is characterized by a solitary trunk
originating from one o f the two PCAs to feed the rostral midbrain
and both thalami (normally there are several bilateral paramedic
arteries originating from the PCAs).

R e c u rre n t A rte ry of
H eu b n er S tro k e
Classic Caudate Infarct
The Artery of H is a deep branch off
the proximal AC A
This thing can get “bagged” during the
clipping of ACOM artery aneurysm.
'j
F e ta l PCOM S tro k e P a tte rn
C ard io e m b o lic S troke:
This pattern demonstrates infarcts
This has the classic pattern of in both the anterior and posterior
multiple foci of restricted diffusion
circulation of the same
scattered bilaterally along multiple
hemisphere.
vascular territories.
This pattern is possible as the
The clinical history is usually A-Fib or variant anatomy with the PCA
endocarditis.
feeds primarily from the ICA.

107
R es tric te d Diffusion: N ot E verything T h a t R e s tricts
is a S tro k e
Acute infarcts usually arc bright from about 30
Bacterial Abscess, CJD (cortical), Herpes,
mins after the stroke to about 2 weeks. Epidermoids, Hypercellular Brain Tumors
Restricted diffusion without bright signal on (Classic is lymphoma), Acute MS lesions,
Oxyhemoglobin, and Post Ictal States. Also
FLAIR should make you think
artifacts (susceptibility and T2 shine through).
hyperacute (< 6 hours).

E nh an cem ent: The rule of 3’s is still useful. Starts day 3, peaks ~ 3 weeks, gone by 3 months.
0-6 hours 6-24 hours 24 hours -1 week
Diffusion Bright Bright Bright
FLAIR NOT BRIGHT Bright Bright
T1 Iso Dark Dark, with Bright Cortical Necrosis
T2 Iso Bright Bright

H e m o r r h a g ic T r a n s fo r m a tio n : P re d ic to rs of
H e m o rrh a g ic
This occurs in about 50% o f infarcts, with the typical time period
T ra n s fo rm a tio n in
between 6 hours and 4 days. If you got TPA it’s usually within 24
P a tie n ts G ettin g TPA
hours of treatment.
• Multiple Strokes,
People break these into (1) tiny specs in the gray matter called • Proximal MCA occlusion,
"petechial” which is the majority (90%) and (2) full on hematoma • Greater than 1/3 of the
- about 10%. MCA territory,
• Greater than 6 hours since
Who gets it? People on anticoagulation, people who get TPA, onset “delayed
people with embolic strokes (especially large ones), people with recanalization”,
venous infarcts. • Absent collateral flow

Venous In farc t:

Not all infarcts are arterial, you can also stroke secondary to venous occlusion (usually the sequelae
of dural venous sinus thrombosis or deep cerebral vein thrombosis). In general, venous infarcts arc at
higher risk for hemorrhagic transformation. In little babies think dehydration, in older children think
about mastoiditis, in adults think about coagulopathies (protein C & S dcf) and oral contraceptives.
The most common site o f thrombosis is the superior sagittal sinus, with associated infarct occurring
75% o f the time.

Venous thrombosis can present as a dense sinus (on non-contrast CT) or “empty delta” (on contrast
enhanced CT). Venous infarcts tend to have heterogeneous restricted diffusion. Venous thrombosis
can result in vasogenic edema that eventually progresses to stroke and cytotoxic edema.

Arterial stroke = Cytotoxic Edema

Venous Stroke = Vasogenic Edema + Cytotoxic Edema

S tig m a ta of chronic venous throm bosis include the development o f a dural AVF, and/or
increased CSF pressure from impaired drainage.

108
ASPECTS (Alberta Stroke Program E arty CT S core)

This was developed to give “providers” a more specific guideline for giving TPA - as an
alternative to the previous 1/3 vascular territory rule. The idea being that the greater the vascular
territory involved, the worse the clinical outcome (post TPA bleed etc..).

The way this works is that you start out with 10 points, and lose points based on findings of acute
cytotoxic ischemia to various locations (example: minus 1 for caudate, or lentiform nucleus, or
insular ribbon, etc.. etc.. so on and so forth).

Testable Pearls:

• This is for MCA ONLY (not other vascular territories)

• This is for ACUTE ischemia (don’t subtract points for chronic lacunar infarcts etc..)

• A score o f 8 or greater has a better chance of a good outcome (score o f 7 or less may
contraindicate TPA — depending on the institutional policy.

CT Perfusion - C rash C ourse


After an arterial occlusion perfusion pressure is going to be rapidly reduced. Millions of neurons will
suddenly cry out in terror then suddenly silenced, unless they are lucky to have arteriolar dilation with
capillary recruitment to bring in as much blood to that area of brain as possible. This process is called
physiologic auto-regulation and should result in an increase in capillary blood pool. The key point is
that you need live neurons (penumbra) to cry out for help. If they cry out and are suddenly silenced
(infarct core) you won’t see any auto regulation attempts. This physiology makes up the basis of
perfusion for stroke.
Parameters:
• Cerebral Blood Flow (CBF): Represents instantaneous capillary flow in tissue.
• Cerebral Blood Volume (CBV): Describes the blood volume of the cerebral capillaries and venules
per cerebral tissue volume.
• Mean Transit Time (M IT) = CBV divided by CBF ; it is the average length of time a certain
volume of blood is present in the capillary circulation.
• Time to Peak (TTP): This is the opposite of CBF. Less flow = Longer Time to reach maximum
concentration of contrast.

The primary role of perfusion is to distinguish between salvagable brain (penumbra), and dead brain.
The penumbra may benefit from therapy. The dead brain will not- "He s Dead Jim ’’ - Dr. McCoy

109
Aneurysm D o lic h o e c ta s ia of th e
B a s ila r A rte ry
Who gets them? People who smoke, people with
polycystic kidney disease, connective tissue disorders This refers to a widened elongated
(Marfans, Ehlers-Danlos), aortic coarctation, NF, FMD, twisty appearance o f the basilar
and AVMs. artery. This is probably the result
of chronic hypertension (abnormal
Where do they occur? They occur at branch points (why vessel remodeling).
do persistent trigeminals get more aneurysms ? -
The height o f the bifurcation and
because they have more branch points). They favor the the more lateral the position of the
anterior circulation (90%) - with the anterior vessel (relative to the clivus) the
communicating artery being the most common site. more severe - so says the Smoker
As a piece of random trivia, the basilar is the most criteria.
common posterior circulation location (PICA origin is
the second most common).

When do they rupture? Rupture risk is increased with


size, a posterior location, history o f prior SAH, smoking
history, and female gender.

Which one did it? A common dilemma is SAFI in the


setting of multiple aneurysms. The things that can help
you are location o f the SAH/Clot, location o f the Complications include: nothing
vasospasm, size, and which one is the most irregular (most have no symptoms),
Focal out-pouching - "Murphy s tit ”) dissection, compression of cranial
nerves (hcmi-facial spasm), stroke
(brainstem), and hydrocephalus.
A n e u ry s m Types:

Saccular (Berry):
A C A ~ 35%
The most common type and the
m ost common cause o f non-
traumatic SAH. They are commonly
seen at bifurcations.

M1/M2 The underlying pathology m ay be a


Junction congenital deficiency o f the internal
~ 30% elastic lamina and tunica media
(at branch points).
ICA / PComm Junction ~ 30%
Basilar Tip ~ 3% Rem ember that most are idiopathic
PICA - 2% (with the associations listed above).
They arc m ultiple 15-20% o f the
time.

*Note that around 90% arise


from the Anterior Circulation

110
A n eu ry sm T yp e s C o n tin u ed :

Fusiform Aneurysm - Associated with PAN,


Connective Tissue Disorders, or Syphilis. These A V M A s s o c ia te d
more comm only affect the posterior circulation. P e d ic le A n e u r y s m :
May mimic a CPA mass.
Aneurysm associated with an AVM.
Pseudoaneurysm - Think about this with an
irregular (often saccular) arterial out-pouching The trivia to know is that it’s found on the
at a strange / atypical location. You m ay see artery feeding the AVM (15% o f the time).
focal hematoma next to the vessel on non­
contrast. These may be higher risk to bleed than the
AVM itself (because they are high flow).
Traumatic - Often distal secondary to
penetrating trauma or adjacent fracture.

Mycotic - Often distal (most com m only in the


MCA), with the associated history of
endocarditis, meningitis, or thrombophlebitis.

A n e u ry s m R u p tu re T riv ia :
Blister Aneurysm - This is a sneaky little dude
(the angio is often negative). It’s broad-based • Aneurysm > 10mm have a 1% risk o f
at a non-branch point (supraclinoid ICA is the rupture per year.
most common site).
• Although controversial, 7 mm is often
Infundibular W idening - Not a true thrown around as a treatm ent threshold
aneurysm, but instead a funnel-shaped for anterior circulation aneurysm s
enlargement at the origin o f the Posterior
• In general, posterior circulation
Com municating Artery at the junction with the
aneurysm s have a higher rate o f
ICA. Thing to know is "not greater than 3
rupture per mm in size.
mm. ”

A n e u r y s m S u b ty p e s S u m m a r y M a x im u m B le e d in g
- A n e u r y s m L o c a tio n
Saccular Branch Points - in the
(Berry) A nterior C irculation
ACOM Interhem ispheric Fissure

Fusiform Posterior C irculation


PCOM Ipsilateral Basal C istern

Pedicle
A rtery feeding the AVM M CA _ . . „.
A neurysm Sylvian ris s u re
Trifurcation

M ycotic Distal M CAs .. Interpeduncular C istern,


B asilar Tip , . .
or Intraventncular

B lister Broad B ased N on-B ranch


Posterior Fossa or
A neurysm Point (Supraclinoid ICA) PICA
Intraventricular
Vascular Malformations
• Most Common Most Common Type of High Flow Arterial Component
• Congenital malformation
• Supratentorial location (Usually) z "?
Nidus X
• Most common complication = bleeding (3% annual)
H ig h F lo w
• Risk increased with: Smaller AVMs (they are under Draining veins
AVM higher pressure), Small Draining Veins (can’t reduce
pressure), Perinidal Aneurysm, and Basal Ganglia Adjacent brain may
location be gliotic (T2 bright)
. Symptoms: Headache (#1), Seizure (#2) and atrophic.

• Flow Rate is Variable (can be high or low flow)


• SPINAL AVFs are actually the most common type of
AVFs - a helpful hint is the classic clinical history of • No Nidus
“gradual onset LE weakness”
• Risk of Bleeding - increased with direct cortical venous • Can be occult on
Dural AVF MR1/MRA - need
drainage.
• These aren’t congenital (like AVMs) but instead are catheter angio if
suspicion high
acquired — classically from dural sinus thrombosis
• Symptoms: Tinnitus — especially if the sigmoid sinus
is involved.

• “Caput medusa” or
“large tree with
• Variation in normal venous drainage multiple small
• Resection is a bad idea = venous infarct branches” -
• Associated with cavernous malformations. collection of vessels
DVA • They almost never bleed in isolation. If you see converging towards
evidence of prior bleeding (blooming on gradient) there an enlarged vein
is probably an associated cavernoma. (seen on venous
phase only).
• Can have a halo of
T2 bright gliosis.

• Low Flow - WITHOUT intervening normal tissue


• Can be induced from radiotherapy
C avernous
• Can ooze some blood, but typically don’t have full-on “Peripheral Rim of
M a lfo rm a tio n catastrophic bleeds. Presence of a “fluid-fluid” level Hemosiderin.”
(cavernoma, or suggests recent intralesional hemorrhage • Best seen on
cavernous angioma) • Single or multiple (more common in Hispanics). gradient
• Classic gamesmanship is to show you a nearby DVA

• Brush-like” or
• Low Flow - WITH intervening normal tissue “Stippled pattern”
C ap illary • Can also be radiation induced of enhancement
• Usually don’t bleed (thought of as an incidental • Best seen on
T e lan g iectasia finding) gradient (slow flow
• Classic Look = Single lesion in the Pons and
deoxyhemoglobin)

• Wastebasket term, most often used for DVA with AV


M ixed shunting or DVAs with telangiectasias

112
C a lc if ic a t io n R a p id R e v ie w I S u m m a r y

Pineal G land - Common in adults, Rare in kids. If you see


calcification in a kid under 7, it could suggest underlying neoplasm.
—Germinoma = “Engulfed" Pattern “ 1”
— Pineoblastoma & Pineocvtoma: “Expanded” Pattern “2”

H abenular - Curvilinear structure (solid white arrow) located a few


millimeters anterior to the pineal body (open arrow). About 1 in 5
normal adults will have calcification here. The trivia is an increased
association with schizophrenia.

Choroid plexus - Common in adults. Remember there is no choroid


plexus in the frontal/occipital horn o f the lateral ventricles or the
cerebral aqueduct.

Dural C alcifications - Common in adults. If the calcs are bulky and


there are a bunch o f tooth cysts (Odontogenic keratocysts) think Gorlin
Syndrome.
Fahr
Basal G anglia - Very common with age, favors the globus pallidus. If
extensive & symmetrical think Fahr disease.

Sturge-W eber - Tram


track / double-lined
gyriform pattern parallel to
the cerebral folds.

Tuberous Sclerosis - Calcifications o f the subependymal Etiology = subcortical


nodules are pathognomonic typically found at the ischemia secondary to pial
caudothalamic groove and atrium. You can see calcified angiomatosis.
subcortical tubers - more typical in older patients.

Congenital C M V - Periventricular calcifications.


Can also have brain atrophy
/4 i i
Congenital Toxo - Basal Ganglia Calcifications + •••*
Hydrocephalus, 0• V

N eurocysticercosis - Etiology: Eating Mexican pork


sandwiches -end-stage will have scattered quiescent
%i l y
Neurocysticercosis
calcified cyst remnants. CMV Toxo

C avernom a — scattered Brain Tumors can calcify. The ones most people talk about are
dots or stippled Old Elephants Age Gracefully
“popcorn” calcification
O: Oligodendroglioma - variable, but “ribbon” pattern is most commoon
E: Ependymoma (Medulloblastomas can also calcify - just less often)
A: Astrocytoma
G: Glioblastoma - mural calcified nodule
AVM — calcifications in
the tortuous veins or the Even though more Oligodendrogliomas calcify, Astrocytoma is still the
nidus most common calcified tumor (because there are alot more o f them).

Craniopharyngioma, Meningioma, Choroid plexus tumors are all known


to calcify as well. Osteosarcoma mets famously calcify.

113
Vasospasm
Vessels do not like to be bathed in blood (SA H ), it m akes them freak out (spasm ). The
classic tim ing for this is 4-14 days after SA H (N O T im m ediately). It usually looks like
sm ooth, long segm ents o f stenosis. It typically involves m ultiple vascular territories. It can
lead to stroke.

Who g ets it? It’s usually in patients w ith SAH and the m ore volum e o f SA H the greater the
risk. In 1980 som e neurosurgeon cam e up w ith this thing called the Fisher Score, w hich
grades vasospasm risk. T he gist o f it is greater than 1 m m in thickness or intraventricular /
parenchym al extension is at higher risk.

Are there N on-SA H causes o f vasospasm ? Yep. M eningitis, PRES, and M igraine H eadache.

C ritical Take H om e P oint - V asospasm is a delayed side effect o f SAH. It does N O T occur
im m ediately after a bleed. You see it 4-14 days after SAH.

Vascular Dissection
Vascular dissection can occur from a variety
o f etiologies (usually penetrating traum a, or a
trip to the chiropractor). /
Penetrating traum a tends to favor the
carotids, and blunt traum a tends to favor the * * mm
vertebrals.

This w ould be w ay too easy to show on CT



as a flap, so if it’s show n it’s m uch m ore
likely to be the T1 bright “crescent sign”, or “Crescent Sign” oi D issection
intram ural hem atom a. - it’s th e T1 bright intram ural blood.

114
Vasculitis
You can have a variety o f causes o f C N S vasculitis. O ne w ay to think about it is by clum ping
it into (a) Prim ary CN S vasculitis, (b) Secondary CN S vasculitis from infection, or sarcoid,
(c) system ic vasculitis w ith C N S involvem ent, and (d) C N S vasculitis from a system ic disease.

Prim ary CN S Vasculitis Prim ary A ngiitis o f the C N S (PA C N S)

Secondary CN S vasculitis M enin gitis (bacterial, TB, Fungal),


from infection, or sarcoid Septic, Em bolus, Sarcoid,

System ic vasculitis w ith


PAN, Tem poral A rteritis, W egeners, T akayasu’s,
CN S involvem ent

CNS vasculitis from a


C ocaine Use, RA, SLE, L ym e’s
System ic D isease

They all pretty m uch look the sam e w ith m ultiple segm ental areas o f vessel narrow ing, w ith
alternating dilation (“ beaded appearance"). You can have focal areas o f vascular occlusion.

Trivia:

PAN is the M ost C om m on system ic vascu litis to involve the CN S


(although it is a late finding).

SLE is the M ost C om m on C ollagen V ascular D isease

Hlisc Vascular Conditions


M oyam oya - This poorly understood entity (originally described in Japan - hence the nam e),
is characterized by progressive non-atherosclerotic stenosis o f the supraclinoid ICA, eventually
leading to occlusion. The progressive stenosis results in an enlargem ent o f the basal
perforating arteries.

Trivia:

A B uzzw ord = “P u ff o f Sm oke ” - for angiographic appearance


W atershed D istribution
A In a child think sickle cell

A O ther notable associations include: NF, p rio r radiation, D ow ns syndrom e


Bi-M odal A ge D istribution (early childhood and m iddle age)

A C hildren Stroke, A dults Bleed

115
C ro s s e d C e r e b e lla r D ia s c h is is (C C D ):

D epressed blood flow and m etabolism affecting the cerebellar hem isphere after a
contralateral supratentorial insult (infarct, tum or resection, radiation).

C reates an A unt M innie A ppearance:

M ech an ism / G am esm anship: W hen I was a m edical student, I


had to m em orize a bunch o f tiny little tracks and pathw ays all over
the brain, cerebellum , and spine. It (like m any things in m edical
school) m ade m e super angry because it w as such a colossal w aste
o f tim e. M ore PhD bullshit, lum ped right in w ith those step 1
‘‘w hat chrom osom e is that on ? ” questions.

R edem ption for the PhD s has arrived. A pparently, one o f these
pathw ays, the “corticopontine-cerebellar pathw ay,” is actually
im portant. S o rta ....

A llegedly, this pathw ay connects one cerebral hem isphere to the


opposite cerebellar hem isphere. If the pathw ay gets disrupted (by
tum or, radiation, e tc ...), then m etabolism shuts dow n in the
opposite cerebellum even though there is nothing structurally
w rong w ith it. T hat is w hy you get this criss-crossed hypo-
m etabolic appearance on FD G -PET.

The trick is to show you the FD G -PE T picture, and try and get
you to say there is a pathology in the cerebellum . T here isn ’t! The
cerebellum is norm al - the problem is in the opposite cerebrum
w here the pathw ay starts.

116
N A S C E T C riteria: The N orth A m erican
Sym ptom atic C arotid E ndarterectom y Trial
(N A SC ET) criteria, are used for carotid
stenosis.

The rule is: m easure the degree o f stenosis A


using the m axim um internal ca rotid artery 1 - —
stenosis ( “A ”) com pared to a p a ra lle l (non­ B
curved) segm ent o f the dista l cervical
internal carotid a rtery (" B ”).

You then use the form ula:

[1- A/B] X 100% = % stenosis

C arotid endarterectom y (C E A ) is often perform ed


for sym ptom atic patients w ith > 50% stenosis.

117
t F a c e
S E C T IO N
a n d T -B o n e
1 1:
S m a s h
IMMMJMlMPJMEl
#
The LeFort Fracture Pattern System: In the dark ages, Rene L eF ort beat the shit out o f
cadavers w ith clubs and threw them o ff buildings — it w as “ science” . He then described three
facial fracture patterns that interns in EN T and people w ho w rite m ultiple test questions think are
im portant. It can be overly com plicated but the m ost com m on w ay a test question is w ritten
about these is either by asking the buzzw ord or the essential com ponent (or show ing them ).

B uzzw ords:

L eFort 1: “The Palate


LeForts
Separated from the M axilla”
or “Floating Palate”

LeFort 2:“T he M axilla


Separated from the Face” or
“ P yram idal”

L eF ort 3: “The Face


Separated from the
C ran iu m ”

E s s e n tia l E le m e n ts : All three fracture types share the pterygoid process fracture. If the
pterygoid process is not involved, you d o n ’t have a LeFort. Each has a unique feature (w hich
lends itse lf easily to m ultiple choice).

* LeFort 1: Lateral N asal A perture


* LeFort 2: Inferior O rbital Rim and O rbital Floor
* LeFort 3: Z ygom atic A rch and Lateral O rbital Rim/W all

M u c o c e le : If you have a fracture that disrupts the frontal sinus outflow tract (usually
nasal-orbital-ethm oid types) you can develop adhesions, w hich obstruct the sinus and result
in m ucocele developm ent. The buzzw ord is “airless, expanded sin u s.” T hey are usually T l
bright, w ith a thin rim o f enhancem ent (tum ors m ore often have solid enhancem ent). The
frontal sinus is the m ost com m on location - occurring secondary to traum a (as described
above). M ore on this la te r....

C S F L e a k : Fractures o f the facial bones, sinus w alls, and anterior skull base can all lead
to CSF leak. T he m ost com m on fracture site to result in a C SF leak is the anterior skull b a se .
"Recurrent bacterial m eningitis ” is a know n association w ith C SF leak.

118
Temporal Bone Fractures:
The traditional w ay to classify these is longitudinal and transverse, and this is alm ost
certainly how the questions w ill be w ritten. In the real w orld that system is old and
w orthless, as m ost fractures are com plex w ith com ponents o f both. The real predictive
finding o f value is violation o f the otic capsule - as described in m ore m odern papers.

L o n g itu d in a l tran sverse


Long Axis o f T-Bone Short Axis o f T-Bone

More Common Less Common

More Ossicular Dislocation More Vascular Injury (Carotid / Jugular)

Less Facial Nerve Damage (around 20%) More Facial Nerve Damage (>30%)

More Conductive Hearing Loss More Sensorineural Flearing Loss

Longitudinal Transverse

T h in g s t o K n o w A b o u t F a c ia l F r a c tu r e s :

• N asal Bone is the m ost com m on fracture

• Z ygom aticom axillary C om plex Fracture (T ripod) is the m ost com m on fracture
pattern, and involves the zygom a, inferior orbit, and lateral orbit.

• Le-Fort Fractures are both a stupid and a high yield topic in facial traum a - for
m ultiple choice. Floating Palate = 1, Pyram idal = 2, Separated Face = 3

• Transverse vs Longitudinal Tem poral Bone Fractures - this classification system


is stupid and outdated since m ost are m ixed and otic capsule violation is a w ay
better predictive fac to r... but this is still extrem ely high yield

119
TEMPORAL BONE

It w ould be very easy to get com pletely carried aw ay w ith this anatom y and spend the next
20 pages talking about all the little bum ps and variants. I’m gonna resist that urge and instead
try and give you som e basic fram ew ork. Then as w e go through the various pathologies I ’ll
try and give “norm al” anatom y com parisons and point out som e landm arks that are relevant
for pathology. A dditionally, I ’m gonna do a full anatom y T-Bone talk for R adiologyR onin
this year - so if your really w ant to understand this deeper, that m ight be helpful.

Epitympanum
“The A ttic”

Mesotympanum

Hypotympanum

A closer look at the middle ear gives us 3 more


At the most basic level you can think about 3 general locations
general locations: External, Middle, and Inner.
The Epitympanum, also called “the attic” is
The External car is everything superficial to the basically everything above the tip o f the scutum
ear drum (tympanic membrane). (see below).

The Inner ear is everything deep to the medial The Hypotympanum is everything below the
wall o f the tympanic cavity. tympanic membrane. This is where the
Eustachian tube arises.
The Middle ear is everything in-between.
The Mesotympanum is everything in-between
(or everything directly behind the ear drum).

The scutum (arrows) is a “shield” like osseous spur formed


via the lateral wall of the tympanic cavity. This anatomic
land mark is often brought up with discussion o f the
erosion pattern o f Cholesteatomas.

120
Middle &Inner Ear Pathology

C h o le s te a to m a - The sim ple w ay to think about this is “a bunch o f exfoliated skin


debris grow ing in the w rong p lace.” It creates a big inflam m ation ball w hich w recks the
tem poral bone and the ossicles.

There are tw o parts to the ear drum , a flim sy w him py part “ Pars
Flaccida” , and a tougher part “ Pars T ensa.” T he flim sy flaccida P' Flaccida
is at the top, and the tensa is at the bottom . P_ Tensa

If you “acquire” a hole w ith som e


inflam m ation / infection involving the Pars
Flaccida you can end up w ith this ball o f
epithelial crap grow ing and causing
inflam m ation in the w rong place.

- Typical location o f this


soft tissue blob with
erosion o f scutum
(arrow)

- They restrict diffusion

There are two subtypes based on the location.

Pars F la c c id a Type P ars T en sa Type:

A cquired Types are m ore com m on - typically involving •The inner ear structures are
the pars flaccida. They grow into P ru ssak ’s Space involved earlier and m ore
• The Scutum is eroded early (m aybe first)- considered often
a very specific sign o f acquired cholesteatom a •T h is is less com m on than
The M alleus head is displaced m edially the Flaccida Type
• The long process o f the incus is the m ost com m on
segm ent o f the ossicu lar chain to be eroded.
Fistula to the sem i-circular canal m ost com m only
involves the lateral segm ent

121
P r u s s a k ’s S p a c e a n d S c u tu m E ro s io n :

This is a coronal view of the T-Bone. To orient you 1


drew I cartoon finger in the ear. That finger is running
right up to the ear drum (tympanic membrane). The
membrane is usually too thin to see, but it’s right around
there. Remember the flimsy Flacida is at the top and the
thicker Tensa is at the bottom.

There are two white arrows here.

The top arrow is pointing to a space between an ossicle


(incus) and the lateral temporal bone. This is called
“Prussak’s space” and is the most common location of
a Pars Flacida Cholesteatoma. Remember the incus was
the most common ossicle eroded.

The bottom arrow is pointing to a bony shield shaped


bone - “the scutum” which will be the first bone eroded
by a pars flaccida.

L a b y r in th in e F is tu la (p e r ily m p h a t ic f is tu la ):

This is a potential com plication o f


cholesteatom a (or other things - iatrogenic,
traum a, e tc ...). W hat w e are talking about here
is a bony defect creating an abnorm al
com m unication betw een the norm ally fluid
filled inner ear and norm ally air filled tym panic
cavity. In the case o f cholesteatom a, the
lateral sem icircular canal (arrow s) is m ost 17y
Coronal Axial
often involved.

The classic clinical history is “sudden fluctuating sensorineural hearing loss and vertigo.’

On CT, you w ant to see the soft


tissue density o f the cholesteatom a
eating through the otic capsule into
the sem icircular canal. The
presence o f air in the sem icircular
canal (p n eum olabyrinth) is
definitive evidence o f a fistula
(although it’s not often seen in the
real w orld).

122
O titis M e d ia (O M ) - This is a common childhood disease with effusion and infection of the
middle ear. It’s more common in children and patients with Down Syndrome because o f a more
horizontal configuration of the Eustachian tube. It’s defined as chronic if you have fluid persisting for
more than six weeks.

It can look a lot like a cholesteatoma (soft tissue density in the middle cart.

T H IS vs THAT: C o m p lic atio n s o f OM

C hronic O titis Coalescent Erosion o f the mastoid septae with or


C holesteatom a
M edia Mastoiditis without intramastoid abscess
Poorly Poorly Secondary to inflammation o f the
Mastoids Facial Nerve
pneumatized pneumatized tympanic segment (more on this later in
Palsy
the chapter).
Middle Ear Can completely Can completely
Opacification opacity opacity Adjacent inflammation may cause
thrombophlebitis or thrombosis o f the
For the Purpose
Non- sinus. This in itself can lead to
o f Multiple Thickened
Dependent complications:
Choice - this mucosa
Mass
could be a hint Dural Sinus Otitic
Venous Infarct:
Thrombosis Hydrocephalus:
Erosions This can occur
Common Lateral sinus
(scutum and Rare ( < 10%) secondary to
(75%) thrombosis can
ossicular chain) dural sinus
alter resorption o f
thrombosis
CSF and lead to
Displacement
hydrocephalus.
o f the ossicular NEVER It can happen
chain Meningitis,
and It can happen
Labyrinthitis

L a b y r in th itis O s s ific a n s -
• Gamesmanship - "history o f childhood meningitis. ” WTF is a “membranous labyrinth ” ?
• You see it in kids (ages 2-18 months).
The w orld “Labyrinth” m ost com m only
• Classic Appearance on CT - Ossification o f the
refers to the tim eless 1986 science
membranous labyrinth. fiction adventure staring
• They present with sensorineural hearing loss. David Bowie as Jareth the M other
• Calcification in the cochlea is often considered a Fucking Goblin King.
contraindication for cochlear implant.
A nother less popular use o f the word
“ Labyrinth” is the anatom ical blanket
term encom passing the Vestibule,
C ochlea, and Sem icircular Canals.

U nder the um brella o f the “Labyrinth”


you can have the bony portion (the
series o f canals tunneled out o f the t-
bone), and the membraneous portion
(w hich is basically the soft tissue lining
inside the bony part).
You can then further divide the
“m em branous” portion into the
cochlear & vestibular labyrinths.

Normal - For Comparison Labyrinthitis Ossificans

123
L a b y r in th itis - This is an inflam m ation o f the
m em branous labyrinth, probably m ost com m only the
result o f a viral respiratory track infection. A cute
otom astoiditis can also spread directly to the inner ear
(this is usually unilateral). B acterial m eningitis can
cause bilateral labyrinthitis.

C lassic Look: The cochlea and sem icircular canals


will be show n enhancing on T l post contrast im aging.

T h e F a c ia l N e r v e (C N 7 )

Enhancement
Most people will describe 6 segments to the facial nerve.
The facial nerve is unique in that
• Intracranial (“Cisternal”) segment portions o f it can enhance normally.
The trick is which parts are normal
• Meatal (“Canalicular”) segment - the part inside the
and which parts arc NOT.
Internal Auditory Canal “IAC”).
Normal Enhancement: Tympanic &
• Labyrinthine segment (LS) - from the IAC to
Mastoid Segments including the
geniculate ganglion (GG). Geniculate Ganglia. The
• Tympanic segment (TS) - GG to pyramidal eminence Labyrinthine segment can also
• Mastoid segment (MS) - from pyramidal eminence sometimes.
to stylomastoid foramen “SMF” No normal enhancement = Cisternal,
• Extratemporal segment - Distal to the SMF Canalicular, or Extratemporal

Axial MR T2 - Cisternal Segment Axial CT- Level o f IAC


-CN8 is posterior also -the bend at the GG = Sagittal CT-
entering the IAC “anterior genu” -

IVhat causes abnorm al enhancem ent? B e lls P a ls y & R a m s e y H u n t


Big one is B e ll’s Palsy. Lymes, R am say
Bells: Etiology is probably viral. Usually a
H unt, and C ancer can do it too. clinical diagnosis. Abnormal enhancement in
When do yo u think C ancer ? the Canalicular Segment (in the IAC) is
probably the most classic finding.
N odular Enhancem ent.

When do y o u dam age the fa cia l nerve? RH: Caused by reactivation varicella zoster
virus. Classic rash around ear. CN 5 is
T-Bone fracture (transverse > longitudinal).
usually also involved.

124
O to s c le r o s is ( F e n e s tr a l a n d R e t r o f e n e s t r a l) :

A better term would actually be “otospongiosis,” as the bone becomes more lytic (instead of sclerotic).
When I say conductive hearing loss in an adult female, you say this.

Fenestral - This is bony resorption anterior to the oval widow at the fissula ante fenestram.
If not addressed, the footplate will fuse to the oval window.

Retro-fenestral - This is a more severe form, which has progressed to have demineralization around
the cochlea. This form usually has a sensorineural component, and is bilateral and symmetric nearly
100% of the time.

Cochlea

Fissula
Ante
Fenestram

Vestibule

Horizontal
Posterior Turn Turn o f the
o f the SCC SCC

B ony
resorption
anterior to the N orm al
oval w idow at C om parison
the fissula ante
fenestram

Treatm ent O ptions:

Early on ( if the focus is sm all) dietary supplem entation w ith Fluoride


m ay be useful. A lthough this is controversial - m ay or m ay not w o rk ...
and m ay or m ay not be part o f a new w orld order D avid Icke R eptilian
conspiracy to low er IQs (A lex Jones has the docum ents).

Slithery space horn


Later on they m ight try a stapedectom y (partial rem oval o f the stapes
illuminati villains
w ith im plantation o f a prosthetic device) or a C ochlear im plant. advocate for
Fluoride treatment.

125
S u p e r io r S e m ic ir c u la r C a n a l D e h i s c e n c e
Pietro Tullio
This is an Aunt Minnie. It’s supposedly from long standing elevated
Mad scientist who drilled
ICP. The most likely way this will be asked is either (1) what is it? with
holes in the semicircular
a picture or (2) “Noise Induced Vertigo” or “Tullio’s Phenomenon.” canals of pigeons then
observed that they became
off balance when he
exposed them to loud
sounds.

He also created a “pigeon


rat” like Hugo Simpson did
in the 1996 Simpsons
Halloween Special.

Normal Anatomy
Note the intact Bony Roof (Arrow) Superior Semicircular Canal
Dehiscence Note the Absence
of a Bony Covering
"Look, I've been practicing”
L a r g e V e s tib u la r A q u e d u c t S y n d r o m e
The vestibular aqueduct is a bony canal that connects the vestibule (inner ear) with the endolymphatic
sac. The enlargement of the aqueduct (> 1.5 mm) has an Aunt Minnie appearance. The classic
history is progressive sensorineural hearing loss. Supposedly the underlying etiology is a failure of
the endolymphatic sac to resorb endolymph, leading to endolymphatic hydrops and dilation.

Trivia:
jP 1 «This is the most common cause o f congenital sensorineural hearing loss
•The finding is often (usually) bilateral,
jlv ‘There is an association with cochlear deformity - near 100%
4y r l (absence o f the bony modiolus in more than 90%)
•Progressive Sensorineural Hearing Loss (they arc NOT bom deaf)

The normaI Vestibular Aqueduct (VA) is Enlarged Vestibular Aqueduct


NEVER larger than the adjacent (larger than PSCC - grey arrow)
Posterior Semicircular Canal (PSCC)

126
C ongenital m alform ations o f the inner ear can be thought about along a spectrum o f severity.
As the construction o f the hearing m achinery is com plicated business, you can im agine that
the earlier things go w rong in this m ulti-stage anatom ical developm ent the m ore severe the
anomaly.

A long those lines, w e can discuss tw o disorders on opposite ends o f that severity spectrum
w ith the M ichel’s A plasia being the earliest and m ost severe, and the C lassic M o n d in i’s
M alform ation (incom plete partition II) being the latest and least severe.

M ondini M a lfo r m a tio n - Type o f cochlear hypoplasia w here the basal turn is norm al, but
the m iddle and apical turns fuse into a cystic apex. This is usually w ritten as “only 1.5 turns" -
instead o f the norm al 2.5. T here is an association w ith an enlarged vestibule, and enlarged
vestibular aqueduct. They have sensorineural hearing loss, although high frequency sounds are
typically preserved (as the basal turn is norm al).

M ic h e l’s A p la s ia - This is also referred to as com plete labyrinthine aplasia or “ C L A .” As


above, this represents the m ost severe o f the congenital abnorm alities o f the inner ear - w ith
absence o f the cochlea, vestibule, and vestibular aqueduct. N o surprise these kids are
com pletely deaf.

A ssociations: A nencephaly, T halidom ide Exposure

G am esm anship: Som e people think this looks like labyrinthitis ossificans. Look for the
absent vestibular aqueduct to help differentiate.

t h is vs THAT: Mondini vs Michel


M o n d in i M ic h e l

Tim ing Late (7th W eek) E arly (3rd W eek)

Severity Som e Preserved High Frequency H earing Total D eafness

Cochlea: C ystic A pex (basilar turn is norm al) A b sent

Vestibule Som etim es E nlarged (can be norm al) A b sent

Vestibular
Large A b sent
A queduct

C om m on
Frequency Rare A s Fuck
(relative to other m alform ations)

127
E n d o ly m p h a tic S a c T u m or
Rare tum or o f the endolym phatic sac and duct. A lthough m ost are sporadic, w hen you see this
tum or you should im m ediately think V on -H ip p el-L in d au .

C lassic Look: They alm ost alw ays have internal am orp hous calcifications on CT. T here are
T2 bright, w ith intense enhancem en t. They are very vascu lar often w ith flow voids, and
tum or blush on angiography.

P a r a g a n g lio m a

On occasion, paragangliom a o f the ju g u la r fossa (glom us ju g u lare or ju g u lotym panic tum ors)
can invade the occipital bone and adjacent petrous apex.

Trivia:

JLv »40% o f the tim e it’s hereditary, and they are m ultiple.
W rl »The m ost com m on presen ting sym ptom is hoarseness from vagal nerve com pression.
A »They are very vascular m asses and enhance avidly w ith a “ salt and p epper”
appearance on post contrast M R I, with flow voids.
*They are FD G avid.

M ore on these later in the chapter

128
Petrous Apex - Anatomic Variations:
Variation can occur in the am ount o f pneum atization, m arrow fat, bony continuity, and
vascular anatom y.

A s y m m e tr ic M arrow : - T ypically the petrous apex contains significant fat, closely


follow ing the scalp and orbital fat (T1 and T2 bright). W hen it’s asym m etric you can have
two problem s (1) falsely thinking y o u ’ve got an infiltrative process w hen you d o n ’t, and (2)
overlooking a T1 bright thing (cholesterol granulom a) thinking it’s fat. The key is to use
STIR or som e other fat saturating sequence.

C e p h a lo c e le s : - A cephalocoele describes a herniation o f C N S content through a defect


in the cranium . In the petrous apex they are a slightly different anim al. They d o n ’t contain any
brain tissue, and sim ply represent cystic expansion and herniation o f the posterolateral portion
o f M eckel’s cave into the superom edial aspect o f the petrous apex. D escribing it as a
herniation o f M eck el’s C ave w ould be m ore accurate. T hese are usually unilateral and are
classically described as “sm o o th ly m arginated lo h u la ted cystic expansion o f the petrous
apex. ”

A b e r r a n t in te r n a l c a r o tid . The classic history is pulsatile tinnitus (although other


things can cause that). T his term is used to describe the situation w here the C l (cervical)
segm ent o f the ICA has involuted/underdeveloped, and m iddle ear collaterals develop
(enlarged caroticotym panic artery) to pick up the slack. T he hypertrophied vessel runs
through the tym panic cavity and joins the horizontal carotid canal. The EN T exam will show a
vascular m ass pulsing behind the ear drum (d o n ’t expect them to m ake it that easy for you).

T he oldest trick in the book is to try and fool you


into calling it a paragangliom a.

Look for the connection to the horizontal carotid


canal - that is the m ost classic w ay to show this.

DO N O T B IO PSY !

129
A p ic a l P e t r o s it is

Infection o f the petrous apex is a rare com plication o f infectious otom astoiditis. It can have
som e bad com plications if it progresses including osteom yelitis o f the skull base, vasospasm o f
the ICA ( if it involves the carotid canal), subdural em pyem a, venous sinus throm bosis, tem poral
lobe stroke, and full on m eningitis.

In children, it can present as a prim ary process. In adults it's usually in the setting o f chronic
otom astoiditis or recent m astoid surgery.

G r a d e n ig o S y n d r o m e Dorello's Canal

This is a com plication o f apical petrositis, w hen D o rello ’s canal The most medial point
of the pertrous ridge -
(CN 6) is involved. They will show you (or tell you) that the patient
between the pontine
has a lateral rectus palsy. cistern and cavernous
sinus

C lassic Triad:
• O tom astoiditis,
• Face pain (trigem inal
neuropathy), and
• Lateral R ectus Palsy
<— \

Look Straight Look Left

130
Petrous Apex- Inflammatory Lesions
C h o le s te r o l G ra n u lo m a

The m ost com m on prim ary petrous apex lesion. M echanism is likely obstruction o f the air
cell, w ith repeated cycles o f hem orrhage and inflam m ation leading to expansion and bone
rem odeling. T he m ost com m on sym ptom is hearing loss.

On CT the m argins will be


sharply defined. On MRI
it’s gonna be T l and T2
bright, w ith a T2 dark
hem osiderin rim , and faint
peripheral enhancem ent.

K ey Point:
C h olesterol G ranulom a =
T l a n d T2 Bright.

The slow grow ing ones can


be w atched. The fast Cholesterol Granuloma = T1 and T2 B right
grow ing ones need surgery.

C h o le s t e a t o m a

This is basically an epiderm oid (ectopic epithelial tissue). U nlike the ones in the m iddle ear,
these are congenital (not acquired) in the petrous apex. T hey are typically slow grow ing, and
produce bony changes sim ilar to cholesterol granulom a.

The difference is their M RI findings; T l d ark . T2 bright, and restricted d iffusion.

K ey Point: C h olesteatom a = T l D ark, T2 Bright, R estricted D iffusion

THIS vs THAT:

C h o lestero l G ranulom a C h o lestea to m a


T l Bright T l D ark

T2 Bright T2 Bright

D o esn ’t R estrict Does R estrict

Sm ooth Expansile B ony C hange Sm ooth E xpansile B ony C hange

I3l
External Ear

R e g u la r a n d N e c r o t iz in g O titis E x te r n a :

O titis Externa - the so called “ sw im m ers ear” is an infection (usually bacterial) o f the
external auditory canal. The m ore testable version N ecrotizin g O titis E xterna (also called
"M a lig n a n t" O titis E xterna - for the purpose o f fucking w ith you) is a m ore aggressive
version seen alm ost exclusively in diabetics.

You are going to see sw ollen EAC soft tissues, probably w ith a bunch o f sm all abscesses, and
adjacent bony destruction.

They alw ays (95% ) have diabetes and the causative agent is alw ays (98% ) Pseudom onas.

E x te r n a l A u d ito r y C a n a l E x o s t o s i s (“S w im m e r s Ear”) - This is an


overgrow th o f tissue in the ear canal, classically seen in Surfers w ho get repeated bouts o f ear
infections. It’s usually bilateral, and w hen chronic will look like bone. U nlike N ecrotizing
O titis, these patients are im m unocom petent and non-diabetes (although they are dirty hippie
surfers).

E x te r n a l A u d ito r y C a n a l O s t e o m a - This is a benign bone tum or, m aybe best


thought o f as an overgrow th o f norm al bone. They are usually incidental and unilateral
(,rem em ber exostosis w as b ilateral) occurring near the ju n ctio n o f cartilage and bone in the
ear canal.

E x te r n a l A u d ito r y C a n a l A tr e s ia :
This is a developm ental anom aly w here the external
auditory canal (secondary crayon storage
com partm ent) d o e sn ’t form . A s you m ight im agine,
this results in a hearing deficit (conductive subtype).
There m ay or m ay not be a m ashed up ossicular
chain.
Normally, there is a place to stick a
crayon right around here (arrow).

Trivia: EN T will w ant to know: (1) if the tissue covering the norm ally open ear hole (atretic
plate) is soft tissue or bone, and (2) if there is an aberrant course o f the facial nerve.

132
S E C T IO N 1 3 :

t S k u l l Base & S in u s e s

P a g e t s - This is discussed in great depth in the M SK chapter. H aving said that, I w ant to
rem ind you o f the Paget skull changes. You can have osteolysis as a w ell-defined large
radiolucent region favoring the frontal and occipital bones. Both the inner and outer table are
involved. The buzzw ord is osteolysis circum scripta.

P aget s Skull related com plications:


* D eafness is the m ost com m on com plication
* C ranial N erve Paresis
* B asilar Invagination -> H ydrocephalus -> B rainstem C om pression
* Secondary (high grade) osteosarcom a.

Pagets - Osteolysis Circumscripta (lytic phase) Thickened Expanded Skull (Sclerotic Phase)

C h o r d o m a (m idline) & C h o n d r o s a r c o m a ( o ff m idline) d iscu ssed in M S K chapter.

F ib r o u s D y s p la s ia - The ground-glass lesion. If you are getting ready to call it Pagets,


stop and look at the age. Pagets is typically an older person (8% at 80), w hereas fibrous
dysplasia is usually in som eone less than 30.

• Classically, fibrous dysplasia o f the skull spares the otic capsule


• M cC une A lbright Syndrom e - M ultifocal fibrous dysplasia, cafe-au-lait spots, and
precocious puberty.
• The outer table is favored (Pagets tends to favor in the inner table)

133
S in u s D is e a s e S tr a te g y Intro: You will see C’T and MRI used in the evaluation o f sinus
disease. It is useful to have some basic ideas as to why one modality might be preferred over the
other (for gamesmanship and distractor elimination). CT is typically used for orbital and sinus
infections. In particular it is useful to see if the spread o f infection involves the anterior 2/3 o f the
orbit. If you wanted to know if the patient has cavernous sinus involvement, or involvement of the
posterior 1/3 (orbital apex) MRI will be superior.

CT also has the ability to differentiate common benign disease - H y p e r D ense Sinus
(inspissated secretion and allergic fungal sinusitis) from the more
• Blood
rare sinus tumors. The trick being that a hyperdense opacified sinus
• Dense (inspissated)
is nearly always benign (tumor will not be dense). In addition to
Secretions
that CT is useful for characterization of anatomical variation • Fungus
(justification o f endoscopic nasal surgery for recurrent sinusitis).

MRI is going to be more valuable for tumor progression / extension (perineural spread, marrow
involvement etc..).

Fungal S in u sitis

This comes in two flavors; the good one (allergic) and the bad one (invasive). The chart below will
contrast the testable differences:

A lle rg ic Fungal S inusitis A c u te In vasive Fugal S inusitis

Opacification o f multiple sinuses. Stranding /


Opacification o f multiple sinuses, ususually bilateral
Extension into the fat around the sinuses in the key
favoring the ethmoid and maxillary sinuses.
finding.

Immunocompromised
Normal Immune System (Asthma is common) - Neutropenic = Aspergillus
- Diabetic in DKA = Zygomycetes / Mucor

CT: Opacified Sinus with is NOT hyperdense. Fat


CT: Hyperdense centrally or with layers. Can stranding in the orbit, masticator fat, pre-antral fat,
erode and remodel sinus walls if chronic. or PPF suggests invasion. This does NOT require
bone destruction.

MRI: T1-T2 Dark - because o f the high protein


content / heavy metals. Can mimic an aerated sinus. MRI: Also can be T1/T2 Dark. However, the
mucosa may not enhance (suggesting it is necrotic).
Inflamed (T2 bright) mucosa which will enhance.
The glob o f fungus snot will not enhance (thats how The extension of disease out of the sinus will be
you know it is not a tumor). bright on STIR and enhance.

C h r o n ic I n f la m m a t o r y S i n o n a s a l D i s e a s e

This is typically thought o f as an inflammation o f the paranasal sinuses


that lasts at least 12 weeks. The causes arc complex and people write
long boring papers about the various cytokines and T-Cell mediation
pathways are involved but from the Radiologist’s point o f view the issue
is primarily anatomical patency o f sinus Ostia (box with arrow).

134
C h r o n ic I n f l a m m a t o r y S in o n a s a l D i s e a s e C o n t in u e d ..

There are several described patterns o f recurring sinonasal disease.

In fu n d ib u la r O s tio m e a ta l U n it P a tte r n S in o n a s a l P o ly p o s is
P a tte rn . P a tte r n .
Second most common pattern. I’m
The most common not going to get into depth on the
The pattern is characterized by a
pattern. various subtypes -
combination o f soft tissue nasal
polyps (found throughout the nasal
In this pattern, just think about
cavity) and variable degrees o f sinus
disease is limited to this as more
opacification. About half the time
the maxillary sinus centered at the
fluid levels will also be present.
and occurs from the middle
obstruction at the meatus (star)
A key feature is the bony remodeling
ipsilateral ostium / with disease
and erosion. In particular the
infundibulum (star). involving the
“widening o f the infiindibula” is the
ipsilateral
classic description. This erosion and
maxillary,
remodeling is important to distinguish
frontal and
between the “expansion” o f the sinus
ethmoid
- which is more classic for a
sinuses.
mucocele.

The contributors to this pattern


Testable associations include CF and
involve all the usually suspects
Aspirin Sensitivity.
(hypertrophied turbinates, anatomic
variants - concha bullosa, middle
turbinates which curl the wrong way
“paradoxical”, and septal deviation).

M u c o c e le

This is how I think about these things. You have an obstructed sinus.
Maybe you had trauma which fucked the drainage pathway or you’ve got
CF and the secretions just clog things up. Mucus continues to accumulate
in the sinus, but it can’t clear (because it's obstructed). Over time the sinus
become totally filled and then starts to expand circumferentially. Hence the
buzzword “expanded airless sinus.” The frontal sinus is the most common
location. It w on't enhance centrally (it is not a tumor), but the periphery
may enhance from the adjacent inflamed mucosa.

135
A n t r o c h o a n a l P o ly p
Seen in young adults (30s-40s), classically presenting w ith nasal congestion / obstruction sym ptom s. Arises
w ithin the m axillary sinuses and passes through and enlarges the sinus ostium (or accessory ostium).

Buzzw ord is “widening of the maxillary ostium/'

Classically, there is no associated bony destruction but instead smooth enlargem ent o f the sinus. The polyp
will extend into the nasopharynx. This thing is basically a m onster inflam m atory polyp with a thin stalk
arising from the m axillary sinus.

J u v e n ile N a s a l A n g io f ib r o m a (J N A )

Often you can get this one right ju st from the history - Male teenager with nose bleeds (obstruction is
actually a m ore com m on sym ptom in real life, but not so m uch on m ultiple choice).

•L ocation = Centered on the sphenopalatine foram en


•B one Rem odeling (not bone destruction)
•Extrem ely vascular (super enhancing) w ith intratum oral Flow Voids on M R
f t •Pre-surgical em bolization is com m on (via internal m axillary & ascending pharyngeal artery)

In v e r t e d P a p illo m a :
This uncom m on tum or has distinctive im aging features (which therefore make it testable). The classic
location is the lateral wall of the nasal cavity - most frequently related to the middle turbinate.
Impaired m axillary drainage is expected.

•A focal hyperostosis tends to occur at the tum or origin.


•A nother high yield pearl is that 10% harbor a squam ous cell CA.
* •MRI “cerebriform pattern” - which sorta looks like brain on T1 and T2.

E s t h e s io n e u r o b la s t o m a :
This is a neuroblastom a o f olfactory cells so it’s gonna start at the cribiform plate. It classically has a
dum bbell appearance w ith grow th up into the skull and grow th dow n into the sinuses, w ith a w aist at the
plate. There are often cysts in the mass. There is a bi-m odal age distribution.

•D um bbell shape with w asting at the cribiform plate is classic


.Intracranial posterior cyst is a “diagnostic” look
M k ‘O ctreotide scan will be positive - since it is o f neural crest origin

S q u a m o u s C e ll I S N U C :
Squam ous cell is the most common head and neck cancer. The m axillary antrum is the m ost com m on
location. It's highly cellular, and therefore low on T2. Relative to other sinus m asses it enhances less.
SNUC (the undifferentiated squam er), is the m onster steroided-up version o f a regular squam ous cell. They
are m assive and seen m ore in the ethmoids.

E p is ta x is (N o s e B le e d s )
This is usually idiopathic, although it can be iatrogenic (picking it too much - or not enough). They could
get sneaky and work this into a case o f HHT (hereditary hemorrhagic telangiectasia). The most common
location is the anterior septal area (Kiesselbach plexus) - these tend to be easy to compress manually. The
posterior ones are less common (5%) but tend to be the ones that “bleed like stink" (need angio). Most
cases are given a trial o f nasal packing. W hen that fails, the N-IR team is activated.

J lv The main supply to the posterior nose is the sphenopalatine artery (terminal internal maxillary
artery) and tends to be the first line target. Watch out for the variant anastomosis between the ECA
and ophthalmic artery (you don’t want to embolize the eye).

136
N asal S e p ta l P e rfo ra tio n

Typically involves the


anterior septal cartilaginous
area.

There are a bunch o f things


that can do this. T hese are
the ones I'd think about:

• Surgery - O ld school
Septoplasty techniques -
essentially resecting the
thing (K illian subm ucous
resection)
• C ocaine use (> 3 m onths)
• Too m uch nose picking (or perhaps not picking it enough)
• G ranulom atosis w ith polyangiitis (W egener gran u lom atosis) — Triad o f renal m asses, sinus
m ucosal thickening and nasal septal erosion, disease, and cavitary lung nodules / fibrosis.
cA N C A positive.
• Syphilis - affects the bony septum (m ost everything else effect the cartilaginous regions).

137
M O U CTTH °& JA W ^ IM M a B B H M M

S ia lo lith ia s is - Stones in the salivary ducts. The


testable trivia includes: (1) M ost com m only in the Submandibular = Wharton
subm andibular gland duct (w harton’s), Parotid = Stenson
(2) can lead to an infected gland “sialoadenitis”, and Sublingual = Rivinus
(3) chronic obstruction can lead to gland fatty atrophy.

O d o n to g e n ic In fe c tio n - These can be dental or periodontal in origin. If I were w riting


a question about this topic I w ould ask three things. The first w ould be that infection is more
com m on from an extracted tooth than an abscess involving an intact tooth.

The second w ould be that the


attachm ent o f the m ylohyoid Sublingual
m uscle to the m ylohyoid ridge Space
dictates the spread o f infection to -Above the Mylohyoid Line
the sublingual and subm andibular (Anterior Mandibular Teeth)
spaces. A bove the m ylohyoid line
(anterior m andibular teeth) goes to
the sublingual space, and below the
m ylohyoid line (second and third
m olars) goes to the subm andibular
space.

The third thing I w ould ask w ould be


that an odontogenic abscess is the Subm andibular
m ost com m on m asticator space Space
“m ass” in an adult. -Below the Mylohyoid Line
(2nd and 3rd Molars)

L u d w ig ’s A n g in a :

This is a super aggressive cellulitis in the floor o f the


m outh. If they show it, there w ill be gas everyw here.

Trivia: m ost cases start w ith an odontogenic infection

138
T o ru s P a la tin u s : O s te o n e c r o s is o f R a n u la :
t h e M a n d ib le
This is a norm al variant that
This is a m ucous retention
looks scary.
cyst. They are typically
T he trivia is m ost likely
lateral. There are two
Because it looks scary some gonna be etiology.
testable pieces o f trivia to
m ultiple choice w riter m ay try
know:
and trick you into calling it
Just rem em ber it is related
cancer.
to prior radiation, licking (1) They arise from the
It’s just a bony exostosis that a radium paint brush, or sublingual gland / space.
com es o ff the hard palate in and
b isphosphon ate
the midline.
treatm ent. (2) Use the w ord
Classic History. “G randm a’s “plunging” once it’s under
dentures w on’t stay in.” the m ylohyoid m uscle.

T h y r o g lo s s a l D u c t C y s t - This can occur anyw here betw een the foram en cecum (the
base o f the tongue) and the thyroid gland. T hey are usually found in the m idline. It looks like a
thin-w alled cyst. F urther discussion in the endocrine & peds chapters.

F lo o r o f M o u th D e rm o id I E p id e rm o id - There isn ’t a lot o f trivia about these other


than the buzzw ord and w hat they classically look like. The buzzw ord is “sack o f m arbles” -
fluid sack with globules o f fat. They are typically m idline. Further discussion in the peds
chapter.

C a n c e r - Squam ous cell is going to be the m ost com m on cancer o f the m outh (and head and
neck). In an older person think drinker and sm oker.

In a younger person think H P V . HPV related SC C s tend to be present w ith large necrotic level
2a nodes (d o n ’t call it a branchial cleft cyst!).

C lassic Scenario = Young adult with new level 1/ neck m ass = H P V related SCC.

139
lesions of the Jaw
T here are a B U N C H o f these and they all look pretty sim ilar. Lesions in the ja w are broadly
grouped into either odontogenic (from a tooth) or non-odontogenic (not from a tooth). The
non-odontogenic stu ff you see in the m andible is the sam e kind o f stu ff you see in other bones
(A B C s, Sim ple Bone C ysts, O steom yelitis, M yelom a / P lasm acytom a e tc ...). I think if a test
w riter is going to show a ja w lesion - they probably are going to go for odontogenic type.
O bviously your answ er choices will help you decide w hat they are going for. T he other tip is
that odontogenic lesions are usually associated w ith a tooth.

I'm going to pick 5 that I think are m ost likely to be asked and focus on how 1 w ould tell them
apart.

P e r ia p ic a l C y s t (R a d ic u la r C y s t) - This is the m ost


com m on type o f odontogenic cyst. They are typically the result
o f inflam m ation from dental caries (less com m only traum a).
The inflam m atory process results in a cystic degeneration
^ around the periodontal ligam ent.

A
Aw ‘ L ocated at the apex o f a non-vital tooth
‘ R ound w ith a Well C orticated B order
• U sually < 2 cm

D e n tig e r o u s C y s t (F o llic u la r C y s t) - This is a


cyst that form s around the crow n o f an un-erupted tooth.
It’s best thought o f as a developm ental cyst (peri-apicals
are acquired). These things like to displace and resorb
adjacent teeth - usually in an apical direction. This is the
kind o f cyst that will displace a tooth into the condylar
^ regions o f the m andible or into the floor o f the orbit.

A
‘ L ocated at the crow n o f an un-erupted tooth
• Tend to displace the tooth

140
K e r a to g e n ic O d o n to g e n ic T u m o r
(O dontogenic K eratocyst) - U nlike the prior two
lesions (w hich w ere basically fluid collections) this
is an actual tumor. They tend to occur at the
m andibular ram us or body. A lthough they can be
uni-locular the classic look is m ulti-locular
( “daughter cysts ”) and th a t’s how I w ould expect
them to look on the test.

• Body / R am us M andible
• They typically grow along the length o f the bone
• W ithout significant cortical expansion
• M ay have daughter cysts
• W hen m ultiple think G orlin Syndrom e

A m e lo b la s to m a (A d am an tin om a o f the
jaw ) - This is another tum or (locally aggressive).
The appearance is variable but for the purpose o f
m ultiple choice I w ould expect the m ost classic
look - m ulti-cystic w ith solid com ponents and
expansion o f the m andible.

O ut o f the four I ’ve discussed, this will be the


m ost aggressive-looking one. If they show you a
really aggressive-looking lesion, especially if it
has m ultiple “soap bubbles” - you should
consider this.

• H allm ark = E xtensive Tooth Root A bsorption


• M andibular E xpansion
• Solid com ponent (show n on M R or C T) favors the Dx o f A m eloblastom a
• A bout 5% arise from D entigerous Cysts

O d o n to m a - This is the easy one to pick out because it’s


m ost likely to be show n in it’s m ature solid form (they start
out lucent). It’s actually the m ost com m on odontogenic
tum or o f the m andible. It’s basically a “tooth ham artom a.”

• R adiodense w ith a lucent rim

141
S E C T IO N 15:

t J Ci m o a u v m i-> M r ^ - i /

SU PRA HYO ID NECK


<
* IMIMJMP1PMPM 4 B

• Can be LA R G E w ith “ fluffy” calcifications

The suprahyoid neck is usually taught by using a “spaces” m ethod. This is actually the best
w ay to learn it. W hat space is it? W hat is in that space? W hat pathology can occur as the
result o f w hat norm al structures are there? Exam ple: lym ph nodes are there - thus you can get
lym phom a or a met.

Parotid Space:

The parotid space is basically


the parotid gland, and portions
o f the facial nerve. You c a n 't
see the facial nerve, but you can
see the retrom andibular vein
(w hich runs ju st m edial to the
facial nerve).

A nother thing to know is that


the parotid is the only salivary
gland to have lym ph nodes, so
pathology involving the gland
itself, and anything lym phatic
related, is fair gam e.

Parotid S p ace C on tains:


- The Parotid Gland
- Cranial Nerve 7 (Facial)
- Retro-m andibular Vein

142
Pathology:
Major Salivary Glands:
P le o m o rp h ic A d e n o m a (b e n ig n m ix e d tu m o r ) -
• Parotid
This is the m ost com m on m ajor (and m inor) salivary gland • Submandibular
tumor. It occurs m ost com m only in the parotid, but can also • Sublingual
occur in the subm andibular or sublingual glands. 90% o f
these tum ors occur in the superficial lobe. They are Minor Salivary Glands
com m only T2 bright, with a rim o f low signal. They have a • Literally 100s of un­
named minor glands
sm all m alignant potential and are treated surgically.

Superficial vs D eep: Involvem ent o f the superficial (lateral to the facial nerve) or deep
(m edial to the facial nerve) lobe is critical to the surgical approach. A line is draw n
connecting the lateral surface o f the posterior belly o f the digastric m uscle and the
lateral surface o f the m andibular ascending ram us to separate superficial from deep.

Apparently, if you resect these like a clow n you can spill them , and they will have a
m assive, ugly recurrence.

W a rth in s : This is the second m ost com m on benign tumor. This one ONLY occurs in the
parotid gland. This one is usually cystic, in a m ale, bilateral (15% ), and in a sm oker. As a
point o f total trivia, this tum or takes up pertechnetate (it’s basically the only tum or in the
parotid to do i t , ignoring the ultra rare p a ro tid oncocytom a).

M u c o e p id e rm o id C a r c in o m a - This is the m ost com m on m alignant tum or o f m inor


salivary glands. The general rule is - the sm aller the gland, the m ore com m on the m alignant
tumors; the bigger the gland, the m ore com m on the benign tum ors. There is a variable
appearance based on the histologic grade. There is an association w ith radiation.

A d e n o id C y s tic C a r c in o m a - This is another m alignant salivary gland tumor, which


favors m inor glands but can be seen in the parotid. The num ber one thing to know is
perineural spread. This tum or likes perineural spread.

W hen I say adenoid cystic, you say perineural spread.

Pearl: I used to think that perineurial tum or spread w ould w iden a neural foram en
(foram en ovale for exam ple). It’s still m ig h t... but it’s been m y experience that a nerve
sheath tum or (schw annom a) is m uch m ore likely to do that. L et’s ju st say for the
^ ^ purpose o f m ultiple choice that neural foram ina w idening is a schw annom a - unless
there is overw helm ing evidence to the contrary.

143
Lym phom a

Because the parotid has lymph nodes (it’s the only salivary gland that does), you can get
lym phom a in the parotid (prim ary or secondary). If you see it and it’s bilateral, you should
think Sjogrens. Sjogrens patients have a big risk (like lOOOx) o f parotid lym phom a. Like
lym phom a is elsew here in the body, the appearance is variable. You m ight see bilateral
hom ogeneous m asses. For the purposes o f the exam , ju st know ing you can get it in the
parotid (prim ary or secondary) and the relationship with Sjogrens is probably all you
need.

S jo g r e n s

A utoim m une lym phocyte-induced destruction o f the gland. “ D ry Eyes and Dry M outh.”
Typically seen in w om en in their 60s. Increased risk (like lOOOx) risk o f non-H odgkins M ALT
type lym phom a. T here is a honeycom bed ap p earance o f the gland.

B e n ig n L y m p h o e p ith e lia l D is e a s e :

You have bilateral m ixed solid and cystic lesions w ith diffusely enlarged parotid glands. This
is seen in H IV . The condition is painless (unlike parotitis - w hich can enlarge the glands).

A c u t e P a r o titis :

O bstruction o f flow o f secretions is the m ost com m on cause. T hey will likely show you a stone
(or stones) in S tensen’s duct, w hich will be dilated. The stones are calcium phosphate. Post
infectious parotitis is usually bacterial. M um ps w ould be the m ost com m on viral cause. A s a
point o f trivia, sialography is contraindicated in the acute setting.

144
Parapharyngeal Space
Also referred to as the
“pre-styloid” parapharyngeal
space - for the purpose of
fucking with you.

The parapharyngeal
The prim ary utility o f the space is primarily a
space is w hen it is ball o f fat with a few
displaced (discussed branches o f the
below ). trigeminal nerves, and
the pterygoid veins.

M ets and infections can


spread directly in a
vertical direction through
this space (squam ous cell
P a ra p h a ry n g e a l Fat (PPF) D is p la c e m e n t
cancer from tonsils,
tongue, and larynx).

A cystic m ass in this


location could be an
atypical 2nd B ranchial
Parotid
C left C yst (but is m ore Carotid
Space =
likely a necrotic lym ph Space =
Medial
Anterior
node). Displacement %
Displacement

The parapharyngeal space is


bordered on four sides by
different spaces. If you have
a m ass dead in the m iddle, it
can be challenging to tell
w here it’s com ing from .
U sing the displacem ent o f Masticator
Space = Superficial
fat, you can help problem
Posterior Mucosal
solve. M uch m ore im portant Medial Space =
than that, this lends itself Displacement Lateral
Displacement
very w ell to m ultiple choice.

Parotid M ass Pushes


M edially (PM PM )

145
Carotid Space:
The carotid space is also sometimes called the
“post styloid” or “retro-styloid’' parapharyngeal
space — for the purpose o f fucking with you.

There arc 3 Classic Carotid Space Tumors:


(1) Paraganglioma
(2) Schwannoma
(3) Ncurofibroma

Although it is worth noting


that this space is
commonly involved in
Carotid Space
secondary spread of
Contains:
aggressive multi-spatial
disease - such as " C arotid artery
infectious path ■ Jugular vein
(necrotizing otitis external) ■ Portions o f CN 9,
or malignant spread
CN 10, CN 11
(nasopharyngeal,
squamous cell etc..). ■ Internal ju g u lar
chain lym ph nodes
Metastatic squamous cell
is what you should think
for nodal disease in this Lesions Displace the Parapharyngeal Fat ANTERIOR
region.

P a r a g a n g lio m a s : T here are three different ones w orth know ing about - based on location.
The im aging features are the sam e. T hey are h yp ervascular (intense tum or blush), w ith a
“Salt and P epper” appearance on M RI from the flow voids. They can be m ultiple and
bilateral in fam ilial conditions (10% bilateral, 10% m alignant, etc.). ,n In-octreotide
accum ulates in these tum ors (receptors for som atostatin).
C arotid Body Tumor = Carotid
Bifurcation (Splaving ICA and
E C A)

G lom us Jugulare = Skull Base


(often with destruction o f jugular
fo ra m e n )
M iddle Ear Floor D estroyed =
G lom us Jugulare.

G lom us Vagale = A bove Carotid


B ifurcation, but below the Jugular
Foram en

G lom us Tym panicum =


C onfined to the m iddle ear.
B uzzw ord is “overlying the
cochlear prom ontory. ”
M iddle Ear Floor Intact =
G lom us Tym panum

146
Schw annom a S u rg ical
M ost com m only in this location w e are talking about vagal nerve P lanning Trivia:
(CN 10), but if the lesion is pretty high up near the skull base it
• Distance o f Skull
could also be involving CN 9, 11, or even 12. T he typical MR Base ( > 1cm =
appearance is an oval m ass, heterogenous (cystic and sold parts) Neck Dissection)
with heterogenous bright signal on T2. • Degree o f
Vascularity
These things enhance a ton (at least the solid parts anyw ay). They
(might need pre­
enhance so m uch you m ight even think they w ere vascular. embolization)
Ironically, schw annom as are considered hypo vascular lesions and
• Relationship to the
the only reason they enhance is because o f extravascular leakage Carotid (Don’t Fuck
(and poor venous drainage). with Big Red)

N e u ro fib r o m a

These are less com m on than the schw annom a. A bout 10% o f the tim e they are related to
NF-1 (in w hich case you should expect them to be bilateral and m ultiple). In contrast to
schw annom as they tend to be m ore hom ogenous, and dem onstrate the classic target sign on
T2 w ith decreased central signal.

N eu ro fibrom a S ch w an nom a P a rag an g lio m a


Mildly Heterogenous Although they enhance intensely Hypervascular
Enhancement they are not vascular on Angio (tumor blush on angio)
T2: Target Sign T2: Moderate to High Signal - T2: Light Bulb Bright with
(bright rim, dark middle) Heterogenous Salt and Pepper (flow voids)
NF-1 Association NF-2 Association ' "In-Octreotide avid

LI
o
N e c k In f e c t io n S y n d r o m e s

L e m ie rre ’s Syndrom e - This is a thrombophlebitis o f the jugular veins with septic emboli in
the lung. It’s found in the setting of oropharyngeal infection (pharyngitis, tonsillitis, peritonsillar
abscess) or recent ENT surgery. Buzzword bacteria = “Fusobacterium Necrophorum”

G risel’s Syndrom e - Torticollis with atlanto-axial joint inflammation seen in H&N surgery or
retropharyngeal abscess

147
Masticator Space:

As the nam e im plies this space contains the


m uscles o f m astication (m asticator,
tem poralis, m edial and lateral pterygoids).

A dditionally, you have the angle and ram us


o f the m andible, plus the inferior alveolar
nerve (branch o f V3).

A trick to be aw are o f is that the space


extends superiorly along the side o f the skull
via the tem poralis m uscle. So, aggressive
neoplasm or infection m ay ride right up there.

Lesions Displace the Parapharyngeal Fat


POSTERIOR and MEDIAL

O d o n to g e n ic In f e c t io n -
In an adult, this is the most common cause of a masticator space mass. If you see a mass here, the next
move should be to look at the mandible on bone windows. Just in general, you should be on the look
out for spread via the pterygopalatine fossa to the orbital apex and cavernous sinus. The relationship
with the mylohyoid makes for good trivia - as discussed above.

S a rc o m a s -
In kids, you can run into nasty angry masses like Rhabdomyosarcomas, You can also get sarcomas
from the bone of the mandible (chondrosarcoma favors the TMJ).

C a v e r n o u s H e m a n g io m a s - Key Point:
Congenital Stuff and
These can also occur, and are given away by the presence of
Aggressive Infection/
phlcboliths. Venous or lymphatic malformations may involve multiple
Cancer tends to be Trans-
compartments / spaces.
Spatial.

P e r in e u r a l S p r e a d - You can have perineural spread from a head and neck primary along V3.

When I say ''perineural spread” you should think two things:


( 1) Adenoid Cystic Carcinoma of the minor salivary gland
(2) Melanoma

N e r v e S h e a th T u m o rs - Since you have a nerve, you can have a schwannoma or


neurofibroma of V3. Remember the schwannoma is more likely to cause the foramina expansion vs
perineural tumor spread.

148
Retropharyngeal Space / Danger Space
The retropharyngeal space has some complex anatomy. Simplified, this is a m idline space, deep
to the oral & nasal pharynx. The retropharyngeal space has an anterior “true” space which
extends caudal to around C6-C7, and a more posterior “danger space” - which is dangerous
because it listens to rap music and plays first person shooter video games - plus it extends into
the m ediastinum - so you could potentially dum p pus, or cancer, right into the mediastinum.

Infectious behind this deep cervical fascia (the P re v e rte b ra l S p ace) are different than the ones
discussed below in that they are not spread from the neck but instead the spine/disc (osteomyelitis).

In fe c tio n - Involvement o f the retropharyngeal N e c r o t ic N o d e s (nodes o f


space most often occurs from spread from the tonsillar Rouviere) - These things are located
tissue. You are going to have centrally low density in the lateral retropharyngeal region.
tissue and stranding in the space. You should evaluate In kids you can see suppurative
for spread o f infection into the mediastinum. infection in these, but around age 4
they start to regress - so adults are
actually much less to get infection in
this region. Now, you can still get
mets (squam ous cell, papillary
thyroid, etc..). Lym phoma can
involve these nodes as well - but
w on’t be necrotic until treated.

Cartoon Axial Cartoon Axial


Showing Infection Showing the nodes
Spreading from a o f Rouviere
Tonsillar Abscess to Peritonsillar Abscess
the Danger Zone

Don 'I Forget:


Delays are often
critical for
differentiate
phlegmon and
drainable abscess
Retropharyngeal Abscess (midline) Suppurative Node o f Rouviere (lateral)

149
S E C T IO N 16:

t 1U U 1U U M P M 1P S q u a m o u s

In
C

.. _ . . „
f r a h y o id
e l l

N
• _v
C A 8c

eck
*

W hen you are talking about head and neck cancer, you are talking about squam ous cell cancer.
Now, this is a big com plex topic and requires a fellow ship to truly understand / get good at.
O bviously, the purpose o f this book is to prepare you for m ultiple choice test questions not
teach you practical radiology. If you w ant to actually learn about head and neck cancer in a
practical sense you can try and find a copy o f H a m sb erg e r’s original legendary handbook
(w hich has been out o f print for 20 years), but w ho has tim e for that ? Now, for the triv ia ....

Lymph Node Anatomy:

Testable Trivia:

A nterior Belly o f Digastric


separates 1A from 1B

Stylohyoid m uscle
(posterior subm andibular
gland) separates 1B from
2A

Jugular Vein (Spina!


A ccesso ry N erve) separates
2A from 2B *see below

Vertical borders:
2-3 = L ow er H yoid
3-4 = L ow er C ricoid

2A: Anterior, Medial, Lateral or Abutting the Posterior Internal Jugular

2B: Posterior to the Internal Jugular, with a clear fat plane between node and IJ

150
Cancers
F lo o r o f t h e M o u th S C C :

I touched on this once already. Just rem em ber sm oker/drinker in an old person. HPV in a
young person. N ecrotic level 2 nodes can be a presentation (not a branchial cleft cyst).

N a s o p h a ry n g e a l SCC:

This is m ore com m on in A sians


and has a bi-m odal distribution:
• group 1 (15-30)
typically C hinese
• group 2 (> 40).

Involvem ent o f the


parapharyngeal space results in
w orse prognosis (com pared to Fossa o f R osenm uller (arrow s)
nasal cavity or oropharynx
The FO R is the M O ST C O M M O N location for
invasion).
N asop h aryn geal C an cer

M ind M aps:

See a U nilateral
M astoid Effusion
Look
at the
FOR
See a Pathologic
R etropharyngeal N ode
“E arliest Sign ” o f nasopharyngeal SCC is
the effacem ent o f the fat w ithin the FO R.

Look at the C livus


A bout 30% o f the patients w ith n aso p h ary n g eal tum ors
See a Pathologic have skull base erosion.
R etropharyngeal or (MR1 ^ C T for skull base invasion)
S upraclavicular N ode

N odal m ets are p resen t in 90% o f


naso p h ary n g eal tu m o rs, w ith the
retro p h ary n g eal nodes u su ally the
first involved. .,,4

Normal Comparison Invasion (loss o f Normal T1


(T1 Bright Marrow Signal) Marrow Signal)

151
Neck Anatomy Blitz

Supraglottic Region: W
Arrow on the Tip of
the Epiglottis

Supraglottic Region:
Valleculae (grey
arrows),
Hypoepiglottic
Ligament (white
arrow). Epiglottis
(multiple tiny white
arrows)

Supraglottic Region:
Pre-Epiglottic Fat
(star), Aryepiglottic
folds (grey arrows),
piriform sinuses
(white arrows)

Supraglottic Region:
Para-Epiglottic
Spaces (open arrows),
Aryepiglottic folds The Para-Epiglottic communicates with the
(grey arrows), Pre-Epiglottic Space / Fat superiorly.
piriform sinuses
(white arrows)
The Pre-Epiglottic fa t is rich in lymphatics,
makes tumor invasion o f these regions
important fo r tumor staging.

Supraglottic Region: Level of the Glottic Region: Level o f the True Sub- Glottic Region: Level
False Cords (white arrows), with Cords (white arrows), with the o f the Cricoid Ring
para-glottic spaces (open arrows), cricoarytenoid joint (black
and arytenoid cartilages (black arrows), and Anterior Commissure
arrows) (open arrow)

152
larynx
L a r y n g o c e le :

When the laryngeal “saccule” dilates with air you call it a


laryngocele. If it is filled with fluid you still might call it a
larygocele (but if saccular cyst is a choice - consider that). If it
is filled with fluid and air you still might call it a larygocele
(but it laryngopyocele is a choice — and you have any hint of
infection - consider that).

What the fu ck is a “saccule ” ? The “saccule” is the


appendix o f the laryngeal ventricle (a blind ending sac that
extends anterior and upwards). It’s usually closed or
minimally filled with fluid (you don’t typically see it in
normal adults).

Why does it dilate ? Usually because it’s obstructed (ball-


valve mechanics at the neck o f the saccule), and the testable
point is that 15% of the time that obstruction is a tumor.
You can also see them in forceful blowers (trumpet players,
glass blowers) — well maybe, this depends on what you
read. Simply read the mind of the question writer to know if
they are on team trumpet player laryngocele.
version o f this (based on
containment of violation of
the thyrohyoid membrane).

V o c a l C o rd P a r a ly s is :

The involved side will have an expanded ventricle (it’s the opposite side with a cancer). If you see
it on the left, a good “next step” question would be to look at the chest (for recurrent laryngeal nerve
involvement at the AP window).

Buzzword = “H oarseness” - If you see “Hoarseness” in the question header, you need to
d k jp O think recurrent laryngeal nerve compression in the AP Window - either from a mass/node
or aortic path. *Hoarseness is also a classic Laryngeal CA buzzword (so look there too).

Vocal Cord Paralysis - Ipsilateral Expanded Ventricle A P Window P E T Avid Node


153
laryngeal Cancer
( Nearly Always ~ 85% Squamous C e ll)
Risk Factors: Smoking, Alcohol, Radiation, Laryngeal Keratosis, HPV, GERD, & Blasphemy against the
correct religion (false religions are safe to talk shit about).
The role of the Radiologist is not to make the primary cancer diagnosis here, but to assist in staging.
Laryngeal cancers are subdivided into (a) supraglottic, (b) glottic, and (c) subglottic types.
“Transglottic” would refer to an aggressive cancer that crosses the laryngeal ventricle.

Epiglottic mass
Supra-G lottic:
spreading anterior
E piglottic C entered
Supra-G lottic across the hypo
-Anterior epiglottic ligament
-M ore A ggressive -Likes to Invade the Pre- into the preepiglottic
-E arly Lymph Epigolttic Space /Fat (which space and into the
is rich in lymphatics paired vallecula.
N ode M ets
-They d o n ’t get
False Cord mass
hoarseness
Supra-G lottic: spreading into the
False Cord / Fold para-glottic space
You always need tofind C entered Paraglottic space
(guess at) the inferior margin
of a supraglottic mass. involvement makes the
-Posterior Lateral tumor T3 and
Partial laryngectomy usually -Likes to Invade the Para- "transglottic: ”
can only be done if the tumor Glottic Space /Fat (which *Best seen in coronals.
is restricted to the communicates superiorly
supraglottis and does not with the Pre-Epiglottic
involve the arytenoids or Aryepiglottic fold
Space mass spreading into
laryngeal ventricle.
the paraglottic space
& piriform sinus
Usually involve the
Glottic mass spreading
G lottic anterior cord and
spread into the forward towards the
-M ost C om m on anterior commissure anterior commissure
(typically defined as Fixation o f the cords
-B est O utcom e
soft tissue thickening indicates at least a T3
-G row Slow ly o f > 2mm) tumor - this is best
-M etastatic D isease assessed with a scope
Classic Clinical History: but can be suspected
is Late
“Progressive and
with disease in the
Continuous Hoarseness. ”
cricoarytenoid joint.

Sub-G lottic
The only reliable sign o f
-Least Common The typical look is cricoid invasion is
soft tissue thickening tumor on both sides of
-Often small between the airway
compared to nodal the cartilage (irregular
and the cricoid ring. sclerotic cartilage can
burden
be normal).
-Bilateral nodal
disease & Invasion o f the cricoid cartilage is a contraindication to all types o f laryngeal
mediastinal conservation surgery (cricoid cartilage is necessary for postoperative stability
extension o f the vocal cords).

154
S E C T IO N 17:

jM d . O r b i t ...

Tumors and Tumor like Conditions:


R e tin o b la s to m a - This is the m ost com m on prim ary m alignancy o f the globe. If you see
calcification in the globe o f a child - this is the answer.

The step 1 question is RB suppressor gene (chrom osom e 13 — “unlucky 13”). T h at’s the sam e
chrom osom e osteosarcom a patients have issues with and why these guys are at increased risk o f
facial osteosarcom a after radiation.
The globe should be norm al in size (or bigger), w here C oats’ is usually smaller. It’s usually seen
before age 3 (rare after age 6). The trivia is gonna be w here else it occurs. They can be bilateral
(both eyes - 30% ), trilateral (both eyes and the pineal gland), and quadrilateral (both eyes,
pineal, and suprasellar).

C o a ts ’ D is e a s e - T he cause o f this is retinal telangiectasia w hich results


CT - Dense
in leaky blood and subretinal exudate. It can lead to retinal detachm ent. It’s T1 - Hyper
seen in young boys and typically unilateral. T he key detail is that it is N O T T2 - Hyper
C A LC IFIED (retinoblastom a is).

Coats disease has a sm a ller globe. R etinoblastom a has a norm al size d globe.

P e r s is te n t H y p e r p la s t ic P r im a r y V it r e o u s (P H P V ) -

This is a failure o f the em bryonic ocular blood supply to regress. It can lead to retinal
detachm ent. The classic look is a sm all eye (m icrop h th alm ia) with increased density o f
the vitreous. No calcification.

Retinal D etachm ent - This can occur


secondary to PHPV or Coats. It can also be
caused by trauma, sickle cell, or just old age.
The imaging finding is a “V” or “Y” shaped
appearance due to lifted up retinal leaves and
subretinal fluid.
Retinal Detachment in the setting of PHPV

Globe Size Comparison


- A Strategy’for Eliminating Distractors

R etinoblastom a - N orm al Size * P H P V - Sm all Size (N orm al B irth A ge)


Toxocariasis - N orm al Size * R etinopathy o f P rem aturity - B ilateral Sm all
C o a ts’ - Sm aller Size

155
M e la n o m a - This is the m ost com m on intra-occular lesion in an adult. If you see an
enhancing soft tissue m ass in the back o f an ad u lt’s eye this is the answer.

Here are 4 w ays you could ask a question about this:


(1) show a picture - w hat is it?,
(2) ask w hat the m ost com m on intra-occular lesion in an adult is?
(3) ask the buzzw ord "collar button s h a p e d ’’ ? - which is related to
B ruch’s m em brane,
(4) strong predilection for liver m ets - next step Liver MR.

O p tic N e r v e G lio m a : These alm ost alw ays (90% )


occur under the age o f 20. You see expansion / enlargem ent
o f the entire nerve. If they are bilateral you think about
N F -1. They are m ost often W H O grade 1 P ilocytic
Astrocytom as. If they are sporadic they can be G B M s and
absolutely destroy you.

O p tic N e r v e S h e a th M e n in g io m a : T he buzzw ord


is "tra m -tra c k ” calcifications. A nother buzzw ord is
“doughnut” appearance, w ith circum ferential en h an cem en t
around the optic nerve. “Tram-Track” = Meningioma

D e rm o id : This is the m ost com m on benign


congenital orbital m ass.

It’s usually superior and lateral, arising from the


frontozygom atic suture, and presenting in the first
10 years o f life. It’s gonna have fat in it (like any
good derm oid). T he location is classic.

Orbital Dermoid - Classic Location

R h a b d o m y o s a rc o m a - M ost com m on extra-occular orbital m alignancy in children


(derm oid is m ost com m on benign orbital m ass in child). Favors the superior-m edial orbit and
classically has bone destruction. Just think “ bulky orbital mass in a 7 year old.”

W hen they do occur - 40% o f the tim e it’s in the head and neck - and then m ost com m only it’s in
the orbit. It’s still rare as hell.

L y m p h o m a - There is an association with C hlam ydia Psittaci (the bird fever thing) and
MALT lym phom a o f the orbit. It usually involves the upper outer orbit - closely associated with
the lacrimal gland. It will enhance hom ogeneously and restricts diffusion - ju st like in the brain.

156
M e t a s t a tic N e u ro b la s to m a - This has a very classic appearance of “Raccoon Eyes” on
physical exam.
The classic location is periorbital tumor infiltration
with associated proptosis. Don’t forget a basilar skull
fracture can also cause Raccoon Eyes... so clinical
correlation is advised. Neuroblastoma mets tend to be
more SI * ' *>

Another thing worth mentioning is the bony


involvement o f the greater wing o f the sphenoid. * i
Neuroblastoma is gonna be bilateral. Ewings favors
this location also - but will be unilateral.

M e t a s t a tic S c irrh o u s (fib ro s in g )


B re a s t C a n c e r -
This is classic gamesmanship here. The important point to
know is that unlike primary orbital tumors that are going to
cause proptosis, classically the breast cancer met causes a
desmoplastic reaction and enophthalmos (posterior
displacement o f the globe).

Trivia: Mets are actually more common to the eye Infiltrative retrobulbar mass +
relative to the orbit (like 8x more common). enophthalmos = scirrhous carcinoma
of the breast

■gG4- Orbit
O rbital Pseudotum or: Tolosa H unt Syndrom e: Lym pho cytic
H ypophysitis:
This is one o f those IgG4 This is histologically the same
idiopathic inflammatory thing as orbital pseudotumor This is the same deal as
conditions that involves the but instead involves the orbital pseudotumor and
extraoccular muscles. It looks like cavernous sinus. Tolosa Hunt, except it’s the
an expanded muscle. The things pituitary gland. Just think
to remember are that this thing is It is painful (just like enlarged pituitary stalk in a
painful, unilateral, it most pseudotumor), and presents postpartum / 3 rd trimester
commonly involves the lateral with multiple cranial nerve woman. It looks like a
rectus and it does NOT spare palsies. It responds to steroids pituitary adenoma, but it
the myotendinous insertions. (just like pseudotumor). classically has a T2 dark rim.
Remember that Graves does not
cause pain, and does spare the
myotendinous insertions. It gets
better with steroids. It’s
classically T2 dark.

T1+C

157
T h y ro id O rb ito p a th y : This is seen in
1/4th o f the Graves cases and is the most
common cause o f exophthalmos. The
antibodies that activate TSH receptors also
activate orbital fibroblasts and adipocytes.

Things to know:
Risk o f com pressive optic
neuropathy
• Enlargement o f ONLY M USCLE
BELLY (spares tendon) - different
than pseudo tumor
N OT Painful - different than pseudo Thyroid Orbit - Spares Tendon Insertion
tum or
Order o f Involvement:
IR > MR > SR > LR > SO /IO
Inferior, Middle, Superior, Lateral, Oblique

Orbital Vascular Malformations


Lym phangiom a V a rix C arotid-C avernou s Fistula

These arc actually a mix of These occur These come in two flavors:
venous and lymphatic secondary to (1) Direct - which is secondary to
malfonnations. They arc ill- weakness in the trauma, and (2) Indirect - which just
defined and lack a capsule. The post-capillary occurs randomly in post menopausal
usual distribution is infiltrative venous wall (gives women.
(multi-spatial), involving, pre- you massive
The direct kind is a communication
septal, post-septal, extraconal, dilation of the
between the intracavemous ICA and
and intraconal locations. valvclcss orbital
cavernous sinus. The indirect kind is
Fluid-Fluid levels are the most veins).
usually a dural shunt between meningeal
classic finding , with regard to branches o f the ECA and the Cavernous
multiple choice. Most likely question
Sinus.
is going to pertain
Do NOT distend with to the fact that they Buzzword: Pulsatile Exophthalmos
provocative maneuvers distend with *although this can also be a buzzword
(valsalva). provocative fo r NF-1 in the setting o f sphenoid wing
maneuvers dysplasia.
(valsalva, hanging
Prominent left
head, etc...).
superior ophthalmic
vein with proptosis
Another piece of
trivia is that they are
the most common
cause o f
spontaneous orbital
hemorrhage. They
can thrombose and
present with pain.
Prominent left
Fluid-Fluid Levels cavernous sinus

158
Orbital Infection

Orbital A n a t o m y Review
Lacrimal Sac

Lacrimal Intraconal
Gland Space - is the
Fossa space inside the
rectus muscle
pyramid.
Extraconal
Space - is the
space outside
the rectus
muscle pyramid.
Differentiation between Orbital (post-septal) and
Periorbital (pre-septal) cellulitis is based on the Intraconal Space
Extraconal Space
relationship to the orbital septum (arrows).

P re -S e p ta l I P o s t-S e p ta l C e llu litis -


As above, location of orbital infections are described by their relationship to the orbital septum. The
testable trivia is probably (1) that the orbital septum originates from the periosteum o f the orbit and
inserts in the palpebral tissue along the tarsal plate, (2) that pre-septal infections usually start in
adjacent structures (likely teeth and the fac e ), (3) post-septal infections are usually from paranasal
sinusitis, and (4) pre-septal infections are treated medically, post septal is surgical.

Trivia: Periorbital abscess can cause thrombosis o f the ophthalmic veins or cavernous sinus (in
extreme examples infection — usually aspergillosis — can even cause a cavemous-carotid fistula).

D a c ry o c y s titis -

This is inflammation and dilation of the


lacrimal sac. It has an Aunt Minnie
look, with a well circumscribed, round
rim enhancing lesion centered in the
lacrimal fossa. The etiology is typically
obstruction followed by bacterial
infection (staph and strep).

Usually this is diagnosed clinically unless there is an associated peri-orbital cellulitis in which can
CT is needed to exclude post septal infection (treated surgically) from simple dacryocystitis
(treated non-surgically).

O rb ita l S u b p e rio s te a l A b s c e s s :

If you get inflammation under the periosteum


it can progress to abscess formation.

This is usually associated with ethmoid


sinusitis. This also has a very classic look.

159
Misc Orbital Conditions

O p tic N e u ritis :

There will be enhancem ent o f the optic


nerve, without enlargement o f the nerve/
sheath complex. Usually (70%) unilateral,
and painful.

You will often see intracranial or spinal cord


dem yelination - in the setting o f Devics
(neurom yelitis optica). 50% o f patient’s with
acute optic neuritis will develop MS.

If the optic nerve is enlarged, think gliom a... then think NF-1.

P a p ille d e m a :

This is really an eye exam thing.


Having said that you can sometimes see dilation/ swelling o f the optic nerve sheath.

JL

In tra o c u la r Lens
E cto p ia L e n tis (lens
Drusen - Mineralization at the Im p la n t -
optic disc. Supposedly there is an d is lo c a tio n ) - Causes include
The standard treatment for
association with age-related Trauma, Marfans, and
cataracts. A replaced lens has
maculopathy Homocystinuria.
a thin linear appearance.

C o lo b o m a :

This is a focal discontinuity o f the globe


(failure o f the choroid fissure to close).
They are usually posterior. If you see a
unilateral one - think sporadic.

If you see bilateral ones - think C H A R G E


(colobom a, heart, G U , ears). Bilateral Coloboma - C H A R G E Syndrome

160
^ S E C T IO N 18: M
S p in e fP

Anatomy Trivia
Cord Blood Supply: There is an anterior blood supply and a posterior blood supply to the cord.
These guys get taken out with different clinical syndromes.

Anterior spinaI artery>- arises bilaterally as two small branches at the level o f the term ination o f
the vertebral arteries. These two arteries join around the level o f the foramen magnum.

Artery o f Adam kiewicz - This is the most notable Anterior


reinforcer o f the anterior spinal artery. In 75%
o f people it comes off the left side of the aorta
between T8 and T i l . It supplies the lower 2/3
o f the cord. This thing can get covered with the
placement o f an endovascular stent graft for
aneurysm or dissection repair leading to spine
infarct.

Posterior Spinal Artery - Paired arteries which


arise either from the vertebral arteries or the
posterior inferior cerebellar artery. Unlike t h e s *
anterior spinal artery this one is somewhat *
discontinuous and reinforced by m ultiple.
\ Pnstprinr
Posterior
segmental or radiculopial branches.

Conus Medullaris: This is the terminal end o f the spinal cord. It usually terminates at around L I .
Below the inferior endplate o f the L2 / L3 body should make you think tethered cord (especially if
shown in a multiple choice setting).

Epidural Fat: The epidural fat is not evenly distributed. The epidural space in the cervical cord is
predominantly filled with venous plexus (as opposed to fat). In the lumbar spine there is fat both
anterior and posterior to the cord. "Epidural Lipomatosis ” = is a hypertrophy o f this fat that only
occurs with patients on steroids (“on corticosteroids” would be a huge clue).

A B
S te n o s is : Spinal stenosis can be congenital (associated w ith short
pedicles) or be acquired. The T org-Pavlov ratio can be used to call it
(cervical canal diam eter to vertebral body w idth < 0.85).
Sym ptom atic stenosis is m ore com m on in the cervical spine (versus
m
the thoracic spine or lum bar spine). You can get som e congenital
stenosis in the lum bar spine from short pedicles, but it’s generally not
sym ptom atic until m iddle age. Sagittal View
B/A < 0.85

161
Degenerative Changes
W hat is this degenerative change you speak o f ? The best w ay to understand this is through
two m echanism s; Spondylosis D eform ans and Intervertebral O steochondrosis.

Spondylosis D eform ans In te rv e rte b ra l O steochondrosis

This is
probably part Pathologic (but
of “normal not necessarily
aging” symptomatic)

“Degenerative Change” or spine


“ Deteriorated Disc” - This process is more
osteoarthritis. This is more rim / margin
centered in the disc space favoring the nucleus
centered. Process is characterized by
pulposus & vertebral body endplates.
osteophyte formation.

T H IS vs THAT: O s teo p h ytes vs S yndesm ophytes

Osteophytes: Syndesmophytes:
• More horizontal / • More vertical
oblique with a symmetric, and
“claw” like thinner
appearance. • Represent
• Formed also the ossification o f the
vertebral margin. annulus fibrosis.
. Seen in “DJD” / • Seen in Ankylosing
Spondylosis Spondylitis.

E n d p la te C h a n g e s : Commonly referred to as “M odic C hanges.”


There is a progression in the MRI signal characteristics
that m akes sense if you think about it. You start out with Type 1 T1 Dark,
degenerative changes causing irritation / inflam m ation “E dem a” T2 B right
so there is edem a (T2 bright). This progresses to chronic
inflam m ation which leads to som e fatty change - ju st T1 Bright,
Type 2 “ Fat”
like in the bowel o f an IBD patient - causing T1 bright T2 B right
signal. Finally, the whole thing gets burned out and Type 3
fibrotic and it’s T1 and T2 dark. As a prom inent factoid, T1 Dark, T2 D ark
“ Scar”
Type 1 changes look a lot like O steom yelitis (clinical
correlation is recom m ended).
Type Type 2 Type 3
A n n u la r F is s u re :
As the disc ages, it tends to dry out making it more friable and easily tom. “Tears” in the annulus
(which are present in pretty much every degenerated disc) aren’t called “tears” but instead “fissures".
People who write the papers on this stuff make a big fucking deal about that - with the idea being that
“tear” implies pathology. Fissuring can be asymptomatic and part of the aging process.

Even though fissures are present


in basically every degenerated
disc you don’t always see them
on MRI. What you do see (some
o f the time) is a fluid signal gap
in the annulus - which has been
given the official vocabulary
word “High In te n s ity
Z one,” and anything with
official vocabulary nomenclature
should be respected as possible
multiple choice fodder.

•Annular fissures may be a source o f pain


(radial pain fibers - trigger “discogenic pain”) but are also seen as incidentals.
»Fissures are found in all degenerative discs but are not all fissures are visualized as HIZs.
JL v »Discography is more sensitive to fissures relative to MRI. but still not 100% sensitive.
M fQ *Also, Dude, “Tear” is not the preferred nomenclature - “Fissure.”

S c h m o rl N ode: S c h e u e r m a n n ’s L im b u s
Intravertebral Herniation
This is multiple levels V e rte b ra
This is a herniation of disc material (at least 3) o f wedged vertebral
through a defect in the vertebral bodies with associated This is a fracture
body endplate into the actual Schmorl’s nodes — mimic that is the
marrow.
result of herniated
Common - like 75% o f people have disc material
them. between the non­
Classic look is to favor the inferior fused apophysis and
endplate of the lower thoracic / adjacent vertebral
upper lumbar spine. When they are body.
acute they can have edema on T2
and be dark on T l - mimicking
osteomyelitis. Chronic versions will
have a sclerotic rim.
Most classically the thoracic
spine o f a teenager, resulting in
kyphotic deformity (40 degrees
in thoracic or 30 degrees in
thoracolumbar).

25% o f patients have scoliosis.

163
Disc Nomenclature:
In order to “ im prove accuracy” in the description o f lum bar spine disc disease, a handful o f
elites gathered at an unknow n location, ate caviar, drank w ine, and then m ade a sacrifice to
M oloch the Ow l G od - after w hich they issued a proclam ation on w hat vocabulary w ords you
are and are not allow ed to use w hen describing degenerative disc herniation.

H ern ia tio n : Bulge:

This is the approved


This is the approved verbiage for the verbiage for the
displacement of LESS THAN 25% of displacement of M ORE
disc material beyond the limits o f the THAN 25% o f disc
disc space. material beyond the limits
o f the disc space.
Protrusion (subtype Extrusion Sequestration Asymmetric Bulge
of herniation) (subtype of Free (broken off)
herniation) disc fragment.

Term used when the Term used when Symmetric Bulge


distance between the the edges o f the
edge o f the disc disc are greater
herniation is less than than the distance of
the distance between the base
the edges o f the base (neck narrower
(base wider than than herniation).
herniation).

~y Disc Level
3 “ Suprapedicle Level
L o c aliza tio n
C ranial -Pedicle Level
C audal P lane - Infrapedicle Level

Extra Foram inal

Foram inal
L o c aliza tio n *Often symptomatic
A x ial P lane because o f the relationship
to the Dorsal Root Ganglia

Sub A rticular
*Most Common Location C entral

164
Which Nerve is CompressedP
There are 31 pairs o f spinal nerves, w ith each pair corresponding to the adjacent
vertebra - the notable exception being the “C 8” nerve. C ervical disc herniations are
less com m on than lum bar ones.

The question is m ost likely to take place in the lum bar spine (the sam e spot m ost disc
herniations occur). In fact m ore than 90% o f herniations occur at L 4-L 5, and L 5-S 1.

A tale o f two herniations. It w as the best o f times, it w as the w orst o f tim es...

Scenario I:

Scenario 2:

A Central or Sub-
L5 Nerve Articular Disc Will
Sm ash the
Descending Nerve.

In this case the S1


Nerve

S1 Nerve

165
LP / Myelogram Technique
Prior to the LP
Absolute Contraindications:
• Increased intracranial pressure or obstructed CSF flow (A CR-ASN R recommendations)
• Bleeding diathesis (hvpocoagulabilitv) STOP Coum adin 4-5 days
• M yelogram Specific — Iodinated contrast allergy STOP Plavix for 7 days
Hold LM W Heparin for 12 hours
Relative Contraindications (vary p er institution): Hold H eparin for 2-4 hours -
• O verlying infection, hem atom a, or scarring docum ent norm al PTT
• M yelogram Specific - R ecent m yelogram (< 1 week) Aspirin and N SA ID s are fine (not
• M yelogram Specific - H istory o f seizures contraindicated)

Legit Indications for Fluoro Guided LP:


• Advanced degenerative spondylosis,
• Post-surgical changes,
• Patient is so fat (“a person of size”), when Dracula sucked his/her blood, he got diabetes.
• Myelogram Specific - MRI contraindication
• Myelogram Specific - Geriatric Professor Emeritus of Neurosurgery wants it, and it is better than
doing the pneumoencephalogram he originally ordered.

NOT Legit Indications for Fluoro Guided LP — all o f which I ’ve heard:
• “The patient is crazy”
• “The patient is crazy & violent”
• “The patient is crazy, violent, and has high viral load HIV.... and Hep C”
• “The patient recently escaped a locked mental institution for the extremely violent and criminally
insane, has both HIV and Hep C. He spits like a camel and has really terrible body odor.”

L2-L3 or L3-L4 are common Technical Overview


entry points. A potential trick Target the The needle will
would be to show you imaging
with a low lying conus (usually
that thing stops at L1-L2).
Remember you need to be
inlerlaminar
space, just off
o f midline.
Always
\ naturally steer toward
the sharper side and
away from the bevel.
So, if you are directing
. the needle, you’ll want
below the conus - so you might aspirate the bevel side opposite
need to adjust down, before you the direction you are
depending on how low it is. inject attempting to steer.
anything.

Myelogram Specific - Contrast should flow freely away from the needle tip, gradually filling the thecal
sac. The outlining o f the cauda equina is another promising sign that you did it right. If contrast pools at
the needle tip or along the posterior or lateral thecal sac without free-flow, a subdural injection or
injection in the fat around the thecal sac should be suspected.

Technical Strategies to Reduce the Incidence of Post Dural Puncture Headache (PDPH):

' Use a sm all needle (25 G), especially for epidural pain injections or myelography. You m ight have
to use a 22G for a diagnostic LP or you are going to struggle to get enough fluid for a sam ple, and
your opening pressures m ay not be accurate.
1 N on-cutting “atraum atic” needle (diam ond shaped tip) reduce incidence o f PDPH
Replace the stylet before you w ithdraw the needle. This isn ’t just for the 1 in a million chance that
you suck a nerve root up in the needle. This has also been show n to reduce incidence o f PDPH
Direction o f the bevel: This actually matters
You want to ran the Perpendicular is wrong. You are
bevel parallel with the
fibers to push them
i
I
going to cut those fibers. Coming in
at a crazy sideways angle is also not Ill
apart...not cut them. i ideal (same reason).
■7

166
Blood Patch
Even a miniscule defect within the thecal sac post LP . Most PDPHs start 24 hours after the
can allow leakage of spinal fluid resulting in puncture (between 24-48 hours) - larger
intracranial hypotension and the dreaded chronic/ leaks can present earlier.
debilitating post dural puncture_headache. . Most people will wait 72 hours after the
headache begins (“conservative
Classic PDPHs are bilateral, better laying down, and therapy”) prior to attempting the patch.
worse sitting up. They are also worse with coughing, . Most people will try at least twice before
sneezing, or straining to push out a large turd (from calling neurosurgery to sew to hole you
chronic opioid abuse). carved out of the dura (you fucking
psycho)
The procedure involves injecting between 3-20cc of the • Severe atypical symptoms should
patients own blood into the epidural space near the prompt a CT (to exclude a subdural from
original puncture site with the hope of sealing the hole. severe hypotension).

“F a ile d B a c k S u r g e r y S y n d r o m e ” (F B S S )
Another entity invented by NEJM to take down the surgical subspecialties. Per the NEJM these
greedy surgeons generally go from a non-indicated spine surgery, to a non-indicated leg amputation,
to a non-indicated tonsillectomy on an innocent child.

Text books will define it as recurrent or residual low back pain in the patient after disk surgery. This
occurs about 40% of the time (probably more), since most back surgery is not indicated and done on
inappropriate candidates. Causes o f FBSS are grouped into early and late for the purpose o f multiple
choice test question writing:

C o m p licatio n s o f S pine S u rg ery


Recurrent Residual Disk Will lack enhancement (unlike a scar - which will enhance on delays)
Epidural Fibrosis Scar, that is usually posterior, and enhances homogeneously
Buzzwords are “clumped nerve roots ” and “empty thecal sac ”,
Arachnoiditis Enhancement for 6 weeks post op is considered normal. After 6 weeks
may be infectious or inflammatory.
12,000 Square Foot As spine surgeons perform more and more unnecessary surgeries they
Mansion Syndrome need something to spend all that money on.

TH IS v.s THAT: S c a r v.v R e sid u a l D isc:

Tl Pre Contrast they will look the same... like a bunch o f mushy crap.
Tl Post Contrast the disc will still look like mushy crap, but the scar will enhance.

C o n jo in e d N e r v e R o o ts :

Two adjacent nerve roots sharing an enlarged common sleeve - at a point during their exit from the
thecal sac. This can be a source of FBSS if it is the source of pain instead of a disc. Alternatively it
could be misidentified as a disc preoperatively. In both cases, the Radiologist will be cast in the roll
o f “Scapegoat” during the malpractice suit.

167
a „ S E C T IO N 19:
%g S e a tb e lts a re fo r P ussies mF

O d o n to id F r a c t u r e J e f fe r s o n F r a c tu r e :
C la s s ific a tio n : This is an axial loading injury (jumping into a
shallow pool) - with the blow typically to the
Type 3 top o f the head.
Type 2
Type 1 (best
(rare) (mos prognosis for
v ' comm on) . >• x The anterior and
healing) posterior arches blow
out laterally.

X • About 30% will also have a C2 Fracture


• Neurologic (cord) damage is rare, because
all the force is directed into the bones.
Could be shown on a plain
Fracture at
upper part o f
Fracture at Fracture
film open mouth odontoid
view.
tlilr
Odontoid Remember the C 1 lateral
(related to the base through dens
masses shouldn’t slide off
avulsion o f the (high non­ into the body Normal Comparison
laterally.
alar ligament) union rate) o fC 2 .

M ay be Increased distance
Unstable Unstable between the lateral
Stable
masses o f C l and
odontoid peg

O s O d o n to id e u m I O s T e r m in a le :
These variants can mimic a type 1 Odontoid fracture. In both cases, you have an ossicle located at the
position of the odontoid tip (the orthotopic position). The primary difference is that with an
Os Odontoideum the base of the dens is usually hypoplastic.
• Prone to subluxation and instability.
• Associated with M orquio’s syndrome.
• Orthotopic is the position on top of the dens.
• Dystopic is when it’s fused to the clivus.
Normal Os Odontoideum
(hypoplastic dens)

H a n g m a n ’s F r a c tu r e :
Seen most commonly when the chin hits the dashboard in an MVA
(“direct blow to the face”). The fracture is through the bilateral pars at
C2 (or the pedicles - which is less likely). You will have anterior
subluxation of C2 on C3 (> 2mm). Cord damage is actually uncommon
with these, as the acquired pars defect allows for canal widening. There is
often an associated fracture of the anterior inferior comer at C2 - from
avulsion of the anterior longitudinal ligament. T raction is
contraindicated.

168
F le x io n T e a r d r o p : E x te n s io n T e a r d r o p :
This represents a teardrop shaped fracture A nother anterior inferior teardrop
fragment at the anterior-inferior vertebral body. shaped fragm ent with avulsion o f the
Flexion injury is bad because it is associated anterior longitudinal ligament. This is
with anterior cord syndrome (85% o f patients less serious than the flexion type.
have deficits). This is an unstable fracture,
associated with posterior subluxation o f the
vertebral body.

Flexion Teardrop E xtension Teardrop


Impaction Injury Distraction Injury
Extremelv Unstable Stable in flexion (unstable in extension)
Hyperflexion Hyperextension
Classic History: “Ran into w all” Classic History “H itfrom behind”

C la y -S h o v e le r ’s F r a c tu r e : This is an avulsion injury o f a


low er cervical / upper thoracic spinous process (usually C7). It is
the result o f a forceful hyperflexion m ovem ent (like shoveling).

The “ ghost sign” describes a double spinous process at C 6-C 7 on


*
AP radiograph.

C h a n c e F r a c tu r e :
These are flexion-distraction fractures that are
classically associated w ith a lap-band seatbelt.
Flexion
There are 3 colum n (unstable) fractures. D istraction
M ost com m only seen at the upper lum bar levels &
thoracolum bar junction.
High association w ith solid organ traum a.

169
F a c e t D is lo c a tio n : This is a spectrum : S ubluxed facets -> Perched -> Locked.

U nilateral: If you have unilateral locked facet (usually from hyperflexion and rotation) the
superior facet slides over the inferior facet and gets locked. The unilateral is a stable injury.
You will have the inverted ham burger sign on axial im aging on the dislocated side.

Bilateral: This is the result o f severe hyperflexion


You are going to have disruption o f the posterior Inverted
ligam ent com plex. W hen this is full on, you are Hamburger
going to have the dislocated vertebra displaced Sign
*
forw ard one - h a l f the A P diam eter o f the vertebral <------
body. This is highly unstable, and strongly Normal
Bun on Bun
associated w ith cord injury.

A t la n t o a x ia l In s t a b ilit y :

The articulation betw een C l and C2 allow s for lateral m ovem ent (shaking your head no). The
transverse cruciform ligam ent straps the dens to the anterior arch o f C l . The distance betw een
the anterior arch and dens sh o u ld n ’t be m ore than 5 m m. The thing to know is the association
with Down syndrom e and ju ven ile RA.

Rotary subluxation can occur in children w ithout a


fracture, w ith the kid stuck in a “cock-robin”
position - w hich looks like torticollis. A ctually
differentiating from torticollis is difficult and m ay
require dynam ic m aneuvers on the scanner.

This never, ever, ever happens in the absence o f a


fracture in an adult (w ho d o e sn ’t have D ow ns or
RA). H aving said that, people over call this all the
tim e in adults w ho have their heads turned in the
scanner.

P a rs In t e r a r t ic u la r is D e f e c t (S p o n d y lo ly s is o r A d u lt Is th m ic
S p o n d y lo lis th e s is ) : T his is also discussed in the Peds chapter (vol 1, page 137).
I’ll m ake a few com m ents for the adult version. D efects in the pars interarticuaris are
usually caused by repetitive m icro-traum a (related to hyper-extension). It is nearly
alw ays at L5-S1 (90% ). Pain is typically a L5 radiculopathy caused by foram inal
stenosis at L 5 -S 1. The term “pseudo-disc” is som etim es used to describe the
deform ed annular fibers seen in the setting o f a related anterolithesis (forw ard
slippage).

170
Instability
You will read different definitions of “instability” as it relates to spinal trauma. The one I prefer
is something along the lines o f “lost capacity to withstand even a normal physiologic load
without: potential damage to the spinal cord, nerve roots, or developing an incapacitating
deformity that forces one to seek employment in a cathedral bell tower. ”

For the purpose of multiple choice you will see the words “stable ” or “unstable ” associated with
specific fracture types. There are also some radiologic “definitions” of instability which seem to
vary depending on who you ask. In general, if you have acute segmental kyphosis greater than 11
degrees, acute anterolisthesis greater than 3-4 mm, or gross motion on flexion / extension imaging
it is probably an unstable fracture. You will also hear people talk about a “power ratio” for
occipitocervical instability, and a spinal column theory for the thoracolumbar injury.

O c c ip ito c e rv ic a l In s ta b ility D enis 3 S pinal C olum n C o n c e p t

This can be traumatic (in which case Most often you will see this idea applied to
the patient rarely lives because they rip thoracolumbar spinal fractures, although
their brainstem in half), or congenital technically it has some validity in the lower
(classically seen with Down cervical segments as well.
Syndrome). Two popular methods for The idea is to divide the vertebral column into 3
evaluating this: vertical parallel columns , with instability
suggested when all 3 or 2 contiguous columns
Powers Ratio (anterior and middle column or middle and
= C-D: A-B posterior column) arc disrupted.
Ratio is greater
than 1.0 =
Ligamentous A nterior:
Instability • Anterior
Longitudinal
Ligament
• Anterior 2/3
Vertebral Body
H arris Lines
Rule o f 12 M iddle:
< 12mm • Posterior
Both the Longitudinal
Basion-Dens Ligament
< 12mm (A) and • Posterior 1/3
Basion- Vertebral Body
Posterior Axial
Posterior:
Distance (B) • Posterior
should be less Ligaments
than 12 mm. A M • Pedicles, Facets,
Lamina, Spinous
Process

The management does typically change with unstable fractures typically stabilized (either internal
fusion or external bracing/reduction).

171
When Does a “Trauma” Indicate Imaging P

C an adian C-Spine Rule: N exus C riteria:


• Age >65 years • Focal neurologic deficit
• Paresthesias in extrem ities • M idline spinal tenderness
• D angerous m echanism : • A ltered level o f consciousness
• Fall >3 ft or 5 stairs • Intoxication (you c a n ’t clear a drunk
• Axial load to the head (em pty guys/girls c-spine w hile they are
sw im m ing pool diving, piano fell on drunk).
head - w hile chasing a road runner) • D istracting injury
• High Speed MVA,
• Pedestrian vs C ar
• Hulk Sm ash

T H IS us THAT: S ta b ility

U n s ta b le S ta b le

Vertebral Overriding > 3mm (“Subluxation”)


Angulation > 11 Degrees

Flexion Tear Drop | Extension Tear Drop (At least in Flexion)

Bilateral Facet Dislocation “Double-Locked” Unilateral Facet Dislocation


Odontoid Fracture Type 2 and 3 Odontoid Fracture Type 1
(most sources will say Type 2 & 3 or deploy the word I (usually stable - flex/extension film s still usually done
“usually”, but fo r sure if there is lateral displacement) \ to exclude atlantooccipital instability )

Two Contiguous Thoracolumbar Columns


(anterior & middle or middle & posterior)
| Isolated Single Thoracolumbar Column Fracture
Three Thoracolumbar Columns
(Chance Fracture, Etc...)

Jefferson Fracture Clay Shoveler's Fracture


Flangman Fracture Transverse Process Fracture

Atlanto-Occipital and Atlanto-Axial Dislocations

N am ed Spine F ra c tu res
Jefferson Burst Fracture o f Cl Axial Loading
Hangman Bilateral Pedicle or Pars Fracture of C2 Hyperextension
Teardrop Can be flexion or extension Flexion (more common)
Clay-Shoveler’s Avulsion o f spinous process at C7 or T1 Hyperflexion
Chance Horizontal Fracture through thoracolumbar spine “Seatbelt”

172
Trauma to the Cord:
There is a know n correlation betw een spinal cord edem a length and outcom e. H aving said
that, you need to know the m ost im portant factor for ou tcom e is the presence o f a
hem orrhagic spinal cord injury (these do very very badly).

S p in a l C ord S y n d ro m e s

U pper Extrem ity


O ld lady w ith
D eficit is w orse than
spondylosis or young
C entral Cord low er (corticospinal
person w ith bad
tracts are lateral in
extension injury.
low er extrem ity)

A nterior C ord Flexion Injury Im m ediate Paralysis

R otation injury or O ne side m otor, other


Brown Sequard
penetrating traum a side sensory deficits

U ncom m on - but
Posterior C ord som etim es seen w ith P roprioception gone
hyperextension

Anterior Cord Syndrome (The Really Bad One):

The anterior portion of the cord is jacked. Motor function and anterior column sensations (pain and
temperature) are history. The dorsal column sensations (proprioception and vibration) are still intact.

This is the reason FLEXION injuries are so bad.

173
IM IM M IM EM M M a S E C T IO N 2 0 .
V a s c u l a r

A VFs I A VM s:

There are 4 types. Type 1 is by far the m ost com m on (85% ). It is a D ural AVF; the result
o f a fistula betw een the dorsal radiculom edullary arteries and radiculom edullary vein /
coronal sinus - w ith the dural nerve sleeve. It is acquired and seen in older patients who
present w ith progressive radiculom yelopathy. The m ost com m on location is the thoracic
spine. If anyone asks, the “gold standard for diagnosis is angiography” , although CTA or
M RA will get the jo b done. You w ill have T2 high signal in the central cord (w hich will be
sw ollen), w ith serpentine perim edullary flow voids (w hich are usually dorsal).

Spinal AVM /AVFs


Type 1 M ost C om m on Type (85% ). D ural AVF - w ith a single coiled vessel

Intram edullary N idus from anterior spinal artery or posterior spinal artery. Can
Type 2 have aneurysm s, and can bleed. M ost com m on presentation is SA H. A ssociated
w ith H H T and KTS (other vascular syndrom es).

Type 3 Juvenile, very rare, often com plex and w ith a terrible prognosis

Intradural perim ed u llary w ith subtypes depending on single vs m ultiple arterial


Type 4
supply. These tend to occur near the conus.

F o ix A la jo u a n in e S y n d ro m e :

This is a congestive m yelopathy associated w ith a Dural AVF. The classic history
is a 45 year old m ale w ith low er extrem ity w eakness and sensory deficits.

You have increased T2 signal (either at the conus or low er thoracic spine), w ith
associated prom inent vessels (flow voids). The underlying pathophysiology is
venous hypertension - secondary to the vascular m alform ation.

Key Finding Swollen High Signal


Blob Like Cord with Serpentine
The vascular malformation flow voids are CSF flow voids along the
- punctate, serpiginous, and serpentine. Pulsation surface of cord
Artifacts
They are NOT blob like
- sorta like what you see with CSF pulsation signal loss.

174
lP IP M IM P M E IM S E C T IO N 2 1 :

t Cord Pathology ^

Syrinx - Also known as “a hole in the c o rd ”. People use the word “syrinx” for all those fancy
French / Latin words (hydrom yelia, syringom yelia, hydrosyringom yelia, syringohydrom yelia,
syringobulbia et c. . They usually do this because they
don’t know what those words mean.

This is the simple version:

• Hydromyelia = Lined by ependyma.


• Syringomyelia = NOT lined by ependyma

These is zero difference clinically - which is why “Central Cord “Syrinx with
everyone just says “syrinx.” The distinction is strictly Dilation ” Myelopathy ”
academic (i.e. m ultiple choice trivia).

Most (90%) cord dilations (healthy and sick ones) are congenital, and associated with Chiari I and II,
as well as Dandy-Walker, Klippel-Feil, and M yelom eningoceles. The other 10% are acquired either
by trauma, tumor, or vascular insufficiency.

In clinical practice, if there is perfectly central m id cord high signal dilation, surrounded by totally
normal cord I call it “central cord dilation” or “benign central cord dilation.” If there is the same
thing but the cord around the dilation looks “sick” - grayish / high signal, or the cord is atrophic, then
I use the word “ m yelopathy” or “myelopathic changes.” M yelopathy is a word for a diseased cord -
usually from disc/osteophyte compression. Although, you can have m yelopathy for any num ber o f
neoplastic, post traum atic, or inflammatory processes.

S p in a l C o rd In f a r c t : - C ord infarct / ischem ia can have a variety o f causes. T he m ost


com m on cause is “ idiopathic,” although I ’d expect the m ost com m on m ultiple choice scenario
to revolve around treating an aneurysm w ith a stent graft, or em bolizing a bronchial artery.
Im pairm ent involving the anterior spinal artery distribution is m ost com m on. W ith anterior
spinal artery involvem ent you are going to have central cord / anterior horn cell high signal on
T2 (because gray m atter is m ore vulnerable to ischem ia).

The “o w l’s eye” sign o f anterior spinal cord infarct is a buzzw ord.

It’s usually a long segm ent, (m ore than 2 vertebral body segm ents). D iffusion using single
shot fast spin echo or line scan can be used w ith high sensitivity (to com pensate for artifacts
from spinal fluid m ovem ent).

175
Demyelinating (T2 / F L A IR H y perintense):

Broadly you can think of cord pathology in 5 categories: Demyelinating, Tumor, Vascular, Inflammatory,
and Infectious.

In the real world, the answer is almost always MS - which is by far the most common cause. The other
three things it could be are Neuromyelitis Optica (NMO), acute disseminating encephalomyelitis
(ADEM) or Transverse Myelitis (TM).

MS in th e Cord: "Multiple lesions, over space and time.” The lesions in the spine are typically
short segment (< 2 vertebral segments), usually only affect half / part of the cord. The cervical cord is
the most common location. There are usually lesions in the brain, if you have lesions in the cord
(isolated cord lesions occur about 10% of the time). The lesions can enhance when acute - but this is
less common than in the brain. You can sometimes see cord atrophy if the lesion burden is large.

o
T ra n sv e rse M y e litis : This is a focal inflammation of the cord. The causes are
numerous (infectious, post vaccination - classic rabies, SLE, Sjogren’s,
Paraneoplastic, AV-malformations). You typically have at least 2/3 of the cross
sectional area of the cord involved, and focal enlargement o f the cord. Splitters will
use the terms “Acute partial” for lesions less than two segments, and “acute
complete” for lesions more than two segments. The factoid to know is that the
“Acute partials” are at higher risk for developing MS.

ADEM: As described in the brain section, this is usually seen after a viral illness or infection
typically in a child or young adult. The lesions favor the dorsal white matter (but can involve grey
matter). As a pearl, the presence o f cranial nerve enhancement is suggestive of ADEM. The step 1
trivia, is that the “anti-MOG IgG” test is positive in 50% o f cases. Just like MS there are usually brain
lesions (although ADEM lesions can occur in the basal ganglia and pons - which is unusual in MS).

NM O (N e u ro m y e litis O p tic a): This is also sometimes called Devics. It can


be monophasic or relapsing, and favors the optic nerves and cervical cord. Tends
to be longer segment than MS, and involve the full transverse diameter of the cord
(mild swelling). Brain lesions can occur (more commonly in Asians) and are
o usually periventricular. If any PhDs ask, the reason the periventricular location
occurs is that the antibody (NMO IgG) attacks the Aquaporin 4 channels - which
are found in highest concentration around the ventricles.

S u b a cu te Com bined D e g e n era tio n : This is a fancy way of describing the


effects of a Vitamin B12 deficiency. The classic look is bilateral, symmetrically
increased T2 signal in the dorsal columns, without enhancement. The
appearance has been described as an “inverted V sign.” The signal change
typically begins in the upper thoracic region with ascending or descending
progression.

H IV V a c u o la r M yelop ath y: This is the most common cause o f spinal cord

C
dysfunction in untreated AIDS. Key word there is “untreated” - this is a late
finding. Atrophy is the most common finding (thoracic is most common). The T2
high signal will be very similar to B12 (subacute combined degeneration) -
symmetrically involving the posterior columns. It can only be shown 2 ways - (a)
by telling you the patient has AIDS or risk factors such as unprotected anal sex at
a truck stop with a man “bear” with a thick mustache while sharing IV drug
needles, (b) not including B 12 as an answer choice.

176
MS: Lesions favor the white matter of the cervical region.
They tend to be random and asymmetric.

“Owl’s Eye ” -Ischem ia -Ischem ia


Vitamin B12 (SCD )
-Classic for Ischemia - More extensive -This time a
HIV
(Anterior) anterior involvement. posterior circulation
-Also seen in Polio -Also seen in NMO, pattern. These tend
Posterior. Can look
TM, or MS to be unilateral.
like an inverted “V”
-MS can also look
like this

Can Enhance/
Usually Short Usually Part o f the Not swollen, or
MS Restrict when
Segment Cord Less Swollen
Acute

Usually Long Usually involves both Expanded,


TM Can Enhance
Segment sides o f the cord Swollen Cord

Usually Long Usually involves both Optic Nerves


NMO
Segment sides o f the cord Involved

Not swollen, or
ADEM
Less Swollen

Usually Long Restricted


Infarct
Segment Diffusion

Expanded,
Tum or Can Enhance
Swollen Cord

177
Inflammatory / Infectious:

A r a c h n o id itis : T his is a general term for inflam m ation o f the subarachnoid space. It can
be infectious but can also be post-surgical. It actu ally occurs about 10-15% o f the tim e
after spine surgery, and can be a source o f persistent pain / failed back.

It’s show n tw o ways:

(1) E m p ty T h eca l S a c S ig n - N erve roots are


adherent peripherally, giving the appearance o f an
em pty sac.

(2) C entral N erve R o o t C lum ping. T his can range in


severity from a few nerves clum ping together, to all
o f them fused into a single central scarred band.

G u illa in B a r r e S y n d r o m e (G B S ) - A lso
know n as “ A cute inflam m atory dem yelinating p olyneuropathy” (A ID P). One o f those w eird
auto-im m une disorders that causes ascending flaccid paralysis. The step 1 trivia w as
C am pylobacter, but you can also see it after surgery, or in patients w ith lym phom a or SLE.

The thing to know is en h an cem en t o f the nerve roots o f the cauda equina.

O ther pieces o f trivia that are less likely to be asked are that the facial nerve is the m ost
com m on cranial nerve affected, and that the anterior spinal roots enhance m ore than the
posterior ones.

C h ro n ic In f la m m a t o r y
D e m y lin a tin g
P o ly n e u r o p a th y (C ID P ) -
The chronic counterpart to GBS.
C linically this has a gradual and
protracted w eakness (GBS
im proves in 8 w eeks, C ID P does
not). The buzzw ord is thickened,
enhancing, “ onion bulb” nerve
roots.

“ D readlocks,” “ L ocs,” “Jata"


som e people call them .
C ID P - D iffuse T h ickening o f the N erve Roots

178
Tumor

The classic teaching w ith spinal cord tum ors is to first describe the location o f the tum or, as
either (1) Intram edullary, (2) E xtram edullary Intradural, or (3) Extradural. T his is often easier
said than done. D ifferentials are based on the location.

Intramedullary Extramedullary Extradural


Intradural

A s tro c y to m a , Schw annom a, D isc D is e a s e

Ependym om a, M e n in g io m a , (m ost common)


H e m a n g io b la s to m a N e u ro fib ro m a , B o n e T u m o rs ,
D ro p M e ts M e ts,
Lym phom a

179
Intramedullary:

A s tr o c y to m a - This is the m ost com m on


Astrocytoma Ependymoma
intram edullary tum or in peds. It favors the
Most common in child Most common in Adults
upper thoracic spine. T here will be fusiform
Eccentric Central
dilation o f the cord over m ultiple segm ents.
Heterogenous Homogenous
They are eccentric, dark on T l , bright on T2,
Enhancement Enhancement
and they enhance. They m ay be associated
More Often Hemorrhagic
w ith rostral or caudal cysts w hich are usually
benign syrinx(es).

E p e n d y m o m a - This is the m ost com m on prim ary


cord tum or o f the low er spinal cord and conus / filum
term inale. You can see them in the cervical cord as well.
This is the m ost com m on in tram edullary m ass in adults.

The “m yxopapillary form ” is exclusively found in the


conus /filum locations. They can be hem orrhagic, and
have a dark cap on T2. They have tum oral cysts about %
o f the tim e. They are a typically long segm ent (averaging
4 segm ents). Ependymoma - Cervical Cord

M yxo p a p illa ry = M ost com m only located in the L um bar spine (conus/filum location)

V H L A ssociations:
H e m a n g io b la s to m a -
These are associated w ith Von H ippel Lindau (30% ). • Pheochromocytoma
•CNS Hemangioblastoma
The thoracic level is favored (second m ost com m on is (cerebellum 75%, spine 25%)
cervical). •Endolymphatic Sac Tumor
•Pancreatic Cysts
The classic look is a w ide cord w ith considerable e d em a.
•Pancreatic Islet Cell Tumors
A djacent serpinginous draining m eningeal varicosities can
•Clear Cell RCC
be seen.

In t r a m e d u lla r y M e ts -
This is very very rare, but w hen it does happen it is usually lung (70% ).

180
Extramedullary Intradural:

S c h w a n n o m a : This is the m ost com m on tum or to occur in the E xtram edullary


Intradural location. They are benign, usually solitary, usually arise from the dorsal nerve
roots. They can be m ultiple in the setting o f N F-2 and the C arney C om plex. The appearance
is variable, but the classic look is a dum bbell w ith the skinny handle being the intraforam inal
com ponent. They are T1 dark, T2 bright, and will enhance. They look a lot like
neurofibrom as. If they have central necrosis or hem orrhage, that favors a schw annom a.

N e u ro fib r o m a : T his is another benign nerve tum or (co m p o sed o f a ll p a rts o f the nerve:
nerve + sheath), that is also usually solitary. T here are tw o flavors: solitary and plexiform .
The plexiform is a m ultilevel bulky nerve enlargem ent that is pathognom onic for N F-1.
T heir lifetim e risk for m alignant degeneration is around 5-10% . T hink about m alignant
degeneration in the setting o f rapid grow th. T hey look a lot like schw annom as. If they have
a hyperintense T2 rim w ith a central area o f low signal - “target sign” that m akes you favor
neurofibrom a.

Schw annom a N e u r o fib r o m a

D oes envelop the adjacent nerve root


Does N O T envelop the adjacent nerve root
(usually a dorsal sensory root)

Solitary Solitary

M ultiple m akes you think N F-2 A ssociated w ith N F -1 (even w hen single)

Cystic change / H em orrhage T2 bright rim , T2 dark center “target sign”

Plexiform = P athognom onic for NF-1

M e n in g io m a : T hese guys adhere to but do not originate from the dura. They are m ore
com m on in w om en (70% ). T hey favor the posterior lateral thoracic spine, and the anterior
cervical spine. T hey enhance brightly and hom ogeneously. T hey are often T1 iso to hypo,
and slightly T2 bright. T hey can have calcifications.

D ro p M e ts : M edulloblastom a is the m ost com m on prim ary tum or to drop. B reast cancer
is the m ost com m on system ic tum or to drop (follow ed by lung and m elanom a). The cancer
m ay coat the cord or nerve root, leading to a fine layer o f enhancem ent (“ zuckerguss”).

181
Extradural:

V e r t e b r a l H e m a n g io m a : T hese are very com m on - seen in about 10% o f the


population. They classically have thickened trabeculae appearing as parallel linear densities
“jail bar” or “corduroy” appearance. In the vertebral body they are T1 and T2 bright,
although the extraosseuous com ponents typically lack fat and are isointense on T 1 .

O s te o id O s te o m a : This is also covered in the MSK. chapter, but as a b rie f review


focusing on the spine, they love to involve the posterior elem ents (75% ), and are rare after
age 30. They tend to have a nidus and surrounding sclerosis. The nidus is T2 bright and will
enhance. The classic story is night pain, im proved w ith aspirin. R adiofrequency ablation can
treat them (under certain conditions).

O s te o b la s to m a : This is sim ilar to an O steoid osteom a but larger than 1.5 cm. A gain,
very often in the posterior elem ents - usually o f the cervical spine.

A n e u r y s m a l B o n e C y s t: These guys are also covered in the M SK chapter. They also


like the posterior elem ents and are usually seen in the first tw o decades o f life. They are
expansile (as the nam e im plies) and can have m ultiple fluid levels on T2. T hey can get big
and look aggressive.

G ia n t C e ll T u m o r: These guys are also covered in the M SK chapter. These are com m on
in the sacrum , although rare anyw here else in the spine. You d o n ’t see them in young kids.
If they show this, it’s going to be a lytic expansile lesion in the sacrum with no rim o f
sclerosis.

C h o rd o m a : This is m ost com m on in the sacrum (they will w ant you to say clivus - that
is actually num ber 2). The thing to know is that a vertebral prim ary tends to be m ore
aggressive / m alignant than its counter parts in the clivus or sacrum . The classic story in the
vertebral colum n is “involvem ent o f tw o or m ore adjacent vertebral bodies w ith the
intervening disc. ” M ost are very T2 bright.

L e u k e m ia : They love to show it in the spine. You have loss o f the norm al fatty m arrow -
so it’s going to be hom ogeneously dark on T 1 . M ore on this in the M SK chapter.

M e ts : T he classic offenders are prostate, V e r t e b r a P la n a :


breast, lung, lym phom a, and m yelom a.
T hink m ultiple lesions with low T1 The pancake flat vertebral body.
Just say Eosinophilic Granuloma in a ^ 3
signal. C ortical breakthrough or adjacent
kid (could be neuroblastoma met), and
paravertebral com ponents are also helpful. Mets / Myeloma in an adult. *

182
B lank for S cribbles:
184
1 1

fM lfrm irfm lfrg lffm lfrm iirm jfT m lfT m llT a in m lfrm lfrm lfrm J irm ifT g ]

M u s c u l o s k e l e t a l

P r o m e t h e u s L io n h a r t , M .D .

185
v _ SECTION 1.
Traum a and O veruse ^ fr

Fracture Vocab
Stress Fracture = Fracture resulting from the Pathologic Open Fracture
mismatch o f bone strength and chronic Fracture: (Com pound
mechanical force. They come in two flavors Fracture):
(A) Fatigue, and (B) Insufficiency. You will sometimes
hear people use this A fracture associated
Fatigue Fracture term synonym ously with an open wound.
(som etime simply Insufficiency with “ Insufficiency Typically these will
called a “stress Fracture Fracture” . However, go to the OR for
fracture"). for the purpose o f reduction and
Abnormal stress on Normal stress on m ultiple choice this washout - given the
Norm al Bone. A bnorm al bone. term will m ost likely obvious risk for
refer to a fracture infection.
Classic Scenario - Classic Scenario - through a lvtic bone
lesion. Tuft Fractures (finger
Insane (but kinda hot) Old lady with horrible tip fracture) with
Type A Female Cross osteoporosis breaks These lytic lesions disruption o f the nail
Country Runner - her back (com pression can be mets or be plate are considered
literally runs until her fracture) by walking benign prim ary bone “open” fractures - and
legs & feet break in down a few steps. She lesions (like an although the typically
half. blam es Obama for the ABC, or Bone Cyst). w on’t go to the OR
fracture. they do get antibiotics
(whereas an intact
nail bed often w on’t).

Phases of Fracture Healing:


Physiology PhD s will describe 3 phases o f bone healing (Inflam m atory, R eparative, and
R em odeling). From a R ad io lo g ist’s perspective the m ost im portant thing to understand
about this process is that around 7-14 days granulation tissue will be form ing betw een the
bone fragm ents. This results in an increased lucency o f the fracture site related to bone
resorption.

In other w ords, a healing fracture will be M O R E L U C E N T at 7-14 days.

This explains the disclaim er cow ardly R adiologists throw out w hen they are afraid they
m issed a fracture “C onsider R epeat in 7-10 days, ” The idea is that in 7-10 days, you
should be able to see the fracture line , if one is p r e s e n t, because o f the increase in bone
lucency that occurs norm ally in the healing process.

186
Fracture Healing Continued -
In general, bones heal in about 6-8 w eeks, but is location
P halanges = Heal Fast (3 W eeks)
dependent. H ealing is the fastest in the phalanges
T ibia = Heal Slow (10 W eeks)
(around 3 w eeks), and the slow est is either the tibia or
E verything Else = 6-8 w eeks
fem oral neck/shaft - depending on w hat you read
(around 2-3 m onths).

Abnormal Healing Vocab


D e la y e d U n io n N o n -U n io n M al-U n io n
fracture not healed within the fracture is not going to heal This is union in poor anatomic
expected time period (but still without intervention. Some position (healed crooked as a
might). Some sources will say sources will say “6-9 months.’ politician).
“twice as long as expected” The classic locations are the
scaphoid, anterior tibia, and
lateral femoral neck.

Risk Factors For Abnormal Healing ( Delayed and Hon-Union)


(these are the ones I think are m ost testable):

V it a m in D D e fic ie n c y G a s t r ic B y p a s s D ru g s I M e d s

Vitamin D plays a vital role in Having your gut rewired results


calcium uptake and metabolism. in altered calcium absorption Tobacco (Smoking or Chewing)
(causes secondary
Vitamin D deficiency is actually hyperparathyroid and stripping NSA1DS
the most common vitamin o f calcium from bones) and
deficiency in America therefore higher rates o f non­ Prednisone (steroids)
(supposedly). union.

187
THIS vs THAT-C o m p r e s s iv e S id e v s T e n s ile S id e :

This comes up in two main areas - the femoral neck and the tibia.

• Fractures of the Compressive side are constantly pushed back together - these do well.

• Fractures of the Tensile side arc constantly pulled apart - these are a pain in the ass to heal.

T ibial Stress Fracture: _ Lateral

This is the most common


site o f a stress fracture in
young athletes.

These arc most common C


Medial
on the compressive side
T (posterior medial) in either
Anterior the proximal or distal third.

Less common are the F em oral Stress Fracture:


tensile side (anterior)
Fractures along the compressive (medial)
Posterior fractures, and these favor
side are more common, typically seen in
Medial the mid shaft. They arc a younger person along the inferior
bad news and don’t heal - femoral neck.
often called “dreaded
black lines.'" Fractures along the tensile (lateral) side
arc more common in old people.

S O N K (S p o n ta n e o u s O s te o n e c r o s is o f t h e K n e e ):
This is totally named wrong, as it is another type of
insufficiency fracture. You see this in old ladies with
the classic history o f “sudden pain after rising from
a seated position.” Young people can get it too
(much less common), usually seen after a m eniscal
surgery.

Key Factoids:
t o • It’s an insufficiency fracture (NOT
osteonecrosis) think SINK not SONK
t o • Favors the medial femoral condyle
(area o f m aximum weight bearing)
t o • Usually unilateral in an old lady without
history o f trauma
• Associated with meniscal injury
t o

SO NK — LOTS OF EDEM A
- Subchondral Deform ity (arrows)

188
N a v ic u la r S t r e s s F r a c tu r e - You see these in runners w ho run on hard surfaces. The
thing to know is that ju st like in the w rist (scaphoid), the nav icu lar is high risk for AVN.

M arch F r a c tu re: T his is a m etatarsal stress fracture w hich is fairly com m on.
C lassically seen in m ilitary recruits that are m arching all day long.

C a lc a n e a l S t r e s s F r a c tu r e - The calcaneus is actually the m ost fractured tarsal


bone. The fractures are usually intra-articular (75% ). T he stress fracture w ill be seen w ith
the fracture line perpendicular to the trabecular lines.

Y ou’ll rue the day you crossed m e T rebek— ular Lines.

Typical C alcaneal
T rabecular Lines

O rientation
o f C alcaneal
Stress Fx
I ’ll take the Penis M ightier for $600

THIS vs THAT: H igh R isk v s L o w R isk S t r e s s F r a c tu r e s :


You can sort these based on the likelihood o f
uncom plicated healing w hen treated conservatively.

H igh R isk L o w R isk

Fem oral N eck (tensile side) Fem oral N eck (com pressive side)

T ransverse Patellar Fracture L ongitudinal P atellar Fracture

A nterior Tibial Fracture (m idshaft) Posterior M edial Tibial Fracture

5th M etatarsal 2nd and 3rd M etatarsal

Talus C alcaneus

Tarsal N avicular

S esam oid G reat Toe

189
Site-Specific Entities - Hand and Wrist
Scaphoid Fracture
Most common carpal
Blood flow is “retrograde” (distal
bone fracture. “Retrograde”
to proximal). This is because the
scaphoid surface is almost (distal to
Typical age group is an proximal) via
adolescents and young entirely (80%) covered with
the Dorsal
adults (Grandma is more cartilage.
Carpal
likely to get a distal As such, the proximal pole most Branch o f
radial fracture with a Radial Artery
susceptible to AVN and Non-
similar mechanism —
Union.
fall).
Distal Pole / The first sign o f AVN = Sclerosis
(the dead bone can't turn over /
Scaphoid Tubercle
recycle)
/ v V Waist
Most common (70 %) fracture Proximal
site = waist Pole is at
Risk for AVN
Displacement o f > 1mm will / Mal-Union
likely get a fixation screw to pull
Proximal
Pole the fragments together.

Trans-Scaphoid Scapholunate Ligament Humpback Deformity


Perilunate Dislocation Disruption This deformity
results from
Capitate angulation o f the
proximal and distal J /
fragments - in the
CJ
W
setting o f a waist
fracture.
/Cff) n
N orm al
Can progress to
progressive collapse
and non-union.
\
Humpback shown
Associated with D1S1 on lateral view

Seen with 10-30% o f distal radius AVN


and/or carpal fractures
As above the proximal pole is at
Perilunate dislocation
greatest risk.
(discussed later in the The SL ligament is composed o f 3
chapter) have a high parts (volar, dorsal, and middle), The first sign of
association (60%) with with the dorsal band being the AVN = Sclerosis
a scaphoid fracture most important for carpal stability (the adjacent bones
(opposite o f luno-triquetral which will demineralize,
is volar). but the avascular
bone will not).
Disruption o f the ligament Later the bone will
predisposes for DISI deformity fragment.
(discussed later in the chapter in
MRI = T1 Dark
greater depth).
Trivia: “Prieser Disease” is an
atraumatic AVN o f the scaphoid

190
SLAC and SNAC Wrists
Both are potential complications of trauma, with
similar mechanisms.

SLAC Wrist (Scaphoid-Lunate Advanced Collapse)


occurs with injury (or degeneration via CPPD) to the
S-L ligament.

SNAC Wrist (Scaphoid Non-Union Advanced


Collapse) occurs with a scaphoid fracture.
SN A C
Just remember that the scaphoid always wants to
rotate in flexion - the scaphoid-lunate ligament is the
only thing holding it back. If this ligament breaks it
will tilt into flexion, messing up the dynamics of the
wrist. The radial scaphoid space will narrow, and the
capitate will migrate proximally. • Radioscaphoid joint is first to
develop degenerative changes
Treatment depends on the occupation/needs of the
wrist. Wrist fusion will maximize strength, but cause • Capitate will migrate proximally and
a loss o f motion. Proximal row carpectomy will there will eventually be a DISI
maximize ROM, but cause a loss o f strength. deformity

S ca p h o lu n a te L igam ent Tear:

The Terry Thomas look (gap between the scaphoid and lunate) on plain film.

There are actually 3 parts (volar, dorsal, and middle), with the dorsal band being the most
important for carpal stability. If they tear the carpals will migrate away from each other.

Predisposed for DISI deformity and all that crap I talked about earlier. More on this
complex carpal instability on the next page.

191
THIS vs THAT- D ISI vs V IS I

This topic can be very confusing. Here is the way I like to think about it.

1 imagine two people (Lunate and Scaphoid) standing on S -L Ligam ent


opposite sides o f a very steep hill. At the apex o f the hill is
a man named “ Scapholunate Ligam ent” - 1 agree, it’s a
strange name. His parents were probably vegetarians. Scaphoid
Lunate
This hill is very steep, so Scapholunate Ligament has
grabbed each o f the people (Lunate and Scaphoid) by the
hand - he was worried they might fall. In fact, the only
thing keeping these two people from tum bling down the
hill is the insane grip strength o f Scapholunate Ligament D O R SA L V O LA R
(rum or has it he can close a #3 Captain o f Crush - which
would certify him as an official Captain o f Crush).
By using this analogy perhaps you can infer that if you
have carpal ligament disruption, the carpal bones will
rotate the way they naturally want to (down the hill).
The reasons for their rotational desires are complex but
basically have to do with the shape o f the fossa they sit
on.
Just rem em ber the scaphoid wants to flex (rock volar)
and the lunate wants to extend (rock dorsal). The only D O RSAL V O LA R
thing holding them back is their ligam entous attachm ent
to each other.

D IS I (Dorsal Intercalated Segmental Instability) V IS I (Volar Intercalated Segmental Instability) - I


- 1 like to call this dorsiflexion instability like to call this volar-flexion (palmar-flexion)
because it helps me rem em ber w hat’s going on. instability because it helps me remember w hat’s
After a “Radial sided injury” (scapholunate going on. After a “Ulnar sided
side), the lunate becomes free o f the stabilizing injury” (lunotriquetral side), the lunate no
force o f the scaphoid and rocks dorsal ly. longer has the stabilizing force o f the
Rem ember SL ligament injury is comm on, so lunotriquetral ligament and gets ripped volar
this is com m on. with the scaphoid (remember the scaphoid stays
up late every night dreaming o f tilting volar).
Remember LT ligament injury is not common,
so this is not common. It's so uncommon in fact
Dorsal Volar
that if you see it - it’s probably a normal variant
due to wrist laxity.

V ISI: Narrowing of
the SL angle - with
Normal volar-flexion o f the
lunate & scaphoid.
Scaphoid-
Lunate Angle
DISI: Widening of the SL Angle < 30 (this acute
is 30-60 angle looks like a V to
angle - with dorsiflexion of degrees
the lunate. me - “V” for “V")

Angle > 60
(some sources say 80) Dorsal Volar

192
C a r p a l D is lo c a tio n s - A spectrum o f severity

Least Severe

Scapho-L unate P eri-L unate M id -C arp al L unate


D issociation D islocation D islocation D islocation

• SL- Wider Than 3 mm Trivia to Know = Trivia to Know = Trivia to Know =


• Clenched Fist View can
worsen it (would make a Note that the Lunate Both Lunate and Lunate moves,
good next step question) stays put - it’s the Capitate lose radial others stay
• Chronic SL dissociation carpal bones around alignment.
can result in a SLAC the lunate (“peri­ It happens with a
wrist lunate bones”) that Associated with Dorsal radiolunate
move. Triquetro-Lunate ligament injury
interosseous ligament
60% associated with disruption “Most Severe”
Scaphoid Fractures
Associated with a
Triquetral Fracture

V u ln e r a b le Z o n e s T h e o r y

Dislocations around the lunate are described


in two flavors

Lesser Arc: Pure Ligament Injury (No


Greater Arc Fractures)

Greater Arc: Associated with fractures.


Space of Poirier Described by saying “trans” the name o f the
fracture then the dislocation. Example
“Trans-scaphoid, peri-lunate dislocation”

Space of Poirier - Ligament free (“poor”)


area, that is a site o f weakness

193
Anatomic Trivia Regarding the Spaces ot the Wrist:
W h ic h s y n o v ia l s p a c e s n o r m a lly c o m m u n ic a t e ?

The answ er is pisiform recess


and radiocarpal joint. le a n Common
CMC
think o f tw o w ays to ask this
(1) related to fluid - the bottom
line is that excessive fluid in the
pisiform recess should not be Midcarpal
Pisiform
considered abnorm al if there is a
radiocarpal effusion, and
(2) that either space can be used
for w rist arthrography.

Radiocarpal
Distal Radioulnar

O th e r jo in t s p a c e s in t h e b o d y , e a s ily le n d in g to m u lt ip le c h o ic e te s tin g :

G lenohu m eral Joint and Should N O T com m unicate. Im plies the presence o f a
Subacrom ial Bursa full thickness rotator c u ff tear.

A nkle Joint and C om m on Should N O T com m unicate. Im plies a tear o f the


(lateral) Peroneal Tendon Sheath calcaneofibular Ligam ent.

A chilles Tendon and Should N O T com m unicate. The A chilles tendon


P osterior Sub talar Joint does N O T have a true tendon sheath.
Pisifrom R ecess and
Should norm ally com m unicate.
R adiocarpal Joint

Anatomic Trivia - Triangular Fibrocartilage Complex — TFCC


I'll begin by saying that this is arguably the m ost com plex anatom y in the entire body
(m aybe second only to the posterior lateral com er). A detailed understanding is well
beyond the scope o f the exam (p ro b a b ly ...). H aving said that, the TFC is specifically
m entioned on the official study guide, so we need to at least talk about it.

The T FC C functions as the prim ary stabilizer and shock absorber o f the distal radial ulnar
jo in t (D R U J). The T FC C is critical for a range o f activities (doing a pushups , punching
G eneral Zod, e tc ...).

194
Anatomic Trivia - Triangular Fibrocartilage Complex — TFCC - Continued
It looks crazy com plicated - but you really only TFC C 5 C om ponents:
need to know at m ost 5 structures, 1. Triangular Fibrocartilage (Articular Disci
2. Volar & Dorsal Radioulnar Ligaments
O f the 5, the Hand Surgeon only really gives a 3. Meniscus Homologuc
4. UCL
shit about the A rticular Disc and R adioulnar 5. Tendon Sheath of the UCU
L igam ents.
MR Signal:
Ulnar Collateral
Ligament (UCL) “TFC Proper” (Articular
Volar Disc) will be dark on every
Lunate Extensor Carpi
Distal sequence.
Radial Ulnaris (ECU)
Tendon / Sheath - The ulnar attachment often
Ulnar looks intermediate in signal,
Ligament Meniscus this is normal related to loose
Homologue connective tissue in the
Triangular region.
Articular Triangular Lig
- The radial attachment will
Disc (styloid
attachment) also have intermediate signal,
but this is from the normal
Ligamentum articular cartilage.
Subcruentum
Dorsal Triangular Lig.
Distal (foveal
Radial PSR = Prestyloid Recess attachment)
Ulnar

Extensor Carpi
Dorsal Distal Radial TFCC Injuries:
Ulnaris (ECU)
Ulnar Ligament Prest loid You can group these into:
Sheath and Tendon
“Class 1” Acute Injuries: Usually
via fall onto extended wrist.
“Class 2” Chronic Degeneration:
These are more common, and
UCL associated with positive ulnar
variance and ulnar impaction.
Central perforations are common
Meniscus - and might even be “expected”
Homologue on an old person.
Volar Distal
Radial Ulnar
Ligament

Central Tear, with Ulnar Positive


Variance and Abutment (cystic change
in the lunate) - more on next page.

195
TFC Vasculature &Healing
Sim ilar to how the knee
m eniscus has “red ” and
“w hite” zones - the ulnar
side o f the TFC is vascular
and m ore likely to heal. Radius
Radial sided injuries are
relatively avascular and
less likely to heal.

Ulnar Variance / Impaction


U ln a r V a r ia n c e -

This is determ ined by


com paring the lengths o f the
ulna and distal radius.

These length differences can


occur congenitally, or be P o s itiv e N e g a tiv e
acquired from im paction / V a r ia n c e : V a r ia n c e :
fracture deform ity. A ssociation: A ssociation:
- U ln ar Im paction - AVN o f the L unate
S yndrom e (“ K ienbock”)

U ln a r Im p a c t io n S y n d ro m e K ie n b o c k s :
(U ln a r A b u tm e n t): AVN o f the lunate, seen in people in their
Seen w ith positive ulnar variance. 20s-40s. The m ost likely testable trivia is
the association w ith negative ulnar
Essentially the distal ulna sm ashes into
v arian ce. It’s going to show signal drop
the lunate, degenerating it (cystic change
out on T 1 .
/ geodes e tc ...) and tears up the TFCC.

Sclerotic on P lain F ilm L ow S ignal on T1

196
Distal Radius / Wrist Fractures:
There are 3 nam ed fractures o f the distal radius / w rist w orth know ing.

D is ta l R a d iu s R a d ia l R im

Volar Dorsal

C o lle s ’ F r a c tu r e S m ith F r a c t u r e B a r to n F r a c t u r e
(Outward) (Inward) (Dorsal or Volar)
"Collie Dogs ” Like it Outside

Distal Metaphysis Fx Distal Metaphysis Fx Radial Rim Fx


Dorsal Angulation Volar Angulation Volar is More Common
Old Lady Fracture Younger Patient Radial-Carpal Dislocation is the
Ulnar Styloid Fx is Ulnar Styloid Fx is “hallmark”
Commonly Associated Commonly Associated Typically Surgical (they have a
high rate o f re dislocation /
mal-union)

Radial Tilt
• There is a norm al volar tilt o f around 11 degrees
• W ith distal radial fractures this can get fucked up
X\ V &
• M ost O rthopods w o n ’t accept anything past neutral
• A TR U E lateral is necessary to m easure it

H ow do y o u know y o u r lateral is “tru e " ?


Volar Dorsal
The volar cortex o f the pisiform overlies the central
1/3 o f the interval betw een the scaphoid and capitate

197
Wrist Tendon Anatomy Review:
W ith regard to the extensor tendons, there are four things to know:

T here are 6 extensor com partm ents


(5 fingers + 1 for good luck).
Lister's Tubercle
First com partm ent (A PL and EPB) are the
WOM /
ones affected in de Q u erv ain ’s
"* 5i"' JL / » <pu
T hird com partm ent has the EPL w hich
courses beside L iste r’s Tubercle. %

The sixth com partm ent (E xtensor Carpi Ulna Radius \\ * It


U lnaris) - can get an early tenosynovitis in
rheum atoid arthritis.

Carpal Tunnel: They could show you the carpal tunnel, but only to ask you about anatomy.

W hat goes through the carpal tunnel (m ore easily asked as “w hat does N O T go through ”) ?

K now ing what is in (and not in) the carpal


tunnel is high yield for m ultiple choice testing.
The tunnel lies deep to the palm aris longus, FCR Median
and is defined by 4 bony prom inences Not in the FPL Nerve FDP + FDS
true tunnel
(pisiform , scaphoid tubercle, hook o f ham ate,
trapezium tubercle), with the transverse carpal
ligament w rapping the contents in a fibrous
sheath.

The tunnel contains 10 things

• 4 Flexor D. Profudus (FDP)

• 4 Flexor D. Superficials (FDS)


D o e s N O T go
• 1 Flexor Pollicis Longus (FPL), and
th r o u g h t h e tu n n e l
• 1 M edian Nerve
-F lexor Carpi Radialis
The Flexor Carpi Radialis (FCR) is N O T truly in the tunnel.
The extensor tendons are on the other side o f the hand. Note that -F lexor Carpi U lnaris
Flexor Pollicis Longus (FPL) goes through the tunnel, but
-Palm aris Longus
Flexor Pollicis Brevis does not (it’s an intrinsic handle m uscle).
(if you have one)
Palm aris longus (if you have one) does N O T go through the
tunnel. -Flexor Pollicis B R EV IS

198
C a r p a l T u n n e l S y n d r o m e (C T S ):
• M edian N erve D istribution (thum b-radial aspect o f 4th digit), often bilateral, and m ay have
thenar m uscle atrophy.
• On U ltrasound, enlargem ent o f the nerve is the m ain thing to look for
• It’s usually from repetitive traum a,
• Trivia = A sso ciatio n w ith D ialy sis. Pregnancy, DM , and H Y PO thyroidism

Classic Findings:
• Increased Signal in the Median Nerve
• The Nerve May Also Be Swollen or
Look Smashed / Flattened
• Bowing o f the Flexor Retinaculum

G u y o n ’s C a n a l S y n d ro m e :
• Entrapm ent o f the ulnar nerve as it passes through G u y o n ’s canal (form ed by the pisiform
and the ham ate - and the crap that connects them ).
• C lassically caused by handle bars “handle bar p a lsy .”
• Fracture o f the hook o f the ham ate can also eat on that ulnar nerve.

S u b -S h e a th T e a r / D is lo c a tio n

This refers to a traum atic dislocation to the


extensor carpi ulnaris (ECU - com partm ent 6) out
o f its norm al groove at the level o f the distal ulna.
This dislocation / subluxation im plies rupture o f
the overlying sheath.

Trivia - the direction o f dislocation is m edial.

199
Tenosynovitis:
This is an inflammation of the tendon, with increased fluid seen around the tendon. This will be
shown on MRI (or US).

D IF F U S E FOCAL
T u b ercu lo u s or
R heum atoid P e n e tra tin g In fe ctio n
N ontuberculous O veruse
A rth ritis : (ca n be fo c a l or d iffu se )
M y c o b a c te ria l

Hand and wrist are the most Tenosynovitis o f any flexor tendon is
common tendons affected Multiple Flexor a surgical emergency as it can spread
Tendons rapidly to the common flexors o f the
or wrist.
Isolated Extensor This is going to
Increased pressure in the sheath can be classic
Diffuse exuberant tenosynovitis Carpi Ulnaris if
cause necrosis o f the tendons. locations like
that spares the muscles. early (ECU =
Compartment 6) Patients with delayed treatment tend 1st extensor
to do terrible compartment for
De Quervains
Usually occurs in patients who Tenosynovitis — discussed
are immunocompromised. can present as an more below.
Myocobacterium Marinum is usually
Discrete filling defects in the early RA direct infection in a fisherman or
fluid filled sheaths (“rice findings (before sushi chef.
bodies”) is a classic TB finding. bone findings).

De Q u erv a in ’s Tenosynovitis: In te rs e c tio n Syndrom e:

So called “Washer Woman’s Sprain” or “Mommy Thumb.”


A repetitive use issue
Occurs from repetitive activity / overuse.
(classically seen in rowers),
The classic history is “new mom - holding a baby.”

Ultrasound: Increased fluid Occurs where the first extensor


First Extensor Compartment within the first extensor tendons, “intersects” the second
(Extensor Pollicis Brevis and tendon compartment extensor compartment tendons.
Abductor Pollicis Longus MRI: increased T2 signal in The result is extensor carpi radialis
the tendon sheath brevis and longus tenosynovitis.

Finkelstein Test = Pain on passive ulnar deviation.

The presence or absence o f an intratendinous septum (between the EPB


and APB) - tendons on the is a prognostic factor. If its absent, this will
nearly always resolve with conservative treatment alone.

to Listers Tubercle

200
B e n n e tt a n d R o la n d o
%
F r a c tu r e s :
• They are both fractures at the base o f %
%
the first m etacarpal £
-o
• The R olando fracture is com m inuted
(B ennett is not)
• Trivia: The pull o f the A bd u cto r
P ollicis Longus <A P L ) tendon is
w hat causes the dorsolateral
B E N N E T T Fx lillj
Rolando Fx
dislocation in the Bennett
Fracture

DO NOT
G a m e k e e p e r ’s T h u m b (S k ie r ): perform
• Avulsion fracture at the base o f the proxim al first phalanx radiographic
stress views
associated w ith ulnar collateral ligam ent disruption.
for
• The frequently tested association is that o f a “S t e n e r Gamekeepers
L e s io n .” A Stener Lesion is w hen the A dductor tendon Thumbs
aponeurosis gets caught in the tom edges o f the UCL.
The displaced ligam ent w o n ’t heal right, and will need
UCL
surgery.
• It m akes a “yo-yo” appearance on MRI - su p p o se d ly ...
• N ext Step - D o n ’t do “ stress view s” that can cause a
stener. M RI is the m ore appropriate test.

Ulnar collateral
V) _1 CO

'55
o o '5o5 ligament is
i_ 0) 3 retracted and
c
tt> _J 3 c 3 displaced
o o 0> o a)
CO 3 c CO c superficial to
o o
Q. a the adductor
< < aponeurosis.
1
Stener

T r ig g e r F in g e r: The idea is the overuse /


repetitive traum a causes scarring in the flexor tendon
sheath. The fancy w ord is “ stenosing tenosynovitis.”
This is m ost com m only show n w ith ultrasound. If
they should you a hand ultrasound think about this.

A nother com m on area o f “ stenosing tenosyn ovitis” is Thick Sheath Normal for
Comparison
at the ankle specifically the flexor hallucis longus
tendon around the ankle in patients w ith the os
trigonum syndrom e.

201
Site-Specific Entities - Elbow&Forearm

G e n e r a l T r iv ia :

• Radial Head Fracture is m ost com m on in adults (supracondylar is m ost com m on in PED s)
• S ail sign - elevation o f the fat pads from a jo in t effusion. Supposedly a sign o f occult
fracture. The testable trivia is (1) the posterior fat pad is m ore specific (posterior is
p o sitive ), and (2) the posterior fat pad can appear falsely elevated (false positive) if the
lateral isn ’t a true 90 degree flexed lateral. "P osterior Positive, P osterior P osition
D ependent ”
• C apitellum fractures are associated w ith posterior dislocation

F o r e a r m F r a c tu r e s / E p o n y m s :

Forearm fractures are “ring” or “ pretzel” type fractures, sim ilar to the pelvis or m andible.
Think about breaking a pretzel, it alw ays snaps in tw o spots (not ju st one). So forearm
fractures are often tw o fractures, or a fracture + dislocation.

T here are 3 French sounding (therefore high yield) fractures o f the forearm w hich follow this
ring / pretzel principal.

Monteggia Fracture (MUGFt):

Fracture of the proximal ulna, with


anterior dislocation of the radial head.

Dislocation of the radial head follows


the angulation of the Ulnar Fx.

Galeazzi Fracture (MUGR)

Radial shaft fracture, with anterior


dislocation of the ulna at the DRUJ.

Essex-Lopresti

Fracture of the radial head + Anterior


dislocation of the distal radial ulnar
joint.

Unstable fracture - With rupture of the


interosseous membrane

202
C u b ita l T u n n e l S y n d ro m e

T here are several causes - the m ost com m on in the real w orld is probably repetitive valgus
stress. The m ost com m on show n on m ultiple choice is probably an accessory anconeus.

W TF is an “A nconeus ” ? It a piece o f shit m uscle that does nothing but get in the w ay o f an
orthopedic scope. It’s norm ally on the lateral side the elbow. You can have an “A ccessory
A nconeus ” - also called an “A nconeus E pitrochlearis ’’ - on the m edial side w hich w ill exert
m ass effect on the ulnar nerve.

A natom ic Trivia: The site w here the ulnar nerve passes beneath the cubital tunnel
retinaculum also know n as the epicondylo-olecranon ligam ent or O sb o rn e’s ligam ent.

NORM AL C u b it a l T u n n e l S y n d r o m e
Secondary to an A ccesso ry A nconeus

Anconeus Ulnar Nerve


Posterior Band o f the UCL
A ccessory’ A nconeus ” A K A
“A nconeus E pitrochlearis ”

L a t e r a l E p ic o n d y litis (m ore com m on than m edial) - seen in Tennis P layers -


• E xtensor Tendon Injury (classically extensor carpi radialis brevis)
• Radial C ollateral L igam ent C om plex - Tears due to varus stress

M e d ia l E p ic o n d y litis (less com m on than lateral) - seen in golfers


• C om m on flexor tendon and ulnar nerve m ay enlarge from chronic injury

203
P a r tia l U ln a r C o lla t e r a l L ig a m e n t T e a r :

For the exam all you really need to know is


that throwers (people who valgus overload)
hurt their ulnar collateral ligament (which
attaches on the medial coronoid - sublime
tubercle). The ligament has three bundles,
and the anterior bundle is by far the most
important. If you get any images it is most
likely going to be o f the partial UCL tear,
described as the “T sign,” with contrast
material extending medial to the tubercle.
Normal T-Sign
“UCL Partial Tear”

L it t le L e a g u e r E lb o w

The children o f insecure men who sucked at sports in high school are most
susceptible to this injury. The mechanism is repetitive micro-trauma from
endless hours o f training (necessary to finally rectify the injustice which
beset their family when dad was benched senior year from the junior varsity
baseball squad).
We are talking about a repetitive
chronic injury to the medial
epicondyle. When I say injury I mean
stress fracture, avulsion, or delayed
closure o f the medial epicondvlar
apophysis. This is usually associated
Posterior
with UCL injury.
Sheer
Children aren’t the only ones who can Valgus (osteophyte
fuck up their elbows pursuing the kind Overload formation)
o f immortality that is only offered to
those worthy enough to step foot on
the field at Yankee stadium. There is a
well described “valgus overload Lateral
syndrome” seen in throwers, Compression
consisting o f a triad of lateral
compression, medial tension, and
posterior sheer. This mechanism
results in UCL injury (often anterior
band), Arthritis at the Posterior
Humerus / Ulna, and the development
of an OCD at the capitellum.

E p itro c h le a r Lym phadeno pathy - This is a classic look for cat-scratch disease.

D ialysis Elbow: This is the result of olecranon bursitis from constant pressure on the area, related to
positioning of the arm during treatment.

204
B ice p s T ear
Tears can be partial or complete. When complete the tear typically occurs in shoulder with the
tendon avulsing off the labrum (or at the level o f the bicipital groove).
Common mechanism is incorrect deadlift form (while doing cross fit like an ape on cocaine). If
you plan on going nuts slinging that shit around consider switching to a double over grip. If you
want to use over under grips - you need strict form (keep your arms locked out dummy). There
are tons of highlight reals on youtube of people tearing biceps while deadlifting - notice every
single one is using an over under grip, and not maintaining straight arm technique.

Partial Biceps Tear - Gamesmanship


A -Partial tears often arc associated with bicipitoradial bursitis

Patients present with a painful mass in the antecubital fossa (rolled up muscle) - with the classic
history of “trying to impress the girl in the pink spandex sports bra with my deadlift.” This rolled
up muscle is sometimes referred to as the “Popeye Deformity” - in reference to my childhood
hero - a heavily muscled blue collar worker, who smokes, and solves his all of his problems with
violence.

Sneaky: Injury to the bicep is associated with median nerve symptoms

T r ic e p R u p tu re
The tricep tendon has the honorable distinction o f being the LEAST
common tendon in the body to rupture. Even tendinopathy is fairly
uncommon relative to other nearby structures. When it does tear you Normal
should be thinking about salter harris II fractures of the olecranon - Striations
that is the classic scenario.
1 think because this is so uncommon that mimics would be more likely
on the exam. So, I’d be aware o f two things: (1) the normal striated
appearance of the insertion at the olecranon, and (2) the common entity
of olecranon bursitis - which you should think o f first if you see a
bunch o f fluid signal in the posterior elbow.

E lb o w D is lo c a tio n

This is the second m ost com m on jo in t dislocated in the adult. The associated fractures
are usually the radial head and the coronoid process.

Instability in the elbow (so called P o s te r io r R o ta r y In s t a b ilit y ) is described in a


pattern starting in the posterior lateral co m e r w ith tearing o f the lateral UCL.

Partial Dislocation Dislocation


Tearing of the Coronoid Perched on Coronoid Posterior
LUCL Trochlea to Humerus with a
(lateral UCL). UCL Tear
(LU C L + LCL + C apsule)

205
Site-Specific Entities - Shoulder

D is lo c a tio n :
* A n terio r inferior (subcoracoid) are by far the m ost com m on (like 90% ).
o H ill-Sachs is on the H um erus.
o H ill-Sachs is on the posterior lateral hum erus, and best seen on internal
rotation view.
O Bankart - anterior inferior labrum
O G reater tuberosity avulsion fracture occurs in 10-15% o f anterior dislocations
in p atien t’s over 40.
* P osterior D islo ca tio n : uncom m on - probably from seizure or electrocution
O R im Sign - no overlap glenoid and hum eral head
O Trough Sign - reverse Hill Sachs, im paction on anterior hum erus
O “Light B ulb Sign ” - Arm m ay be locked in internal rotation on all view s
* Inferior D islocation (lu.xatio erecta hum eri) - this is an uncom m on form , w here the
arm is sticking straight over the head. The thing to know is 60% get neurologic
injury (usually the axillary nerve).

Posterolateral hum eral head M emory>Tool


H ill-Sachs im paction fracture (anterior
dislocation) 1 rem em ber that hip dislocations are
posterior - from the straight leg
A nterior G lenoid Rim dashboard m echanism .
B ankart
(anterior dislocation)

A nterior hum eral head im paction Then I ju st rem em ber that shoulders
Trough Sign are the opposite o f that (the other
fracture (posterior dislocation)
one, is the other one).
R everse Posterior G lenoid Rim
B ankart (posterior dislocation) Sh ou ld er = U sually A n terior

l^lnl
Internal
Rotation
(Hill-Sachs)

206
P ro x im a l H u m e r u s F r a c tu r e :

This is usually in an old lady falling on an out stretched arm . O rthopods use the N eer
classifications (how m any parts the hum erus is in ?). T hree or four part fractures tend to do
worse.

T h e P o s t O p S h o u ld e r (P r o s th e s is )

There are 4 M ain Types: H um eral H ead R esurfacing, H em i-A rthroplasty, Total Shoulder
A rthroplasty, and the R everse Total S houlder A rthroplasty.

W hat is this “ reverse total sh ou ld er” ?

P f
A conventional total shoulder m im ics norm al
anatom y. A reserve total shoulder is the bizarro
version; w ith a plastic cup on the hum eral head
and m etallic sphere on the glenoid.

Conventional Reverse

W ho gets w hat? -

The surgical choice


depends on tw o m ain
factors:

(1) Is the C u ff Intact?


(2) Is the G lenoid
T rashed ?

C om plications / Trivia:

— Total Shoulder M ost C om m on C om plication = L oosening o f the G lenoid C om ponent


— Total Shoulder C om plication - “A n terio r E scape ” - This describes anterior m igration o f
the hum eral head after subscapularis failure.

— Reverse Total S houlder D oes N O T require an intact rotator c u ff - patient rely heavily on
the deltoid.
— Reverse Shoulder C om plication - P osterior A crom ion F racture - from excessive deltoid
tugging.

207
Im pingem ent / Rotator C uff Tears:

This is a high yield / confusing subject that is w orth talking about in a little m ore detail. In
general, rotator cu ff pathology is the result o f overuse activity (sports) or im pingem ent
m echanism s. There are two types o f im pingem ent with tw o m ajor sub-divisions w ithin those
types. Like m any things in Radiology, if you get the vocabulary down, the pathology is easy to
understand.

External: This refers to im pingem ent o f the rotator cu ff overlying the bursal surfaces
(superficial surfaces) that are adjacent to the coracoacrom ial arch. A s a rem inder, the arch is
m ade up o f the coracoid process, acrom ion, and coracoacrom ial ligament.

P rim ary E xternal Causes (Abnorm al Coracoacrom ial Arch) :

• The hooked acrom ion (type III Bigliani) is m ore associated with external im pingem ent
than the curved or flat types.

• Subacrom ial osteophyte form ation or thickening o f the coracoacrom ial ligam ent

• Subcoracoid im pingem ent: Im pingem ent o f the subscapularis betw een the coracoid
process and lesser tuberosity. This can be secondary to congenital configuration, or a
configuration developed post traum atically after fracture o f the coracoid or lesser
tuberosity.

Secondaiy External Causes (N orm al Coracoacrom ial Arch):

• “M ultidirectional G lenohum eral Instability” - resulting in m icro-subluxation o f the


hum eral head in the glenoid, resulting in repeated m icro-traum a. The im portant thing to
know is this is typically seen in patients with generalized jo in t laxity, often involving both
shoulders.

Internal: This refers to im pingem ent o f the rotator cuff on the undersurface (deep surface)
along the glenoid labrum and hum eral head.

• Posterior Superior: This is a type o f im pingem ent that occurs when the posterior superior
rotator c u ff (junction o f the supra and infraspinatus tendons) com es into contact with the
posterior superior glenoid. Best seen in the A BER position, w here these tendons get
pinched betw een the labrum and greater tuberosity. This is seen in athletes w ho m ake
overhead m ovem ents (throw ers, tennis, swim m ing).

• A nterior Superior: This is internal im pingem ent that occurs when the a n n is in horizontal
adduction and internal rotation. In this position, the undersurface o f the biceps and
subscapularis tendon m ay im pinge against the anterior superior glenoid rim.

208
Im pingem ent Continued..

Primary
’ Abnormal Coracoacromial Arch ------ ► Subacrom ial
— Hook Shaped (B3) *Fs With Supra S
— Osteophytes
—Post Traumatic ► Subcoracoid
— Thickened Ligaments ’F ’s With Sub S
External

Secondary

\ ’ Multidirectional Instability
—Labrum Often Normal
—“Increased Glenohumeral
Volume” - with injection

Posterior Superior +------------------------ If the exam writers


’ Throwers just say “Internal
s' —F’s with Infraspinatus (and posterior Siupra) Impingement” -
—Posterior Superior Labrum Torn this is the one
—Cystic Change in Greater Tuberosity they are talking
Internal about

Anterior Superior

\ —Associated with Sub Scapular damage


(Maybe the cause rather than the result)
—Anterior Superior Labrum Torn

High Yield Trivia Points on Im pingem ent

Subacrom ial Im p ingem ent - m ost com m on


form , resulting from attrition o f the D am ages S up rasp in atu s Tendon.
coracoacrom ial arch.

D am ages Su b scap u laris (rem em ber the


Subcoracoid Im p ingem ent - Lesser
c o ra co id is anterior - a n d so is the
tuberosity and coracoid do the pinching.
subscapularis).

Posterior Sup erior “ Internal'” Im p ingem ent


-A th le te s w ho m ake overhead m ovem ents. D am ages Infraspinatus (and posterior
G reater tuberosity and posterior inferior fibers o f the supraspinatus).
labrum do the pinching.

209
Rotator C u ff T e a rs:

People talk about these tears as


either “ Bursal Sided” (m eaning the
top part), or “ A rticular
Sided” (m eaning the undersurface).

A tear o f the articular surface is


m ore com m on (3x m ore) than the
bursal surface. The underlying
m echanism is usually degenerative,
although traum a can certainly play a
role.

The m ost com m on o f the four m uscles to tear is the S u p rasp inatus - w ith m ost tears
occurring at the " c r itic a lz o n e ” - 1-2 cm from the tendon footprint. This relatively
avascular “critical zone” is also the m ost com m on location for C alcium H ydroxyapatite
(H A D D ) - or “calcific tendinitis.” The Teres M inor is the least com m on to tear.

A partial tear that is > 50% often results in a surgical intervention.

“M assive rotator c u ff tear ” - refers to at least 2 out o f the 4 rotator c u ff m uscles.

A final general piece o f trivia is that a tear o f the fibrous rotator c u ff interval (junction
betw een anterior fibers o f the Supraspinatus and superior fibers o f the subscapularis), is still
considered a rotator c u ff tear.

H ow do y o u know i t ’s a f u l l thickness tear? You will have high T2 signal in the expected
location o f the tendon. On T1 you will have G ad in the bursa.

Full
Thickness
Tear

- With Gad
crossing over
the cuff into
the bursa.

210
A d h e s iv e C a p s u litis “F ro zen S h o u ld e r”

An inflam m atory condition characterized by a global decrease in m otion. You can have
prim ary types, but a m ultiple choice key w ould be a history o f traum a or surgery.

It m ost com m only effects the rotator c u ff interval - and that is the m ost likely spot they
will show it. The classic look is a T1 (or non-fat sat T2) in the sagittal plane show ing loss
o f f a t in the rotator c a ff interval (the spot w ith the biceps tendon - betw een the Supra S,
and the Sub Scap).

G rey Sm udgy Shit Instead o f C lean Fat in the R otator C u ff Interval

BUZZW ORD- ■‘D ecreased G lenohum eral Volume ” - w ith injection

' ** R em em ber in M ulti-D irectional Instability the volum e w as increased.

B U Z Z W O W ^" "T hickened in ferio r a n d P o sterio r C apsule ”

"E nhancem ent o f the R otator C u ff In terva l - Post gad

211
In ju ry to th e L ab ru m :

S L A P : Labral tears favor the superior m argin and track anterior to posterior. As this tear
involves the labrum at the insertion o f the long head o f the biceps , injury to this tendon is
associated and part o f the grading system (type 4).

Things to know about SL A P tears:

W hen the SLA P extends into the biceps anchor (type 4), the surgical m anagem ent
changes from a debridem ent to a debridem ent + biceps tenodesis.

The m echanism is usually an over-head m ovem ent (classic = sw im m er)

• People over 40 usually have associated R otator C u ff Tears

• N O T a sso c ia te d w ith In s ta b ility (usually)

S L A P M im ic - T h e S u b la b ra l R ecess. This is essentially a norm al variant w here you have


incom plete attachm ent o f the labrum at 12 o'clock. The 12 o'clock position on the labrum
has the shittiest blood flow - that's w hy you see injury there and all these developm ent
variants.

Sublabral Recess SLAP

Follows Contour of Glenoid Extends Laterally


SMOOTH Margin Ratty Margin
Located at Biceps Anchor Located at Biceps Anchor & Posteriorly

212
Labral Tear M im ic - T he Sublabral Foram en
- This is an unattached (but present) portion o f
the labrum - located at the anterior-superior
labrum (1 o'clock to 3 o ’clock).

As a rule it should N O T extend below the


equator (3 o'clock position).

Labral Tear M im ic - T he B uford C om plex - A com m only tested (and not infrequently
seen) variant is the B uford C om plex. It’s present in about 1% o f the general population. This
consists o f an absent anterior/su p erior labrum (1 o'clock to 3 o ’clock), along w ith a
thickened m iddle glen ohum eral ligam ent.

Buford Complex:
- Thick Middle GH Ligament
- Absent Anterior Superior Labrum

213
B a n k a r t L e s io n s :

T here is an alphabet soup o f B ankart (anterior dislocation) related injuries.

f *
\ ji

> > v > ^


G LA D Perthes A LPSA Bankart Bankart
(Cartilaginous) (Osseous)

G L A D = G lenolabral A rticular D isruption. It’s the m ost m ild version, and it’s basically a
superficial anterior inferior labral tear w ith associated articular cartilage dam age
(“ im paction injury w ith cartilage defect” ). N ot typically seen in patients w ith underlying
laxity. It’s com m on in sports. No instability (a re n 'ty o u G LA D there is no instability)

Memory Aid:
-The detached
Perthes = D etachm ent o f the anteroinferior labrum
labrum sorta
(3-6 o'clock) w ith m edially stripped looks like a
but intact p e rio steu m . ...------ —J P

A L P SA = A nterior Labral Periosteal Sleeve A vulsion. M edially displaced labroligam entous


com plex w ith absence o f the labrum on the glenoid rim . Intact p erio steu m . It scars dow n to
glenoid.

True Bankart: Can be cartilaginous or osseous. The p erio steu m is disru p ted . T here is
often an associated Hill S a c h ’s fracture.

GLAD Perthes ALPSA True Bankart

Superficial partial Avulsed anterior Similar to perthes but Tom labrum


labral injury with labrum (only with “bunched up”
cartilage defect minimally displaced). medially displaced
inferior GH complex
Inferior GH complex
still attached to
periosteum

No instability Intact Periosteum Intact Periosteum Periosteum Disrupted


(lifted up)

214
P o s te r io r G le n o h u m e r a l In s t a b ilit y

As I m entioned previously, anterior shoulder dislocations are w ay m ore com m on than posterior
shoulder dislocations. T herefore the B ankart, A L PSA , Perthes, e tc ... are the ones you typically
think o f as the stigm ata o f prior dislocation.

However, all that shit can happen in reverse w ith a posterior dislocation.

R everse O sseous Bankart: PO LPSA :

T his is the bizarro


version o f the
A L PSA , w here the
A fracture o f the
posterior labrum and
posterior inferior
the posterior
rim o f the
scapular periosteum ■•"'"x /
glenoid. /
(still intact) are *

« A stripped from the


glenoid resulting in
a recess that
com m unicates w ith
the jo in t space.
“ Bennett L esion ”

An extra-articular curvilinear
calcification - associated w ith posterior “ K im ’s L esion”
labral tears tm avbe the PO LPSA ).
An incom pletely
It’s related to injury o f the posterior avulsed / flattened / ^ * 0*
band o f the inferior glenohum eral m ashed posterior-
ligam ent. inferior labrum .

A key (testable)
\ . \ point is the glenoid
few cartilage and
p osterior labrum
' \ J i relationship is
preserved.

215
HAG L:

A non-B ankart lesion that is frequently tested is the H A G L (H um eral avulsion


glenohum eral ligam ent). This is an avulsion o f the inferior glen ohum eral ligam ent, and
is m ost often the result o f an anterior shoulder dislocation (just like all the above bankarts).
The “J Sign” occurs w hen the norm al U -shaped inferior glenohum eral recess is retracted
aw ay from the hum erus, appearing as a J.

Axial MR - Showing the IGHL


Tom at its Humeral Attachment

S u b lu x a tio n o f t h e B ic e p s T e n d o n : T he subscapularis attaches to the lesser


tuberosity. It sends a few fibers across the bicipital groove to the greater tuberosity , w hich
is called the “transverse ligam ent” . A tear o f the subscapularis opens these fibers up and
allow s the biceps to dislocate (usually m edial). Sub scap ularis Tear = M edial D islocation
o f the L ong H ead o f the Biceps Tendon.

Sub Scap Tendon - Forming Occurs with a Tear of the Subscapularis


portions of the "Transverse
Ligament" that holds the biceps
tendon in the groove

216
Nerve Entrapment: High Yield Trivia:

S u p r a s c a p u la r N o tc h v s S p in o g le n o id N o tc h : A cyst at the level o f the


suprascapular notch w ill affect the supraspinatus and the infraspinatus. At the level o f the
spinoglenoid notch, it will only affect the infraspinatus.

Suprascapular
Notch

Spinoglenoid
Notch

Cyst in the spinoglenoid notch causing


fatty atrophy of the Infraspinatous

***Dotted Line = Suprascapular

Q u a d r ila te r a l S p a c e S y n d ro m e : C om pression o f the A xillary N erve in the


Q uadrilateral Space (usually from fibrotic bands). They will likely show this w ith atrophy
o f the teres m inor. A nother classic question is to nam e the borders o f the quadrilateral
space: Teres M inor A bove, Teres M ajor Below, H um eral neck lateral, and T riceps m edial.

Humerus

Quadrilateral Space Syndrome


-Atrophy of Teres Minor

P a r s o n a g e -T u r n e r S y n d ro m e : This is an idiopathic involvem ent o f the brachial


plexus. Think about this w hen you see m uscles affected by pathology in tw o or m ore nerve
distributions (suprascapular and axillary etc..).

217
Site-Specific Entities - Hip / Femur / Sacrum

F e m o r a l S h a ft F r a c tu r e s :

• On the inside (m edial) is the classic


stress fracture location

* On the outside (lateral) is the classic


b isphosphon ate related fx location.
As show n in the im age, you see cortical
thickening (w hite arrow ) along the lateral
fem ur, eventually progressing into a
fracture.

H ip F r a c t u r e / D is lo c a tio n :

You see these w ith dash board injuries. The


posterior dislocation (alm ost alw ays associated
with a fracture as it’s driven backw ards) is much
m ore com m on than the anterior dislocation.

BUZXWORD:
C '- 'n jO "Foot in internal rotation ”

5 *°
A nterior Colum n vs P osterior Column - the
acetabulum is supported by two colum ns o f bone
that m erge together to form an “ inverted Y ”

Iliopectineal Line = A nterior


Ilioischial Line = Posterior (rem em ber you sit on your ischium )

The both colum n fracture by definition divides the ilium proxim al to the hip joint, so you have no
articular surface o f the hip attached to the axial skeleton (th at’s a problem ).

Corona M ortis: The anastom osis o f the inferior epigastric and obturator vessels som etim es rides
on the superior pubic ram us. During a lateral dissection - som etim es used to repair a hip fracture
- this can be injured. 1 talk about this m ore in the vascular chapter.

Hip Fracture Leading to AVN: The location o f the fracture m ay predispose to AVN. It’s
im portant to rem em ber that, since the fem oral head gets vascular flow from the circum flex
fem orals, a displaced intracapsular fracture could disrupt this blood supply - leading to
AVN. Testable Point: Degree o f fracture displacem ent corresponds with risk o f AVN.

218
A v u ls io n In ju ry :

This is seen m ore in kids than adults. A dult bones are stronger than their tendons. In kids it’s the
other w ay around. O ne pearl is that if you see an isolated “ avu lsion ” o f the lesser trochan ter
in a seem ingly m ild trau m a / injury in an adult - query a pathologic fracture. Now , to
discuss w hat 1 believe to be one o f the highest yield topics in M SK , “w here d id the avulsion
com e fro m ? ”

The easiest w ay to show this is a plain film pelvis (or M RI) w ith a tug/avulsion injury to one o f
the m uscular attachm ent sites. The question will m ost likely be "w hat attaches there? ” or
“w hich m uscle g o t avulsed? ”

llliac Crest
Abdominal Muscles

ASIS
Sartorius
Tensor Fascia Lata

AIIS
Rectus Femoris

Greater Trochanter
Gluteal Muscles

Symphysis
ADDuctor Group

Ischial Tuberosity
Hamstrings
Lesser Trochanter
lllioDsoas

219
S n a p p in g H ip S y n d ro m e :

The clinical sensation o f “ snapping” or “ clicking” w ith hip flexion and extension.
The key point is that it is clinical. This is the w ay people w ork this thing up:

C linical Eval for the “E x te r n a l T y p e ”


(IT band “ snapping” over the G reater T rochanter)

This is to evaluate for the “In tra -A rtic u la r T yp e”


Looking for Hip D egen / L oose Bodies, Etc

I IF NO Degen
X N EX T S T E P = Ultrasound

This is to look for the “In te r n a l T y p e ”


Look for D ynam ic “ S napping” o f the
Iliopsoas over the Uiopectineal em inence or fem oral head
*This has to he show n with a C IN E - because the fin d in g is a dynam ic m oving o f a tendon
I f y o u see a hip ultra so u n d f o r sn a pping - this is w hat they are g o in g f o r

I IF US Negative
X N EXT S T E P = MRI Arthrogram

This is to evaluate for the “ In tra -A rticu lar Type" ... A gain
T his tim e looking for Labral Tears

Trivia:

• “ Snapping H ip” is a “clinical sensation” - they have to tell you that patient feels “ snapping”
• The 3 Types, and the W ork-U p A lgorithm Above:
• Types:
• External (m ost com m on) = Iliotibial Band over G reater Trochanter
• Internal = Iliopsoas over Uiopectineal em inence or fem oral head
• Intra-A rticular = Labral tears / jo in t bodies

220
IT B a n d S y n d ro m e

This is a repetitive stress syndrome seen most classically in runners.


The key finding is fluid on both sides of the IT band, extending
posterior and lateral.

Fluid in the joint does not exclude the diagnosis, but for the purpose of
multiple choice if you see fluid around the band and none in the joint
you can be fairly certain this is the pathology the question writer is
after.

H ip L a b ru m

This is com plicated and I'm not going to go into depth talking about all the little bum ps
and variants. I w ould ju st know a few things:

1: A nterior-Superior Tears (w hite


arrow s) are by far the m ost com m on.

2: Paralabral C ysts (black arrow ) are


associated w ith tears and likely a hint
that a tear is present.

3: Just like a shoulder intra-articular Coronal View Sagittal View


contrast will increase your sensitivity. -Showing the classic anterior superior cleft o f a tear

Gamesmanship: A fluid signal “mass” with anterior to


the femur (adjacent to the psoas tendon) at the level o f
Ilio p s o a s B u rs a the ischial tuberosity is likely Iliopsoas Bursitis

• Largest bursa o f the entire body.

• C om m unicates with the jo in t in 15% o f the population

• Seen A nterior to the hip

• Trivia: The illiospsoas tendon runs anterior to the labrum


on axial and can m im ic a tear.

221
F e m o r o a c e t a b u la r Im p in g e m e n t (F A I): This is a syndrome o f painful hip movement.
It’s based on hip / femoral deformities, and honestly might be total BS. Supposedly it can lead to early
degenerative changes. There arc two described subtypes: (A) Cam and (B) Pincher (technically there is a
mixed type - but I anticipate multiple choice to make it more black and white).

CAM Type: This is an osseous “bump” along the femoral head-neck junction.

M emory A id

I remember that the femoral one (cam-type)


is more common in men because the femoral

c a lc ® ] head kinda looks like a penis.

Normal Cam-Type Be honest, you were thinking that too.

Pincer Type: Whereas the CAM type is a deformity o f the femur, the pincer type represents a deformity of
the acetabulum. Whereas the CAM type is more common in a young athletic male, the pincer is more
common in a middle aged woman (insert sexist jo ke here).

The most classic way to show or ask this is the so-called


“cross over sign”, where the acetabulum is malformed -
causing the posterior lip to “Cross over” the anterior lip. A
Key point is that the coccyx needs to be centered at the
symphysis pubis to even evaluate this (rotation fucks things
up).

The other associated finding) s) of the pincer subtype worth


knowing are the acetabular over coverage buzzwords
(Coxa Profunda and Protrusio), and the Ischial Spine Sign: C ro ss-
Over

Normal Coxa Profunda Acetabular Protrusion Prominent


(Acetabulum projects medial to (Femur projects medial to the Ischial Spine
the ilioischial line) ilioischial line)

C lassic FAI A ssociation: Os A c e ta b u li


This is an unfused secondary >=>
• O s A cetabuli (40% ) ossification center. It’s actually normal
in kids (should fuse by adult hood).
• Labral Tears
It has several testable associations
• Early A rthritis including FAI and Labral Tears

222
Total Hip Arthroplasty
Bone Remodeling / Stress Shielding: Proximal stress shielding -
greater trochanter
The stress is transferred through the bone resorption
metallic stem, so the bone around it is not
loaded. Orthopods call this “Wolff’s Law’
- where the unloaded bone just gets
resorbed.
Calcar
Happens more with uncemented Resoption
arthroplasty. To some degree this is a
normal finding - but when advanced can
predispose to fracture.

Potentially Asymptomatic
Complications of Hip
Distal stress loading: cortical
Arthroplasty:
thickening & pedestral (around
• Stress Shielding the bottom)
• Aggressive Granulomatosis “Zone 4“

Heterotopic Ossifications: This is very common (15-50%). It’s usually asymptomatic. The trivia
regarding multiple choice tests is that “hip stiffness” is the most common complaint.
Also in Ank Spon patients, because they are so prone to heterotopic ossifications, they sometimes
give them low dose prophylactic radiation prior to THA.

Aseptic Loosening: This is the most common indication for revision. The criteria on x-ray is > 2
mm at the interface (suggestive). If you see migration of the component, you can call it
(migration includes varus tilting o f the femora! stem).

Subsidence: Basically an arthroplasty


that is sliding downward. This is a
described reason for early failure of
THA. You see this most often in collared
stem
arthroplasty implants without a
collar. baseline

Greater than 1 cm along the


Distance from the tip o f the greater
femoral com ponent, or
trochanter to the superolateral
progression after 2 years are shoulder o f the stem is increased
indications of loosening. relative to baseline.

Wear Patterns: It is normal to have a little bit of thinning in the area of


weight bearing - this is called “Creep.” It is not normal to see wear
along the superior lateral aspect.

• Wear = Pathologic

• Creep = Normal

223
Total Hip Arthroplasty Continued -
Particle Disease (Aggressive Granulomatosis): Any component o f the device that sheds will
cause an inflammatory response. The more wear that occurs the more particles — wear is the
primary underlying factor. Macrophages will try and eat the particles and spew enzymes all over
the place. This process can cause progressive lytic focal regions around the replacement and joint
effusions.

Things to know about particle disease (in THA):

Most commonly seen in non-cementcd hips


Tends to occur 1-5 years after surgery
— “late complication”
X-ray shows “smooth” endosteal scalloping
(distinguishes from infection)
Aseptic - ESR & CRP will be normal
Produces no secondary bone response
— no sclerosis
Can be seen around screw holes (particles are
transmitted around screws)

Wear Particle Disease Osteolysis

Wear = Arrow

S a c ru m :
You can get fractures o f the sacrum in the setting o f traum a, but if you get show n or asked
anything about the sacrum it’s going to be either (a) SI degenerative change - discussed later,
(b) unilateral SI infection, (c) a chordom a - discussed later, (d) sacral agenesis, or (e) an
insufficiency fracture. O ut o f these 5 things, the insufficiency fracture is probably the m ost
likely.

Sacral Insufficiency Fracture - The m ost


com m on cause is postm enopausal osteoporosis.
You can also see this in patients w ith renal failure,
patients w ith RA, pelvic radiation, m echanical
changes after hip arthroplasty, or extended
steroid use. They are often (usually) occult on
plain film s.

They will have to show this either w ith a bone V


scan, or M RI. The classic “H onda S ign ” from the
“H ” -shaped appearance is probably the m ost likely
. * . * , H o n d a S io n
presentation on a m ult,pie choice test. Sacra| |nsu„ iciency Fx

224
Site-Specific Entities - Knee / Tibia / Fibula

S e g o n d F r a c tu r e :
This is a fracture o f the Lateral Tibial Plateau
(icommon distractor is m edial tibia). The thing to
know is that it is associated with ACL tear (75% ),
and occurs with internal rotation.

R e v e rs e S e g o n d F r a c tu r e :
This is a fracture o f the M edial Tibial Plateau. The
thing to know is that it is associated with a PCL
tear, and occurs with external rotation. There is J Arcuate
-PCL
also an associated medial meniscus injury. Reverse Segond
Fx
A r c u a t e S ig n :
This is an avulsion o f proximal fibula (insertion o f
arcuate ligament complex). The thing to know is
that 90% are associated with cruciate ligam ent
injury (usually PCL)

D e e p In t e r c o n d y la r N o tc h S ig n :
Deep Lateral
This is a depression o f the lateral femoral condyle Femoral Notch
(terminal sulcus) that occurs secondary to an impaction - ACL Insufficiency
injury. This is associated with ACL tears. - Acute ACL Tear

A n a to m y B litz :

Ligam ents: ACL & PCL are extrasynovial and intra-


articular. The synovium folds around the
• ACL: Composed of two bundles (anteromedial & ligaments. This is why a tom ACL won’t
posterolateral). The tibial attachment is thicker then heal on its own (usually).
the femoral attachment. Both the ACL and PCL are
intra-articular and extrasynovial. The ligament can be tom even if the
synovium is intact - this is why the
• PCL: The strongest ligament in the knee (you don’t “taunt” angle of the ligament is a key
feature o f integrity - more on that later
want a posterior dislocation of your knee resulting
in dissection of your popliteal artery).

• MCL: The MCL fibers arc laced into the joint capsule
at the level o f the joint, with connection to the medial
meniscus. Unlike the ACL and PCL, the MCL is an
0
extra-articular structure.
Biceps
• Conjoint Tendon: Formed by the biceps femoris IT Band
Femoris
inserting
tendon and the LCL. LCL
on Gerdy’s
Tubercle Proper

225
M a g ic A n g le P h e n o m e n o n

The PCL and Patellar tendon m ay have foci o f interm ediate signal intensity on sagittal
im ages w ith short echo tim e (TE) sequences w here the tendon form s an angle o f 55 degrees
w ith the m ain m agnetic field (m a g ic a n g le p h e n o m e n o n ).

This will N O T be seen on T2 sequences (w ith long TE). This phenom enon is reduced at
higher field strengths due to greater shortening o f T2 relaxation tim es.

Magic Angle: You sec it on short TE sequences (T l, PD, GRE). It goes away on T2

A C L T e a r : A C L tears happen all the tim e, usually in


people w ho are stopping and pivoting.

• Associated with Segond Fracture (lateral tibial


plateau) and tibial spine avulsion
• ACL Angle lesser than Blum ensaat’s Line
• O ’donoghue’s Unhappy Triad: ACL Tear, MCL
Tear, M edial M eniscal Tear
• Classic K issing Contusion Pattern: The lateral
femoral condyle (sulcus terminals) bangs into the
posterior lateral tibial plateau. This is 95% specific
in adults.
• Anterior D raw er Sign = Ortho Physical Exam A C L Tear
Finding suggesting ACL Tear. -Kissing Contusion Pattern

A C L M u c o id D e g e n e r a tio n :

This can mimic acute or chronic partial


tear o f the ACL. There will be no
secondary signs o f injury (contusion
etc..). It predisposes to ACL ganglion
cysts, and they are usually seen
together. The T2/STIR buzzword is
“celery stalk” because o f the striated
look. T h e T l buzzword is
“drum stick” because it looks like a
drum stick. Mucoid Degeneration of the A C L
- “Drumstick / Celery Stick”
226
A C L R e p a ir:

ACL can be repaired with two primary methods. Method 1: Using the middle one-third of the
patellar tendon, with the patella bone plug attached to one end and tibial bone plug attached at the
other. Method 2: Using a graft made o f the semitendinosus or gracilis tendon, or both. The graft is
then attached with all sorts o f screws, bolts, etc... There is a lower reported morbidity related to
harvest site using this method.

G raft E valuation:

There are two tunnels (tibial and femoral) between which the graft
runs. Here are the testable pearls:

Tibial Tunnel: Should parallel the roof of the femoral intercondylar


notch. Too Steep = Impinged by femur on extension. Too Flat = Lax
& won’t provide stability. Too Far Anterior (“Intersection with
Blumensaat line”) = Can lead to pinching at the anterior inferior Normal ABnormal
, , r f .„ Tibial Tunnel Steep & Anterior to B
intercondylar root. Buzzword Root Impingement. ^ are pacjt

Femoral Tunnel: Supposedly the primary factor for maintaining length and tension during range of
motion. This is referred to as “maintained isometry.”

Femoral Tunnel = Maintains Isometry. Tibial Tunnel = Roof Impingement.

“Arthrofibrosis” Can be focal or diffuse


(focal is more common). The focal form
is the so called “Cyclops” lesion - so
named because o f its arthroscopic
appearance. It’s gonna be a low signal
mass-like scar in Hoffa’s fat pad. It’s bad
because it limits extension.

Buzzword “palpable audible clunk”

Seen around 16 weeks - it obviously


won’t occur immediately post op because
you have to build up your scar. C y clo p s Lesion - S ca r Associated with Ventral Graft

“Graft Tear ” - Usually Ortho can just pull on his fucking leg F em ur F em ur
is see if the graft is trashed (anterior drawer sign). For
imaging, the simple way to understand this: “flat angle = tear.
The ACL should parallel the roof o f the intercondylar notch.
If the angle becomes flat, a tear is likely.
Tibia
Trivia: The graft is most susceptible to tear in the remodeling
process (4-8 months post op). Normal Angle Flat

Trivia: Other signs o f graft tear: grossly high T2 signal (some is ok), fiber discontinuity, uncovering
of the posterior horn o f the lateral meniscus (secondary sign), anterior tibial translation (secondary
sign).

227
P o s te r io r L a t e r a l C o r n e r (P L C ): T he m ost com plicated anatom y in the entire body.
M y G od this posterior lateral corner! Just think about the LCL, the IT band, the biceps
fem oris, and the popliteus tendon. The m ost likely w ay to show this on a single im age
(m ultiple choice style) is edem a in the fibu lar head.

Who cares? M issed PLC injury is a very com m on cause o f A C L reconstruction failure.

P C L T e a r : The posterior collateral ligam ent is the strongest ligam ent in the knee. A tear
is actually uncom m on, it’s m ore likely to stretch and appear thickened ( > 7 m m ). PCL tears
should m ake you think about posterior dislocation as the m echanism o f injury..

N ext S te p /A s s o c ia tio n = If you see a PCL Tear - look at the popliteal flow void. If the knee
dislocated posterior, a dreaded consequence is vascular com prom ise. D epending on the w ording
o f the question they m ight need a ru n -o ff (w atch your back).

M e n is c a l A n a to m y :

The m eniscus is “ C Transverse Ligam ent



shaped”, thick along the
periphery and thin
centrally.

M edial m eniscus is
thicker posteriorly.

Lateral m eniscus has


equal thickness betw een
anterior and posterior
portion.

M eniscal H ealing

The Peripheral “ Red Z one” is vascular and


m ight heal.

The C entral “w hite zone” is avascular and will


not heal. The blood supply com es from the
geniculate arteries (w hich enter peripherally).

228
M e n is c a l T e a rs :
As stated, the peripheral meniscus (red zone) has better vasculature than the inner 2/3s (white zone) and
might heal on its own. In general, you can group tears based on their general direction (as seen on a
sagittal section MRI - i.e. the triangles and bowties) - as either vertical (top-to-bottom) or horizontal
(front-to-back). You can then sub-group them depending how they look on subsequent sections.

Radial Tear:
- Bad because they
cause “loss of hoop
strength.” Radial Tearing
- Can lead to extrusion, Cuts the Circular
Hoop Fibers that
early OA etc..
Hold the Meniscus
Together

Flap Tear (Parrot Beak):


- Radial Tear that Changes
Direction into the
longitudinal direction

Idealized Vertical Tear


Longitudinal Tear:
- Can be vertical or
horizontal (or mixed
oblique patterns)
- Defined by a long
extension in the axial
direction
- Vertical Types can flip
(bucket- handle)

Horizontal Cleavage Tear:


- Pure cleavage tears extend to
the apex
- Associated with Meniscal Cysts
- Most common in posterior horn
of the medial meniscus

R a d ia l T e a rs: T here are 3 classic Signs - tw o o f w hich are usually present.

Truncated Triangle Cleft - Most Reliable “G h o st” or Absent Triangle

229
Bucket Handle Tear:
Discoid Meniscus:
This is a tom meniscus (usually medial -
This is a normal variant of the lateral meniscus 80%) vertical longitudinal sub-type, that flips
that is prone to tear. It’s not C-shaped, but medially to lie anterior to the PCL.
instead shaped like a disc. In other words, it’s too
big (too many bowties!). Gamesmanship - Most likely shown as the
classic Aunt Minnie appearance o f a “double
Gamesmanship - “Pediatric Patient with Meniscal PCL.”
Tear”.

Trivia - There are three types, with the most rare


and most prone to injury being the Wrisberg
Variant.

Gamesmanship "Bow Ties” If shown on sagittal


they have to show you 3 or more “bow ties” /
double triangles.

Buckle Hand Tear


-Double PCL Sign

Gamesmanship - Can also be shown as “not


enough bowties,” the opposite o f the “too
many bowties” look o f a discoid meniscus.

Only 1
bowtie -
Normal Meniscus will have 2 bowtie shapes in the instead o f
sagittal plane - assuming 3mm slices with 1mm the
normal 2.
gap-

Discoid Meniscus will have 3 or more bowties The


m iddle o f
the second
bowtie is
flipped
Gamesmanship: If medially.
shown on coronal they
need to show you a
meniscus stretching
into the notch. Trivia: The appearance o f a double PCL can
only occur in the setting of an intact ACL,
otherwise it w on’t flip that way. Just know it
sorta indirectly proves the ACL is intact (I
can just see some knucklehead asking that).

D iscoid M eniscus
-Extending into Notch

230
M e n is c a l C y s ts :
B akers Cyst:

M ost often seen near the lateral m eniscus and are often O ccurs betw een the
associated w ith horizontal cleavage tea rs. sem im em branosus and the
M ED IA L head o f the gastroc

M e n is c o c a p s u la r S e p a r a tio n :

This is a rare (in real life - m aybe not on exam s) injury. T he idea is that the deepest layer o f the
M C L com plex (capsular ligam ent) is relatively w eak and is the first to tear. This deep tearing
m ay result in the separation o f the m eniscus and the M CL. I’ve never seen it occur in isolation
(theoretically it can). T he im portant things to rem em ber are probably (1) it happens m ore with
proxim al M CL tears, and (2) this is a serious injury — requires im m obilization or surgery.

M e n is c a l O s s ic le :

This is a focal ossification o f the posterior horn o f the m edial m eniscus, that can be
secondary to traum a or sim ply developm ental. T hey are often associated w ith radial root
tears.

M e n is c o fe m o r a l L ig a m e n ts :

There are 2 (W risberg, H um phry) w hich can be m im ics o f m eniscal tears. W risberg is in
the back ( “hum ping H u m p h r y ”). You could also rem em ber that “ H” com es before “ W ” in
the alphabet.

M e n is c a l F lo u n c e :

This an uncom m on finding o f a “ ru ffle d ”


appearance o f the m eniscus that m im ics a tear.

It’s N O T associated w ith an increased incidence o f


tear - but can look like one, if you d o n ’t have any
idea w hat one looks like.

“Flounce”
Ruffled = Not a Tear

231
P a te lla D islo catio n :

D islocation o f the patella is usually lateral


because o f the shape o f the patella and femur.
The contusion pattern is classic.

• It’s Lateral
• C ontusion Pattern - Classic
•A ssociated tear o f the M PFL
(m edial patellar fem oral ligam ent)
•A ssociated w ith “Trochlear
D ysplasia ” - the trochlea is too flat.

Patellar Dislocation
Classic Contusion Pattern (arrows)
P a te lla A lt a / B a ja :

The patella will m ove up or dow n in certain


traum atic situations. If the quadricep tendon
tears you w ill get unopposed pull from the %

t
patellar tendon resulting in a low patella
(Baja). If the patella tendon tears you will get
1
unopposed quadriceps tendon pull resulting in
a high patella (Alta).
I
The “ classic” association w ith patellar tendon
tear (A lta) is S L E , (also can see in elderly,
traum a, athletics, or RA). P a t e l l a A lt a P a t e lla B a ja

'B ilateral p a tella r rupture ” is a buzzw ord for chronic steroids.

Normal
Comparison

Prepatellar Fat Im pingem ent Jum pers Knee


Bursitis Syndrom e
High T2 Signal +
Fluid superficial High T2 Signal in Hoffa’s Thickening o f the
to the Patella Fat Inferior to the Patella Inferior Patella

232
T ib ia l P la te a u F r a c tu r e : This injury m ost com m only occurs from axial loading (falling
and landing on a straight leg). The lateral plateau is w ay m ore com m on than the m edial. If
you see m edial, it’s usually w ith lateral. Som e dude nam ed Schatzker m anaged to get the
classification system nam ed after him , o f w hich type 2 is the m ost com m on (split and
depressed lateral plateau).

P ilo n F r a c t u r e ( T ib ia l p la fo n d f r a c t u r e ) : This injury also m ost


com m only occurs from axial loading, w ith the talus being driven into the
tibial plafond. The fracture is characterized by com m inution and articular
im paction. A bout 75% o f the tim e you are going to have fracture o f the distal
fibula.

T ib ia l S h a ft F r a c tu r e : This is the m ost com m on long bone fracture. It w as also listed


as the m ost highly tested su b ject in orthopedic O IT E exam (with regard to traum a), over the
last 8 years. A pparently there are a bunch o f w ays to put a nail or plate in it. It d o e sn ’t seem
like it could be that high yield for the C O R E com pared to other fractures w ith French or Latin
sounding nam es. I w ill point out that the tibia is one o f the slow est healing bones in the body
(10 weeks).

T illa u x F r a c tu r e s : T his a Salter-H arris 3,


through the anterolateral aspect o f the distal tibial
epiphysis.

Trivia: This pattern requires an open physis along the


lateral distal tibia. This is w hy you see this fracture
pattern in the w indow betw een the start o f m edial \i
physis fusion and the com plete fusion o f the lateral
physis (lateral physis typically closes around 12-15).

Trivia: The distal tibial grow th plate closes from


m edial to lateral (m edial first).

T r ip la n e F r a c tu r e : This is a Salter-H arris


4, w ith a vertical com ponent through the
epiphysis, horizontal com ponent through the
physis, and oblique through the m etaphysis.

*The addition o f the fracture p la n e in the


p o sterio r distal tibial m etaphysis (coronal plane)
distinguishes this fro m the Tillaux.

233
M a is o n n e u v e F r a c tu r e :

This is an unstable fracture involving the m edial tibial m alleolus and/or disruption o f the
distal tib iofibu lar syndesm osis.

The m ost com m on w ay to show this is to first show you the ankle w ith the
w idened m ortis, and “next step? ” get you to ask for the proxim al fibula - w hich
D will show the fracture o f the proxim al fibu lar shaft.

This fracture pattern is unique as the forces begin distally in the tibiotalar jo in t and then ride
up the syndesm osis to the proxim al fibula.

Trivia: The fracture does not extend into the hindfoot.

W ide M edial M alleolu s Proxim al F ib ula F racture


(+/- M edial M alleolus Fracture) — From u p w a rd fo rce extension
— D istal Tibiofibular syndesm osis ( “the rippin a n d the tearin ”) via the syndesm osis
+/- D e lto id L ig Injury^

234
Site-Specific Entities - Ankle (the rest of it 1/ Foot

C a s a n o v a F r a c t u r e - I f you see bilateral calcaneal fractures, you should “n ext s te p ? ” look


at the spine (T 12-L 2) for a com pression or burst fracture. T hese tend to occur in axial loading
patterns (possibly from ju m p in g out a w indow to avoid an angry husband).

Trivia:
• Peroneal tendons can becom e entrapped w ith lateral calcaneal fractures.
• C alcaneal fractures are the m ost com m on (60% ) Tarsal Bone Fx
• Fractures o f the calcaneus are either extra-articular or intra-articular - depends on su b ta la r jo in t
involvem ent. Intra-articular fractures w ill have a fracture line through the “critica l angle o f
G issane ”

Bohler's A ngle - The line


draw n betw een the anterior
and posterior borders o f the
calcaneus on a lateral view.
An angle less than 20
degrees, is concerning for a
Bohler’s Angle (20-40) Critical Angle of Gissane
fracture.
“More Flat” (Less than 20) “More Flat” (More than 130)
Equals Calcaneal Fracture Equals Depression of the Posterior Facet

S tr e s s F r a c tu r e o f t h e 5 th M e t a t a r s a l:
This is considered a high risk fracture (hard to heal).

J o n e s F r a c tu r e : This is a fracture
at the base o f the fifth m etatarsal, 1.5cm
distal to the tuberosity. T hese are placed
in a non-w eight bearing cast (m ay
require internal fixation- because o f risk
o f non-union.

A v u ls io n F r a c t u r e o f t h e 5 th
M e t a t a r s a l:
This is m ore com m on than a jo n es
fracture. The classic history is a dancer. Avulsion Fx
It m ay be secon dary to tug from the
lateral cord o f the plantar Os Peroneum
aponeurosis or peroneus brevis (this is (within p e ro n e u s long us)
controversial).

235
P a in fu l O s P e ro n e u s S y n d r o m e (P O P S )

*Os Peroneus (accessory ossicle) is within the Peroneus LONGUS

•This ossicle is seen in about 10% o f gen pop

•Stress reaction and pain can progress to tendon disruption = POPS

M R Key Findings: Edema in the os peroneus just before the peroneus


longus tendon enters the cuboid tunnel

L is fr a n c In ju ry : This is the m ost com m on dislocation o f the


foot. The Lisfranc jo in t is the articulation o f the tarsals and
m etatarsal bases. This jo in t is recessed creating a “ keystone”
locking m echanism , and w ould m ake a good place to am putate if
you w ere a surgeon assisting in the N apoleonic invasion o f
Russia. The Lisfranc ligam ent connects the m edial cuneiform to
the 2nd m etatarsal base on the plantar aspect.

•C a n 't exclude it on a non-w eight bearing film


•A ssociated fractures are m ost com m on at the base o f the 2nd M T - "F leck Sign ”
•Fracture non-union and post traum atic arthritis are gonna occur i f you m iss it
(plus a law suit).

"F leck Sign ” - This is a sm all bony fragm ent A “fleck” o f bone near
in the Lisfranc Space (betw een 1st M T and 2nd the base of the 2nd MT
MT) - that is associated w ith an avulsion o f the can sometimes be the
only clue.
LF ligam ent.

Normal Homo-Lateral Divergent


M echanism =
E xtrem e Plantar Flexion + 3 Ligaments make up the
A xial Load complex between the
medial cuneiform and
2nd MT.

The plantar band is the


strongest 2nd MT

236
A natom ic Trivia Achilles Tendon
This is the largest tendon in the
body. It represents the fused
tendons o f the gastrocnemius and
the soleus muscles.
\ bunch of e ^ves Tibialis Anterior
tendons «» ° ne 8 I It does NOT have a tendon
a shit about sheath, so it cannot have a
tenosynovitis (fluid in the sheath).

Instead inflammatory change


around the tendon is referred to
as a “paratendinitis.”

Tibialis Posterior “Tom”


Flexor Digitorum Longus
Peroneus Tibial Nerve
Longus Flexor Hallucis Longus “Harry”
Peroneus
Plantaris
Brevis
Achilles

The M ythical M a s t e r K n o t off H e n r y - T his has a funny sounding nam e, therefore it’s high
yield. This is w here D ick (FD L ) crosses over H arry (FH L ) at the m edial ankle.

Harry (w hite) starts out lateral relative to Dick (black). They cross at the “m aster knot” and then
Harry (w hite) ends up m edial on it’s w ay to the big toe (H arry = H allucis).

What is the M aster K not o f H enry? It’s a “ H arry D ick”

237
L ig a m e n to u s In ju r y : The highest yield fact is that the anterior talofib u lar ligam ent
is the w eakest ligam ent and the m ost frequ en tly injured (usually from inversion).

P o s te r io r T ib ia l T e n d o n In ju r y I D y s fu n c tio n : This results in a progressive


flat foot deform ity, as the PTT is the prim ary stabilizer o f the longitudinal arch. W hen
chronic, the tear is m ost com m on behind the m edial m alleolus (this is w here the m ost
friction is). W hen acute, the tear is m ost com m on at the insertion into the navicular bone.
A cute Flat Arch should m ake you think o f PTT tear.

You will also have a hindfoot valgus deform ity (from unopposed
peroneal brevis action). The other point o f trivia to know is that
the spring ligam ent is a secondary supporter o f the arch (it holds
up the talar head), and it will thicken and degenerate w ithout the
help o f the PTT. D o n ’t get it tw isted though, the spring ligam ent is
very thick and strong and alm ost never ruptures in a foot/ankle
traum a.
Hindfoot Valgus
“Too M any Toes”
I Say A cute F lat Foot, You Say P osterior Tibial Tendon In ju ry

S in u s T a r s i S y n d ro m e : *N ever m ake this dia g n o sis in the se ttin g o f a cu te traum a

The space betw een the lateral talus and calcaneus. The sinus tarsi is not ju st a jo in t space.
It is an im portant source o f proprioception and balance. Fucking it up has consequences (if
your goal is to m ake prim a ballerina assoluta).

The “ syndrom e” is caused by hem orrhage or inflam m ation o f the synovial recess w ith or
w ithout tears o f the associated ligam ents (talocalcaneal ligam ents, inferior extensor
retinaculum ). T here are associations w ith rheum atologic disorders and abnorm al loading
(flat foot in the setting o f a posterior tibial tendon tear).

MRI find ing is obliteration o f fat in the sinus tarsi space, and replacem ent w ith scar.

Loss of Normal T1 Bright Fat (arrow) Normal For Comparison

238
P la n ta r F a s c iitis :

This is an inflammation o f the fascia secondary to either repetitive trauma


(overuse via endless rounding on fat diabetic, smokers as a m edicine intern), B uzzw ord is
abnormal m echanics (pes cavus, etc), or arthritis (Reiters, etc...). "'most severe
in the
The pain is localized to the origin o f the plantar fascia, and worsened by
dorsiflexion o f the toes. This is usually a clinical diagnosis. m orning."1

Plain film might show heel spurs (which are not


specific), but could be a hint. A bone scan may
As a rapid anatomy review, the
show increased tracer in the region o f the
plantar fascia consists o f 3 bands with
calcaneus (from periosteal inflammation).
the central / lateral part normally
thicker than the medial part the
thinnest.
MRI may show:
Coronal T l diagram through the heel
a thickened fascia
(> 4 m m ), most
often the central
band

with increased T2
signal, most
significant near
its insertion at the
heel.

CLASSIC - FLAT FOOT PROGRESSION

P osterior Tibial
Show up in the
Tendon G oes Out
ER at 3 am
because y o u r
You start
f e e t hurt a n d
w alking like
Spring Ligam ent O ut y o u n e e d an
an idiot, ■ ^ Plantar . ^
excuse to not
h eel strikin g Fasciitis
go to w ork
over a n d
tom orrow
Sinus Tarsi gets ja c k e d over again
(a n d y o u w ant
(including those little
a Sprite a n d a
proprioception nerves
C heeseburger)
that are in it)

239
S p lit P e ro n e u s B re v is :

You can see longitudinal splits in the peroneus in


people w ith inversion injuries. The history is
usually “ chronic ankle pain” .

The tendon will be C shaped or b oom erang


shaped w ith central thinning and partial
envelopm ent o f the peroneus longus.
A lternatively, there m ay be 3 instead o f 2 tendons.
The tear occurs at the lateral m alleolus.

T here is a strong (80% ) association w ith lateral Split Peroneus Brevis


ligam ent injury. -B oom erang

A n t e r o la t e r a l Im p in g e m e n t S y n d ro m e :
Injury to the anterior talofibular ligam ents and tibiofibular ligam ents (usually from an
inversion injury) can cause lateral instability, and chronic synovial inflam m ation.

You can eventually produce a “m ass” o f hypertrophic synovial tissue in the lateral gutter.

The M RI find ing is a “ m eniscoid m ass” in the lateral gutter o f the ankle, w hich is a
balled up scar (T1 and T2 dark).

T a r s a l T u n n e l S y n d ro m e :

Pain in the distribution o f the posterior tibial


nerve (first 3 toes) from com pression as it
passes through the tarsal tunnel (behind the
m edial m alleolus).

It’s usually unilateral (unlike carpal tunnel Tarsal tunnel is a covered by the flexor
retinaculum (arrows) and includes tom, dick,
w hich is usually bilateral), unusually harry the posterior tibial artery and nerve.
“ idiopathic” although pes planus (hindfoot
valgus) can predispose by tightening the You can see atrophy o f multiple foot muscles
retinaculum . (not just minimi as seen with “Baxter”).

H aving said that, any m ass lesion (ganglion cysts, neurogenic tum ors, varicosities, lipom as,
severe tenosynovitis, and accessory m uscles) can cause com pression o f the nerve in the
tunnel.

2 40
M o r to n ’s N e u ro m a : Soft tissue mass (tear drop shaped) shown between the 3rd and 4th
metatarsal heads (third intermetatarsal space) is most likely a M orton’s Neuroma (especially on
multiple choice tests). The proposed pathology results from compression / entrapmento f the plantar
digital nerve in this location by the intermetatarsal ligament. Over time this results inthickening and
development o f perineural fibrosis.

“Mulder's S ign” - is a physical exam (a sonographic sign) where you squeeze the patients
foot and reproduce the pain (or see the scar pop out under ultrasound).

Trivia: Morton’s Neuroma is NOT a Neuroma (a tumor).


It’s a scar.

Classic Look: It is a scar, so it’s gonna be dark on T 1 and T2


(usually). It is tear drop shaped and projects downward.
Mortons: T1 Dark Below the
People make a big deal about this thing staying below the Plantar Ligament
plantar ligament.

The reason is that your primary differential is in term eta ta rsa l b u rsitis - which will extend above
the transverse ligament, be fluid signal, and have a more cystic look. Small bursa in this location can be
normal as long as the stay smaller than 3mm.

Bursitis:
Bursitis: T2 Bright
Above the
Ligament
(dumbbell
shaped)

H a g lu n d ’s S y n d r o m e I D e fo r m ity

This is also called the “Mulholland deformity” for the


purpose o f fucking with you. Depending on what you
read there are either 3 or 4 classic features:

• Retro-Achilles bursitis, /
• Retrocalcaneal bursitis,
• Thickening o f the distal Achilles tendon
(insertional portion)
• Calcaneal Bony Prominence “prominent posterior
superior os calcis”

The deformity is the “bump.”


The “syndrome” is the bursitis
and Achilles tendon thickening.
They call this thing the “pump Retrocalcaneal Bursitis (black arrow)
bumps,” because wearing high- Retro-Achilles / Adventitial Bursitis
heeled shoes is supposedly a (white arrow)
predisposing factor

241
Os T rig o n u m S y n d ro m e What is this
“Synchondrosis ” ?
The idea is that the Os Trigonum (accessory
ossicle) puts the smash on the FHL (“Harry”) This is a joint that has
during extreme ankle flexion — toe pointing essentially no
shit (“Pointe technique”) that ballet dancers movement and is
d o ... or other repetitive micro trauma. lined with cartilage.

Classic findings are going to be

(1) “Stenosing” tenosynovitis /


collection o f fluid around the FHL, and

(2) edema within the Os Trigonum and


across the synchondrosis between the
Os and the Posterior Talus.

B uzzw ord is
“B allet D an cer ' ’ Axial T2 - Fluid Around the FHL
Edema in the Os and Posterior

A c h ille s T e n d o n In ju ry :
A chilles Tendon Tear
X anthom a
Acute rupture is usually obvious with a fluid filled gap. (partial / sm all gap)
The gap size will determine treatment (big gaps need Thick Tendon Thick Tendon
surgery). The tear is usually 4 cm above the calcaneal ( > 7mm) (> 7mm)
insertion and the classic history is an unconditioned middle
aged fake athlete (“weekend warrior”) with acute pain and Unilateral Bilateral (usually)
loss o f the ability to plantar flex.
Associated with being Associated with
a fake athelete having very high
Without a large gap these things can be very hard to tell cholesterol
apart from a X anthom a (both can just look like a very
thick tendon). There are a few differences that can be used Step 1 Trivia:
Fluoroquinolone
to differentiate - per my chart.
antibiotics

P la n ta r is R u p tu re
(“Tennis Leg”): This is usually presented as the classic trick: “Achilles
tendon ruptured but can still plantar flex.” Remember not everyone
has this tendon (it’s absent in 10% of the population). The classic look
on MRI is focal fluid collection between the solcus and the medial
head of the gastrocnemius. There is an association with ACL tears.

A vu lsion s o f th e C a lca n ea l T uberosity:


This is sort of an Aunt Minnie with the back of the bone totally ripped
off via the Achilles. The classic association is diabetes. When you see
this you have to think diabetes. Calcaneal Avulsion

242
^ S E C T I O N 2:
O s t e o p o r o s i s , O s t e o p e n i a , & AVN p p

O s te o p e n ia : This ju st m eans increased lucency o f bones. A lthough this is m ost


com m only caused by osteoporosis, that is not alw ays the case.

O s te o m a la c ia : This is a soft bone from excessive uncalcified osteoid. T his is


typically related to vitam in D issues (either renal causes, liver causes, or other m isc
causes). It generally looks ju st like diffuse osteopenia. For the purpose o f m ultiple choice
you should think about 4 things: Ill-defined trabeculae, Ill-defined corticom edullary
junction, bow ing, and “ Loosers Z ones.”

Looser Zones: T hese things are w ide lucent


bands that transverse bone at right angles to
the cortex. These things can happen in lots \ /
:/ x
o f different locations - but the classic two
^SSi I 7 :
are the fe m o r a l neck and the p u b ic ram i.
Typically there is sclerosis surrounding the
.1
lucency. You should think tw o things: m
osteom alacia and rickets. Less com m on is /
01. The other piece o f trivia is to
understand they are a type o f insufficiency
fracture. Looser Zones
— Raise your suspicion for this shit when
you see (1) Symmetric Findings
(2) The “90 Degree” to the cortex line

O s te o p o ro s is : The idea is that you have low bone density. Bone density peaks around
30 and then decreases. It decreases faster in w om en during m enopause. The im aging
findings are a thin sharp cortex, prom inent trabecular bars, lucent m etaphyseal bands, and
spotty lucencies.

Causes: A ge is the big one. M edications (steroids, heparin, dilantin), E ndocrine issues
(cushings, hyperthryoidism ), A norexia, and O steogenesis Im perfecta.

C om plications: Fractures - M ost com m only o f the spine (2nd m ost com m on is the hip,
3rd m ost com m on is the w rist).

243
DEXA:
This is a bone m ineral density test and an excellent source o f m ultiple choice trivia.

G eneral Things to know about DEXA


* T score = D ensity relative to young adult
* T score defines osteopenia vs osteoporosis
* T score > -1.0 = N orm al, -1.0 to -2.5 = O steopenia, < -2.5 O steoporosis
* Z score = D ensity relative to age-m atched control “to Z a Z am e A ge”
* False negative / positive (see below )

F alse P ositive / N egative on D E XA: D EX A w orks by m easuring the density. A nything


that m akes that higher or low er than norm al can fool the m achine.

False Positive:
* A bsent N orm al Structures: Status post lam inectom y

False N egative:
* Including excessive O steophytes, derm al calcifications, or m etal
* Including too m uch o f the fem oral shaft w hen doing a hip - can elevate the
num ber as the shaft norm ally has denser bone.
* C om pression Fx in the area m easured

FRAX:
The Fracture R isk A ssessm ent T ool is a clinical risk tool used to predict fractures by
using clinical risk factors (age, sex, race, BM I, fam ily history, personal fracture history,
prior steroid use, w here the patient lives, e tc ...) w ith or w ithout fem oral neck bone
density. The fracture risk is calculated as a ten year fracture probability.

Trivia:

• FRA X calculates fracture risk at a 10 year probability


• FRA X adds “value” by helping to identify the subset o f osteopenic patients w ho are at a
higher risk for fracture - and m ight benefit from pharm acologic intervention
• FRA X is N O T supposed to be used in patient w ho have already been placed on m eds for
osteoporosis. The entire point o f the FR A X is to m ake big pharm a m ore m o n e y ... 1
m ean help identify those w ho w ould benefit m ost from pharm acologic intervention -
those already on m eds d o n ’t need identified.
• FR A X is applicable for m en and w om en
• FR A X is recom m ended to calculate 10 year fracture risk in patients w ith a T-Score
betw een -1 and -2.5.
• Som e guidelines suggest pharm acologic intervention for patients w ith a FRA X
calculated 10 year hip fracture risk o f > 3% or m ajor fracture risk o f > 20%

2 4 4
Reflex Sympathetic Dystrophy (RSD):

A lso called “C om plex R egional P ain Syndrom e” — w hich m akes it sound like som e
R heum atology Psycho-som atic bullshit (i.e. fibrom yalgia).

A lso called “Sudeck A trophy” - w hich m akes it sound serious - like som e incurable
neurodegenerative death sentence.

The classic clinical vignette is a history o f traum a or infection.

On plain film , it can cause severe osteopenia (like disuse osteopenia). Som e people say it looks
like unilateral R A , w ith preserved join t spaces. H and and shoulder are the m ost com m on sites
o f involvem ent.

It’s one o f the m any causes o f a 3 phase hot bone scan. In fact, intra-articular uptake o f tracer on
bone scan is typically seen (on m ultiple choice) in patients w ith RSD (secondary to the increased
vascularity o f the synovial m em brane), and this is som ew hat characteristic.

245
T ra n sie n t O steop o ro sis:

There are tw o types o f presentations.:

Transient osteoporosis o f the hip: For the purpose o f m ultiple choice tests, by far you
should expect to see the fem ale in the 3'" trim ester o f pregnancy w ith involvem ent o f
the left hip. H aving said that, it’s actually m ore com m on in m en in w hom it’s usually
bilateral. The jo in t space should rem ain norm al. It’s s e lf lim iting (hence the w ord
transient) and resolves in a few m onths. Plain film show s osteopenia, M R I sh o w s Edem a,
B one scan show s increased uptake focally.

R egion al m igratory osteoporosis - T his is an idiopathic disorder w hich has a very classic
history o f pain in a jo in t, w hich gets better and then show s up in another jo in t. It’s
associated w ith osteoporosis - w hich is also self-lim iting. It’s m ore com m on in men.

H ip E dem a - S trategy Session — This vs That - Transient O steoporosis vs A V N vs Fx

On radiograph, transient osteoporosis and AVN


look totally different. Transient Osteoporosis is
super lucent - so lucent that sometim es you can
barely see the femoral head. AVN on the other
hand, will have patchy areas o f sclerosis.

Osteoporosis
On MRI, the story is different. These things can look similar. They both have edema on STIR, and
they arc both are dark on T 1 . The difference is that AVN should be shown with a serpiginous dark
line (double line if you are lucky) - that represents infarct core. Joint effusions can be seen in both -
so this isn’t helpful.

Now - if these assholes want to take it to the twilight zone, they can add “ insufficiency fracture” to
the list o f distractors. This is really a dirty trick as both Transient Osteoporosis and AVN are
susceptible to this. The distinction is that this fracture line should be less serpiginous and instead
parallel the subchondral bone o f the femoral head.

T1

STIR

Normal Transient Insufficiency


O steoporosis Fracture
246
O steoporotic C o m p re ssio n Fractu re: Super C om m on. O n M R you w ant to see
a "b a n d lik e " fracture line - w hich is typically T l dark (T2 is m ore variable). The non­
deform ed portions o f the vertebral body should have no n n al signal.

N eo p lastic C o m p ressio n
What is this “Abnorm al M arrow S ig n a l” ?
Fractu re: M ost vertebral m ets
d o n ’t result in com pression fracture Normally (in an adult) the marrow of the spine is
until nearly the entire vertebral fatty - so it should be Tl bright. The internal control
is an adjacent nonnal disc (not a desiccated disk).
body is replaced w ith tumor. If you
see abnorm al m arrow signal (not If you see dark stuff - it might just be red marrow.
band like) w ith involvem ent o f the BUT if it is darker than the adjacent (nonnal) disc,
posterior m argin you should think you have to assume that it’s a bad thing.
about cancer.

D N ext Step ? - Look at the rest o f the spine - m ets are often m ultiple.

THIS VS THAT- O steoporotic F x vs N eoplastic Fx

If I w as going to show a N eoplastic vs O steoporotic Fracture case, this is how 1 w ould do it.
I w ould have 2 sequences, a T l and a STIR. T he STIR w ould be positive (bright on both).
The O steoporotic Fracture w ould have a T l dark line. T he N eoplastic Fracture w ould be
diffusely low T l signal and blobby - and I w ould stick a few lesions in other vertebral bodies.

* -%
Ic ^3 I CXI
*
T1 STIR T1 STIR T1 STIR

NORMAL Osteoporotic Fx Neoplastic Fx

247
OCDS/ OCLS
O s t e o c h o n d r it is D i s s e c a n s (OCD):
OCD Trivia:
The new term inology is actually to call these “O C L s” (the “L ” is for
Most common in
Lesion). This a spectrum o f aseptic separation o f an osteochondral
males under 18
fragm ent w hich can lead to gradual fragm entation o f the articular surface
Most common in
and secondary OA. M ost o f the tim e it is secondary to traum a, although it
the lateral aspect
could also be secondary to AVN. of the medial
femoral condyle
Where it happens: C lassic locations include the fem oral condyle (m ost
com m on site in the knee), patella, talus, and capitellum .

Staging: There is a staging system , w hich you probably need to know exists.

* Stage 1: Stable - C overed by intact cartilage, Intact w ith H ost Bone


* Stage 2: Stable on Probing, Partially not intact w ith host bone.
* Stage 3: U nstable on Probing, C om plete discontinuity o f lesion.
* Stage 4: D islocated fragm ent
Treatment / Who cares? If the fragm ent is unstable you can get secondary OA. You w ant to
look for high T2 signal und ercutting the fragm ent from the bone to call it unstable
(edem a can force a false positive). Thus, the absence o f high T2 signal at the bone fragm ent
interface is a good indicator o f osseous bridging and stability. G ranulation tissue at the
interface (w hich will enhance w ith G d), does not m ean it’s stable.

THIS v s THAT: C a p ite llu m L e s io n s

O steo ch o n d ritis
P ann er’s D isease Pseudo-Lesion
D issecans

C a p ite llu m o f th e d o m in a n t arm A ls o in th e c a p ite llu m o f


in th ro w e rs th ro w e rs

A n te rio r c o n v e x m a rg in o f th e
c a p ite llu m . U n s ta b le le s io n s are
E n tire C a p ite llu m
c h a ra c te riz e d b y h ig h s ig n a l
is a b n o rm a l in s ig n a l d flP P o s te rio r C a p ite llu m
flu id th a t e n c irc le s th e
(lo w T 1, h ig h T2)
o s te o c h o n d ra l
fra g m e n t o n T 2 W im a g e

C an lead to in tra -a rtic u la r L o o s e b o d y fo rm a tio n is A c o ro n a l im a g e th ro u g h th e


lo o se b o d ie s N O T seen (usually) p o s te rio r c a p ite llu m c a n m im ic
a d e fe c t. T h is o c c u rs b e c a u s e
th e m o s t p o s te rio r p o rtio n o f
5 to 10 y e a rs o ld th e c a p ite llu m h a s an a b ru p t
S lig h tly o ld e r p a tie n ts
“P e te r Pan w anted to stay s lo p e .
(1 2 -1 6 years)
young”

248
O steo ch o n d ro ses:
These are a group o f conditions (usually seen in childhood) that are characterized by
involvem ent o f the epiphysis, or apophysis w ith findings o f collapse, sclerosis, and
fragm entation - suggesting osteonecrosis.

Kohlers Tarsal Navicular Boys 4-6. Treatment is not surgical.


Second Metatarsal Adolescent Girls
Freiberg Infraction
Head - can lead to secondary OA
Calcaneal Some say this is a normal
Sever’s
Apophysis “growing pain”
Kid 5-10 “Thrower”;
Panner’s Capitellum
does not have loose bodies.
Perthes (LCP) Femoral Head White kid; 4-8.
Associated with negative ulnar
Kienbock Carpal Lunate
variance. Seen in adults 20-40.
Causes kyphosis. 3 adjacent levels
Scheuermann Thoracic Spine with wedging, plus a thoracic kyphosis
o f > 40 degrees (normal 20-40)
Adolescents (10-15) who jum p and
Osgood-Schlatter Disease (OSD) Tibial Tubercle kick. Need Fragmentation + Soft
Tissue Swelling.
Sinding-Larsen-Johansson (SLJ) Inferior Patella Adolescents (10-15) who jump.

Freiberg
- Flat 2nd
/ Metatarsal
Kienbock: head
Kohlers
- Lunate - Flat —
- Associated
with Negative \ Perthes Navicular
Severs
Ulnar - Femoral ^ Calcaneal
Variance Head Apophysis

S LJ:
- Inferior
Patella
Scheuerm ann
- Thoracic
Panner’s: Os 9 ° ° d - Wedging
- Capitellum \ Schlatter: - 3 Levels
- No loose - Tibial - Kyphosis
bodies Tubercle > 40 degrees
- Age 5-10 - Soft Tissue
(younger Swelling
than OCD)

249
S E C T I O N 3•

t U M ililllllill . . , .
IN F E C T IO N
, ,
v,j«
liiiiig n iin A

W ith regard to osteom yelitis, radiographs will be norm al for 7-10 days. Essentially,
osteom yelitis can have any appearance, occur in any location, and occur at any age. C hildren
have hem atogenous spread usually hitting the long bones (m etaphysis). A dults are m ore
likely to have direct spread (in diabetic).

I Say This, You Say THAT


H ow ever, you can
have hem atogenous
O steom yelitis in Spine = IV D rug User
spread in certain
O steom yelitis in Spine w ith K yphosis (G ibbus D eform ity) = TB
situations as well (IV
U nilateral SI jo in t = IV D rug U ser
D rugs).
Psoas M uscle A bscess = TB

G eneral Rule: Septic jo in ts m ore com m on in adults. O steom yelitis m ore com m on in kids.

Classic Look: H allm arks are destruction o f bone and periosteal new bone form ation.

V ocab:

S eq u estru m
SB Sequestru m = Piece o f necrotic bone
surround by granulation tissue

Involucrum = T hick sheath o f


Involu cru m
periosteal bone around sequestrum

C loaca = D efect in the periosteum


(bone skin) caused by infection
C loaca
Sinus Tract: A channel from the
bone to the skin (lined with
A b scess granulation tissue).

C hronic O steom yelitis: This is defined as osteom yelitis lasting longer than 6 w eeks.
Som e trivia w orth know ing:
• D raining sinus tracts are a risk factor for squam ous cell CA
• M ost specific sign o f active chronic osteom yelitis is the presence o f a
sequestrum (best show n w ith com puted tom ography)
• M RI diagnosis o f healed osteom yelitis is based on the return o f norm al fatty m arrow

250
A c u t e B a c t e r ia l O s t e o m y e lit is can be thought o f in three different categories:
1) hem atogenous seeding (m ost com m on in child), 2) contiguous spread, and 3) direct
inoculation o f the bone either from surgery or traum a.

A cute hem atogenous osteom yelitis has a predilection for the long bones o f the body,
specifically the m etaphysis, w hich has the best blood flow and allow s for spreading o f the
infection via small channels in the bone that lead to the subperiosteal space.

* A ge < 1 m onth = M ulti-centric involvem ent, often Infant


with join t involvem ent. Bone scan often negative
(75% ) at this age

* A ge < 18 m onths = Spread to epiphysis through Child


blood

* A ge 2-16 years = T rans-physeal vessels are closed


(prim ary focus is m etaphysis).

In the slightly older baby (<18 m onths) these vessels from the m etaphysis to the epiphysis
atrophy and the grow th plate stops the spread (although spread can still occur). T his creates
a “ septic tank” effect. This sam e thing happens w ith certain cancers (leukem ia); the garbage
gets stuck in the septic tank (m etaphysis). O nce the grow th plates fuse, this obstruction is
no longer present.

M R I fin d in g s o f osteom yelitis: Low signal in the bone m arrow on T l im aging adjacent to an
ulcer or cellulitis is diagnostic.

STIR - High Signal in


Bone Adjacent to Ulcer
(more sen sitive sign)

T1 - Corresponding Low Signal


(more specific sign)

Ulcer - N ext to “Foot Bone"

T he G host Sign: N europathic B one vs O steom yelitis in a N europathic B one

A bone that becomes a ghost (poor definition o f margins) on Tl imaging, but then re-appears
(more morphologically distinct) on T2, or after giving IV contrast, is more likely to have
osteomyelitis.

251
D is c itis / O s te o m y e litis :

M e c h a n is m (A d u lt)

(1) Seeding o f the (2) Eruption and (3) Eventual


Infection o f the disc and
vertebral endplate crossing into the involvement o f the
infection of the vertebral
(which is vascular) disc space adjacent vertebral body
body nearly always go
together.

The reason has to do


with the route o f seeding;
which typically involves

T y p ic a l L o o k & T r iv ia

Early: Later Plain Film: Later MR:


• Plain Film: Hard to see • Adjacent • T1 - Dark
• MR1: Paraspinal and irregular Marrow
Epidural inflammation, endplate
T2 bright disc signal, destruction.
• T2 - Bright
Marrow 0
and disc enhancement. • Disc Space • Post Contrast
Narrowing Enhancement
T1 T2

Trivia: Epidural Abscess


• Adults: the source is usually from a This is an infected
recent surgery, procedure, or systemic collection between the
infection. dura and periosteum.
• Children (younger than 5) it’s usually
Classic Appearance:
from hematogenous spread.
. T1 Dark, T2 Bright,
• Step 1 trivia: Staph A is the most • Peripheral
common bug, and always think about Enhancement, &
an IV drug user. • Restricted
• Almost always (80% o f the time) the Paraspinal Epidural Diffusion.
ESR and CRP are elevated. Abscess Abscess Classic Scenario:
• Gallium is superior to WBC scan in • HIV patient
the spine. • Bad Diabetic.

P e d ia tric D is c itis / O s te o m y elitis


Children have direct blood supply to the intervertebral disc, so they can get isolated discitis.
Isolated Discitis is basically never seen in adults. The classic scenario is: kid (younger than 4)
with a upper respiratory infection, now with back pain — usually lumbar.

252
TB

This is a special topic (high yield) w ith regard to M SK infection. It’s not that com m on,
with < 5% o f patients w ith TB having M SK involvem ent. A lthough on m ultiple choice
tests, I think y o u ’ll find it appears w ith a high frequency.

Pott D isease
(tuberculosis o f the spine) “ G ibbus D e fo rm ity ”

The vertebral body is involved w ith sparing o f This is a focal


the disc space until late in the disease (very kyphosis seen in “ Pott
different than m ore com m on bacterial infections). D isease” , am ong
m any other thing
It tends to spare th e disc space
t o It tends to have m ulti-level thoracic "skip ”
involvem ent
t o B uzzw ord “Large p a ra sp in a l abscess
B uzzw ord “C alcified Psoas A bscess ”
t o
B uzzw ord "G ibbus D eform ity ” - w hich is
a destructive fo c a l kyphosis
t o

M im ic - B rucellosis (unpasteurized m ilk


from an A m ish P e rs o n ), can also have disc
space preservation.

“ Tuberculosis D actylitis ” (Spina Ventosa)


“R ice B odies ”
Typically seen in kids w ith involvem ent o f the
short tubular bones o f the hands and feet. These are sloughed, infarcted
synovium seen w ith end stage
It is often a
RA, and TB infection o f joints.
sm oldering infection
w ithout periosteal
reaction.
C lassic look is a
diaphyseal
expansile lesion
w ith soft tissue
i w
sw elling.

253
S e p t ic A r t h r it is

You see this the m ost in large jo in ts w hich have an abundant blood supply to the m etaphysis
(shoulder, hip, knee).

I V drug users w ill g et it in the S I joint, a n d sternoclavicular jo in t.

C onventional risk factors include being old, having A ID S, RA, and prosthetic joints.

On plain film you m ight see a jo in t effusion, or


Pneum oarth rogram Sign
MRI will show synovial enhancem ent. If
untreated this will ja c k your jo in t in less than If you can dem onstrate air w ithin a jo in t
you can exclude a jo in t effusion.
48 hours.
No jo in t effusion

N o septic jo in t

N e c r o tiz in g F a s c iitis :

This is a very bad actor that kills very quickly. The good new s is that it’s pretty rare,
typically only seen in HI Vers, T ransplant patients, diabetics, and alcoholics. It’s usually
polym icrobial (the second form is G roup A Strep).

G as is only seen in a m inority o f cases, but if you see gas in soft tissue this is w h at they
want. D iffuse fascial enhancem ent is w hat y o u ’d see if the ER is dum b enough to order
cross sectional im aging (they often are).

Fournier G angrene is w hat they call it in the scrotum ( "testes satchel” ).

254
^ m u H n n n i^ a g g r e s s iv e V e s io n s

There are tons o f primary osseous malignancies, the most common are myeloma/plasmacytoma (27%),
Osteosarcoma (20%) and Chondrosarcoma (20%). I’ll discuss myeloma/plasmacytoma later in the chapter
when I get to lucent lesions. This discussion will focus more on the bone forming aggressive tumors.
I guess before I do that though, I should review what “aggressive” means. In the real world, dealing with
bone lesions is simple - it’s either aggressive, not aggressive, or not sure. Even though multiple choice is
very different than actual clinical radiology, that first mental calculus o f - aggressive vs not aggressive -
may still be useful in eliminating distractors.

What makes a lesion “aggressive ” ?

According to Helms, the wide zone of transition is the


best sign that a lesion is aggressive. This is actually a
useful pearl. The simplest way to conceptualize this is
to ask yourself if you can trace the edges of the lesion
with a pencil. If you can the lesion is probably benign.
If the edges are blurry or there is a gradient to the edge
- this is a more likely an aggressive lesions. Narrow Zone - Wide Zone -
Benign Aggressive

The reason is that margins reflect the growth the lesion. Bones are dumb. They really only know how to
do two things - make bone and destroy bone. The margins are the reflection of bone formation. If the
margins are sharp and sclerotic, this means the bone has time to adjust to the irritation and lay down a
coat of mature bone. If the margins are not distinct (zone o f transition is wide) this indicates a faster
growing lesion and therefore a higher probability of malignancy (or infection).

If the tumor grows rapidly enough it can break through the cortex and destroy the
newly formed bone capsule / lamellated bone. When this happens you end up with
a triangular structure - called the Codman triangle (as shown in diagram).

When we think about bone lesions, we often imagine lytic holes or bone destruction. Bone destruction
occurs from complex methods best understood as either direct obliteration via the tumor or pissed off
osteoclasts enraged by the uninvited tumor / hyperemia. Trabecular bone loss occurs more rapidly
(relative to cortical bone), but is noticed later because cortical bone is more smooth and organized.
Supposedly you need to destroy 70% o f the trabecular bone before it is noticed radiographically.

Bone destruction that occurs in a uniform geographic pattern


(especially with a sharp well defined border) is more suggestive j - f'S'f
of a benign slow growing lesion. A moth-eaten (cluster o f small ^ *
lytic holes) or permeative (ill-defined tiny oval or streak like I4 1 *‘j (* 1
luccncies) suggests rapid infiltrative tumor growth — as seen in jf 1 • *f
myeloma, lymphoma, and Ewings sarcoma. It is worth noting
that osteomyelitis and hyperparathyroidism can also demonstrate Moth Permeative
these aggressive patterns - pre-test probably is always important. ® P Eaten

255
Osteosarcoma:
T here are a bunch o f subtypes, but for the purpose o f this discussion there are 4.
C onventional Intram edullary (85% ), Parosteal (4% ), Periosteal (1% ), T elangiectatic (rare).
All the subtypes produce bone or osteoid from neoplastic cells. M ost are idiopathic but you
can have secondary causes (usually seen in elderly) XRT, Pagets, Infarcts, e tc ...

C o n v e n tio n a l In tr a m e d u lla r y : M ore com m on, and higher grade than the surface
subtypes (periosteal, and parosteal). Prim ary subtypes typically occur in young patients
(10-20). The m ost com m on location is the fem ur (40% ), and proxim al tibia (15% ).

B uzzw ords include various types o f aggressive periosteal reactions:

“S u n b u r s t”- periosteal reaction that is aggressive and looks like a sunburst


C odm an triangle - W ith aggressive lesions, the periosteum does not have tim e to
ossify com pletely w ith new bone (e.g. as seen in single layer and m ulti-layered
periosteal reaction), so only the edge o f the raised periosteum will ossify - creating
the appearance o f a triangle.
* L a m ella ted (onion skin reaction) - m ulti layers o f parallel periosteum , looks like an
o n io n ’s skin.

I Trivia:

O steosarcom a m et to the lung is a “classic” cause o f occult pneum othorax.

“ R everse Z on ing P h en om en on ” - m ore


dense m ature m atrix in the center, less
peripherally (opposite o f m yositis ossificans)
Progress Over Time
“Reverse Zoning” = Bad

256
P a r o s te a l O s te o s a r c o m a : G enerally low grade, B U L K Y parosteal bone form ation.
Think B ig ... ju st say Big. This guy loves the posterior distal fem ur (because o f this location
it can m im ic a cortical d e sm o id "tug lesion ” early on). The lesion is m etaphyseal 90% o f the
tim e. The buzzw ord is “ string sign ” - w hich refers to a radiolucent line separating the bulky
tum or from the cortex.

THIS vs THAT
P e r io s te a l P a r o s te a l P e r io s t e a l
O s te o s a r c o m a : Early A dult / M iddle Age A ge G roup (15-25)
W orse prognosis than parosteal
M etaphyseal (90% ) D iaphyseal
but better than conventional
Likes P osterior D istal Fem ur Likes M edial D istal Fem ur
osteosarcom a. Tends to occur
in the diaphyseal regions, M arrow extension (50% ) U sually no m arrow extension
classic m edial distal femur. Low G rade Interm ediate G rade

F L U ID -F L U ID L E V E L S D D x :
T e la n g ie c t a t ic
O s te o s a r c o m a :
A bout 15% have a narrow zone o f
transition. Fluid-Fluid levels on Telangiectatic
Osteosarcoma
M RI is classic. T hey are H igh on
Aneurysmal Bone Cyst
T1 (from m ethem oglobin). C an be
Giant Cell Tumor
differentiated from A BC or G C T
(m aybe) by tum or nodularity and
enhancem ent.

Parosteal Periosteal Telangiectatic C la s s ic (Intramedullary) Secondary O S


4% 1% Rare A s Fuck 85% Started off as Pagets

More Evil than Skeletor

Cystic on Plain Film


M etaphyseal ,v , , Fluid Levels on MR
r, *1 ■ Diaphyseal Fem ur - 40%
Posterior ,
rv . i c Medial - Tibia - 15%
Distal Femur rv * r c
Distal Femur

257
Other Bad Actors
C h o n d ro s a rc o m a :
Usually seen in older adults (M>F). Likes flat bones,
limb girdles, proximal tubular bones. Can be central
(intram edullary) or peripheral (at the end o f an THIS vs THAT
osteochondroma). Most are low grade. Enchondroma vs Low Grade
Chondrosarcoma
Risk Factors: Pagets, and anything cartilaginous
(osteochondrom as, M affucci e tc ...) Factors Favoring
C hondrosarcom a:
Gamesmanship: If you want to say chondroblastom a but
it’s an adult think clear cell chondrosarcom a Pain
Cortical Destruction
Gamesmanship: Scalloping o f > 2/3 o f the cortex
>5cm in Size
If shown with CT - they have to “Changing Matrix”
show you some “chondroid
m atrix” - “arcs and rings”

Ew ings:

Permeative lesion in the diaphvsis o f a child = Ewings


(could also be infection, or EG).

Extremely rare in African-Americans. Likes to met bone to bone (skip lesions are more common
in Ewings, relative to Osteosarcoma). Does NOT form osteoid from tumor cells, but can mimic
osteosarcoma because of its marked sclerosis (sclerosis occurs in the bone only, not in the soft
tissue - which is NO T the case in osteosarcoma).

C hordom a:

Usually seen in adults (30-60), usually slightly younger in the clivus and slightly older in the
sacrum. Most likely questions regarding the chordoma include location (most common sacrum,
second most common clivus, third most common vertebral body), and the fact that they are verv
T2 bright.

Chordoma Most Commons:


• Most common primary malignancy o f the spine.
• Most common primary malignancy o f the sacrum.
• When involving the spine, most common at C2.

I Say Chordoma, You Say "Midline, Midline, M idline!’’


Why is it always midline? It’s made o f cells left over from the “notochord” (some embryology
bullshit that was involved in making MIDLINE structures).
Chordomas arc NEVER EVER seen off the midline (NEVER in the hip, leg, arm, hand, e tc ...).

258
S E C T IO N 5: JttT

t ^ L u c e n t L e s io n s in n jiin in it

FEGNOM ASHIC is a “useful” m nem onic for lucent bone lesions made popular by Clyde Helms. As
it turns out, you can rearrange the letters o f FEGNOM ASHIC to form a word FOGM ACHINES. I
find it a lot easier to rem em ber a mnem onic if it actually forms a real word. Having said that, the
whole idea o f m em orizing a list o f 11 or 12 barely related things is stupid. You would never give a
differential that included all o f those, as they occur in different places, in different ages, and often
look very different.

Differentials (for people who know what they are looking at) are usually never deeper than 3 or 4
things. If you are giving a differential o f 12 things, ju st say you don’t know what it is. Seriously in
the real world bone lesions only come in 3 flavors: 1 - Bad (cancer or infection), 2 - Obviously
benign (not sure I ’ll waste saliva m entioning it in my re p o rt), 3 - Ehh hard to tell - get a follow up.
This is actually pretty much true o f lesions everywhere in the body. Realizing this allows for the
following paradigm shift:

Eyeball: Oh Shit! A lesion that has non-aggressive Eyeball: Oh Shit! A lesion that has non-
features. I wonder what it is.... aggressive features. I wonder what it is....
Neuron 1: We better look it up and give it a name. Lion Neuron 1: Who gives a shit? It’s
Neuron 2: Maybe we should give a list of possible benign... next case.
names. Then people will read our report and think Lion Neuron 2: This is an ortho study.
we are smart. Literally no one will ever read this report.
Bystander Neurons: Initiate Waffle Protocol Bystander Neurons: Bro... can we finish this
fucking list already? We need to look at that
new Tesla Roadster, 0-60 is sub 2 seconds.

B ut... we aren’t training for real life. The realm o f multiple choice is obviously different.

For multiple choice, when you encounter a lucent bone lesion you can expect one o f two questions
(1) what is it ? or (2) what is it associated with ? In either case you are going to need to figure out
what it is. In the real world you would probably have to give a short differential, but in the world o f
multiple choice you will have to come up with one answer. D on’t fret, they have to give you clues
so you can pick just one. A useful mental exercise when elim inating m ultiple choice distractors is to
ask yourself “why is it NOT this?” It’s an exercise that is often not perform ed at the workstation -
but very valuable in the test environm ent - especially for these types o f questions.

Here is my suggested method:


Age - Key Facts
(1) Age o f the p a tien t? - If you are lucky they will tell . < 3 0 = EG, ABC, NOF,
you. If you are less lucky you will have to guess. Chondroblastoma, and Solitary
Growth plates open = kid. Growth plates closed with Bone Cysts
no degenerative change = young adult. Growth plates • Any Age = Infection
closed with degenerative changes = older than 40. • > 40 = Mets and Myeloma
(unless it’s neuroblastoma mets).

(2) Location o f the lesion ? M etaphysis, Epiphysis,


Diaphysis? Is this an epiphyseal equivalent discussion (see the next page for discussion).

(3) Classic Locations and Looks ? See my summ ary at the end o f this section

259
location:

E p ip h y s is :

In general, only a few lesions tend to arise in the epiphysis. The “ four horsem an o f the
(e)apophysis” is the m nem onic 1 like to use, and I think about the com pany A IG that w as
involved in som e scandal a few years ago. A IG “ the evil” C om pany.

A B C, Infection, G iant C ell, and C hondroblastom a.


* The caveat is that A B C is usually m etaphyseal hut a fter the E p iphyseal Equivalents:
grow th p la te closes it can e x te n d into the epiphysis. (bones that will have the
same lesions as
the epiphysis)
For the purpose o f m ultiple choice tests, it is im portant to not
forget about the m alignant tum or at the end o f the bone
C arpals,
(epiphysis) - C lear Cell C hondrosarcom a. This guy is slow
Patella,
grow ing, w ith a variable appearance (lytic, calcified, lobulated,
G reater Trochanter,
ill defined, e tc ...). Just rem em ber if they say m alignant
C alcaneus
epiphyseal you say C lear C ell C hondrosarcom a.

M e ta p h y s is

The m etaphysis is the fastest grow ing area o f a bone, w ith the best blood supply. This
excellent blood supply results in an increased predilection for M ets and Infection. M ost o f the
cystic bone lesions can occur in the m etaphysis.

D ia p h y s is

Just like the m etaphysis, m ost entities can occur in the diaphysis (they ju st do it less).

260
Pathology-For Trivia
F ib ro u s D y s p la s ia :

Fibrous dysplasia is a skeletal developmental anomaly of osteoblasts - failure


o f normal maturation and differentiation which results in replacement of the
nonnal medullary space.

Famously “can look like anything”, with phases like Pagets (lytic, mixed,
blastic) - although the most classic appearance is a "long lesion, in a long
bone, with ground glass matrix. ” Sometimes the vocabulary “lytic lesion Shepherd Crook
with a hazy matrix” is used instead of the word “ground glass” - for the -Coxa Varus Angulation
purpose o f fucking with you. The discriminator used by Helms is “no -Classic fo r FD (but can be
periosteal reaction or pain.” seen in Paget and Oi)

Likes the ribs and long bones. If it occurs in the pelvis, it


also hits the ipsilatcral femur (Shepherd Crook deformity). THIS vs THAT
If it’s multiple it likes the skull and face (Lion-like faces). M cC une
M azabrau d
A lb rig h t
Polyostotic FibrousPolyostotic Fibrous
The disorder can occur at any age - but the multiple lesion
Dysplasia Dysplasia
variety “polyostotic” - tends to occur earlier.
Girl Woman (middle aged)
Cafe au lait spots Soft Tissue Myxomas
You could think monostotic (20’s & 30’s) or polyostotic (<
Increased Risk Osseous
10 year old). When you see the polyostotic form (often with a Precocious Puberty Malignant
mangled horrible horrible face... a face that only a Transformation
mother could love) - you should think syndromes.

A d a m a n tin o m a :
A total zebra (probably a unicorn). A tibial lesion that resembles Fibrous dysplasia
(mixed lytic and sclerotic). It is potentially malignant.

N o n o s s ify in g F ib r o m a (N O F ):

These are very common. They arc seen in children, and will spontaneously regress (becoming more
sclerotic before disappearing). They are rare in children not yet walking. Just like GCTs they like to
occur around the knee. They are classically described as eccentric with a thin sclerotic border
(remember GCTs don’t have a sclerotic border). They are called fibrous cortical defects when
smaller than 2 cm.

Vocab: NOFs are the larger version (> 3cm) o f a fibrous cortical defect (FCD). A wastebasket term
for the both o f them is simply “fibroxanthoma."

J a f f e - C a m p a n a c c i S y n d ro m e : Syndrome o f multiple NOFs, cafe-au-lait


S? sf spots, mental retardation, hypogonadism, and cardiac malformations.
\\ %.V

261
E n c h o n d ro m a :

This guy is a tumor o f the medullary cavity composed o f hyaline cartilage. They become progressively
more common with age - peaking around 10-30 years old.

The sneaky thing about this lesion is that it looks different depending on the body part it is in.

• Humerus or Femur = Arcs and Rings


• Fingers or Toes = Lytic

The ARCS AND RINGS is the more classic textbook look with the irregularly
speckled calcification of chondroid matrix. Just don’t forget that this classic matrix
is not found in the fingers or toes.

The enchondroma is actually the most common cystic lesion in the hands and feet. Just like fibrous
dysplasia, this lesion does not have periostitis.

Differentiating Enchondroma vs Low Grade Chondrosarcoma


▼ f Strategies to Deal with the Chondroid Matrix Lesion in a Long Bone

Primary Tactic: History of pain — Enchondroma vs a Low Grade Chondrosarcoma


Not Painful PAINful

Secondary Tactic: Size — Enchondroma vs a Low Grade Chondrosarcoma


1-2 cm > 4-5 cm

Tertiary Tactic: Glitch in the Matrix — Enchondroma vs a Low Grade Chondrosarcoma


Arcs & Rings Arcs & Rings Pattern
Pattern does Changes - moves
NO T Change around grows etc...

When multiple — especially when in the hands you should think syndromes:

THIS vs THAT
M a ffu c c i Syndrom e
O ilie r D isease “Marffucci Has More”
More Cancer Risk and More Vascular Malformations

Multiple Enchondromas (3 or more) Multiple Enchondromas

Hemangiomas
(bunch lucent centered calcifications)
Increase risk in Chondrosarcoma
Slight increase risk in Chondrosarcoma
(probably more than Oilier)

262
E o s in o p h ilic G r a n u lo m a (E G ):

This is typically included in every differential for people less than 30 (peak age is 5-10).
It can be solitary (usually) or m ultiple.

T here are 3 classic appearances - for the purpose o f m ultiple choice:


(1) Vertebra plana in a kid
(2) Skull w ith lucent “beveled edge” lesions (also in a kid).
(3) “ Floating Tooth” w ith lytic lesion in alveolar ridge — this w ould be a differential case

The appearance is highly variable and can be lytic or blastic, w ith or w ithout a sclerotic border,
and w ith or w ithout a periosteal response. C an even have an osseous sequestrum .

C lassic DD x for V ertebra Plana C lassic DDx for O sseous Sequestrum :


(M E L T ) O steom yelitis
M ets / M yelom a Lym phom a
* EG Fibrosarcom a
L ym phom a * EG
* T raum a / TB *O steoid O steom a can m im ic a sequestrum

G ia n t C e ll T u m o r (G C T ):
This guy has som e key criteria (w hich lend them selves well to m ultiple choice tests).
They include:
- Physis M U ST be closed
- N on Sclerotic B order
kK k - A buts the articular surface

A nother trick is to show you a pulm onary m et, and ask if it could be a G C T ? T he answ er is
yes (although this is rare) G C T is considered “q u asi-m alig n an f ’ because it can be locally
invasive and about 5% w ill have pulm onary m ets (w hich are still curable by resection). As a
result o f this, it should be resected w ith w ide m argins.

Things to know about GCTs:


iLw - M ost com m on in the knee - abutting the articular surface
w jv - M ost com m on at age 20-30 * physis m ust be closed
- T here is an association w ith A B C s (they can turn into them )
* - They are “quasi-m alignant” - 5% have lung m ets
- Fluid levels on MRI

263
Associations of
O s te o id O s te o m a “Pain at night, relieved by aspirin. " Osteoid Osteoma
Painful Scoliosis
- Most Classic Age: “A dolescent” — 10-25 ish.
Growth Deformity:
- Most Classic Look: Oval lytic lesion (“ lucent nidus”) Increased length and
surrounded by dense sclerotic cortical bone (“periosteal girth o f long bones
reaction”). Synovitis: Can be seen if
- M ost Classic Locations: (1) M eta/diaphysis o f long bones intra-articular, joint
(femoral neck = most comm on) and (2) Posterior elem ents o f effusions
the spine (lum bar > cervical > thoracic). Technically the Arthritis: Can occur from
fingers arc more common than the spine, but that’s rarely show
primary synovitis, or
on multiple choice.
secondarily from altered
joint mechanics.
M odality Trivia:

• MRI: “Lots o f edem a.” I’ll say that again “large a m o u n t o f edem a f o r the size o f the lesion. ”
Adjacent soft tissue edema is also common - don’t let that fool you.

• Nuke Bone Scan: “ Double Density Sign” - very intense central activity at nidus, surrounded
by less intensity o f reactive bone. A common distractor is a stress fracture. Stress fractures
are linear. O.O. should be round.

Scoliosis Trivia: W hen you have them in the spine (most comm on in C o n vex
the posterior elements o f the lum bar spine), you frequently have an
associated painful scoliosis with the convexity pointed away from
the lesion.

Treatment:

These can be treated with percutaneous radiofrequency ablation (as long as it’s not within 1 cm
o f a nerve or other vital structure - typically avoided in hands, spine, and pregnant patients).

O s te o b la s to m a :

Basically it’s an osteoid osteom a that is larger than 2 cm. It’s seen in patients < 30 years old.
They are m ost likely to show this in the posterior elem ents. It also occurs in the long bones
(35% ) and w hen it does it is usually diaphyseal (75% ).

264
A n e u r y s m a l B o n e C y s t (A B C ): Classic DDx for
Lucent Lesion in
A neurysm al bone cysts are aneurysm al lesions o f bone w ith thin- Posterior Elements:
w alled, blood-filled spaces (fluid-fluid level on MR1). Patients are O steoblastom a
usually < 30. They m ay develop follow ing traum a. ABC
TB
Location: Tibia > Vert > Fem ur > H um erus

They can be described as prim ary A B C , presum ably arising denovo or secondary A B C ,
associated w ith another tum or (classic G C T). They are com m only associated w ith other
benign lesions.

Things to know about ABC :


- Up to 40% o f secondary A B C's are associated w ith giant cell tum or o f bone.
- It's on the DD x for Fluid - Fluid Level on MRI
- Patient < 30
A - Tibia is the m ost com m on site

S o lit a r y ( U n ic a m e r a l) B o n e C y s t:

It w ould be unusual to see one o f these in a patient older than 30. M ost com m on in the
tubular bones (90-95 % ) usually hum erus or femur. U nique feature: “ A lw ays located
centrally.”

It’s going to be show n one o f tw o ways: (1) W ith a fracture through it in the hum erus
(probably w ith a fallen fragm ent sign) or (2) As a lucent lesion in the calcaneus (probably
w ith a fallen fragm ent sign).

The fallen fragm ent sign (bone fragm ent in the dependent portion o f a lucent bone lesion) is
pathognom onic o f solitary bone cyst.

B ro w n T u m o r ( H y p e r p a r a th y r o id is m ):

The “brow n tum or” represents localized accum ulations o f giant cells and fibrous tissue (in
case som eone asks). T hey appear as lytic or sclerotic lesions w ith other findings o f
hyperparathyroidism (subperiosteal bone resorption). In other w ords, they need to tell you he/
she has hyperparathyroidism first. They m ay ju st straight up tell you, or they w ill show you
som e bone resorption first (classically on the side o f a finger, edge o f a clavicle, or under a
rib).

These things have different stages o f healing / sclerosis. They resorb and can becom e totally
sclerotic / healed, w hen the H yper PTH is treated.

More on this in the arthritis section — later in the chapter

265
Chondroblastom a:

This is seen in kids (90% age 5-25). T hey classically show it in tw o w ays
(1) in the epiphysis o f the tibia on a 15 year old, or (2) in an epiphyseal equivalent.

So w hat are the epiphyseal equivalents???


* P atella
* C alcaneus
* C arpal Bones
* A n d a ll the A pophyses (greater a n d less trochanter, tuberosities, etc...)

Features o f the tum or include; A thin sclerotic rim , extension across the physeal plate
(25-50% ), periostitis (30% ). A ctual location: fem ur > hum erus > tibia . T his m ay show bone
m arrow edem a, and soft tissue edem a on MR1 (M R I can m islead you into thinking it’s a bad
thing). This is one o f the only bone lesions that is often N O T T2 bright. T hey tend to
reoccur after resection (like 30% o f the tim e).

n _ G am esm anship Hip: W hen you have a chondroblastom a in the hip, it tends \o fa v o r
y / ® the g rea ter trochanter (m ore than the fem o ra l epiphysis).

C h o n d ro m y x o id F ib ro m a :

This is the least com m on benign lesion o f cartilage. It is usually in patients younger than 30.
The typical appearance is an osteolytic, elongated in shape, eccentrically located,
m etaphyseal lesion, w ith cortical expansion and a “bite” like configuration. Sorta looks like
an N O F - w ith the classic location in the proxim al m etaphyseal region o f the tibia.

T h e H ip

G reater Trochanter - R em em ber this is an epiphyseal equivalent and the


chrondroblastom as prefer it to the fem oral epiphysis. You can get all the other D D xs
(A BC , Infection, GCT, e tc ... here as w ell). Plus, you can have avulsions o f the gluteus
m edius and m inim us.

L esser Trochanter - A n avulsion here w ithout significant clinical history should m ake you
think pathologic fracture.

The Intertrochanteric Region: C lassic DDx here: Lipom a, Solitary Bone Cyst, and
M onostotic Fibrous D ysplasia.

266
C lassic (& s n e a k y non-classic) Lesions P osterior M etaphysis S uperior E piphysis

th a t c an be show n in th e c alc an eu s .
The suggested Promethean method is to first use
location within the bone, and then use characteristic
appearance as a secondary discriminator.
First let us take a closer look at the calcaneus. A nterior
Remember this thing is an epiphyseal equivalent, but E piphysis
only in certain locations. It’s probably better to think
about the bone like a hybrid long bone - complete
with a diaphysis, two metaphysis, and three A nterior M etaphysis
P osterior
epiphysis. E piphysis

08
C hondroblastom a: G ia n t C ell Tum or: Can also involve the epiphysis
(although it typically starts out metaphyseal and grows into the
epiphysis). Remember these things required a closed physis.
The Posterior Metaphysis / Epiphysis is favored.

O steoid O steom a:
Talus > Calcaneus
Similar to Chondroblastoma in
Most classic epiphysis lesion - with a favoring the superior epiphysis
preference for the superior epiphysis near near the talocalcaneal
the talocalcaneal articulation (although articulation.
they can be at any o f the 3 epiphysis). Distinction is the sclerotic
Lucent lesion, that can have some thickening o f the adjacent bone
internal calcifications. and the radiolucent nidus.

G eode: Older Patient + Subtalar degenerative change / Obvious Arthritis

O steo m y e litis & M ets: O steom yelitis


The calcaneal apophysis (equivalent to the metaphyseal region o f long bones)
will have a similar predilection for collection hematogenous spread o f both
infection or cancer (GU or Colon).
In both cases the involvement favors the posterior meta-epiphvseal region
(which has the richest blood supply), with lesions potentially growing large
enough to involve the entire calcaneus.

S o lita ry Bone Cyst: The typical location for this lesion is the Younger than 20
diaphysis (anterior 1/3 laterally). This will have sharp edges. A Fallen Fragment
thick sclerotic edge with a multiloculated appearance is helpful. The Tl Dark, T2 Bright
“fallen fragment” will be more in the bottom if shown - although
fractures in the calcaneus are much less common than in the arm.

In tra o ss e o u s Lipom a: This is also typically located in the


diaphysis (anterior 1/3 laterally). If they show you this, it will either Older than 20
have (a) fat density on CT or MR1, or (b) a central fragm ent - Central Fragment
stuck within the middle o f the fat. This calcification / fat necrosis Isointense to Fat
occurs about 50% o f the time in the real world (nearly 100% on on all sequences
pictures shown on tests), and is secondary to fat necrosis.

Pseudo-cyst: This is a variation on the normal trabecular


pattern, which creates a central triangular radiolucent area. This area
is sometimes called the “pseudo-cyst triangle” and is obnoxiously
located in the same anterior 1/3 as the SBC and Lipoma. Supposedly
the persistence o f thin trabeculae, visible nutrient foramen, and the
classic location are helpful in telling it from the other benign
entities.

267
M e t a s t a t ic D is e a s e : Should be on the differential for any patient over 40 w ith a lytic
lesion. A s a piece o f trivia renal cancer is ALWAYS lytic (usually).

• C lassic Blastic Lesions: Prostate, C arcinoid, M edulloblastom a


• Classic Lytic Lesions: Renal and T hyroid

N ext Step - Prostate M et vs Bone Island??? - G et a Bone Scan


• Bone Island should be m ild (or not active)
• Prostate M et should be HOT

M u ltip le M y e lo m a (M M ): Plasm a cell proliferation increases surrounding osteolytic


activity (in case som eone asks you the m echanism ). U sually in older patient (4 0 ’s-8 0 ’s).
Plasm acytom as can precede clinical or hem atologic evidence o f m yelom a by 3 years.

They usually have discrete m argins, and can be solitary or m ultiple. Vertebral body
destruction w ith sparing o f the posterior elem ents is classic. Bone Scan is often negative,
skeletal survey is better (but horribly painful to read), and M RI is the m ost sensitive.

A dditional classic (testable) scenario: M M m anifesting as D iffuse O steopenia

M yelom a R elated C onditions:

P lasm acytom a (usually under 40): This is a discrete, solitary m ass o f neoplastic m onoclonal
plasm a cells in either bone or soft tissue (extram edullary subtype). It is associated w ith latent
system ic disease in the m ajority o f affected patients. It can be considered as a singular
counterpart m ultiple m yelom a. The lesions look like a geographic lytic area, som etim es w ith
expansile rem odeling.

“M ini B rain A p p e a ra n c e ” - Plasm acytom a in vertebral body

P O E M S : This is basically “M yelom a with Sclerotic M ets.” It’s a rare m edical syndrom e with
plasm a cell proliferation (typically m y e lo m a ), neuropathy, and organom egaly.

268
L u c e n t Lesion C lassic Looks and L o c atio n s

Long Lesion in a Long Bone Fibrous Dysplasia


Ground Glass Fibrous Dysplasia
Lytic lesion with a hazy matrix Fibrous Dysplasia
Chondroid Matrix in the Proximal Humerus or
Enchondroma
Distal Femur
Lucent Lesion in the Finger or Toe Enchondroma
Epiphyseal Tibial Lesion in a Teenager Chondroblastoma
Chondroblastoma or Giant Cell Tumor
Epiphyseal Equivalent Lesion **technicaily GCTs grow into the Epiphysis
Chondroblastoma
Lucent Lesion in the Greater Trochanter

Lucent Lesion with a Fracture Solitary Bone Cyst


(Fallen Fragment) in the Humerus
Calcaneal Lesion with Central Calcification Lipoma
Lucent Lesion in the Skull EG
Vertebra Plana in a Kid EG
Vertebra Plana in an Adult Mets
Sequestrum / Nidus in the Tibia / Femur Osteoid Osteoma
“Painful Scoliosis” Osteoid Osteoma
Calcified Lesion in the Posterior Element
Osteoblastoma
of the C-Spine

S o lita ry vs M u ltip le (G e n e ra liza tio n fo r M u ltip le C h o ice T riv ia )

Multiple Sclerotic Lesions Mets


Multiple Sclerotic Lesions Centered Around a
Osteopoikolosis
Joint
Mets, Myeloma, Metastatic Non-Hodgkin
Multiple Lucent Lesions (older than 40) Lymphoma

S ize M a tte rs

Nidus < 2.0 cm Osteoid Osteoma

Nidus > 2.0 cm Osteoblastoma


Well-defined lytic lesion in the cortex of a long
Fibrous cortical defect
bone with a sclerotic rim < 3 cm
Well-defined lytic lesion in the cortex of a long
Nonossifying fibroma
bone with a sclerotic rim > 3 cm
Chondral lesion in a long bone 1-2 cm Probably an Enchondroma

Chondral lesion in a long bone > 4-5 cm Increased risk of low-grade chondrosarcoma

269
r a n d o m A s ? C o l S ion o f T r .v i a ^

L ip o s c le ro s in g M y x o fib ro m a :
Very characteristic location - at the intertrochanteric region o f the fem ur.
Looks like a geographic lytic lesion with a sclerotic margin. Despite non-
aggressive appearance. 10% undergo m alignant degeneration so they need to %
be followed.

O s te o c h o n d ro m a :
Some people think o f this as more o f a developm ental anom aly (although
they still always make the tum or chapter). Actually, it’s usually listed as the I
most common benign tum or (“exostosis” ). They can be radiation induced,
making them the only benign skeletal tum or associated with radiation.
They have a very small risk o f m alignant transform ation (which supposedly /
can be estimated based on size o f cartilage cap).
Supposedly a cap > 1.5 cm is concerning.
Key Points:
• They point away from the joint
• The bone marrow flows freely into the lesion *Away from Joint

M u ltip le H e r e d ita r y E x o s to s is :
AD condition with m ultiple osteochondromas.
They have an increased risk of m alignant transform ation.

T re v o r D is e a s e (D y s p la s ia E p ip h y s e a lis H e m im e lic a - D EH ):
This is a disease characterized by the developm ent o f osteochondrom as develop at the epiphysis
which result in significant joint deform ity (most comm on in ankle and knee) — m aking you
terrible at tennis and soccer. Instead o f pointing away from the joint (like a normal
osteochondroma) these assholes point into the joint — this is why you have so m any joint
issues. You see this is young children. The osteochondrom a looks more like an irregular mass.
They tend to be treated with surgical excision.

S u p ra c o n d y la r S p u r (A v ia n S pur):
This is an Aunt M innie, and normal variant. This is an ^ 'an
osseous process, that usually does nothing, but can
compress the median nerve if the Ligam ent o f Ligament o f
Struthers smashes it. Struthers

Notice this thing points towards the joint, that is how


you know it is not an osteochondroma. Also - it is not Supracondylar
a Trevor Disease thing - because Canal
(1) o f the characteristic location and
(2) it is not originated from the epiphysis.

270
P e r io s te a l C h o n d ro m a ( J u x t a - C o r t ic a l
C h o n d ro m a ): W hen you see a lesion in the finger
o f a kid think this. It’s a rare entity, o f cartilaginous
origin. “ Saucerization” o f the adjacent cortex w ith
sclerotic periosteal reaction can be seen.

O s te o fib ro u s D y s p la s ia : This is a benign lesion found exclusively in the tibia or


fibula in children (10 and under - usually). It looks like an N O F , but is centered in the
anterior tibia, and has associated anterior tibial bow ing. It can occur w ith A dam antinom a,
and the tw o cannot be differentiated w ith im aging.

When I say looks like N O F in the anterior tibia with an terio r bowing, y o u say O steofibrous
D ysplasia.

D is ta l F e m o r a l M e ta p h y s e a l Ir r e g u la r it y ( C o r t ic a l D e s m o id ):

This is a lucency seen along the back o f the


posteriom edial aspect o f the distal fem oral
m etaphysis. If they show you a lateral knee x-
ray, and there is an irregularity or lucency on the
back o f the fem ur this is it. It’s often bilateral.

Buzzw ords include “ Scoop like defect” w ith an


“irregular but intact cortex.”

This is a total incidental finding and is a d o n ’t


touch lesion. D on ’t biopsy it, D o n ’t M RI it.

Just leave it alone. If you really w ant to know,


it’s probably a chronic tug lesion from the
adductor m agnus.
Cortical Desmoid - Scoop Like Defect

271
C a lc iu m H y d r o x y a p a tite : M ost pathologic calcification in the body is calcium
hydroxyapatite, w hich is also the m ost abundant form o f calcium in bone.

C alcium hydroxyapatite deposition disease = calcific tendinitis.

The calcium is deposited in tendons around the joint. The m ost com m on location for
hydroxyapatite deposition is the shoulder. Specifically, the su p raspinatus ten don is the
m ost frequ en t site o f calcification, usually at its insertion near the greater tuberosity. The
longus colli m uscle (the m uscle anterior to atlas -> T3) is also a fa vo rite location fo r m ultiple
choice test writers. It m ay be prim ary (idiopathic) or secondary. Secondary causes w orth
know ing are: chronic renal disease, collagen-vascular disease, tum oral calcinosis and
hypervitam inosis D.

O s te o p o ik ilo s is : It’s ju st a bunch o f bone islands. U sually in epiphyses (different from


blastic m ets or osteosarcom a m ets). It can be inherited or sporadic but if you are forced to
pick a pattern - I 'd go w ith autosom al dom inant.

M ets vs O steopoikilosis - O steopoikilosis tends to be jo in t centered (clustered around


centered). Sclerotic m ets will be all over the place. Sclerotic m ets believe in nothing
Lebow ski.

Trivia - O steopoikilosis patients tend to be keloid form ers.

O s te o p a th ia S t r ia t a : Linear, parallel, and longitudinal lines in


m etaphysis o f long bones. D o e sn ’t m ean shit (usually - but can in som e
situations cause pain).

E n g e lm a r m ’s D is e a s e : T his is also know n as progressive diaphyseal


S '" ! dysplasia or PDD. W hat you see is fusiform bony enlargem ent w ith sclerosis o f
;5 j ;
“ the long bones. This is a total zebra that begins in childhood.

Things to know:

* It's B ila tera l a n d Sym m etric


* It likes the long bones - usually show n in the tibia
* I t s hot on bone scan
* It can involve the sk u ll —a n d can cause optic nerve com pression

272
T h a la s s e m ia : This is a defect in the hem oglobin chain
T h alassem ia Sickle Cell
(can be alpha or beta - m ajor or m inor). From the M SK
R adiologist prospective, w e are talking about “hair-on-end’
W ill N ot
skulls, expansion o f the facial bones, “rodent faces,” W ill O bliterate
O bliterate
expanded ribs “jail-b a rs” . It is frequently Sinuses
Sinuses
associated w ith extram edullary Lytic U sually A sym ptom atic
hem atopoiesis.
M ixed E levated A lkaline
(reparative) Phosphate. Fractures
Sclerotic E levated H ydroxyproline.
(latent M ore fractures. Sarcom as
A V N o f t h e H ip :
inactive) m ay develop.
Variety o f causes including Perthes in kids,
sickle cell, G au ch er’s, steroid use e tc .... It can also be traum atic w ith fem oral neck
fractures (degree o f risk is related to degree o f displacem ent / disruption o f the retinacular
vessels). AVN o f the hip typically involves the superior articular surface,
^ beginning m ore anteriorly.

D ouble Line Rim Sign: C rescen t Sign:


Sign: Best seen on T2; high T2 signal line Seen on X-ray (optimally frog
Best seen on T2; inner sandwiched between two low signal leg); Refers to a subchondral
bright line (granulation lines. This represents flu id between lucency seen most frequently in
tissue), with outer dark sclerotic borders o f an osteochondral the anterolateral aspect of the
line (sclerotic bone). fragm ent, and implies instability. proximal femoral head. It
(Stage III). indicates imminent collapse.

Plain Film Stages o f O steonecrosis


(there are M R stages also - p re su m e d to be b eyo n d the scope o f this exam ):
o Z ero = N orm al
o One = N orm al x-ray, edem a on M R
o Two = M ixed Lytic / Sclerotic
o Three = C rescent Sign, A rticular C ollapse, Joint Space Preserved
o Four = Secondary O steoarthritis

273
P a g e t D is e a s e ( O s t e it is D e fo rm a n s ):

A relatively com m on condition that affects 4% o f people at 40, and 8% at 80 (actually 10%,
but easier to rem em ber 8%). M > F. M ost people are asym ptom atic. The pathophysiology o f
Paget is not well understood.

The bones go through three phases w hich progress from lytic to m ixed to sclerotic (the
latent inactive phase). T he phrase “ W ide Bones w ith T h ick T rabecula” m ake you
im m ediately say Pagets (nothing else really does that).

C om es in two flavors: (1) M onostotic and (2) Polyostotic - w ith the poly subtype being
m uch m ore com m on (80-90% ).

Buzzwords / Signs

•Blade o f Grass Sign: Lucent leading edge in a long bone


*Osteoporosis Circumscripta: Blade o f Grass in the Skull
•Picture Frame Vertebra: Cortex is thickened on all sides (Rugger Jersey is only superior and inferior
cndplates)
• Cotton Wool Bone: Thick disorganized trabeculae
• Banana Fracture: Insufficiency fracture o f a bowed soft bone (femur or tibia).
*Tam O ’Shanter Sign: Thick Skull - with the frontal aspect “falling over the facial bones”
*Saber Shin: Bowing o f the tibia
• Ivory Vertebra: This is a differential finding, including mets. Pagets tends to be expansile.

Complications: Deafness is the most common


complication. Spinal stenosis from cortical thickening
is very characteristic. Additional complications include
cortical stress fracture, cranial nerves paresis, CHF C lassic Look:
(high output), secondary hyperparathyroidism (10%),
Secondary development of osteosarcoma (1%) - E xpanded
which is often highly resistant to treatment. As a
Bone
piece o f ridiculous trivia - giant cell tumor can arise
from Paget.
C oarse or
Trivia: O f all the tumors to which Paget may devolve T hick
to, Osteosarcoma is the Most Common.
T rabecular
Total Trivia: Paget bone is hypervascular and may be 5 Pattern
degrees hotter than other bone (get your thermometer
ready). Aik Phos will be elevated (up to 20x) in the
reparative phase.

274
S k u ll:
Long B ones
Large Areas of Osteolysis
in the Frontal and Occipital A dvancing m argin o f
Bones “Osteoporosis
lucency from one end to
Circumscripta”, in the lytic
phase. the other is the so-called
“blade o f grass” or
“ flam e.” W ill often spare
the fibula, even in diffuse
The skull will look disease.
“cotton wool” in the
mixed phase. Tibia B ow ing “ saber
shin” is also classic.

Thickened sclerotic
appearance is a good chronic look. Involves P e lv is :
BOTH inner and outer table (Fibrous Dysplasia
favors the outer table)
M ost com m on bone involved. “ A lw ays”
involves the iliopectineal line on the
pelvic brim .

C an cause advanced arthritis and


acetabular protrusio.

Has a classic look


on bone scan.

Tam O ’Shanter Sign: Skull sorta looks like one


o f those stupid hats with the frontal aspect
“falling over the facial bones”

S p in e :

C ortical T hickening can cause a “ picture


fram e sign” (sam e as osteopetrosis). A lso
can give you an ivory vertebral body.

D iscussed m ore on the next page

275
THIS vs THAT Pagets Spine vs O ther Spine Changes

Pagets - This is discussed in detail in the M SK chapter, but is such a high yield topic that it’s
w orth touching on again. The incidence increases w ith age (around 8% at age 80). It’s at
increased risk for fracture, and has a 1% risk o f sarcom a degeneration (usually high grade).

It’s show n two w ays in the spine:


(1) An enlarged “ ivory vertebrae” ,
Ivory |. Piclure
(2) Picture fram e vertebrae (sclerotic border) Frame
— w ith central lysis (m ix e d p h a s e )

Renal O steodystroph y - A nother high yield topic covered in depth in the M SK chapter. The
w ay it’s show n in the spine is the “ R ugger Jersey Spine” - w ith sclerotic bands at the top and
bottom o f the vertebral body. You could also have paraspinal soft tissue calcifications

This vs That: R ugger Jersey vs “ Paget” Picture Frame:

Rugger Jersey = Top and B ottom O nly


Paget Picture Fram e = All 4 m argins o f the vertebral body

O steopetrosis - A nother high yield topic covered in depth in the M SK chapter. This is a
genetic disease w ith im paired osteoclastic resorption. You have thick cortical bone, w ith
dim inished m arrow. On plain film or C T it can look like a R ugger Jersey Spine or Sandw ich
vertebra. On M R you are going to have loss o f the norm al T1 bright m arrow signal, so it will
be T1 and T2 dark.

“ H -Shaped V ertebra” - This is usually a buzzw ord for sickle cell, although it’s H
only seen in about 10% o f cases. It results from m icrovascular endplate infarct.
If you see “ H -Shaped vertebra,” the answ er is sickle cell. If sickle cell isn ’t a I I
choice the answ er is G auchers. A nother tricky w ay to ask this is to say w hich o f
the follow ing causes “w id en in g o f the disc sp ace.” W idened disc space is
another w ay o f describing a “ H Shape” w ithout saying that. H

276
Paget on Other Imaging M odalities:

M R I: T here are three m arrow patterns


Lytic / Early H eterogenous T2; T1 is
that closely (but not exactly) follow the
M ixed isointense to m uscle, w ith a
phases on x-ray.
“ speckled ap p earan ce”
Late M ixed M aintained fatty high T1 and
T2 signals
Sclerotic Low signal on T1 and T2

This vs That: M alignant T ransform ation vs A ctive D isease:


• Both are T2 Bright and Will Enhance
• M alignant T ransform ation will lose the N orm al T1 signal (just like a cancer w ould)
• B est Sequence f o r D istinguishing the two? T1 Pre C on

N u c le a r M ed icin e: The prim ary utility o f a bone scan is in


defining the extent o f disease and to help assess response
R
to treatm ent. The characteristic look for Paget is “ W hole
Bone Involvem ent.”

For exam ple, the entire vertebral body in clud ing the
posterior elem ents, or the entire pelvis. T he classic
teaching is that Paget is hot on all three phases (although
often decreased or norm al in the sclerotic phase).

277
Tibial Bowing

M ost likely show n as an A unt M innie: NF-1 anterior w ith a fibular pseudoarthrosis, Rickets
with w ide grow th plates, or B lounts tibia vara. Pagets can also cause this.

The m ost likely p u re trivia question is that ph ysio lo g ic bow ing is sm ooth, lateral, a n d occurs
from 18 m onths - 2 years.

NF-1 A n terior L ateral - U nilateral M ay be unilateral. M ay


have hypoplastic fibula w ith
pseudarthrosis.

f 'V .

Lateral Frontal

Foot D eform ities Posterior

Physiologic B ow ing Lateral - Bilateral Sym m etric S e lf lim iting betw een 18
m onths and 2 years.

H ypophosphatasia Lateral “ R ickets in a new born”

Rickets Lateral F raying o f the m etaphyses


and w idening o f the grow th
plates.

Seen best in “ fast grow ing


b ones” - knee, w rist

Blount Tibial Vara - O ften Early w alking, Fat, black


asym m etric kid.

Proxim al tibia posterom edial


physeal grow th disturbance
\ f ) resulting in deform ity

O steogenesis Im perfecta Involves all long bones

D w arfs Short Lim bs

278
S E C T IO N 7:
^
M tt

t S O FT T IS S U E M ASSES * B M IM IM M

Studying / learning M SK soft tissues m asses / tum ors tends to create tons o f anxiety. T his is
because A cadem ic R adiology tends to overcom plicate the issue.

Here is the sim ple part - only about 20-30% o f them can be accurately diagnosed on M RI. T h a t’s
because they are alm ost all T2 bright and enhance. This is actually good new s for the purpose o f
m ultiple choice, because you only need to learn the ones that d o n ’t behave like that - or are
overw helm ing likely due to epidem iological stats (w hich has to be provided for you - either
directly “the patient is 65” or w ith clues - “arthritis = old” , “no arthritis = not old” ).

Here is the list I w ould know:

• M FH — M alignant Fibrous H istiocytom a aka - Pleom orphic U ndifferentiated Sarcom a


• Synovial Sarcom a
• Lipom a, A typical Lipom a, L iposarcom a
• H em angiom a
• M yxom a

M a lig n a n t Fibrous H is tio c y to m a (M F H )

Yes - they changed the nam e to Pleom orphic U ndifferentiated Sarcom a "P U S .” H ow ever, I w ant
you to continue to think o f this as M FH because it will help you rem em ber som e o f the im aging
features.

First, the generalizations — This is very com m on. It’s seen in old people. It’s seen in a central
location (proxim al arm s and legs).

F eatures - A bout h a lf the tim e it’s dark to in term ed iate on T 2 (rem em ber m ost soft tissue
tum ors are T2 bright). T he w ay 1 rem em ber this is the w ord “fibrous” - m akes m e think scar
(w hich is dark).

G am esm anship - These things are often associated w ith spontaneous hem orrhage - they outgrow
their blood supply. The history is often “old lady, stood up from a chair” - has a big proxim al
m uscular hem atom a — under that hem atom a is the M FH.

Trivia: Bone infarcts can turn into M FH - ‘'sarcom atous transform ation o f in fa rct"
Trivia: R adiation is a risk factor.

279
S yn ovial S a rc o m a :

G eneralization - Seen m ost com m only in the p e rip h e ra l low er extrem ities o f patients aged
20-40.
Baker’s C y st Fuckery
G am esm anship - They occur close to the jo in t (but n o t in th e
jo in t). To confuse the issue they m ay have secondary Baker’s Cyst MUST be
located between the medial
invasion into the jo in t (10% ), how ever for the purpose o f
head of the gastrocnemius
m ultiple choice tests they “ never involve the jo in t.” A and the semimembranosus.
com m on trick is to show an ultrasound o f the leg w ith looks
like a B ak er’s cyst - but the m other fucker is too com plex - or If it’s NOT - you should
think Synovial Sarcoma -
has flow in it. *N ot everything in the popliteal fossa is a
and “next step” MRI.
B ak er’s cyst - especially on m ultiple choice.

Besides the “n o t-a-B ak er’s cyst” trick - there are 3 o ther w ays to show this. (1) as the “trip le
sig n ” , w hich is high, m edium , and low signal all in the sam e m ass (probably in the knee) on
T2, (2) as the “ bow l o f g r a p e s ” w hich is a bunch o f fluid -flu id levels in a m ass (probably in
the knee), or (3) as a plain x-ray w ith a soft tissue com ponent and calcifications - this w ould
be the least likely w ay to show it.

Synovial Sarcom a Trivia:


* M ost sarcom as d o n ’t attack bones; Synovial Sarcom a Can
* M ost sarcom as present as painless m ass; Synovial Sarcom as Hurt
* Soft tissue calcifications + Bone Erosions are highly suggestive
* They are slow grow ing and sm all in size, often leading to people thinking they are B9.
90% have a translocation o f X - 18.
* M ost com m on m alignancy in teens/young adults o f the foot, ankle, and low er
extrem ity

When I sa y “B all-like tu m or" in the extremity' o f a y o u n g adult, y o u sa y Synovial Sarcoma.


When I sa y “Soft Tissue Tumor in the F o o t ” o f a y o u n g adult, y o u sa y Syn o via l Sarcoma.

MFH (PUS) Synovial Sarcoma


OLD YOUNG
Central (Upper Thigh, Upper Arm) Peripheral (Foot, Knee)
T2 - Variable (Sometime Dark) T2 - “Triple Sign”

2 80
Lipom a vs A ty p ic a l L ip o m a vs L ip o s a rc o m a

These exist on a spectrum , w ith L ipom a being totally benign, and L iposarcom a being a bad
bad boy. A pearl is that, histologically, A typical L ipom a behaves and looks ju st like a low
grade Liposarcom a. It w ould be total horse-shit to ask you to tell those apart. It’s m ore likely
the distinction will be either L ipom a vs Liposarcom a.

Think about them like this:

Atypical Lipoma / Low Grade High Grade Liposarcoma


Lipoma
Liposarcoma “The One That Fucks You Up”
May not even have fat (for the
May have parts that are slightly
Signal Intensity parallels fat on exam it will have some -
darker (or brighter) than fat on
all sequences. otherwise you can’t even tell for
T1.
sure that it is a Liposarcoma)
May incompletely fat sat (or not
Will Fat Sat Out May incompletely fat sat
fat sat at all)
No Sepations (or thin ones) Thick Chunky Septations Thick Nodular Complex Stuff

Enhancing Components

Pearls:
•L iposarcom as tend to be D EEP (retroperitoneum )
% * ‘ L iposarcom as tend to be BIG
‘ Lipom as tend to be Superficial

Trivia: M yxoid L iposarcom a is the m ost com m on liposarcom a in patients < 20. T hey can be
T2 Bright (expected), but T1 dark (confusing) - d o n 't call it a cyst. A lso, d o n ’t call it a
com eback (I’ve been here for years). T h e y ’ll need gad+

281
Hemangioma
M a za b rau d S ynd ro m e
These are common.

Here are the tricks: It’s a totally zebra syndrom e - which


• T2 bright (like most tumors) m akes it totally appropriate for an
• Flow voids. They have to show you flow voids “ intermediate level exam .” It has 3 main
(buncha dark holes). findings:
• Hem angiom as don’t respect fascial boundaries
- they will infiltrate into stuff (this is a (1) Polyostotic Fibrous Dysplasia -
somewhat unique feature). which makes y o u ugly
• Enhances Intensely - Duh - they are a vascular (2) M ultiple Soft Tissue M yxomas
tumor (3) Difficulty finding a date to the prom
• They can contain fat - and likely will on — *see finding “ 1”
multiple choice.

Next Step: A great next step question would be to ask for a plain film. W hy a plain film?
phleboliths my friend — If they show you soft tissue phleboliths then hem angiom a is the answer.

M yxo m a

If this show s up on the exam , it is alm ost certainly going to be show n in the setting o f
M azabraud Syndrom e.

What do M yxom as Look Like? They are T2 bright (like every tumor), but tend to be lower signal
than muscle on T1 - which makes them sorta unique.

What does M arsellus Wallace Look Like ? *Hint - D on’t say “w hat?”

C T v s M RI T r e a t m e n t H ig h Y ie ld T r iv ia :
f o r L e s io n C h a r a c te r iz a tio n :
• O steosarcom a: C hem o first (to kill m icro
m e ts ) , follow ed by w ide excision
CT is Good for:
-Occult Bone Destruction •E w ings: Both C hem o and R adiation,
-Matrix and Mineralization — Example, follow ed by w ide excision.
better look at the lucent nidus o f an • C hondrosarcom a: U sually ju st w ide
osteoid osteoma.
excision (they are usually low grade, and
M R is Good for: m ain concern is local recurrence).
-Staging — specifically local extend and • G iant C ell Tumor: B ecause it extends to
tumor spread.
the articular surface usually requires
-Follow up - to assess response to
therapy. arthroplasty.

282
Other Soft Tissue Masses (and related conditions)

P ig m e n te d V illo n o d u la r S y n o v itis (P V N S ) : PV NS is an uncom m on benign


neoplastic process that m ay involve the synovium o f the jo in t diffusely or focally. It can also
affect the tendon sheath.

Intra-Articular D isease : Basically, it’s Synovial Proliferation + H em osiderin Deposition.


The knee is by far the m ost com m on jo int affected (65-80% ). On plain film, features you will
probably see are a joint effusion with or without m arginal erosions. O sseous erosions with
preservation o f the joint space and norm al m ineralization is typical. It is not possible to
distinguish PVNS from synovial chondrom atosis (see below) on plain film. MRI will be
obvious with bloom ing on gradient echo, and this is the m ost likely w ay they will show this.
Treatm ent is with com plete synovectom y, although recurrence rate is 20-50% .

Trivia: Unusual in kids, but when present is typically polyarticular.

Giant Cell Tumor o f the Tendon Sheath (P V N S o f the tendon): Typically found in the hand
(palm ar tendons). Can cause erosions on the underlying bone. Will be soft tissue density, and be
T1 and T2 dark (contrasted to a g lo m u s tu m o r which is T1 dark, T2 bright, a n d w ill enhance
uniformly).

P rim a ry S y n o v ia l C h o n d ro m a to s is : There are both prim ary and secondary types;


secondary being the result o f degenerative changes in the joint. The prim ary type is an
extrem ely high yield topic. It is a m etaplastic / true neoplastic process (not inflam m atory) that
results in the fonnation o f m ultiple cartilaginous nodules in the synovium o f joints, tendon
sheaths, and bursea. These nodules will eventually progress to loose bodies. It usually affects
one joint, and that one joint is usually the knee (70% ). It is usually a person in their 4 0 ’s or 5 0 ’s.

Joint bodies (which are usually m ultiple and uniform in size) m ay dem onstrate the ring and arc
calcification characteristic o f chondroid calcification. Treatm ent involves rem oval o f the loose
bodies with or w ithout synovectom y.

PVNS Synovial C h on drom atosis


Benign N eoplasia B enign N eoplasia
A ssociated w ith H em arthrosis N O T A ssociated w ith H em arthrosis
N ever C alcifies M ay C alcify

S e c o n d a r y S y n o v ia l C h o n d ro m a to s is : A lower yield topic than the prim ary type.


This is secondary to degenerative change, and typically seen in an older patient. There will be
extensive degenerative changes, and the fragm ents are usually few er and larger w hen com pared
to the prim ary subtype.

283
D ia b e t ic M y o n e c r o s is :

T his is basically infarction o f the m uscle seen in poorly controlled type 1 diabetics. It alm ost
alw ays involves the thigh (80% ), or c a lf (20% ). M RI will show m arked edem a w ith
enhancem ent and irregular regions o f m uscle necrosis. You should N O T biopsy this: it
delays recovery tim e and has a high com plication rate.

L ip o m a A rb o r e s c e n s :

This is a zebra that affects the synovial lining o f the jo in ts and bursa.
\\ kV

The buzzw ord is “fron d-like” deposition


o f fatty tissue.

It's seen in late adulthood (5 0 ’s-7 0 ’s), with


the m ost com m on location being the
suprapatellar bursa o f the knee. A lthough
it can develop in a norm al knee, it’s
often associated with O A , C hronic RA,
or prior traum a. It’s usually unilateral.
On M RI it’s going to behave like fat - T l
and T2 bright w ith response to fat
saturation.

A sneaky trick is to show this on gradient


- and have you pick up the chem ical shift
Lipoma Arborescens
artifact at the fat-fluid interface.

T his could also be show n on ultrasound w ith a “ frond-like hyperechoic m ass” and associated
jo in t effusion.

284
B one B io p sy
The route o f biopsy should be discussed w ith the orthopedic surgeon, to avoid
contam inating com partm ents not involved by the tum or (or not going to be used in the
resection process).

Special considerations:
• Pelvis: Avoid crossing gluteal m uscles (m ay be needed for reconstruction).
Knee: Avoid the jo in t space via crossing suprapatellar bursa or other
com m unicating bursae. Avoid crossing the quadriceps tendon unless it is
involved.
Shoulder: Avoid the posterior 2/3rd (axillary nerve courses post -> anterior,
therefore a posterior resection w ill denervate the anterior 1/3).

“D on’t Touch Lesions”


C haracteristically B enign Lesions that look A ggressive but are N O T
- and should N O T be biopsied because o f possibly m isleading pathology.

C ircum ferential Can look scary on M RI if


M yositis O ssificans calcifications w ith a lucent im aged early because o f
center edem a, and avid enhancem ent

Typical location near the Can have an aggressive


Avulsion Injury
pelvis periosteal reaction

C an be hot on bone scan.


C haracteristic location on
the posterior m edial
N O T a desm oid (despite the
epicondyle o f the distal
C ortical D esm oid nam e). It’s actually a tug
femur.
lesion from the m edial
gastrocnem ius and A D D uctor
Bilateral 30% o f the tim e.
m agnus.
C haracteristic location in the
anterosuperior fem oral neck.
Lytic appearing lesion

Synovial H erniation Pit


“ P itt’s Pit”
-f* A ssociated w ith fem oral
acetabular syndrom e
(probably).

285
S E C T IO N B'
t
A R T H R IT l's ^

A rthritis is tricky. A nne B row er w rote a book c a lle d A rthritis in B lack a n d W hite, w hich is
probably the best book on the subject. The problem is that book is 415 pages. So, I ’m going
to try and offer the 10 page version.

Epidem iology’

A lthough there are over 90 different rheum atic diseases recognized by the A m erican C ollege
o f R heum atology, only a few tend to show up on m ultiple choice tests (and at the view box).

You can broadly categorize arthritis into 3 categories:


D egenerative (O A , N europathic)
Inflam m atory (RA and Variants)
M etabolic (G out, C PPD )

D e g e n e ra tiv e :

Osteoarthritis is the most common cause. The pathogenesis is that you have mechanical breakdown
(hard work) which leads to cartilage degeneration (fissures, micro-fractures) and fragmentation of
subchondral bone (sclerosis and subchondral cysts). You get all the classic stuff, joint space
narrowing (NOT symmetric), subchondral cysts, endplate changes, vacuum phenomenon, etc... The
poster boy is the osteophyte.

Neuropathic Joint. The way the case is classically shown is a bad joint followed by the reason for a
bad joint (syringomyelia, spinal cord injury, e tc...). A way to think about this is “osteoarthritis with a
vengeance A The buzzword is “Surgical Like Margins.” Basically nothing else causes this kind o f
destruction. I like to describe the joints as a deformity, with debris, and dislocation, having dense
subchondral bone, and destruction o f the articular cortex. The classic scenario is a shoulder that looks
like it’s been amputated, and then they show you a syrinx.

Charcot Foot - The classic example of a diabetic neuropathic foot with


the deformity, with debris, and dislocation, having dense subchondral
bone, and destruction of the articular cortex - favoring the midfoot
eventually causing a “rocker-bottom deform ity” of the foot resulting
from the collapse of the longitudinal arch.

This vs That - Charot vs Infection: Diabetics get neuropathic feet and infections - so there can be overlap.
To tell them apart you can look for the presence o f an ulcer or sinus tract (that infers infection).
Location is helpful - charcot prefers the midfoot (osteomyelitis prefers the pressure points o f the forefoot -
metatarsal heads, IP joints - and the posterior plantar aspect o f the calcaneus).

286
In fla m m a to ry :

Erosive O steoarthritis (Inflam m atory


O steoarthritis). The buzzw ord is “gull w ing”,
w hich describes the central erosions. It is seen in
postm enopausal w om en and favors the D IP joints.

R heum atoid A rthritis: T here is a ton o f trivia


mm
HHHH i HflH
Erosive OA - Gullwing

related to this disease. It’s not a disease o f bone production. Instead it is characterized by
osteoporosis, soft tissue sw elling, m arginal erosions and uniform jo in t space narro w in g . It’s
often bilateral and sym m etric. C lassically spares the DIP jo in ts (opposite o f erosive OA ).

Trivia: The 5th M etatarsal head is the first spot in the foot

RA in the H a n d P earls - Expect the PIP jo in ts to be involved A F T E R the M CP jo in ts. The


First CM C is classically spared (or is the last carpal to be involved). The first C M C should
N O T be first. O bviously O A loves the first C M C so this is helpful in separating them .
Psoriasis, on the other hand, also tends to m ake the first C M C go last.

Felty Syndrom e: RA > 10 years + Splenom egaly + N eutropenia

* C aplan Syndrom e: R A + P neum oconiosis

The distribution o f RA vs OA in the


hip is a classic teaching point:
Axial

OA RA

287
Rheum atoid V ariants:

* Psoriatic A rthritis
* R eiter’s syndrom e (Reactive arthritis)
* A nkylosing Spondylitis
* Inflam m atory Bowel D isease

Psoriatic A rthritis: This is seen in 30% o f patients w ith psoriasis. In alm ost all cases
(90% ) the skin findings com e first, then you get the arthritis. A s a point o f trivia, there is a
strong correlation betw een involvem ent o f the nail and involvem ent o f the DIP jo in t. The
classic description is “erosive change w ith bone proliferation (IP jo in ts > M CP joints).
The erosions start in the m argins o f the jo in t and progress to involve the central portions
(can lead to a “pencil sharpening” effect). The hands are the m ost com m only affected
(second m ost com m on is the feet). Up to 40% o f cases will have SI jo in t involvem ent
(asym m etric).

A dditional B uzzw ords


* “ Fuzzy A ppearance” to the bone
around the jo in t (bone proliferation)
Sausage D igit - w hole digit has soft * * *
# ** * *
tissue sw elling ★
*
Ivory Phalanx - sclerosis and/or *
*
bone proliferation (m ost com m only
the great toe)
Pencil in C up D eform ities IP Joints Ray Pattern Pencil Mouse
* A nkylosis in Finger in Cup Ears
* “ M ouse Ears”
A cro-osteolysis

When I say A n kylosis in the H and, You S a y (1) E rosive OA o r (2) Psoriasis
..............
RA P s o ria s is M u tila n s

Symmetric Asymmetric Severe bone resorption leading to


Proximal Distal soft tissue “telescoping” collapse.
(favors MCP, carpals) (favors IP joints)
Trivia: If you
Osteoporosis No Osteoporosis pulled on the
patient’s fingers
they would
Bone Proliferation
No Bone Proliferation lengthen - but who
- the form of periostitis
would want to
Can Cause “Mutilans” Can Cause “Mutilans” touch a patient ?
When Severe When Severe Yuck!

288
R e ite r ’s (R eactive A rthritis): R eiter’s Triad:

Urethritis
A pparently R eiter w as a Nazi (killed a bunch o f people w ith
Conjunctivitis
typhus vaccine experim ents). So, people try not to give him any Arthritis
credit for things (hence the nam e change to R eactive arthritis).
Regardless o f w hat you call it, it’s a very sim ilar situation to (Can't See or Climb a
Tree to Pee on a Nazi
Psoriatic arthritis - both have bone proliferation, erosions, and
named Reiter).
asym m etric SI jo in t involvem ent.

You Know Who Else


The difference is that R eiter’s is rare in the hands (tends to
Was A Nazi?
affect the feet m ore). Just rem em ber R e ite r’s favors things below
the w aist (like the penis = urethritis, and the foot). H enry Ford
- Google It.

A n k y lo s in g S p o n d y litis : This disease favors the spine and SI jo ints. The classic
buzzw ord is “bam boo sp in e” from the syndesm ophytes flow ing from adjacent vertebral
bodies. Shiny com ers is a buzzw ord, for early involvem ent. A s you m ight im agine, these
spines are susceptible to fracture in traum a. SI jo in t in volvem en t is usually the first site
(sym m etric). The jo in t actually w idens a little before it narrow s. A s a point o f trivia, these
guys can have an upper lobe predom inant interstitial lung disease, w ith sm all cystic spaces.

N ext Step - A ny significant A nk Spon / D ISH + Even M inor T raum a = W hole Spine CT

R andom High Yield Topic: A n k ylosin g Spon dylitis in the Hip

W hen the peripheral skeleton is involved in p a tie n t’s w ith A nk Spond, think about the
shoulders and hips (hips m ore com m on). Hip involvem ent can be very disabling.

H eterotopic O ssification tends to occur post hip replacem ent or revision. It occurs so
m uch that they often get postoperative low dose radiation and N S A lD s to try as
prophylactic th era p y ______________________________________ ____________________________
If they show you norm al SI jo in ts - then show you anything in the spine it’s not AS. It has
to hit the SI jo in ts first (especially on m ultiple choice).

In fla m m a to r y B o w e l D is e a s e ( E n t e r o p a t h ic ) - A llegedly 20% o f p a tie n t’s


w ith C rohns & UC have a chronic inflam m atory arthritis. T he im aging findings occurs in
two distinct flavors.

(A): A xial A rthritis (favors SI jo in ts and spine) - often unrelated to bow el disease
(B): Peripheral A rthritis - this one varies depending on the severity o f the bow el disease.

289
S I Jo in t Involvem ent P a tte rn s ( R h e u m a t o id V a ria n ts )

P soriasis, Reiters Inflammatory Bowel, A S

Psoriatic A rthritis Reiters (R eactive) A nkylosing Spondylitis


M=F M>F M>F
Asymmetric SI Joint Asymmetric SI Joint Symmetric SI Joint
Hands, Feet, Thoracolumbar Spine Feet, Lumbar Spine, SI joint SI joint, Spine (whole thing)

M e ta b o lic :

G o u t: This is a crystal arthropathy from the deposition o f uric acid crystals in and around the
joints. It’s alm ost alw ays in a m an over 40. The big toe is the classic location.

Buzzw ords / Things to K now :


* E arliest Sign = Joint Effusion G out on MR

Spares the Joint Space (until late in the disease); Juxta- • Juxta-articular soft
articular Erosions - aw ay from the joint. tissue mass (LOW
ON T2).
“Punched out lytic lesions”
• The tophus will
* “ O verhanging E dges” typically enhance.
Soft tissue tophi

G out M im ickers:
There are 5 entities that can give a similar appearance to a gouty arthritis, although they are much
less common. This is the mnemonic I was taught in training:

“American Roentgen Ray Society Hooray”


Am yloid
RA (cystic)
Reticular Histocytosis (the most rare)
Sarcoid
Hyperlipidemia

290
C PPD : C alcium Pyrophosphate D ihydrate D isease is super com m on in old people. It often
causes chondocalcinosis (although there are other causes). Synovitis + C PPD =
“ Pseudogout.” C PPD loves the triangular fibrocartilage o f the w rist, the peri-odontoid tissue,
and intervertebral disks. A nother im portant phrase is “degen erative change in an
uncom m on jo in t” - shoulder, elbow, patellofem oral jo in t, radiocarpal jo in t. H aving said
that, pyrophosp hate arth rop athy is m ost com m on at the knee.

* I f y o u see iso la ted disease in the patellofem oral, radiocarpal, o r talonavicular jo in t,


think CPPD.
* H o oked M C P O steophytes with chondrocalcinosis in the TFC C is a classic took
(although hem ochrom atosis can also look that way).

CPPD can (and does com m only) cause SLA C w rist by degenerating the SL Ligam ent.

TH IS vs THAT — OA vs CPPD ?
There are m any overlapping features including jo in t space narrow ing, subchondral
sclerosis, subchondral cyst, and osteophyte form ation. H ow ever, C PPD has som e unique
features, such as an “ atypical jo in t distribution” - favoring com partm ents like the
patellofem oral or radiocarpal. Subchondral cyst form ation can be bigger than expected.

H e m o c h ro m a to s is :

This iron overload disease also is know n for calcium


pyrophosphate deposition and resulting
chondrocalcinosis. It has a sim ilar distribution to V
CPPD (M C P joints). Both C PPD and
H em ochrom atosis will have “hooked osteophytes”
at the M CP joint.

THIS v s THAT CP PD a n d H em ochrom atosis: Hemo cppd


Hemo CPPD
H em ochrom atosis has uniform jo in t space loss at Hemo
ALL the M CP jo in ts. C PPD favors the index and Hemo
m iddle finger M CPs.
/
Trivia: As a point o f trivia, therapy for the
system ic disease does N O T affect the arthritis. H o o k e d O s te o p h y te s
—C P P D vs Hemochromatosis

291
“ M ilw a u k e e S h o u ld e r ” - This is an apocalyptic
destruction o f the shoulder (a lm o st looks n e u ro p a th ic )
secondary to the dem on m ineral h y d ro x y a p a tite .

The articular surface changes will be very advanced,


and you have a lot o f intra-articular loose bodies.
Humeral Head has
The hum eral head will look like it needed the U nited been massacred.
N ations to delivery a binding resolution to keep the
-I’m talking scorched
Earth Mother Fuckers
hydroxyapatite from fucking destroying it.

C lassic History’: O ld w om en w ith a history o f traum a to


that joint.

H y p e r p a r a th y r o id is m - A s you m ay rem em ber from m edical school, this can be


prim ary or secondary, and its effects on calcium m etabolism typically m anifest in the bones.
Here are your buzzw ords: “ Subperiosteal bone resorption' ’ o f the radial aspect o f the 2nd and
3rd fingers, rugger-jersey spine, brow n tum ors, and term inal tuft erosions.

The classic w ays this can be show n:


’ Superior a n d inferior rib notching - bone resorption
R esorption along the radial aspect o f the fin g e r s w ith brown tum ors
* Tuft Resorption
R ugger Je rse y Spine
P elvis with N arrow ing o r “C onstricting ” o f the fe m o r a l necks, a n d w ide S I jo in ts.

Hyperparathyroidism

Subperiosteal Resorption, Tuft Resorption Rugger Jersey Spine Brown Tumor


and brown tumors

292
P r o b le m S o lv in g :

If you are given a picture o f a hand or foot and asked w hat the arthritis is, it w ill probably be
obvious (they show a gull-w ing for erosive O A , or bad carpals for RA, or the pencil in cup for
psoriasis, or the 5th m etatarsal for RA). If it’s not m ade obvious w ith an “ A unt M innie”
appearance, I like to use this approach to figure it out (I also use this in the real w orld).

Infection

Has one or more: One Joint

-Symmetric Joint
* No Bony Proliferation
Proximal Distribution
RA
Inflam m atory
Space Narrowing
-Erosions /
Multiple Joints
AS
\ Bony Proliferation
Distal Distribution
p SOriasis
R eactive
Inflam m atory
Bowel Related

Typical Joints (Hands, Knees) OA


Has one or more:
/
-Post Traum atic
-Asymmetric Joint ►
D egenerative Atypical Joint -G out or CPPD
Space Narrowing Atypical Age -H em ophilia

\
-Osteophytes

Atypical Joint
Atypical Age N europathic
SEVERE or “Surgical Destruction"

293
Spine D egenerative C hange:

In the real w orld it’s usually ju st m ultilevel degenerative change. But in m ultiple choice
w orld you should be thinking about other things. Shiny com ers w ith early A S, or flow ing
syndesm ophytes w ith later AS. DISH w ith the bulky osteophytes sparing the disc space.
The big bridging lateral osteophyte is classically show n for psoriatic arthritis.

V ertebral O ssifications C ervical Spine:


G a m e sm a n sh ip
“ Flow ing A nkylosing
B am boo Spine
S yndesm ophytes” Spondylitis
Fusion: Either
congenital (Klippel-
D iffuse Paravertebral Feil) or Juvenile RA.
DISH O ssification o f A L L
O ssifications
Erosions of the
Dens: CPPD and RA
Focal Lateral
O ssification o f famously do this.
Paravertebral Psoriatic A rthritis
A nnulus Fibrosis
O ssification Bad Kyphosis = NF1

DISH (D iffuse Idiopathic Skeletal H y p e r o sto sis):

You see ossification o f the anterior longitudinal ligam ent involving m ore than 4 levels with
sparing o f the disc spaces, you say DISH. T he thoracic spine is m ost com m only used.
These guys often have bony proliferation at pelvis, ischial tuberosities, trochanters, and iliac
crests. There is no sacroiliitis (helps you differentiate from AS).

O PL L (O ssification o f the Posterior L on gitudinal L igam ent):

This is an ossification o f the posterior longitudinal ligam ent. It is associated w ith DISH,
ossification o f the ligam entum flavum , and A nkylosing Spondylitis. It favors the cervical
spine o f old A sian men. It can cause spinal canal stenosis, and can lead to cord injury after
m inor traum a. A key point is that it’s bad new s in the cervical spine (w here it is m ost
com m on); in the thoracic spine it is usually asym ptom atic.

D estructive Spondyloarthropathy.:

This is associated w ith patients on renal dialysis (for at least 2 years), and it m ost com m only
affects the C -spine. It looks like bad degenerative changes or CPPD . A m yloid deposition is
supposedly w hy it happens.

294
Misc Stuff That’s Sorta in the Arthritis Category:

System ic Lupus E rythem atosus:

The Aunt Minnie look is reducible deformity


of joints without articular erosions. Joint
space narrowing and erosions are uncommon
findings. They can show you the hands with
ulnar subluxations at the MCPs on Norgaard
view, then they reduce on AP (because the
hands are flat).

This ligamentous laxity also increases risk o f


patellar dislocations.

S L E : Shows Reversible Ulnar Deviation

Jaccou d ’s A rthropathy: This is very similar to SLE in the hand (people often say them together).
You have non erosive arthropathy with ulnar deviation o f the 2nd-5th fingers at the MCP joint. The
history is post rheumatic fever.

M ixed C on nective T issue D isease: One unique feature is that it is positive for some antibody -
Ribonucleoprotein (RNP) - and therefore serology is essential to the diagnosis.

Juvenile Idiopathic A rthritis: This occurs before age


16 (by definition). What you see is a washed out hand
that has a proximal distribution (carpals are jacked),
and is ankylosed (premature fusion of growth plates).
Serology is often negative (85%). In the knees, you see
enlargement o f the epiphyses and widened intercondylar
notch - similar to findings in hemophilia.

Buzzword: “E piphyseal O vergrow th”

A m yloid A rthropathy: This is seen with patients on dialysis (less commonly in patients with chronic
inflammation such as RA). The pattern o f destruction can be severe - similar to septic arthritis or
neuropathic spondyloarthropathy. The distribution is key, with bilateral involvement of the shoulders,
hips, carpals, and knees being typical. Carpal tunnel syndrome is a common clinical manifestation.
The joint space is typically preserved until later in the disease. When associated with dialysis, it’s rare
before 5 years of treatment, but very common after 10 years (80%).

Pituitary G igantism : If they happen to show you x-rays o f Andre the Giant, look for “widening of
the joint space in an adult hip” - can be a classic buzzword. Late in the game, the cartilage will
actually outgrow its blood supply and collapse, leading to early onset osteoarthritis. The formation of
endochondral bone at existing chondro-osseous junctions results in widening o f osseous structure.

295
This is a confusing topic and there are entire books on the subject. I ’m going to attem pt to
hit the m ain points, and sim plify the subject.

Bone m arrow consists o f three com ponents: (1) T rabecular Bone - the support structure, (2)
Red M arrow - for m aking blood, and (3) Yellow M arrow - f a t for a purpose unknow n at this
tim e.

M arrow C onversion: The basic rules are that


yellow m arrow increases w ith age, in a
predictable and progressive way. This is usually
com pleted by the m id 20s. You are bom w ith all
red m arrow , and the conversion o f red to yellow
occurs from the extrem ities to the axial skeleton
(feet and hands first). W ithin each long bone the
progression occurs epiphyses / apophyses first ->
diaphysis -> follow ed by the distal m etaphysis ,
and finally the proxim al m etaphysis. Red Red Marrow Coverts to Yellow Marrow from
Distal to Proximal
m arrow can be found in the hum eral heads
and fem oral heads as a norm al varian t in
adults.

As a child, you have


diffuse red m arrow
except for ossified
epiphyses and apophyses.

Yellow As adults, you have


Marrow
Expands yellow m arrow
out from the everyw here except in the
Middle
axial skeleton, and
proxim al inetaphyses o f
proxim al long bones.

Baby Child

296
F e w P e a r ls o n M a r r o w :

* Yellow m arrow increases w ith age (as trabecular bone decreases w ith osteoporosis,
yellow m arrow replaces it).
* T 1 is your m oney sequence: Yellow is bright, R ed is darker than yellow (near iso­
intense to m uscle).
Red m arrow should never be darker than a norm al disk or m uscle on T1 (think about
m uscle as your internal control).
* Red m arrow increases if there is a need for m ore hem atopoiesis (reconversion -
occurs in exact reverse order o f norm al conversion)
* M arrow turns yellow w ith stress / degenerative change in the spine

T h r e e m o s t c la s s ic m a r r o w q u e s tio n s :

( I Q ) What is the norm al p a ttern o f conversion ?

(A) The epiphyses convert to fatty m arrow alm ost im m ediately after ossification. Distal then
proceeds m edial / proxim al (diaphysis first, then m etaphysis).

(2 Q) What is the norm al p a tte rn o f R e c o n ve rsio n ? Spine

(A) The pattern o f reconversion: T his occurs in the reverse


Flat Bones
order o f norm al m arrow conversion, beginning in the axial
skeleton and heading peripheral. The last to go are the m ore
distal long bones. Typically, the epiphyses are spared unless Long Bone Metaphysis
the hem atopoietic dem and is very high.
Long Bone Diaphysis

I
Long Bone Epiphyses

(3 Q) W hat areas are sp a red / norm al variants?

(A) Patchy areas o f red m arrow m ay be seen in the proxim al fem oral m etaphysis o f
teenagers. D istal fem oral sp arin g is seen in teen agers and m en stru atin g w om en.

297
Leukem ia:

Proliferation o f leukem ic cells results in replacem ent o f red m arrow. M arrow w ill look
darker than m uscle (and norm al disks) on T l. On STIR , m arrow m ay be brighter than
m uscle because o f the increased w ater content. T2 is variable, often looking like diffuse red
marrow.

They can show leukem ia in tw o m ain ways:

(1) Lucent m etaphyseal bands in a kid ^

(2) T l-w e ig h ted M RI show ing m arrow darker than


adjacent disks and m uscle. R em em ber that Red
M arrow is still 40% fat and should be brighter than
m uscle on T l.

M ost infdtrative conditions affect the m arrow diffusely.


The exceptions are m ultiple m yelom a, w hich has a
predilection fo r fo c a l “sp e ck led ” deposits, a n d
W aldenstrom's m acroglobulinem ia, w hich causes infarcts.
T1 S a g - Marrow is Darker
than Disk —

C h lo ro m a ( G r a n u lo c y tic S a r c o m a )

Just say “destructive m ass in a bone o f a leukem ia


patient. ” It’s som e kind o f colloid tumor.

eg. skull

298
S E C T I O N 1 O:
U ltr a so u n d

It’s absolutely incredible that 1 even need to go over this, but dinosaur radiologists love this
stuff. Plus it is popular in E urope, so logically it belongs on an interm ediate exam in the US.

A n is o tro p y : The m ost com m on and


m ost problem atic issues with
Normal
ultrasounding tendons is this thing called Appearing
“anisotropy.” The tendon is norm ally Hyperechoic
hyperechoic, but if you look at it w hen
Tendon

it’s N O T perpendicular to the sound


beam it can look hypoechoic (injured?).
Exact same
It s the biggest p a in in the ass: tendon - now
appearing
"S upraspinatus tendon - as it curves hypoechoic -
when
along the contours o f the hum eral head scanned non
*Long H ead o f the Biceps - In the perpendicular
bicipital groove

T e a r s : The tendon is usually hyperechoic. Focal hypoechoic areas are tears. It can be
really tricky to tell if it’s partial or com plete (th a t’s w hat M RI is for).

299
T e n o s y n o v itis : A s discussed above, there are a variety o f causes. If they show it on
ultrasound, you are looking for increased fluid w ithin the tendon sheath. You could also see
associated peritendinous subcutaneous hyperem ia on Doppler.

Tenosynovitis - Increased fluid in the tendon sheath

P la n ta r F a s c iitis : T his is another


pathology that lends itse lf to a “w hat is it ? ”
type o f ultrasound question. H opefully, they at . - -+
least tell you this is the foot (they could label
the calcaneus). The finding will be thickening
PF +
o f the plantar fascia (greater than 4 m m ), w ith CALC

loss o f the norm al fibrillar pattern. If you see


Plantar Fasciitis
calipers on the plantar fascia - this is going to
be the answer.

Trivia - M ost com m only involves the central band (there are 3 bands - people w ho d o n ’t
know anatom y think there are two).

C a lc ific T e n d o n itis : As described above, this is very com m on and related to


hydroxyapatite. The m ost com m on site is the supraspinatus tendon, near its insertion. It will
shadow ju st like a stone in the GB.

Calcific Tendonitis - Shadowing calcification in the classic location (supraspinatus)

300
a r-. ,1J . L
m P R O C ED U R ES

An im portant point to rem em ber is that the target is not actually the joint. The target is the
capsule. In other words, you ju st need the needle to touch a bone w ithin the capsule. The trick
is to do this w ithout causing contam ination or dam aging an adjacent structure (like an artery).
G eneral Tip - Avoid putting air in the joint, as this will cause susceptibility artifact.

Hip: The general steps are as follows: ( l ) M ark the


fem oral artery. (2) Internally rotate the hip (slightly)
to localize the fem oral head-neck junction (your
target). (3) Clean and num b the skin. (4) A dvance a
20-22 gauge spinal needle into the jo in t - straight
down on the superior head neck junction. (5) Inject a
small am ount o f contrast to confirm position. Contrast
should flow away from the tip. If the contrast just
stays there it’s not in the capsule. (6) Put the rest o f the
contrast in.

Trivia: Capsule is w idest at the head-neck junction.

Trivia: The cocktail injected is around 14 cc total


(4 cc Lidocaine, 10 cc Visipaque, and only about 0.1 cc Gd).

Shoulder: The general steps are as follows: (1)


Supinate the hand (externally rotate the shoulder) (2)
Clean and num b the skin. (3) A dvance a 20-22 gauge
spinal needle into the jo in t - straight dow n on the
junction betw een the m iddle and inferior third o f the
hum eral head - 2 m m inside the cortex. (4) O nce you
strike bone, pull back 1 m m and turn the bevel tow ards
the hum eral head - this should drop into the jo int (5)
Inject a small am ount o f contrast to confirm position.
Contrast should flow away from the tip. If the contrast
ju st stays there it’s not in a space. (6) Put the rest o f
the contrast in.

Trivia: The cocktail injected is around 12cc total


(4cc Lidocaine, 8cc Visipaque, and only about 0.1 cc Gd).

301
S p e c ia l S itu a tio n s - P ro b le m S olvin g

So there are three m ain reasons that you m ight get asked to put a needle in a jo in t

1 - N eeds Arthrogram to E valuate the Lahrum

2 - N eeds S teroid / L idocaine Injection f o r P ain M anagem ent

3 - P ossible Jo in t Infection - N eeds A spiration

Only one o f these is going to end up getting im aged (the arthrogram ), the other two ju st need
the needle in the correct spot - and it d o e sn ’t m atter w hat you had to do to get it there.

Scen ario 1 - Patient is allergic to (has a phobia of) G d, but needs an arthrogram ?

You could try a CT A rthrogram . So - not G ad, ju st V isipaque and Lidocaine.

Scen ario 2 - Patient is allergic to (has a phobia of) C T C ontrast (Visi or O m ni), but needs a
steroid jo in t injection?

You can inject air into the jo in t (instead o f visi) to confirm placem ent - then put the steroid in.

Scen ario 3 - Patient is allergic to (has a phobia of) C T C ontrast (Visi or O m ni), but needs an
arthrogram ?

This situation is different. You c a n ’t inject air because you w ill end up w ith a big bloom ing
m ess on MR. CT isn ’t an option either - because obviously you need the C T C ontrast or it’s
not an arthrogram . Your only choices w ould be to either (1) pre-m edicate them , or (2) use the
force (trust your fe e lin g s...) and hope you can get in the jo in t w ithout confirm ing positioning
with fluoro.

302
B la n k fo r S c r ib b le s
304
12
rrrnirimiiEiiimirrairfmiiTmiiTmiirairfmiiEirteiirmirEirEJirmi

Va scular
ITB]ffMirr^lfTglfTm]fTglfrB]frg]fTMirE]n^]fTM]fTg][rmJiraJfTMI

P r o m e t h e u s L io n h a r t , M .D .

305
S E C TIO N 1‘

t A n a t o m y 1111111111111 ^ P *

A orta: The thoracic aorta is divided anatomically into four regions; the root, the ascending aorta,
the transverse aorta (arch), and the descending aorta. The “root” is defined as the portion o f the aorta
extending from the aortic valve annulus to the sino-tubular junction. The diameter o f the thoracic
aorta is largest at the aortic root and gradually decreases (average size is 3.6 cm at the root, 2.4 cm in
the distal descending).

• Sinuses o f Valsalva: There are 3 outpouchings (right,


left, posterior) above the annulus that terminate at the Isthmus
ST Junction. The right and left coronaries come off the
right and left sinuses. The posterior cusp is sometimes Ductus
Bump
called the “non-coronary cusp.”

• Isthmus: The segment o f the aorta between the origin


of the left Subclavian and the ligamentum arteriosum.

• Ductus bump: Just distal to the isthmus is a contour


bulge along the lesser curvature, which is a normal
structure (not a pseudoaneurysm).

A o rtic Arch V a ria n ts : There are 4 common variations:

Normal (75%), Bovine Arch


( 15%) - common origin of
brachiocephalic artery and left
common carotid artery, left
common carotid coming off
the brachiocephalic proper
(10%), and in 5% o f people
the left vertebral artery
originates separately from the
arch. Branching with regards Left CC off Separate Origins
Normal Bovine
to right arch, left arch and Brachiocephalic
double arch was discussed in
more detail in the cardiac chapter.

Pulmonary Sling: Aberrant Left pulmonary artery coming off the


right pulmonary artery.
-Unique as the only anomaly to create indentations in the posterior
trachea and anterior esophagus.
-Unique as the only anomaly that can cause stridor in a patient with
a normal left sided arch.

306
A d a m k iew ic z:

The thoracic aorta puts off multiple


important feeders including the great
anterior medullary artery (Artery of
Adamkiewicz) which serves as a dominate
feeder o f the spinal cord.

This thing usually comes off on the left side


(70%) between T9-T12.
H airpin T urn”

“Bew are o f the H airpin Turn”


-The classic angiographic appearance of the artery is
the "hairpin turn" as its anastomosis with the anterior
spinal artery.

M e s e n te ric Branches:

The anatomy o f the SMA and IMA is high yield, and can be shown on a M1P coronal CT, or
Angiogram. I think that
Inferior SMA knowing the inferior
Pancreatico­ pancreaticoduodenal
Arc of Riolan
duodenal . comes off the SMA first,
and that the left colic
Middle (from IMA) to the
Colic Left Colic middle colic (from
SMA) make up the Arc
Right- Marginal of Riolan are probably
Colic Artery the highest yield facts.

Sigmoid
lleo- Colic
Branches

Appendicular A.
\,ett
YteP,at'c
C elia c B ranches:

The classic branches o f the celiac axis are the


common hepatic, left gastric, and the splenic
arteries.
Celiac
The “common” hepatic artery becomes the Trunk

“proper” hepatic artery after the GDA.


GDA
This “traditional anatomy” is actually only seen in SMA

55% of people.

307
C e lia c Anatom y - R em em ber this can be show n w ith an angiogram , CTA, or M RA .

Left Hepatic Artery

Left Gastric Artery


/
Right Hepatic Artery
? *
\
/ * Splenic Artery
Heoatic Artery
Common Hepatic Arterv /
Celiac Trunk

V a r ia n t H e p a tic A r t e r y A n a to m y - The right hepatic artery and left hepatic arteries


m ay be “replaced” (originate from a vessel other than the proper hepatic) or duplicated -
w hich anatom ist called “accessory.” This distinction o f “replaced” vs “accessory” w ould
m ake a great m ultiple choice question.

Trivia to know:

• R eplaced = D ifferent O rigin, usually o ff the left gastric or SM A

• A ccessory = D uplication o f the Vessel, w ith the spare com ing o ff the left gastric or SM A

• If you see a vessel in the fissure o f the


Iigam entum venosum (w here there is
not norm ally a vessel), it’s probably an
accessory or replaced left hepatic
artery arising from the left gastric
artery.

• The proper right hepatic artery is


anterior to the right portal vein,
w hereas the replaced right hepatic
artery is posterior to the m ain portal
vein. T his positioning o f the replaced
Vessel in the Ligamentum Venous
right increases the risk o f injury in (probably rep la ced left hepatic)
pancreatic surgeries.

308
Ilia c A n a to m y : The branches o f the internal iliac are high yield, w ith the m ost likely
question being “w hich branches are from the p osterior or anterior div isio n s?” A useful
m nemonic is “I L ove Sex, ” Illiolumbar, L a tera l Sacral, S u perior Gluteal, fo r the p o sterio r
division.

M y trick for rem em bering that the m nem onic is for posterior and not anterior is to think o f
that super religious girl I knew in college — 1 Like Sex in the butt / posterior.

I d o n ’t think they will actually show a picture, it’s w ay m ore likely to be a w ritten question.

Anterior Division Posterior Division


Umbilical Iliolumbar
Superior Vesicular (off umbilical) Lateral Sacral
Inferior Vesicular Superior Gluteal
Uterine (if you have a uterus) Inferior Gluteal *** sometimes
Middle Rectal
Internal Pudendal
Inferior Gluteal
Obturator
Trivia: The ovarian arteries arise from the anterior-m edial aorta 80-90% o f tim e.

309
P e r s is te n t S c ia t ic A r t e r y - An anatom ic variant, w hich is a continu ation o f the
internal iliac. It passes posterior to the fem ur in the thigh and then w ill anastom ose w ith the
distal vasculature. C om plications w orth know ing include aneurysm form ation and early
atherosclerosis in the vessel. T he classic vascular surgery boards question is “external iliac is
acutely occluded, but there is still a strong pulse in the foot” , the answ er is the patient has a
persistent sciatic.

Persistent Sciatic Artery

Mesenteric Arterial Collateral Pathways:


C eliac to SM A: T he conventional collateral pathw ay is C eliac -> C om m on H epatic ->
G D A -> Superior Pancreatic D uodenal -> Inferior Pancreatic D uodenal -> SM A.

Arc ofB uhler: This is a variant anatom y (seen in like 4% o f people), that represents a
collateral pathw ay from the celiac to the SM A. The arch is independent o f the G D A and
inferior pancreatic arteries. This rare collateral can have an even m ore rare aneurysm , w hich
occurs in association w ith stenosis o f the celiac axis.

Hepatic Celiac

310
S M A to IM A : The conventional collateral pathw ay is SM A -> M iddle C olic -> Left
Branch o f the M iddle C olic -> Arc o f Riolan (as below ) -> Left C olic - > IM A.

Arc o f Riolan - A lso referred to as the


m eandering m esenteric artery. C lassically a
connection betw een the m iddle colic o f the
SM A and the left colic o f the IM A.

M arginal Artery’ o f D rum m ond - This is


another SM A to IM A connection. The Arc of Riolan
Marginal Artery
anastom osis o f the term inal branches o f the of Drummond
ileocolic, right colic and m iddle colic arteries
o f the SM A , and o f the left colic and sigm oid
branches o f the IM A , form a continu ous
arterial circle or arcade along the inner
border o f the colon.

IM A to llia c s : The conventional collateral pathw ay is IM A -> S uperior Rectal -> Inferior
Rectal -> Internal Pudendal -> A nterior branch o f internal iliac.

IMA
W in s lo w P a t h w a y - This is a collateral pathw ay that is seen in the setting o f aorto-iliac
occlusive disease. The pathw ay apparently can be inadvertently cut during transverse
abdom inal surgery. The pathw ay runs from subclavian arteries -> internal thoracic (m am m ary)
arteries -> superior epigastric arteries -> inferior epigastric arteries -> external iliac arteries.

C o ro n a M o r tis - C lassically described as a vascular connection betw een the obturator


and external iliac. Som e authors describe additional anastom otic pathw ays, but you should
basically think o f it as any vessel coursing over the su perior pubic rim , regardless o f the
anastom otic connection. The “crow n o f death” is significant because it can (a) be injured in
pelvic trau m a or (b) be injured during surgery - and is notoriously difficult to ligate.
Som e authors report that it causes 6-8% o f deaths in pelvic traum a. The last piece o f trivia is
that it could hypothetically cause a type 2 endoleak.

312
Upper Extremity Anatomy:
The scalene muscles make a triangle in the neck. If you have ever had the pleasure o f reading a
brachial plexus MRI finding this anatomy in a sagittal plane is the best place to start (in my
opinion). The relationship to notice (because it’s testable) is that the subclavian vein runs anterior to
the triangle, and the subclavian artery runs in the triangle (with the brachial plexus).

Trivia to Remember: The subclavian artery runs posterior to the subclavian vein.

Transverse Process C2-C3

Brachial Plexus

Middle Scalene Muscle


Anterior Scalene Muscle

First Rib

Subclavian VEIN
I

Phrenic Nerve
Subclavian ARTERY

The subclavian artery has several major branches: the vertebral, the internal thoracic, the
thyrocervical trunk, the costocervical trunk, and the dorsal scapular.

As the subclavian artery progresses down the arm,


anatomists decided to change it’s name a few times. This
name changing makes for great multiple choice fodder.

The highest yield thing you can know with regard to


upper extremity vascular anatomy is when stuff becomes
stuff:

• Axillary Artery: Begins at the first rib


Axillary A.
• Brachial Artery: Begins at the lower border o f the
teres major (major NOT minor!)
• Brachial Artery: Bifurcates to the ulnar and radial

313
H o w T o T e ll T h e U l n a r F r o m T h e R a d ia l A r t e r y O n A n g io g r a m O r C T A :

A round the radial head the brachial artery splits into the radial and ulnar arteries. I have
three tricks for telling the radial from the ulnar artery apart (in case it’s not obvious).

1. The ulnar artery is usually bigger.

2. The ulnar artery usually gives o ff the com m on interosseous

3. The ulnar artery supplies the superficial palm ar arch (usually), and therefore the
radial supplies the deep arch (usually).

U p p e r E x tre m ity N o rm a l V a ria n ts :

* A nterior Interosseous Branch (M edian A rtery) persists and supplies the deep p alm ar arch
o f the hand.

* “ H igh O rigin o f the R adial A rtery” - Radial artery com es o ff either the axillary or
high brachial artery (rem em ber it norm ally com es o ff at the level o f the radial head).

Deep

314
lower Extremity Anatomy:
Every m edical student know s the aorta bifurcates into the right and left com m on iliac
arteries, w hich subsequently bifurcate into the external and internal iliac arteries. The
nom enclature pearl for the external iliac is that it becom es the com m on fem oral once it
gives o ff the inferior epigastric (at the inguinal ligam ent).

Inferior Epigastric

Aorta Common Iliac External Iliac Common


Femoral

O nce the inferior epigastric com es o ff (level o f the inguinal ligam ent) you are dealing w ith
the com m on fem oral artery(C FA ). The CFA divides into the deep fem oral (profunda) and
superficial fem oral. The deep fem oral courses lateral and posterior. The superficial fem oral
passes anterior and m edial into the flexor m uscle com partm ent (A D D uctor / H u n te r’s Canal).
At the point the vessel em erges from the canal it is then the popliteal artery. A t the level o f
the distal border o f the popliteus m uscle the popliteal artery divides into the anterior tibialis
(the first branch) and the tibioperoneal trunk. T he anterior tibialis courses anterior and
lateral, then it transverses the interosseous m em b ran e, running dow n the front o f the anterior
tibia and term inating as the dorsalis pedis. T he tibioperoneal trunk bifurcates into the
posterior tibialis and fibular (peroneal) arteries. A com m on quiz is “w hat is the m ost m edial
artery in the leg?” , w ith the answ er being the p o sterio r tibial (felt at the m edial m alleolus).
N otice how lateral the AT is - you can im agine it running across the interosseous m em brane,
ju st like it’s suppose to.

Interosseous
Membrane

Anterior

Peroneal

315
Mesenteric Venous Collaterals:
G a s tr ic V a r ic e s : As described in m ore detail in the GI chapter, portal hypertension
shunts blood aw ay from the liver and into the system ic venous system . Spontaneous portal-
system ic collaterals develop to decom press the system . The thing to know is that m ost
gastric varices are form ed by the left gastric (coronary v e in ) . T hat is the one they
alw ays show big and dilated on an angiogram . Isolated gastric varices are secondary to
splenic vein throm bosis. G astric Varices (80-85% ) drain into the inferior phrenic and then
into the left renal vein, form ing a gastro-renal shunt.

• Left Gastric
(Coronary) = Cardia

• Posterior and Short


Gastric = Fundus

Gastroepiploic Vein

S p le n o r e n a l S h u n t: A nother feature o f portal hypertension, this is an abnorm al


collateral betw een the splenic vein and renal vein. This is actually a desirable shunt because
it is not associated with GI bleeding. H ow ever, enlarged shun ts are associated with
hepatic en cephalopath y (discussed in greater detail in the BRTO section o f the IR chapter).
A com m on w ay to show this is an enlarged left renal vein and dilatation o f the inferior vena
cava at the level o f the left renal vein.

316
Caval Variants

L e ft S id e d S V C - T he m ost com m on
congenital venous anom aly in the chest. In a few
rare cases these can actually result in a right to
left shunt. T hey are only seen in isolation in 10%
o f cases (the other 90% it’s “ duplicated” ). The
location and appearance is a total A unt M innie.

Trivia to know:

• M ost com m only associated C H D is the A SD

• A ssociated w ith an unroofed coronary sinus


Left Sided SVC - When you see that
• N early alw ays (92% o f the tim e) it drains into son of a bitch right there, that thing
can only be one of two things - a
the coronary sinus
lymph node or a duplicated SVC.

D u p lic a te d S V C - T he above discussed “ Left Sided SV C ” alm ost never occurs in


isolation. Instead, there is alm ost alw ays a “n orm al” right sided SV C , in addition to the left
sided SV C. It is in this case (w hich is the m ajority o f cases o f left sided SV C ) that the
term inology “duplicated SV C ” is used. It is so com m on that people w ill use the term s
duplicated and left sided interchangeably. But, technically to be duplicated you need a right
sided and left sided SVC (even if the right one is a little sm all - w hich is often the case in the
setting o f a left SVC).

D u p lic a te d IV C - T here are tw o m ain points


w orth know ing about this: (1) that the appearance
is an A unt M innie, and (2) it’s associated with
\i/
Renal stuff. Renal associations include
horseshoe and crossed fused ectopic kidneys.
|
A lso these dudes often have circum aortic renal
collars (see below ).
Duplicated IVC (IVCs are the bread,
Aorta is the cheese or peanut butter...
or bacon)

317
C ir c u m a o r t ic V e n o u s C o lla r - Very com m on variant w ith an additional left renal
vein that passes posterior to the aorta. It only m atters in tw o situations (a) renal transplant,
(b) IVC filter placem ent. The classic question is that the anterior lim b is superior, and the
posterior lim b is inferior.

A z y g o s C o n tin u a tio n - This is also know n as absence o f the hepatic segm ent o f the
IVC. In this case, the hepatic veins drain directly into the right atrium . O ften the IVC is
duplicated in these patients, w ith the left IVC term inating in the left renal vein , w hich then
crosses over to jo in the right IVC.

The first thing you should think w hen I say azygous continuation is polysplenia (reversed
IV C /A orta is m ore com m only associated w ith asplenia).

Azygos Continuation - N o IVC in th e Liver, D ila te d A z y g o s in th e C h e s t

318
S E C T I O N 2* J fc

t Acute Aortic S yndrom es '■F

T here are 3 “ acute aortic syndrom es” , aortic dissection,


intram ural hem atom a, and penetrating ulcer. Lets take these one at
a tim e and talk about the trivia.

A n a to m y R e v ie w : Intima
M edia
R em em ber vessels have 3 layers: Intim a, M edia, and that other E xterna
one no one gives a shit about.

Penetrating Ulcer:
This is an ulceration o f an atherom atous plaque that has eroded the inner elastic layer o f the
aortic w all. W hen it reaches the m edia it produces a hem atom a w ithin the m edia.

• #1 R isk F actor = A therosclerosis


(D elicious B urger K ing and Tasty C igarettes)

C lassic Scenario: E lderly patient w ith hypertension and


atherosclerosis usually involving the descending thoracic
aorta

G enesis: Eating like a pig and sm oking results in


atherosclerosis. N asty atherosclerotic plaque erodes
through the intim a. H em atom a form s in the m edia (in tra­
m ural hem atom a). W ith severe disease can eventually
progress to a pseudo aneurysm (and m aybe even rupture)

Pearl: Look for a gap in the intim al calcifications (th a t’s how you know it’s truly
penetrated through the intim a, and not ju st som e funky contour abnorm ality).

C lassification: All 3 A A Ss can be classified as type A or B Stanford, based on their


locations before (type A) or after the tak eo ff the o f the left subclavian (type B).
T hese things often result in a saccu lar m orp hology around the arch. In general, sac
like aneurysm above the diaphragm is related to penetrating ulcer. Sac like aneurysm
below the diaphragm is gonna be septic (“ m ycotic”).

319
R e la tio n s h ip b e t w e e n P e n e tr a tin g U lc e r a n d D is s e c tio n :
— C ontroversial (w hich usually m eans it w on't he tested) - fam ous last w ords

If forced to answ er questions on this relationship, 1 w ould go w ith the follow ing:
• Penetrating U lcers are caused by atherosclerosis (this is a fact)
• Penetrating U lcer can lead to D issection (this is probably true in som e cases)
• A therosclerosis does N O T cause D issection (w hich is confusing, m ay or m ay not be true,
and is unlikely to be tested). W hat is true is that the presence o f dense calcified plaque can
stop extension o f a dissection tear.
• D issections often occur in the aortic root - w here you have the highest flow pressures
• Penetrating U lcers nearly never occur in the root - as these flow pressures prevent
atherosclerosis (w ash those cheeseburger crum bs aw ay).

T r e a t m e n t o f P e n e tr a tin g U lc e r ?

• “ M edical” = Sim ilar to Type B D issections. If they do get treated (grafted e tc ...) they tend
to do W O R SE than dissections (on average)

• Q: W hen are they Surgical ?


• A: H em odynam ic instability, Pain, Rupture, Distal Em boli, R apid E nlargem ent

Dissection
• The m ost com m on cause o f acute aortic syndrom e (70% )
• H ypertension is the m ain factor - leads to an intim al tear resulting in tw o lum ens
• M arfans, Turners, and other C onnective Tissue D iseases increase risk

C lassic Testable Scenarios:


• Pregnancy — know n to increase risk
• C ocaine Use in a young otherw ise healthy person
• Patient w ith “ H ypertension” and a sub-sternal “Tearing S ensation.”

C hicken vs E g g : Som e people say that hypertensive pressures kill the vasa vasorum (the
little vessels inside the vessel w alls) leading to developm ent o f intram ural hem atom a w hich
then ruptures into the intim a. This is the “ inside out” thinking. O ther people think the
hypertensive forces tear the inner layer directly (“ outside in” thinking).
H o n estly ... w ho gives a shit? N ot even sure w hy I m entioned that.

320
Dissection Continued...
There are two general ways to classify these things:
(1) Time: Acute (< 2 weeks), or Chronic
(2) Location:
• Stanford A: Account for 75% of dissections and involves the ascending aorta and arch proximal
to the take-off o f the left subclavian. These guys need to be treated surgically.

• Stanford B: Occur distal to the take-off o f the left subclavian and arc treated medically unless
there are complications (organ ischemia etc...)

THIS vs THAT: Floating V isc e r a Sign:


True Lumen False Lumen This is a classic angiographic sign of
Continuity with “CobWeb Sign” - slender abdominal aortic dissection.
undissected portion of linear areas of low
It is shown as opacification of
aorta attenuation
abdominal aortic branch vessels during
Smaller cross sectional Larger cross section area aortography (catheter placed in the aortic
areas (with higher velocity (slower more turbulent true lumen), with the branch vessels—
blood) flow) ( celiac axis, superior mesenteric artery,
Surrounded by Beak Sign - acute angle at and right renal artery) arising out of
calcifications (if present) edge o f lumen - seen on nowhere.
axial plane
They appear
f False to be
floating , with
^ T r u e '^ ( o ) little or no
antegrade
Usually contains the origin Usually contains the origin opacification
of celiac trunk, SMA, and o f LEFT renal artery o f the aortic
RIGHT renal artery true lumen.
Surrounds true lumen in
Type A Dissection
**just rem em b er fa lse is left (like left h a n d e d p e o p le are
evil, o r fa lse) - then e v e ry th in g else is true.

Dissection Flap in the Abdomen - Vocab Trivia:


Static = dissection flap in the feeding artery (usually treated by stenting)
• Dynamic = dissection flap dangling in front o f ostium (usually treated with fenestration).
It can be h a rd to tell these apart. I f a s k e d I 'd exp e ct them to j u s t use the vocab w ords.

THIS vs THAT: Aneurysm with Mural Thrombus VS Thrombosed Dissection


• The dissection should spiral, the thrombus tends to drop straight down
• Intimal Calcs - the dissection will displace them.

321
For the purpose o f multiple choice - the cause is HTN

W ays th is can be shown:

1
Crescent Sign o f IMH

Hyperdense on Non Con Contrast CT - Difficult to T l Bright Crescent


distinguish from plaque

T re a tm e n t:
A lso uses the S tanford A vs B idea
S om e p eo p le w ill say Type A = Surgery, Type B m edical
• T his is co n tro v ersial and unlik ely to be tested

P re d ic to rs of S h itty O utcom e:
• M ost o f these w ill sp o n tan eo u sly regress. T hese are the things that m ake that less likely:
• H em atom a T h ick n ess G reater than 2 cm
• A sso ciatio n w ith aneu ry sm al dilatio n o f the aorta - 5 cm or m ore
• P rogression to d issectio n or p en etrating ulcer
• IM H + P en etratin g U lcer has a w o rse outcom e com pared to IM H + D issection

322
S E C T IO N 3:

t A n e u r y s m , M i s c ...

THIS v s THAT: A n e u r y s m v s P s e u d o - a n e u r y s m - The distinction betw een a true


and false aneurysm lends itse lf well to m ultiple choice testing. A true aneurysm is an
enlargem ent o f the lum en o f the vessel to 1.5 tim es its norm al diam eter. True = 3 layers are
intact. In a false (pseudo) aneurysm all 3 layers are N O T intact, and it is essentially a
contained rupture. The risk o f actual rupture is obviously higher w ith false aneurysm . It
can som etim es be difficult to tell, but as a general rule fusiform aneurysm s are true, and
saccular aneurysm s m ight be false. C lassic causes o f pseudoaneurysm include traum a,
cardiologists (groin sticks), infection (m ycotic), pancreatitis, and som e vasculitides. On
ultrasound they could show you the classic yin/yang sign, w ith “to and fro” flow on pulsed
Doppler. The yin/yang sign can be seen in saccular true aneurysm s, so you sh o u ld n ’t call it
on that alone (unless th a t’s all they give you). To and Fro flow w ithin the aneurysm neck +
clinical history is the best w ay to tell them apart.

SVC S y n d r o m e - O ccurs secondary to com plete or near com plete obstruction o f flow
in the SVC from external com pression (lym phom a, lung cancer) or intravascular
obstruction (C entral venous catheter, or pacem aker w ire w ith throm bus). A less com m on
but testable cause is fibrosing m ediastinitis (just think histoplasm osis). The dude is gonna
have face, neck, and bilateral arm sw elling.

T r a u m a tic P s e u d o a n e u r y s m -
Again a pseudoaneurysm is basically a
contained rupture. T he m ost com m on place
to see this (in a living patient) is the aortic
isthm us (90% ). This is supposedly the
result o f tethering from the ligam entum
arteriosum . The second and third m ost
com m on sites are the ascending aorta and
diaphragm atic hiatus - respectively.
A scending aortic injury is actually probably
num ber one, it ju st kills them in the field so
you d o n ’t see it. They could show you a
CX R w ith a w ide m ediastinum , deviation
Classic Isthmus Pseudoaneurysm
o f the N G Tube to the right, depressed left
m ain bronchus, or left apical cap and w ant
you to suspect acute injury.

323
A s c e n d in g A o r t ic C a lc if ic a t io n s - There are only a few causes o f ascending aortic
calcifications, as atherosclerosis typically spares the ascending aorta. T akayasu and
Syphilis should com e to m ind. The real-life significance is the clam ping o f the aorta m ay
be difficult during CA BG .

A n e u r y s m - D efined as enlargem ent o f the artery to 1.5 tim es its expected diam eter
(> 4 cm A scending and T ransverse, > 3.5 cm D escending, > 3.0 cm A bdom inal).
A therosclerosis is the m ost com m on overall cause. M edial degeneration is the m ost
com m on cause in the ascending aorta. Patients w ith connective tissue (M arfans, Ehlers
Danlos) diseases tend to involve the aortic root. When I sa y cystic m edial necrosis you
should think Marfans. A neurysm s m ay develop in any segm ent o f the aorta, but m ost
involve the infra-renal abdom inal aorta. T his varies based on risk factors, rate o f grow th,
e tc ... but a general rule is surgical repair for aneurysm s at 6cm in the chest (5.5 cm w ith
collagen vascular disease) and 5 cm in the abdom en.

S in u s o f V a ls a lv a A n e u r y s m - A neurysm s o f the valsalva sinus (aortic sinus) are


rare in real life, but have been know n to show up on m ultiple choice tests. Factoids w orth
know ing are that they are m ore com m on in A sian M en, and typically involve the right
sinus. They can be congenital or acquired (infectious). V SD is the m ost com m on associated
cardiac anom aly. R upture can lead to cardiac tam ponade. Surgical repair w ith Bentall
procedure.

Sinus of Valsalva Aneurysm

E n d o le a k s - T here are 5 types, and type 2 is the m ost com m on. These are discussed in
detail in the IR chapter.

324
R u p tu re / Im p e n d in g R u p tu r e - Peri-aortic stranding, rapid enlargem ent (10 m m or
m ore per year), or pain are w arning signs o f im pending rupture. A retroperitoneal hem atom a
adjacent to an A A A is the m ost com m on im aging finding o f actual rupture. T he m ost
com m on indicator for elective repair is the m axim um diam eter o f the aneurysm , “ Sac Size
M atters,” w ith treatm ent usually around 6 cm (5.5 cm in patients w ith collagen vascular
disease). A thick, circum ferential m ural throm bus is thought to be protective against rupture.
E nlargem ent o f the patent lum en can indicate lysis o f throm bus and predispose to rupture.

Findings o f Im p en d in g R upture

Posterior wall o f the aorta drapes over the vertebral


Draped Aorta Sign column.

Increased Aneurysm Size 10 mm or more increased per year

Focal Discontinuity in Circumferential Wall


Calcifications

Well-defined peripheral crescent o f increased


Hyperdense Crescent Sign attenuation. One o f the most specific manifestations
o f impending rupture.

M y c o tic A n e u r y s m - These are m ost often saccu lar and m ost often
pseudoaneurysm s. T hey are prone to rupture. They m ost often occur via hem atogenous
seeding in the setting o f septicem ia (en docard itis). T hey can occur from direct seeding via a
psoas abscess or vertebral osteom yelitis (but this is less com m on). M ost occu r in the
thoracic or supra-renal aorta (m ost atherosclerotic aortic aneurysm s are infra-renal).
Typical findings include saccular shape, lobular contours, peri-aortic inflam m ation, abscess,
and peri-aortic gas. T hey tend to expand faster than atherosclerotic aneurysm s. In general
sm all, asym ptom atic, and unruptured.

A v Gam esm anship - If you see a saccular aneurysm o f the aorta (especially the
If abdom inal aorta) you have to lead w ith infection.

N F 1 - O ne o f the m ore com m on neurological genetic disorders, w hich you usually think
about causing all the skin stu ff (C afe au lait spots and freckling), and bilateral optic gliom as.
A lthough uncom m on, vascular findings also occur in this disorder. A n eu rysm s and stenoses
are som etim es seen in the aorta and larger arteries, w hile dysplastic features are found in
sm aller vessels. Renal artery stenosis can occur, lead in g to ren ovascu lar hypertension
(found in 5% o f children w ith N F). T he classic look is orificial renal artery stenosis
presenting with hypertension in a teen ager or child. T he m echanism is actually D ysplasia
o f the arterial w all itse lf (less com m on from peri-arterial neurofibrom a).

325
M a rfa n S y n d ro m e - Genetic disorder caused by m utations o f the fibrillin gene (step 1
question). There are lots o f system ic m anifestations including ectopic lens, being tall, pectus
deformity, scoliosis, long fingers e tc ... Vascular findings can be grouped into aneurysm ,
dissection, and pulm onary artery dilation:

• Aneurysm: Dilation with M arfans is classically described as “ A nnuloaortic ectasia”,


with dilatation o f the aortic root. The dilation usually begins with the aortic sinuses, and
then progresses into the sinotubular junction, ultim ately involving the aortic annulus.
Dilatation of the aortic root leads to aortic valve insufficiency. Severe aortic
regurgitation occurs that m ay progress to aortic root dissection or rupture. The
m echanism for all this nonsense is that disruption o f the m edia elastic fibers causes
aortic stiffening, and predisposes to aneurysm and dissection. The buzzw ord for the
M arfans ascending aneurysm is “tulip bulb.” They are usually repaired earlier than
normal aneurysm (typically around 5.5 cm).

Marfan’s - Annuloaortic Ectasia, with dilation of the aortic root

• Dissection: Recurrent dissections are com m on, and even “triple barreled dissection” can
be seen (dissections on both sides o f a true channel).

* Pulmonary A rtery Enlargement: Just like dilation o f the aorta, pulm onary artery
enlargem ent favors the root.

L o e y s D ie tz S y n d ro m e - Despite the name,


Loeys Dietz Classic Triad:
this is actually not a Puerto Rican DJ. Instead think o f
this as the really shitty version o f M arfans. They 1. Hypertelorism (frog eyes),
have a terrible prognosis, and rupture their aortas all 2. Bifid uvula or cleft palate
3. Aortic aneurysm with tortuosity
the time. Vessels are very tortuous (twisty). They
also have crazy w ide eyes (hypertelorism ).

C lassic Look: C razy tw isty vertebral arteries

326
E h le rs -D a n lo s - This one is a disorder in collagen, w ith lots o f different subtypes. They
have the stretchy skin, hyperm obile jo in ts, blood vessel fragility w ith bleeding diatheses.
Invasive diagnostic studies such as conventional angiography and other percutan eou s
procedures should be avoided because o f the excessive risk o f arterial dissection. Im aging
characteristics o f aortic aneurysm s in E hlers-D anlos syndrom e resem ble those in M arfan
syndrom e, often involving the aortic root. A neurysm s o f the abdom inal visceral arteries
are com m on as well.

S y p h ilitic ( L u e t ic ) A n e u r y s m - T his is super rare and only seen in patients w ith


untreated tertiary syphilis. T here is classically a saccular appearance and it involves the
ascending aorta as well as the aortic arch. C lassic description “saccu lar asym m etric aortic
aneurysm with involvem ent o f the aortic root branches. ” O ften heavily calcified “tree
bark” intim al calcifications. C oronary artery narrow ing (at the ostium ) is seen 30% o f the
tim e. A ortic valve insufficiency is also com m on.

A o r t o e n t e r ic F is tu la - These com e in tw o flavors: (a) Prim ary, and (b) Secondary.

* Prim ary: Very, very, very rare. R efers to an A -E fistula w ithout history o f
instrum entation. They are only seen in the setting o f aneurysm and atherosclerosis.

* Secondary: M uch m ore com m on. T hey are seen after surgery w ith or w ithout stent
graft placem ent.

The question is usually w hat part o f the bow el is involved, and the answ er is 3 rd and 4th
portions o f the duodenum . The second m ost likely question is A -E fistula vs perigraft
infection (w ithout fistula)? The answ er to that is unless you see contrast from the aorta into
the bow el lum en (usually duodenum ), you c a n ’t tell. Both o f them have ectopic perigraft gas
> 4 w eeks post repair, both have perigraft fluid and edem a, both lose the fat place betw een
the bow el and aorta (tethering o f the duodenum to the anterior w all o f the aorta), both can
have pseudoaneurysm form ation.

Aortoenteric Fistula - Primary Type

327
In fla m m a to r y A n e u ry s m s - M ost are sym ptom atic, more comm on in young men,
and associated with increased risk o f rupture regardless o f their size. Unlike patients with
atherosclerotic AAA, most with the inflamm atory variant have an elevated ESR. Their etiology
is not well understood but m ay be related to periaortic retroperitoneal fibrosis or other
autoimm une disorders (SLE, Giant Cell, RA). Sm oking is apparently a strong risk factor,
and smoking cessation is the first step in medical therapy. In 1/3 o f cases hydronephrosis or
renal failure is present at the time o f diagnosis because the inflam m atory process usually
involves the ureters. Imaging findings include a thickened wall and inflamm atory or fibrotic
changes in the periaortic regions. Often there is asymm etrical thickening o f the aorta with
sparing o f the posterior wall (helps differentiate it from vasculitis).

L e ric h e S y n d ro m e - Refers
to complete occlusion o f the aorta
distal to the renal arteries (most
often at the aortic bifurcation). It
is often secondary to bad
atherosclerosis. There can be
large collaterals.

Clinical Triad:

. Limp Dick (impotence)


• Ass claudication
• Absence femoral pulses
Leriche Syndrome - C om plete occlusion of the
aortal distal to the renal arteries

M id A o rtic S y n d ro m e (C o a rc ta tio n of
th e A b d o m in a l A o r ta ) - Refers to progressive
narrowing o f the abdominal aorta and its m ajor
branches. Com pared to Leriche, this is higher, and
longer in segment. It’s also a total freaking zebra. It
tends to affect children / young adults. This thing is
characterized by progressive narrowing o f the aorta.
It is NOT secondary to arteritis or atherosclerosis
but instead the result o f some intrauterine insult
(maybe) with fragmentation o f the elastic media.

This also has a clinical triad:

• HTN (most comm on presenting sym ptom ) - this Mid Aortic Syndrome - Narrow aorta
is the m ost common cause o f death i f not treated without arteritis or atherosclerosis
• Weak or Absent Femoral Pulses
• Claudication
• Renal failure

328
A o r t ic C o a r c t a t io n -
THIS vs THAT: C o a rc ta tio n of th e A o rtic
• There are tw o subtypes (N a rro w in g th e o f th e A o rtic Lum en)
(as show n in the chart)
In fa n tile A dult
• Strong A ssociation w ith Turners
Syndrom e (15-20% ). Presents with heart Leg Claudication
failure within the first BP differences between
• Bicuspid A ortic valve is the m ost com m on week o f life. arms and legs.
associated defect (80% ).
Pre-Ductal (Before the Post-Ductal (Distal to
• They have m ore berry aneurysm s. left Subclavian A.) left Subclavian A.)

• Figure 3 sign (appearance o f CX R ). Aortic Arch = Aortic Arch =


Hypoplastic Normal Diameter
• Rib Notching: m ost often involves 4th -
Collateral Formation is
8th ribs. It does N O T involve the l sl and More Likely
2 nd because those are fed by the
costocervical trunk.

P se u d o co a rctatio n - This is a favorite o f m ultiple choice w riters. You will have


elongation w ith narrow ing and kinking o f the aorta. It really looks like a coarctation, BU T
there is NO pressure grad ien t, collateral form ation, or rib notch in g - that is the m ost
likely question. The second m ost likely question is the area o f aneurysm al dilation m ay
occur distal to the areas o f narrow ing in pseudocoarctation, and they m ay becom e
progressively dilated and should therefore be follow ed.

T h o ra c ic O utlet Syndrom e - C ongenital or acquired com pression o f the Subclavian


vessels (artery and vein), and brachial plexus nerves as they pass through the thoracic inlet.
It is a spectrum : N erve (95% ) » » » S u bclavian Vein » S u bclavian A rtery. W ith
sym ptom s varying depending on w hat is com pressed. C om pression by the anterior
scalene m uscle is the m ost com m on cause. H ow ever, cervical rib, m uscular hypertrophy,
fibrous bands, Pagets, tum or e tc ... can all cause sym ptom s. T reatm ent is usually surgical
rem oval o f the rib / m uscle. T he classic w ay to show this is arm s up and arm s dow n
angiography (occlusion occurs w ith arm s up).

Paget S ch ro e tte r-T h is is essen tially thoracic outlet syndrom e, w ith d evelop m ent
o f a venous throm bus in the Sub clavian vein. It’s som etim es called “effort throm bosis’’
because it’s associated w ith athletes (pitchers, w eightlifters) w ho are raising their arm s a lot.
They will use catheter directed lysis on these dudes, and surgical release o f the offending
agent as above. Stenting isn ’t usually done (and can only be done after surgery to avoid
getting the stent crushed).

329
Pulm o nary A rte ry A n eurysm /P seud oaneurysm - Think about three things for
multiple choice; (1) Iatrogenic from Swan Ganz catheter *most common (2) Behcets, (3)
Chronic PE. When they want to lead Swan Ganz they may say something like “patient in the 1CU.”
The buzzwords for Behcets are: “Turkish descent”, and “mouth and genital ulcers.”

• Hughes-Stovin Syndrome: This is a zebra cause o f pulmonary artery aneurysm that is similar
(and maybe the same thing) as Behcets. It is characterized by recurrent thrombophlebitis and
pulmonary artery aneurysm formation and rupture.

• Rasmussen Aneurysm: This has a cool name, which instantly makes it high yield for testing.
This is a pulmonary artery pseudoaneurysm secondary to pulmonary TB. It usually involves
the upper lobes in the setting o f reactivation TB.

• Tetralogy of Fallot Repair Gone South: So another possible testable scenario is the patch
aneurysm, from the RVOT repair.

S plenic A rte ry A neurysm : The most common visceral arterial aneurysm (3rd most
common abdominal - behind aorta and iliac).

Etiology of these things depends on who you ask. Some source - High Risk For R upture -
will say arteriosclerosis is the most important cause. However, it
• Liver Transplantation
seems that most sources will say that arteriosclerosis less • Portal Hypertension
important and things like portal hypertension and a history of • Pregnancy
multiple pregnancy are more important. More common in • Connective Tissue Disorders
pregnancy, and more likely to rupture in pregnancy. • Alpha 1 Antitrypsin Def

Most are located in the distal artery. False aneurysms are


associated with pancreatitis.

An important mimic is the islet cell pancreatic tumor (which is hypervascular). Don’t be a dumb
ass and try to biopsy the aneurysm. If you are forced to choose which ones to treat I guess I’d go
with: anything over 2 cm, any pseudoaneurysm, and any in a women planning on getting pregnant.

SMA A neurysm : All SMA aneurysms should be treated - as there is a high rate of rupture and
association with mesenteric ischemia.

H e p a tic A rte ry A neurysm : Treated if the patient is symptomatic or size exceeds 2cm (just
like the spleen). In patients with FMD or polyarteritis nodosa - typically they are treated regardless
o f size.

M edian A rc u a te L ig am en t S yndrom e (Dunbar


Syndrome) : This is compression o f the celiac artery by the
median arcuate ligament (fibrous band that connects the
diaphragm). Most people actually have some degree o f
compression, but it’s not a syndrome until there are symptoms
(abdominal pain, weight loss). Typical age is 20-40 years old.
The buzzword is “hooked appearance.” It’s classically shown
on angiography and they will want you to know that it gets
worse with expiration. It can actually lead to the development
of pancreaticoduodenal collaterals and aneurysm formation. It’s
treated surgically.

330
M esenteric Is c h em ia

This can be broadly classified as acute or chronic.

C h ro n ic : S ignificant Stenosis o f 2 out o f 3 m ain m esenteric vessels + sym ptom s (“ food


fear” ) , LU Q pain after eating, pain out o f proportion to exam ). Som e practical pearls are
that you can have bad disease and no sym ptom s if you have good collaterals. A lternatively
if you have bad one-vessel disease you can have sym ptom s if you have crappy collaterals.
R em em ber that the splenic fle x u r e ("G riffith s P o in t ”) is the m ost com m on because it’s the
w atershed o f the SM A and IM A.

A c u te : This com es from 4 m ain causes. A rterial, Venous, N on-occlussive, and


Strangulation.

* A rterial: O cclusive em boli (usually m ore distal, at branch points), or T hrom bus
(usually closer to the ostium ). V asculitis can also cause it. T he SM A is m ost
com m only affected. Bow el typically has a th in n er w all (no arterial inflow ), and is
N O T typically dilated. A fter reperfusion the bow el wall w ill becom e thick, w ith a
target appearance.

* Venous: D ilation w ith w all th ick en in g (8-9m m , with < 5m m being norm al) is
m ore com m on. Fat stranding and ascites are especially com m on findings in venous
occlusion.

* N on-O cclusive: Seen in patients in shock or on pressors. T his is the m ost difficult
to diagnose on CT. The involved bow el segm ents are often thickened.
E nhancem ent is variable. L ook f o r d e la y e d fillin g o f the p o rta ! vein at 70 seconds.

* Strangulation: This is alm ost alw ays secondary to a closed loop obstruction. This
is basically a m ixed arterial and venous picture, w ith congested dilated bow el.
H em orrhage m ay be seen in the bow el w all. The lum en is often fluid-filled.

Trivia: M esenteric Ischem ia has a described association w ith SM A A neurysm .

331
M e s e n t e r ic Is c h e m ia
A rterial Venous Strangulation N on -O cclusive

T hin Bowel Wall


(thick after T hick Bowel Wall T hick Bow el Wall T hick Bow el Wall
reperfusion)

D im inished
Variable Variable Variable
Enhancem ent

Severe D ilation (and


Bowel N ot D ila ted M oderate D ilation Bow el N ot D ilated
fluid filled)

H azy w ith A scites,


M esentery N ot Hazy M esentery N ot H azy
H azy w ith A scites and “w hirl sign” with
(until it infarcts) (until it infarcts)
closed loop.

Griffith’s Point
• SMA-IMA -
Watershed
• “Most Common
Location for
Ischemia”

Sudeck’s Point
• IMA - Iliac Watershed
• Highly Susceptible to
Ischemia

332
S E C T IO N 5:

t [M M M IM M M BM M isc, S o
E
on

TC.. E
r-r/~
and

TC..
cr-i-/~
S o Fo r th , ^

This is m y general algorithm if I see angry (thick w alled) bow el

Infection
-diffuse or Tl

Embolic
Colitis / Enteritis Inflammatory -Brach points
-T l Occlusive
Arterial N Thrombotic
-Closer to ostium

Ischemic Non-Occlusive
-think hypotension (watershed)
Venous

C o lo n ic A n g io d y s p la s ia - T his is the second m ost com m on cause o f colonic arterial


bleeding (diverticulosis being num ber one). T his is prim arily right sided w ith angiography
dem onstrating a cluster o f sm all arteries during the arterial phase (along the antim esenteric
border o f the colon), w ith early opacification o f d ila ted draining veins that persists late into
the venous phase. T here is an association w ith aortic stenosis w hich carries the eponym
H eyde Syndrom e (w hich instantly m akes it high yield for m ultiple choice).

Colonic Angiodysplasia

333
O s ie r W e b e r R en d u ( H e r e d ita r y H e m o r r h a g ic T e la n g ie c t a s ia ) - This is an
AD m ulti-system disorder characterized by m ultiple AVMs. On step 1 they used to show you
the tongue / m outh with the telangiectasis and a history o f recurrent bloody nose. Now, they
will likely show m ultiple hepatic AVMs or m ultiple pulm onary AVMs. Extensive shunting in
the liver can actually cause biliary necrosis and bile leak. They can have high output cardiac
failure. **M ost die fro m stroke or brain abscess.

Gamesmanship “Next S te p ” - If the syndrom e is suspected, these guys need CT o f the


C Lung & Liver (with contrast), plus a Brain M R / M RA

R e n a l A r t e r y S te n o s is - N arrow ing o f the renal artery m ost com m only occurs


secondary to atherosclerosis (75% ). This type o f narrow ing is usually near the ostium , and can
be stented. EM D is the second m ost com m on cause and typically has a beaded appearance
sparing the ostium (should not be stented). A dditional m ore rare causes include PAN,
Takayasu, NL-1, and Radiation.

F M D (F ib r o m u s c u la r D y s p la s ia ) - A non-atherosclerotic vascular disease, prim arily


affecting the renal arteries o f young w hite wom en.

Things to know:

- R enovascular HTN in Young W omen = LMD

- Renal arteries are the m ost com m only involved (carotid #2, iliac #3)

- There are 3 types, but ju st rem em ber m edial is the m ost com m on (95%)

- They are predisposed to spontaneous dissection

- B uzzw ord = String o f Beads

- Treatm ent = A ngioplasty W ITH O U T stenting. Eating w alnuts helps (seriously, there is a
paper on it). Eating cashew s does not help (sam e paper), which is too bad because
cashew s are delicious... and w alnuts taste like shit.

334
N u t c r a c k e r S y n d r o m e - U sually a healthy fem ale 30s-40s. The left renal vein gets
sm ashed as it slides under the SM A , w ith resulting abdom inal pain (left flank) and
hem aturia. T he left renal vein gets sm ashed a lot, but it’s not a syndrom e w ithout
sym ptom s. Since the left gonadal vein drains into the left renal vein, it can also cause left
testicle pain in m en, and LLQ pain in w om en.

Aorta
Left RV
Nutcracker

Duodenum
SMA
Syndrome

Nutcracker: Renal Vein, Smashed by SMA. N utcracker D iagnostic G old Standard:


Retrograde venography with renal / caval
Note the prominent venous collateral (arrow) pressure measurements

S A M (S e g m e n ta l A r t e r ia l M e d io ly s is ) - Targets WTF is a
“S p la n c h n ic ” A rtery ?
the splanchnic arteries in the elderly, and the coronaries in
young adults. N ot a true vasculitis, w ith no significant
Typically the splanchnic
inflam m ation. It’s com plicated but essentially the m edia o f circulation o f the GI tract
the vessel turns to crap, and you get a bunch o f aneurysm s. refers to the C eliac,
The aneurysm s are often m ultiple. The w ay this is show n is SM A , and IM A.
m ultiple abdom inal splanchnic artery saccu lar
aneurysm s, d issections, and occlusion - this is the disease
hallm ark. Can also be show n as spontaneous in tra -abdom inal hem orrhage.

P e lv ic C o n g e s tio n S y n d r o m e - T his is a controversial entity, som etim es grouped


in the fibrom yalgia spectrum . Patients often have “chronic abdom inal p ain .” T hey also
often w ear a lot o f rings and drink orange soda. T he classic dem ographic is a depressed,
m ultiparous, pre-m enopausal w om en w ith chronic pelvic pain. Venous obstruction at the left
renal vein (nutcracker com pression) or incom petent ovarian vein valves leads to m ultiple
dilated p arauterine veins. This very “real” diagnosis can be treated by y our local
Interventional R adiologist via ovarian vein em bolization.

335
T e s t ic u la r V a r ic o c e le - A bnorm al dilation o f veins in the pam piniform plexus. M ost
cases are idiopathic and m ost (98% ) are found on the left side (left vein is longer, a n d
drains into renal vein a t right angle). T hey can also occur on the left, secondary to the
above m entioned “nutcracker syndrom e.” T hey can cause infertility. “ Non-
decom pressible” is a buzzw ord for badness. Som e sources state that neoplasm is actually the
m ost likely cause o f n on-decom p ressib le varicocele in m en over 40 years o f age; (left renal
m alignancy invading the renal vein). R ight-sided varicocele can be a sign o f m alignancy as
well. W hen it’s new, and on the right side (in an adult), you should raise concern for a pelvic
or abdom inal m alignancy. N ew right-sided varicocele in an adult should m ake you think
renal cell carcinom a, retroperitoneal fibrosis, or adhesions.

N on -D ecom pressible = Bad

R ight = Bad

Left = O k

Bilateral = O k (probably)

A G am esm anship - This diagnosis is the classic next step question o f all next step
questions because you need to recognize w hen this com m on diagnosis is
^ " associated w ith som ething bad.

• Isolated Right V aricocele = G et an A B D CT (or M R, or US)

• N on-decom pressible V aricocele = G et an A B D CT (or M R, or US)

• Bilateral D ecom pressible V aricoceles = M ight need treatm ent if infertile etc.., but d o e sn ’t
need additional cancer hunting im aging.

• Isolated Left V aricocele = M ight need treatm ent if infertile etc.., but d o e sn ’t need
additional cancer hunting im aging.

U t e r in e A V M - This can present with life threatening m assive genital bleeding. R arely
they can present w ith CHF. T hey com e in tw o flavors (a) C ongenital, and (b) A cquired.
A cquired occurs after D & C, abortion, or m ultiple pregnancies. They are m ost likely to
show this on color D oppler w ith serpiginous structures in the m yom etrium w ith low
resistance high velocity patterns. This one needs em bolization. C ould look sim ilar to
retained products o f conception (clinical history will be different, and R P O C is usually
centered in the endom etrium rather than the m yom etrium ).

336
Extremity

M a y T h u r n e r - A syndrom e resulting in DVT o f the left com m on iliac vein. The pathology
is com pression o f the left com m on iliac vein by the right com m on iliac artery. Treatm ent
is throm bolysis and stenting. I f they show y o u a sw ollen left leg, this is probably the answer.
PE and acute lim b ischem ia from severe venous obstruction (Phlegm asia cem lea dolens) are
two described com plications.

P o p lite a l A n e u ry s m - This is the m ost com m on peripheral arterial aneurysm (2nd m ost
com m on overall, to the aorta). The m ain issue with these things is distal throm boem bolism ,
which can be limb threatening. There is a strong and frequently tested association with AAA.

* 30-50% o f patients with popliteal aneurysm s have a AAA

* 10% o f patients with AAA have po p litea l aneurysm s

* 50-70% o f po p litea l aneurysm s are bilateral

The m ost dreaded com plication o f a popliteal artery aneurysm is an acute lim b from
throm bosis and distal em bolization o f throm bus pooling in the aneurysm .

P o p lite a l E n tr a p m e n t - Sym ptom atic com pression or occlusion o f the popliteal artery
due to the developm ental relationship with the m edial head o f the gastrocnem ius (less
com m only the popliteus). M edial deviation o f the popliteal artery is supposedly diagnostic.
This usually occurs in young m en (<30). These patients m ay have norm al p u lses that decrease
with pla n ta r fle x io n or dorsiflexion o f the fo o t. They will show you either a M RA or
conventional angiogram in rest and then stress (dorsi / plantar flexion) to show the artery
occluding.

H y p o th e n a r H a m m e r - Caused by blunt traum a (history o f w orking with a jack-


ham m er) to the ulnar artery and superficial palm ar arch. The im pact occurs against the hook o f
the ham ate. Arterial wall dam age leads to aneurysm form ation with or w ithout throm bosis o f
the vessel. Emboli m ay form , causing distal obstruction o f digits (this can cause confusion with
the m ain DDx Buergers). Look for corkscrew configuration of the superficial palm ar arch,
occlusion of the ulnar artery, or pseudoaneurysm o f the ulnar artery.

V e n o u s T h ro m b o e m b o lis m (V T E ) - Blanket term used to refer to PE and DV T (PE


and DVT are both form s o f VTE).

Trivia: You are m ore likely to develop VTE if you are paraplegic vs tetraplegic.

Having said that, VTE is m ore likely in “m otor com plete” AIS A patients (those that have lost
m otor and sensory) vs m otor incom plete AIS B,C ,D (those with som e residual sensory or
m otor function) -- which is m ore intuitive. Q uad being w orse than tetra for VTE risk m akes
zero sense (and therefore m akes a good trivia question).

337
P e r ip h e r a l V a s c u la r M a lfo r m a tio n s - A bout 40% o f vascular m alform ations
involve the extrem ities (the other 40% are head and neck, and 20% is thorax). D ifferent than
hem angiom as, vascular m alform ations generally increase proportionally as the child grow s.
This dude Jackson classified vascular m alform ation as either low flow or high flow. Low
flow w ould include venous, lym phatic, capillary, and m ixes o f the like. H igh flow has an
arterial com ponent. T reatm ent is basically determ ined by high or low flow.

K lip p e l-T r e n a u n a y S y n d r o m e (K T S ) - T his is often com bined w ith Parkes-


W eber w hich is a true high flow AV m alform ation . KTS has a triad o f port w ine nevi,
bony or soft tissue hypertrophy (localized gigantism ), and a venous m alform ation. A
persistent sciatic vein is often associated. The m arginal vein o f Servelle (som e superficial
vein in the lateral c a lf and thigh) is pathognom onic (it’s basically a great saphenous on the
w rong side).

A dditional trivia: 20% have GI involvem ent and can bleed, if the system is big enough it can
eat your platelets (K asabach M erritt). B asically, if you see a M R A /M R V o f the leg with a
bunch o f superficial vessels (and no deep drainage) you should think about this thing.

• KTS = Low Flow (venous)

• Parkes Weber = H igh Flow (arterial)

• “K lippel Trenaunay Weber" = Som ething people say w hen they (a) d o n ’t know w hat they
are talking about, or (b) d o n ’t know w hat kind o f m alform ation it is and w ant to use a
blanket term .

A B Is - So basic fam iliarity w ith the so called “ A nkle to Brachial Index” can occasionally
com e in handy, w ith regard to peripheral arterial disease. This is basically a ratio o f systolic
pressure in the leg over systolic blood pressure in the arm . D iabetics can som etim es have
unreliable num bers (usually high), because dense vascular calcifications w o n ’t let the vessels
com press.

O pinions vary on w hat the various cut o ff num bers m ean. M ost people will agree that you
can safely deploy the phrase “peripheral arterial d isease” if the resting ABI is less than 0.90

You can also deploy the follow ing generalizations: 0.5-0.3 = claudication, < 0.3 = rest pain

**M ore on this in the 1R chapter

338
In tim a l H y p e r p la s ia - “The bane o f endovascular intervention.” T his is not a true
disease but a response to blood vessel wall dam age. B asically this is an exuberant healing
response that leads to intim al thickening w hich can lead to stenosis. You hear it talked
about the m ost in IR after they have revascularized a lim b. R e-Stenosis that occurs 3-12
m onths after angioplasty is probably from intim al hyperplasia. It’s sneaky to treat and
often resists balloon dilation and/
or reoccurs. If you put a bare
stent in place it m ay grow through
the cracks and happen anyway. If
you put a covered stent in, it m ay
still occur at the edges o f the
stent. The take hom e point is that
it’s a pain in the ass, and if they
show an angiogram w ith a stent
in place, that now appears to be Intimal Hyperplasia
-dark stuff growing along the inside of the stent walls
losing flow, this is probably the
answer.

C y s tic A d v e n t it ia l D is e a s e - T his uncom m on d isorder classically affects the


popliteal artery, o f you n g m en. B asically you have one or m ultiple m u coid-filled cysts
developing in the outer m edia and adventitia. A s the cysts grow, they com press the artery.

MIP CTA showing Vascular Narrowing Fluid sensitive MR showing a bunch of


cysts around the vessel, extrinsically
narrowing it

339
S E C T I O N 7:

t V a s c u l i t i s

B asically all vasculitis looks the sam e, w ith wall thickening, occlusions, dilations, and
aneurysm form ation. The trick to telling them apart is the age o f the patient, the gender /
race, and the vessels affected. C lassically, they are broken up into large vessel, m edium
vessel, sm all vessel A N C A +, and sm all vessel A N C A negative.

Large:
T a k a y a s u - “The pulseless disease.” T his vasculitis loves young A sian girls (usually
15-30 years old). If they m ention the w ord “A sian,” this is likely to be the answer. A lso, if
they show you a vasculitis involving the aorta this is likely the answer. In the acute phase
there will be both w all th ickening and w all enhancem en t. T here can be occlusion o f the
m ajor aortic branches, or dilation o f the aorta and its branches. The aortic valve is often
involved (can cause stenosis or AI). In the late phase there is classically diffuse narrow ing
distally. The pulm onary arteries are com m only involved, w ith the typical appearance o f
peripheral pruning.

If anyone w as a big enough je rk to ask, there are 5 types w ith variable involvem ent o f the
aorta and its branches. W hich type is w hich is beyond the scope o f the exam , ju st know type
3 is m ost com m on - involves arch and abdom inal aorta.

Takayasu - Wall Thickening Involving the Aorta

34-0
G ia n t C e ll (G C A ) - The m ost com m on prim ary system vasculitis. T his vasculitis loves
old m en (usually 70-80)** although there are a fe w p a p e rs that w ill sa y this is slig h tly m ore
com m on in wom en. T his vasculitis involves the aorta and its m ajor branches particularly
those o f the external carotid (tem poral artery). T his can be show n in tw o w ays: (1) an
ultrasound o f the tem poral artery, dem onstrating w all thickening, or (2) CTA / M R A or even
angiogram o f the arm pit area (S ubclavian/ A xillary/ B rachial), dem onstrating wall
thickening, occlusions, dilations, and aneurysm . T hink about it as the part o f the body that
w ould be com pressed by crutches (old m en need crutches).

Trivia w orth know ing:

• ESR and C R P are m arkedly elevated,

• D isease responds to steroids.

• “G old S tandard” for diagnosis is tem poral artery biopsy (although it’s often negative).

• Clinical connection betw een G C A and polym yalgia rheu m atica. (they m ight be different
phases o f the sam e disease). H istory m ight be “m orning stillness in shoulders and hips.”

Giant Cell - “A rm p it” Vessel Thickening

C o g a n S y n d r o m e - Total Z ebra probably not even w orth


m entioning. It is a large vessel vasculitis that targets children and young
adults. It likes the eyes and ears causing optic neuritis, uveitis, and
audiovestibular sym ptom s resem bling M enieres. T hey can also get
aortitis, and those that do have a w orse prognosis.

Basically, kid with eye and ear sym p tom s + or - aortitis.

341
Medium

PAN ( P o ly a r t e r it is N o d o s a ) - This is one o f tw o vasculitides (the other being


Buergers) that is m ore com m on in m en. PAN is m ore com m on in a M A N . T his can effect
a lot o f places w ith the big 3 being Renal (90% ), C ardiac (70% ), and GI (50-70% ). Typically
we are talking about m icroaneurysm form ation, prim arily at branch points, follow ed by
infarction. I w ould expect this to be show n either as a CTA or angiogram o f the kidneys
with m icroaneurysm s, or a kidney w ith areas o f infarct (m ultiple w edge shaped areas).

Trivia to know is the association with Hep B.

A lso, as a point o f trivia the m icro-aneurysm form ation in the kidney can also be seen in
patients w ho abuse C rystal M eth (som etim es called a “ speed kidney”).

K a w a s a k i D is e a s e - Probably the m ost com m on Clinical Trivia:


vasculitis in children (H SP also com m on). T hink about this as
“Fever for 5 D ays”
a cause o f coronary vessel aneurysm . A calcified coronary
artery aneurysm show n on C X R is a very rare aunt • Strawberry Tongue
M innie. • Neck Lymph Nodes
• Rash of Palms of
-C oronary A rtery A neurysm s > 8m m are “G iant” and prone Hands / Soles of Feet
• Sore Throat Diarrhea
to badness including Ml
• “Etiology Unknown”
-C oronary A rtery A neurysm s < 8m m m ay regress

342
S m a ll V e s s e l D is e a s e (A N C A +)

W e g e n e r s - 1 think about upper respiratory tract As I ’ve previously mentioned, you


(sinuses), low er respiratory tract (lungs), and aren’t supposed to say “Wegener” -
kidneys. cA N C A is (+) 90% o f the tim e. W ays this because apparently he ate his boogers,
is show n are the nasal perforation (like a cocaine and was a Nazi. He also could not
addict) and the cavitary lung lesions. (more likely deliberately chose not to)
correctly pronounce the word “Gyro.”
C h u rg S tr a u s s - This is a necrotizing No matter how many times people
corrected him. Dude- it is “YEE-roh.”
pulm onary vasculitis w hich is in the spectrum o f
E osinophilic lung disease. T hey alw ays have asthm a Wegener was truly one o f histories
and eosinophilia. T ransient perip heral lung greatest assholes.
consolidation or ground glass regions is the m ost
So we don’t say his name. Instead say
frequent feature. C avitation is rare (this should m ake
“Granulomatosis with polyangiitis.”
you think W egeners instead). They are pA N C A (+)
75% if the tim e. You know who else was a Nazi??
Henry Ford. No one ever talks about
M ic r o s c o p ic P o ly a n g iit is - A f f e c ts the th at... Google it - he wrote a book
kidneys and lungs. D iffuse pulm onary hem orrhage called “ The International Jew, the
is seen in about 1/3 o f the cases. It is pA N C A (+) World's F orem ost Problem , ” a n d g o t
80% o f the tim e. an a w a rd from Hitler.

S m a ll V e s s e l D is e a s e (A N C A -)

H S P (H e n o c h -S c h o n le in P u r p u ra ) - The m ost com m on vasculitis in children


(usually age 4-11). A lthough it is a system ic disease, G1 sym ptom s are m ost com m on
(painful bloody diarrhea). It is a com m on lead point for intussusception. T hey could show
this tw o classic w ays: (1) ultrasound w ith a d ough nut sign for in tu ssu scep tion , or (2) as a
ultrasound o f the scrotum show ing m assive skin edem a. A less likely (but also possible)
w ay to show this case w ould be m ulti-focal bow el w all thickening, or a plain film w ith
thum bprinting.

B e h c e ts - C lassic history is m outh ulcers and genital ulcers in som eone w ith Turkish
descent. It can cause thickening o f the aorta, but for the purpose o f m ultiple choice test I
expect the question will be pulm onary artery aneurysm .

B u e r g e r s - This vasculitis is strongly associated w ith sm okers. It affects both sm all and
m edium vessels in the arm s and legs (m ore com m on in legs). A lthough it is m ore com m only
seen in the legs, it is m ore com m only tested w ith a hand angiogram . The characteristic
features are extensive arterial occlusive disease w ith the developm ent o f corkscrew collateral
vessels. It usually affects m ore than one lim b. B u zzw ord = A u to-am p utation.

343
G a m e s m a n s h ip H an d A n g io g ra m s

If they are show ing you a hand angiogram , it’s going to be either B uergers o f H ypothenar
H am m er Syndrom e (H H S).

M y strategy centers around the ulnar artery.

(1) U lnar artery involved = HH S. The m ost helpful finding is a pseudo-aneurysm o ff the
ulnar artery - this is a slam dunk for HHS.

(2) U lnar artery looks ok - then look at the fingers - if they are out, go w ith Buergers. It
sure w ould be nice to see som e “corkscrew collaterals” - to m ake it a sure thing.

Be careful, because the fingers can be out w ith HH S as w ell (distal em boli), but the ulnar
artery should be fucked. Look at that ulnar artery first.

L o c a tio n - L o c a tio n

C entral =
Think Takayasu

M id-C lavicle =
T hink Thoracic
Syndrom e

A rm pit =
Think G iant Cell Y

Takayasu

344
Large Vessel

Takayasu Young A sian Fem ale - thickened aneurysm al aorta

O ld Person w ith involvem ent o f the “ crutches” / arm pit region


G iant Cell
(Subclavian, axillary, brachial).

C ogan Syndrom e Kid w ith eye and ear sym ptom s + A ortitis

M edium Vessel

PAN is m ore com m on in a M A N (M > F). Renal M icroaneurysm


PAN
(sim ilar to speed kidney). A ssociated w ith Hep B.

K aw asaki C oronary A rtery A neurysm

Sm all Vessel (A N C A + )

W egeners N asal Septum E rosions, C avitary Lung Lesions

C hurg Strauss T ransient peripheral lung consolidations.

M icroscopic Polyangiitis D iffuse pulm onary hem orrhage

Sm all Vessel (A N C A -)

HSP Kids. Intussusception. M assive scrotal edem a.

Behcets Pulm onary artery aneurysm

Buergers M ale sm oker. H and angiogram show s finger occlusions.

345
^ CA R O Tm D O PPLER i * . SB1® B * S“ "

G e n e ra l V a s c u la r U ltra s o u n d C o n c e p ts — S te n o s is :

The w aveform will go through changes before entering a stenosis, w ithin the stenosis, and after
exiting the stenosis.

Proximal (Upstream) S ten o sis Distal (Downstream)

Waveform can be normal, Waveform usually has a Waveform can be Tardus


monophasic decreased high velocity jet -might Parvus (prolonged “slow”
peak systolic velocity, or see aliasing. systolic acceleration /
loss of diastolic flow. upstroke, and small
High velocity = low systolic peak - “rounding
Diastolic flow is reduced in pressure (Bernouli’s effect of the peak”)
proportion to the severity mother fuckers)
of the stenosis.

Fuckery with words: It w ould be correct to say that Tardus Parvus is found dow nstream from a
stenosis. It w ould also be correct to say that Tardus Parvus is the result o f upstream stenosis.
See w hat I did right there? I'm not the only one w ho can pull som e shit like th a t...

34-6
High Yield Topics of Carotid Doppler

Stenosis: They will show you an elevated velocity (normal is < 125cm/s). They may also show
you the ICA/CCA ratio (normal is < 2), or the ICA end diastolic velocity (< 40 cm/s is normal).

Here are the rules:

• Less that 50% stenosis will not alter the peak systolic velocity

• 50-69% Stenosis: ICA PSV 125-230 cm/s , ICA/CCA PSV ratio: 2.0-4.0 . ICA EDV 40-100

• >70 % Stenosis: ICA PSV > 230 cm/s, ICA/CCA PSV ratio: > 4.0 , ICA EDV >100

Proximal Stenosis: OK here is the trick; they will show a tardus parvus waveform. If they show it
unilateral, it is stenosis of the innominate. If it's bilateral then it’s aortic stenosis.

Subclavian s te a l: This is discussed in greater detail in the cardiac chapter, but this time lets
show it on ultrasound. As a refresher, we are talking about stenosis and/or occlusion o f the
proximal subclavian artery with retrograde flow in the ipsilateral vertebral artery.

How will they show it? They are going to show two things: (1) Retrograde flow in the left vertebral,
and (2) a stenosis o f the subclavian artery with a high velocity.

How they can get really sneaky? They can


show this thing called “early steal.” Steal is
apparently a spectrum, which starts with mid-
systolic deceleration with antegrade late-
systolic velocities. Some people think the
“early steal” waveform looks like a rabbit.
Early Steal

347
THIS vs THAT: G a m e s m a n s h ip I n te r n a l C a ro tid vs E x te r n a l C a ro tid

This really lends itse lf w ell to m ultiple choice test questions. The big point to
understand is that the brain is alw ays on. You need blood flow to the brain all the tim e,
w hich m eans diastolic flow needs to be present all the tim e, and thus continuous color
flow throughout the cardiac cycle. T he external carotid feeds face m u sc le s... they only
need to be on w hen you eat and talk.

Internal C arotid E xternal C arotid

Low Resistance High R esistance

Low Systolic Velocity High Systolic Velocity

D iastolic velocity does not return to D iastolic velocity approaches zero


baseline baseline

C ontinuous color flow is seen throughout C olor flow is interm ittent during the
the cardiac cycle cardiac cycle

Temporal Tap - It is a technique Sonographers use to tell the external carotid from the
internal carotid. You tap the tem poral artery on the forehead and look for ripples in the
spectrum . The tech will usually w rite “ T T ” on the strip - w hen they do this.

*You can also look f o r branches to tell the external ca ro tid v.v the internal.

348
A o r tic R e g u r g ita tio n : - Just like aortic stenosis they are going to show you bilateral
CCAs. In this case you are going to get reversal o f d iastolic flow .

B ra in D e a th - A pparently in the ever-feuding m onarchies o f E urope, ultrasound can be


used for brain death studies. A loss o f d iastolic flow suggests cessation o f cerebral blood
flow.

A n e u r y s m s - In case som eone asks you, distal form ation o f an aneurysm (such as one in
the skull) cannot be detected by ultrasound, because proxim al flow rem ains norm al.

In tr a - A o r tic B a llo o n P u m p - R em em ber these guys are positioned so that the


superior balloon is 2 cm distal to the take o ff o f the left subclavian artery, and the inferior
aspect o f the balloon is ju st above the renals (you d o n ’t w ant it occluding im porting stu ff
w hen it inflates). W hen the balloon does inflate it will displace the blood in this segm ent o f
the aorta - sm ashing it superior and inferior to the balloon. The balloon will inflate during
early diastole (right after the aortic valve closes) because this is w hen the m axim um am ount
o f blood is available for displacem ent.

What does this do to the internal ca rotid (ICA) w aveform ? You are going to see an extra
bum p or “ augm entation” as the balloon inflates and displaces blood superior.

W hich w ave w o u ld y o u m easure to evaluate the velocity?


The first one (the one that is not assisted).

349
D o p p le r E v a lu a tio n in t h e LV A D

Bro... WTF is a "LVAD” ?

The Left V entricular A ssist D evice is a surgically


im planted device that helps pum p blood from the left
ventricle to the aorta. It's done in the setting o f severe
heart failure, typically as a bridge to cardiac
transplantation, or in those w ho are sim ply too evil to die
o f natural causes (D ick C heney) and require an
interm ediate step w hile the Darth Vader suit is prepped.

Testable D oppler Changes:

LVAD W aveform s will lose the norm al high resistance spiked look o f the ICA , C C A ,
Vertebral A rteries. Instead they are m ostly flat, w ith a tardus parvus look. The flow is
continuous through systole and diastole. T he little spikes you see during systole are from
the sm all am ount o f residual LV function.

ICA - LVAD Patient

ICA - Normal Patient

350
C la s s ic C a r o tid D o p p le r C a s e s

Normal Peak Velocity 60-100cm/s


|}
I^J- Continuous Diastolic Flow

J - Lower Peak Velocity

High Velocity Continuous Diastolic Flow


}
Higher Peak Velocity

J - Less Diastolic Flow

ICA O cclusion
-The C C A looks like the E C A , with a high
resistance waveform, and loss of diastolic
flow

Aortic Regurgitation
-With Classic Reversal of Diastolic Flow
-Most Likely Shown Bilaterally

Aortic Regurgitation
-This time showing the "Pulsus Bisferiens"
or double systolic peak. This is also seen in
hypertrophic obstructive cardiomyopathy.

Aortic Stenosis
-Characteristic Tardus Parvus waveform
-This will be shown B ILA T E R A L - to prove it’s
the aortic valve. Unilateral will be amore central
vascular stenosis.

D issection - with Flap

351
352
13
h b ] rr^ i n ^ i rrm i rrm i rte j i r^J nm ] rrm i rrs i rrs i rrg irr^ j rr^ i rrm i rrm i

In t e r v e n t i o n a l

P r o m e t h e u s L i o n h a r t , M . D .

353
fM IM
-- MMMM
——MM |, NSTRUMENTS
SECTo pIO |NNTERVENT|ON
1:

t
---- ||El|lKd|EU|ElHH!llEUlEUlHl

Puncture Needles:

• The smaller the “gauge” number, the bigger the needle. It’s totally P uncture N eedle sizes
counterintuitive. For example, an 8G Needle is much bigger than are designated by the
a 16G Needle. *This is the o p p o site o f a “French, ” which is u sed
O U T E R diam eter
to d escrib e the size o f a ca th eter o r dilator. The larger the French,
the larg er the catheter.
C atheter and D ilator
• The Gauge “G” refers to the OUTER diameter o f the needle. sizes are designated by
the O U T E R diam eter
Wires:

Just some general terminology: Sheaths are designated


by their IN N E R lumen
• 0.039 inch = 1mm size, (the m axim um
* 0.035 inch is the usual size for general purposes
• 0.018 and 0.014 are considered microwircs capacity o f a diam eter
* “Glide Wires” are hydrophilic coated wires that allow for they can accom m odate)
easier passage of occlusions, stenosis, small or tortuous
vessels.

Catheters - General

• 3 French = 1 mm (6 French = 2 mm, 9 French = 3 mm) Diameter in mm = Fr / 3


• Important trivia to understand is that the French size is the external diameter o f a catheter (not the
caliber of the internal lumen).
• The sta n d a rd 0.035 wire w ill f i t through a 4F ca th eter (or larger)

Sheaths
• Sheaths arc used during cases that require exchange o f multiple catheters. The sheath allows you to
change your catheters / wires without losing access.
• They are sized according to the largest catheter they will accommodate.
• The outer diameter o f a vascular sheath is usually 1,5F to 2F larger than the inner lumen.

Sheath - Size is Given by


~_J~
L 1F
(outer \ INNER Diameter
diameter) \
Add 2 F for the Outer
\ 2F
Diameter (1F + 1F = 2F) if
(Total Outer
/ Diameter) you want to know how big
the hole in the skin will be.
1F
• This would be a 6 F Sheath
> (outer
diameter) • The hole in the skin would be 8 F

354
G a m e s m a n s h ip — Various Forms o f Fuckery That Can Be Performed ~

There are a few classic w ays this can be asked. You can get all these questions right if you
understand the follow ing trivia:

• 3 French = 1 m m , so 1 French = 0.3 mm


• Puncture N eedles, G uide W ires, and D ilators are designated w ith sizes that describe their
O U TER diam eters.
• Sheaths are designated w ith sizes that describe their IN N E R diam eter
• The rubber part o f the sheath is about 2F (0.6 m m ) thick, so the hole in the skin is about
0.6m m bigger than the size o f the sheath.
• W ire DIAM ETERS are given in IN C H ES (exam ple “0.035 w ire” is 0.035 inches thick)
• W ire LEN G TH S is typically given in C E N T IM E T E R S (exam ple “ 180 w ire” is 180 cm
long)

Exam ple Q uiz 1: W hat is the size o f a puncture hole in m m o f a 6 F rench s h e a th ?

D iam eter in mm = F r / 3 • 6 French Describes the Inner Diameter


2 .7 mm = (6 + 2) / 3 • Add 2 French for the thickness o f the rubber
• Total is 8 French
Answer = 2.7 mm

E x a m p le Q u iz 2: What is the size o f a puncture hole in mm o f a 6 French sheath, p la c e d


coaxially into a short access sheath ?

**This is som e seco n d level fuckery.

6 French sheath is being p la c e d inside o f another sheath


6 French sheath is actually 8 French Big (rem em ber yo u a d d 2F f o r the rubber wall).
So you n eed a sheath that can accom m odate an 8 French Diameter. R em em ber that
sheaths are n a m e d fo r what they can accom m odate, so you n eed an 8 French Sheath.
8 French sheath is actu ally 10 French Big (rem em ber yo u a d d 2 F fo r the rubber
wall).

8F* 10F*

D iam eter in mm = Fr / 3
8 French Describes the Inner Diameter
3.3 mm = (8 + 2) / 3 Add 2 French for the thickness of the rubber
Total is 10 French
Answer = 3.3 mm

355
P u n c tu re N e e d le s - T h e L eg e n d C o n tin u es:

Some Conversions:

• 16G n e e d le h a s an o u te r d ia m e te r o f 1.65 m m , = 5 F c a th e te r;

• 2 0 G n e e d le h a s an o u te r d ia m e te r o f 0 .9 7 m m , = 3 F c a th e te r.

Some Needle Wire Rules:

O ld School S e ld in g e r T e c h n iq u e :
*Remember 0.035 is probably the
• 18G n e e d le w ill a c c e p t a 0 .0 3 8 in ch g u id e w ire most common wire used. Thus the
19G is the standard needle in
• 19G n e e d le w ill a llo w a 0 .0 3 5 in c h g u id e w ire many 1R suites.

M ic r o P u n c tu r e S ty le :

• In itia l p u n c tu re is p e rfo rm e d w ith a 2 1 G (ra th e r th a n a ty p ic a l 18G o r 19G ) n e e d le .

• 2 1 G n e e d le w ill a llo w a 0 .0 1 8 in ch g u id e w ire

• A fte r y o u h a v e th a t tin y w ire in , y o u c a n e x c h a n g e a fe w d ila to rs u p to a s ta n d a rd


4 F -5 F s y s te m w ith th e p o p u la r 0 .0 3 5 w ire .

M icro Puncture is Good when.... M icro Puncture is Bad when....

• A ccess is tough (exam ple = a fucking • Scarred Up Groins


antegrade fem o ra l puncture) • Big Fat P eople
• You suck ("lack experien ce") • When you try and upsize, som etim es
• Anatom ically sensitive areas (internal that flim sy 0.018 w o n ’t give enough
jugular, dialysis access) support fo r antegrade p a ssa g e o f a
dilator.

356
G u id e w ire s - T h e L eg e n d C o n tin u e s to C o n tin u e:

There are tw o m ain flavors o f guidew ires:

(1) N on-Steerable - T hese are used as supportive rails for catheters. These are N O T for
negotiating stenosis or selecting branches

(2) Steerable - These have different shaped tips that can be turned or flipped into tight spots.
W ithin this category is the “hydrophilic” coated w hich are used to fit into the tightest
spots.

H yd ro p h ilic G uidew ires - “Slippery when w e t" . T hey are sticky w hen dry, and super slippery
w hen w et. At m ost academ ic institutions dropping one o f these slippery strings on the floor
will result in “ not m eeting the m ilestone” and “ additional training” (w eekend PIC C w orkups).

• N ext Step Questions: C ould revolve around the need to “ w ipe the w ire with a wet sponge
each tim e it is used.”

• N ext Step Q uestions: Pretty m uch any situation w here you c a n ’t get into a tight spot. This
could be a stenotic vessel, or even an abscess cavity.

L ength - H ere is the testable trivia regarding w ire length.

• R em em ber D iam eter is in INCH ES, L ength is in C E N T IM E T E R S

• 180 cm is the standard length

• 260 cm is the long one. These are used if you are w orking in the upper extrem ity (from a
groin access), w orking in the visceral circulation and need to exchange catheters, using a
guide cath that is longer than 90 cm , through-and-through situation (“ body flossing” ).

• M inim al guidew ire length = length o f catheter + length o f the guidew ire in the patient.

Floppy T ips - A lot o f w ires have pointy ends and soft floppy ends. T he floppy ends are

usually available in different sizes. T he testable point is that the sh orter the floppy part the

greater the chance o f vessel dissection . For ex am p le, a 1 cm floppy tip has a greater risk o f

dissection com pared to a 6 cm floppy tip. T he practical tip is to choose a w ire w ith a long

floppy tip (unless you are trying to squeeze into a really tight spot).

35 7
G u id e w ire s - T h e L eg e n d C o n tin u e s to Continue to C o n tin u e

Stiffness: I feel like there are tw o prim ary w ays to ask questions about stiffness:

(1) Your classic "right tool f o r the right jo b " questions.


U nfortunately, these are often "read my mind, and understand my prejudices ” questions.
The follow ing are probably the clearest scenarios:

• Bentson (floppy tip) = C lassic guidew ire test for acute throm bus lysability

• L underquist (super stiff) = “T he coat hanger.” T his thing is pretty m uch only for aortic
stent grafting.

• H ydrophilic = Trying to get into a tight spot. Yes a B entson is also an option, but this is
m ore likely the “read my m ind” choice.

(2) Which is "Stiffer ? ” or Which is "Less S tif f ? ” types o f question. B asically ju st have a
general idea o f the progression. A second-order style question w ould be "Which is more or
less likely to cause dissection ? ” R em em ber the rule is “ m ore stiff = m ore dissection.”

Le ss More Stiff

“Noodle Like” Normal Supportive Stiff Hulk Smash !!

Bentson Hydrophilic Stiff Hydrophilic Flexfinder Lunderquist

Standard 0.035 J or “Heavy Duty” J or Amplatzer Stiff or Backup Meier


Straight Straight Extra Stiff
0.018 Platinum
Plus
V 18 - shapeable tip

Trivia: Stiff guidew ires should N E V E R be steered through even the m ildest o f curves. You
should alw ays introduce them through a catheter (that was originally placed over a
conventional guidew ire).

J Tip T erm inology: A “J S haped” Tip supposedly has the advantages o f not digging up
plaque and o f m issing branch vessels. O ften you will see a num ber associated with the J
(exam ple 3 m m , 5 m m , 10 m m , 15 mm e tc ...) . This num ber refers to the radius o f the
curve. Sm all curves m iss sm all branch vessels, larger curves m iss larger branch vessels. The
classic exam ple is the 15mm curve that can be used to avoid the profunda fem oris during the
dreaded arterial antegrade stick.

358
C a th e te rs - T h e L e g e n d C o n tin u e s
— F u c k e ry w ith N u m b e rs

If you look at a “buyers guide” or the packaging o f an angiographic catheter you may (if you
look hard enough) find 3 different num bers. 1 think it would be easy to write a question asking
you to ID the num bers, or asking what size sheath or wire you can use with the catheter.

The three numbers that you are going to see on the package are: the outer diam eter size (in
French), the inner diam eter size (in INCHES), and the length (in CEN TIM ETERS).

E x a m p le Q u e s tio n 1: What size is this catheter?

VANDELAY INDUSTRIES
FINE ANGIOGRAPHIC CATHETERS
Answ er = 4F
4, 110, 0 .0 3 5

Rem ember that the outer diam eter of a catheter defines it’s size (unlike the sheath which is
defined by the inner diam eter), and that these sizes are given in French. 4F catheters are very
commonly used. 110 and 0.035 are not catheter sizes available for hum ans existing outside of
middle earth.

Rem ember that length of the catheter is given in centim eters. The standard lengths vary from
about 45 cm to 125 centim eters.

Lastly the inner diam eter o f a catheter is given in inches and will pair up with the size wire.
For exam ple, the largest wire a 0.035 catheter will accom m odate is a 0.035.

E x a m p le Q u e s tio n 2: What size sheath and guidew ire can you use with this catheter?

VANDELAY INDUSTRIES Answ er =


FINE ANGIOGRAPHIC CATHETERS • 4F Sheath or Larger
*£ ' ' .' '■ - '
, ;
’ ^ ' • 0.035 wire or Sm aller
*a 0.038 wire would N O T fit
4, 110, 0 . 0 3 5

It’s a 4F sheath because sheaths are defined by their INNER diameter. So a 4F snake can crawl
through a 4F tube. Obviously a bigger tube (5F, 6F, e tc ...) will also have enough room for a 4F
snake.

It’s a 0.035 wire because the inner diam eter m easurem ents are given in inches, just like the
guidewires. In this case the tube is the catheter and the wire is the snake. So a bigger snake (any
wire thicker than 0.035) w ouldn’t fit.

359
C a th e te rs - S e le c tiv e vs N o n -S e le c tiv e

Just like G uidew ires can be grouped into “ steerable” or “non-steerable” , catheters can be
grouped into “ non-selective (flush) catheters” and “ selective catheters” .

• N on-Selective C atheters: These things are used to inject contrast into m edium and large
shaped vessels. This is w hy y o u ” ll hear them called “ flush catheters.”

• Selective C atheters: These things com e in a bunch o f different shapes/angles w ith the
goal o f “ selecting” a branch vessel (as the nam e w ould im ply).

N on -S elective C ath eters

Pigtail: For larger vessels this is the m ain w orkhorse. It’s called a “pigtail” because the
distal end curls up as you retract the w ire. T his curled m orphology keeps it out o f small
branch vessels. The catheter has both side and end holes.

Q: W hat m ight happen if you consistently inject through the pigtail like a pussy?

A: All the contrast will go out the proxim al side holes and not the tip. Eventually, if
you keep flushing like a pansy you will end up w ith a clot on the tip.

Q: W hat should you do prior to giving it the full on alpha m ale injection ?

A: G ive a sm all test injection to m ake sure you a re n ’t in or up against a sm all branch
vessel. Pigtails are for use in m edium to large vessels.

Q: W hat if the pigtail fails to form as you retract the w ire?

A: Push the catheter forw ard w hile tw isting.

Straight C atheter: This one d o e sn ’t curl up as you retract the w ire. O therw ise, it’s the
sam e as a pigtail w ith side holes and an end hole. The utility o f this catheter is for sm aller
vessels (w ith the caveat that they still need decent flow ).

The classic location is the iliac.

360
C a th e te rs - S e le c tiv e vs N o n -S e le c tiv e c o n t..

Selective C ath eters

Selective catheters com e in tw o m ain flavors: (1) end hole only, or (2) side + end holes
(,th a t’s what she said).

S id e + E n d H o le s
E n d H ole O n ly “Girl who went to catholic school”

Hand Injection Only


*high flow injection can displace the catheter, or
Works fine with Pump Injected runs
(can handle a rapid bolus without displacing)
cause dissection

Utility = Diagnostic Angiograms and


Utility = Classic would be a SMA Angiogram
Embolization Procedures
NEVER use with Embolotherapy.
The fucking coils can get trapped in the
sideholes or the particulate matter/mush may go
out a side hole and go crush the wrong vessel.
“Non-targeted” they call it.

I think the above chart is probably good enough to get m ost reasonable selective catheter
questions correct. U nfortunately, it’s also possible that you could be asked a "read m y mind,
understand my prejudices ” type o f question in the form o f "which cath eter w o u ld yo u use? ”

This is how I w ou ld guess - i f forced.

Acute Angle ( < 60) Angle of 60-120 Obtuse Angle ( > 120)
Example = Renals,
Example = Aortic Arch Vessels Example = Celiac, SMA, IMA
Maybe SMA and Celiac

“Angled Tip Catheter” “Curved Catheter” “Recurved”

“FtDC” Renal Double Curve, Sidewinder (also called a


-Berenstein or Headhunter
or a “Cobra" Simmons), or a “Sos Omni”

361
WTF is a “Recurve” ?

For whatever reason Academ ic Angio guys tend to spaz if residents do n ’t


understand why a “recurve”is different than a regular curved catheter.
Basically any curved catheter has a “prim ary” curve and a “secondary” curve.
On a regular curved cath both are in the same direction. However, on a
recurved cath the primary goes one way, and the secondary goes the other.

These catheters are good for vessels with an obtuse angle. You pull the
catheter back to drop into them.

I trained under a guy who was always very


impressed with himself when he could perform a
basic catheter maneuver like this. After dropping
into the IMA he would som etim es slowly turn to
one of the fem ale techs and say “that’s why I’m
the king. ”
I was really afraid he was going to break his own
arm, jerking himself off that hard.

That guy (or someone like him) is probably Recurve Curve


writing questions for the IR section (guessing /
not confirmed - but makes me smile to think so).

V o ca b
“Co-Axial Syste m s” - Basically one catheter inside another catheter/sheath. The most
basic example would be a catheter inside the lumen of an arterial sheath.

“Guide Catheters” - These are large catheters meant to guide up to the desired vessel.
Then you can swap them for something more conventional for distal catheterization.

“Introducer Guide” - This is another name for a long sheath. The assholes are trying to
trick you.

“Microcatheter” - These are little (2-3 French). They are the weapon of choice for tiny
vessels (example “super-selection” of peripheral or hepatic branches).

“Vascular Sheath” - It’s a sheath (plastic tube) + hemostatic valve + side-arm for
flushing

362
Flow Rate
“Give m e 2 0 f o r 30 ” — typical angio lingo for a run at 20cc/sec for a total o f 30cc.

H ow do y o u decide w hat the correct flo w rate is ? For the purpose o f m ultiple choice, I ’ll
ju st say m em orize the chart below. In real life you have to consider a bunch o f factors:
catheter size, catheter pressure tolerance, flow dynam ics, vessel size, volum e o f the distal
arterial bed (hand arteries can tolerate less blood displacem ent com pared to som ething like
the spleen), and interest in the venous system (a com m on concern in m esenteric angiography
- hence the relatively increased volum es in the SM A , IM A , and C eliac on the chart).

B igger A rtery = H igher Rate. You w ant to try and displace 1/3 o f the blood per second to get
an adequate picture.

T y p ic a l R ates / V olum es

Rate 1-2mL/sec Rate 4-8m L/sec Rate 5-7m L/sec Rate 20-30mL/sec
Volume 4-10 m L Volume 8-15 m L Volume 30-40 m L Volume 30-40 m L
Bronchial Artery Carotid Celiac Aorta,
Intercostal Artery Subclavian SMA Aortic Arch,
Renal IVC,
Femoral Pulmonary Artery
IMA

*IMA are typically *Abdom inal Aorta has


given a higher a slightly low er rate
volum e (15-30 mL) (15-20 m L /sec) a s it is
sm aller than the
Thoracic Aorta

Maxim um Flow Rates:

T h e s e are d e te r m i n e d b y th e I N T E R N A L d i a m e te r , le n g th , a n d n u m b e r o f size
h o le s . In g e n e r a l, ea ch F rench s iz e g iv e s y o u a b o u t 8 m l/s.

T h e s e a re the n u m b e r s I w o u l d g u e s s i f f o r c e d to on m u ltip le c h o ic e . In th e real


w o r ld its (a) w ritte n o n th e p a c k a g e a n d (b) a r a n g e o f n u m b e r s

3 F = 8 m l/s , 4 F = 16 m l/s , 5 F = 2 4 m l/s .

363
Catheter Flushing

D ouble Flush T echnique - This is used in situations w here even the sm allest throm bus or air
bubble is going to fuck w ith so m eo n e’s g o lf gam e (neuro 1R / cerebral angiogram s). The
technique is to (1) aspirate the catheter until you get blood in the catheter, then (2) you attach
a new clean saline filled syringe and flush.

Single Flush Technique - This is used everyw here else (below the clavicles). T he technique
is to (1) aspirate until you get about 1 drop o f blood in a saline filled syringe, and (2) tilt the
syringe 45 degrees and flush w ith saline only.

W hat i f y o u accidentally m ix e d the b lo o d in with the saline?

D iscard the syringe and double flush

W hat i f you are unable to aspirate any blood ?

H opefully you are ju st jam m ed against a side w all. Try pulling back or m anipulating the
catheter. If that d o e sn ’t w ork then you have to assum e you have a clot. In that case your
options are to (1) pull out and clear the clot outside the patient, or (2) blow the clot inside the
patient - you w ould only do this if you are em bolizing that location anyw ay (and a few other
situations that are beyond the scope o f this exam ).

364
Q F T T I O M 9*

t Va sc u l a r Ac cess

A rte ria l A c c e s s

G eneral “N ext Step ” Scenarios:

• You m eet resistance as y o u thread the guidew ire. N ext Step = STOP! “R e sista n c e" is an
angio buzzw ord for som ething bad. Pull the w ire out and confirm pulsatile flow.
R eposition the needle if necessary.

• The wire w ill not advance b eyond the top o f the needle even a fter y o u p u ll the wire back
and have norm al p u lsa tile flow . N ext Step = Flatten the needle against the skin. You are
assum ing the need to negotiate by a plaque.

• The wire stops after a sh o rt distance. N ext Step = L ook under fluoro to confirm correct
Fem oral ic
anatom A rtery
pathwAay.ccess
If it- This is al
is norm thethen
m ostyou
com m on put
could arterial
a 4F access
sheath route.
in and inject som e
A natom y review = the external iliac becom es the CFA after it gives the
contrast. A fter that m onkeying around with a hydrophilic w ire is o ff conventional answ er.
the inferior epigastric.

The ideal location is over the fem oral head (w hich gives you som ething to com press against),
distal to the inguinal ligam ent / epigastric artery and proxim al to the com m on fem oral
bifurcation.

* If you stick too high (above inguinal ligam ent): You risk retroperitoneal bleed
* If you stick too low, you risk AV Fistula
* If you stick at the bifurcation: You risk occluding branching vessels w ith y our sheath.

Too High Good Too Low

Inguinal - /4 5
Ligament IP:0e
CFA F. Vein (medial) 1

I
External
liac
="i£
^ %i
Femur

365
B rachial A ccess - Possible situations w hen you m ight w ant to do this:

• Fem oral A rtery is dead / unaccessible.

• The patien t’s abdom inal pannus, vagina, or ball sack is really stinky.

• U pper lim b angioplasty is needed

Special Testable Facts/Trivia:

• H olding pressure is often difficult. Even a sm all hem atom a can lead to m edial brachial
fascial com p artm en t syndrom e (cold fingers, w eakness) - and is a surgical em ergency
w hich m ay require fasciotom y.

• The risk o f stroke is higher (relative to fem oral access), if the catheter has to pass across
the great vessels / arch.

• A sheath larger than a 7F m ay require a surgical cut dow n.

• The vessel is sm aller and thus m ore prone to spasm . Som e people like to give prophylactic
“ G T N ” - glyceryl trinitrate, to prevent spasm .

Which arm ?

• Left Side if headed south (abdom inal aorta or low er extrem ity).

• Right Side if headed north (thoracic aorta or cerebral vessels).

• All things equal = Left side (it’s usually non-dom inant, and avoids the m ost cerebral
vessels).

• Blood pressure difference greater than 20 Systolic suggest a stenosis (choose the other
arm).

Radial A ccess - This is also a thing. T here are tw o pieces o f trivia that I think are the m ost
testable about this access type.

(1) Bedrest is not required after com pression.

(2) You need to perform an “A llen Test” prior to puncture. T he “A llen Test” confirm s
collateral flow via the ulnar artery to the hand (just in case you occlude the radial artery).
The test is done by m anually com pressing the radial and ulnar arteries. A pulse ox placed
on the m iddle finger should confirm desaturation. Then you release the ulnar artery and
saturation should im prove, proving the ulnar artery is feeding the hand.

366
Translum bar A ortic P uncture - This w as m ore com m only perform ed in the dark ages /
C retaceous period. You still see them occasionally done during the full-on thrash that is the
typical type 2 endoleak repair.

Trivia:

• The patient has to lay on his/her stom ach (for hours!) during these horrible thrashes

• H em atom a o f the psoas happens pretty m uch every case, but is rarely sym ptom atic.

• Known supraceliac aortic aneurysm is a contraindication

• Typically “high” access - around the endplate of T12 - is done. Although you can
technically go “low” - around L3.

• The patient “Self compresses” after the procedure by rolling over onto his/her back.

• Complaining about a “mild backache” occurs with literally every one of these cases
because they all get a psoas hematoma.

Pre Procedure Trivia: Prior to an arterial stick you have to know som e anticoagulation
trivia.

• Stop the heparin 2 hours prior to procedure (PT T 1.2x o f control or less; norm al
25-35 sec)
• IN R o f 1.5 is the num ber I'd pick if asked (technically this is in flux)
• Stop C oum adin at least 5-7 days prior (vitam in K 25-50 m g IM 4 hours prior, or FFP/
Cryo)
• Platelet count should be > 50K (som e texts say 75)
• Stop A S A /Plavix 5 days prior (according to SIR)
• Per the A C R - diagnostic angiography, routine angioplasty, and throm bolysis are
considered “ clean procedures.” T herefore, antibiotic prophylaxis is unnecessary.

Post P rocedure Trivia: By the book, you w ant 15 m inutes o f com pression. You can
typically pull a sheath w ith an A C T o f < 150-180. H eparin can get turned back on 2 hours
post (assum ing no com plications). G roin check and palpate pulses should be on the post
procedure nursing orders.

C losure Devices: N ever used if there is a question o f infection at the access site.

367
V en o us A c c e s s :

P IC C lines: U se the non-dom inant arm . The preference is basilic > brachial > cephalic.
You d o n 't place these in patients w ith CRF, on dialysis, or m aybe going to be on dialysis.

Median Cubital Vein Brachial V.

Cephalic V.

C entral Lines/P ort: The right IJ is preferred. E xternal ju g u la r veins can be used.
Subclavian access is contraindicated in patients w ith a contraindication to PICC lines.
D on’t place any tunneled lines/ports in septic patients (they get tem porary lines).

N ational K idney F oundation-D ialysis O utcom e Q uality Initiative (N K F -K D O Q I): O rder o f


preference for access: RIJ > LIJ > REJ > LEJ. “F istula F irst B reakthrough I n i t i a t i v e is
the reason you d o n ’t place PIC C s in dialysis patients.

W hat is the p re fe rred access site f o r a dialysis catheter? T he right IJ is the preferred
access, because it is the shortest route to the preferred location (the cavoatrial junction). It
w ill throm bose less than the subclavian (and even if it does, you d o n ’t lose drainage from
the arm - like you w ould w ith a subclavian). Fem oral approach is less desirable because
the groin is a dirty dirty place.

368
B leeding Applying Pressure
- W here dat hole b e ?
The w ord “h y p o te n sio n ” in the clinical vignette after an
arterial access should m ake you think about high sticks / The hole in the skin and
retroperitoneal bleeds. the hole in the artery don’t
typically line up.

Things that w on't help: • A n tegrade Puncture =


Below the skin entry point
• Yelling “ M other Fucker!!” - trust m e, I’ve tried this • A n tegrade Puncture on a
Fatty = Well Below the
skin entry point
Things that m ight help:
• R etrograde Puncture =
Above the skin entry
• Placing an angioplasty balloon across the site o f the
bleeding (or inflow ) vessel.

P seu d oan eu rysm T reatm ent: A s described in the vascular chapter, you can get a
pseudoaneurysm after a visit to the cardiology cath lab (or other rare causes). A lot o f the
tim e, sm all ones (< 2 cm ) will undergo spontaneous throm bosis. T he ones that will typically
respond to interventional therapy are those w ith long narrow necks, and sm all defects. There
are 3 m ain options for repair: (1) open surgery, (2) direct ultrasound com pression, or
(3) throm bin injection.

N ext S tep : Pain disproportionate to that expected after a percutaneous stick = G et an US to


look for a pseudoaneurysm

Direct com pression o f the Painful for the Patient (and


neck ( if possible avoid the R adiologist), can take 20
com pression o f the sac). m ins to an hour.
D irec t C o m p re ssio n
Enough pressure should be
applied to stop flow in the D o n 't com press if it’s above
neck. the inguinal ligam ent.

N eedle into apex o f cavity C ontra in d ica tio n s: Local


(aim tow ards the inflow infection , R apid
defect) - inject 0.5-1.0 ml E nlargem ent, D istal Limb
T h ro m b in In je c tio n (500-1000 u n its). Ischem ia, Large N eck (risk
for propagation),
Do N O T aspirate blood into Pseudoaneurysm cavity size
syringe - will clot. < 1cm.

M ay be needed if throm bin injection fails, there is


S u rg e ry infection, there is tissue breakdow n, or the aneurysm neck
is too w ide.

369
Pseudoaneurysm Treatm ent Cont:

W hich option do y o u p ick? For the purpose o f m ultiple choice I w ould suggest the follow ing:

Standard Starting Point: Cardiac Cath Lab

Infected Depends -
Yes Actively Bleeding No Yes
Combined Probably a
Surgical Skin Necrosis AV Fistula ? covered
Thrombotic Event stent

Got
2 cm
Better?

Repeat US
1 Week
Neck Size
< 1cm ?

Above vs Below
Thrombin Inguinal
Ligament
Below.
rAbove
Got Repeat US
Better? 2 Days Compression Depends -

i4-- f with US Probably a


*might be worth covered stent
a try, but wide
Failure neck reduces
T ry
success rate
a g a in

Surgical Surgical
*if the neck is very
wide (15mm or
more), you might
consider going
straight to surgery

370
P seu d oan eu rysm T reatm en t C ont:

This algorithm on the prior page assumes the Test Writer agrees that Thrombin is superior to
Compression (it hurts less, and probably has a higher success rate). Most modernly trained guys
will think like this. However, some conservative strategies fa vo r trying to compress all the ones
below the Inguinal ligament first - then trying thrombin second line. Simply read the test writers
mind to know his / her bias prior to answering.

Throm bin Injection - W here do you stick the needle ?

The needle should be placed


in the apex of the cavity (tip
directed towards the inflow
defect).

U ltrasound C om pression - W here do you com press ?

Orthogonal plane to the neck of the


pseudoaneurysm. Pressure is
directed to obliterate flow in the
neck / sac.

Femur

Trivia - Seriously Watch the Wording Carefully:

• Anticoagulation has no effect on thrombin injection treatment - primary success**


• Anticoagulation does* increase the risk of recurrence (10%?) after thrombin injection treatment
• Anticoagulation is NOT a contraindication to attempting direct compression, although it DOES
reduce success rate and most people will tell you to stop them prior to the procedure (if possible).
• Failure to respond to thrombin = Occult vascular issue (big puncture site laceration, infection)
• Untreated Pseudoaneurysm for greater than 30 days tend to resist compression and thrombin
therapy to variable degrees. They do best if treated within 2 weeks.
• Attempted compression o f a Pseudoaneurysm above the inguinal ligament can cause a RP bleed.
It is still safe to try and thrombin inject

371
S E C T I O N 3*
p t a and sten ts * ™ ™ “

G eneral Tips/Trivia regarding angioplasty: T he balloon should be big enough to take out
the stenosis and stretch the artery (slightly). T he ideal balloon dilation is about 10-20% over
the norm al artery diam eter. M ost IR guys/gals w ill claim success if the residual stenosis is
less than 30% . O bviously you w ant the patient anticoagulated, to avoid throm bosis after
intim al injury. The typical rule is 1-3 m onths o f anti-platelets (aspirin, clopidogrel) follow ing
a stent.

“ Prim ary Stenting": This is angioplasty first, then stent placem ent. You w ant to optim ize
your result. Stenting after angioplasty usually gives a better result than ju st angioplasty alone
(with a few exceptions - notably FM D - to w hich stenting adds very little). An im portant
idea is that a stent c a n 't do anything a balloon c a n ’t. In other w ords, the stent w o n 't open it
any m ore than the balloon w ill, it ju st prevents recoil.

Balloon E xpanding vs S e lf E xpanding: Stents com e in tw o basic flavors, balloon


expandable or se lf expandable. L ocation determ ines the choice.

S e lf E xpandable stents are good for areas that m ight get com pressed (superficial locations).

• Classic Exam ples = C ervical C arotid or SFA.

Balloon E xpandable stents are good for m ore precise deploym ent

• Classic E xam ple = Renal ostium

C losed vs O pen C ell Stents - V ascular stent designs m ay be categorized as (a) closed-cell -
w here every stent segm ent is connected by a link (less flexible, w ith better radial force) or
(b) open-cell in w hich som e stent segm ent connections are deliberately absent (flexible/
conform s to tortuous vessels, less radial force).

N itinol (m agic?): N itinol is said to have a “ therm al m em ory.’’ It is soft at room


tem perature, but can becom e m ore rigid at body tem perature. T his is exploited for self­
expanding stents.

D rug E luting Stents - These things have been used for C A D for a w hile. The purpose o f
the “ drug” is to retard neointim al hyperplasia.

372
Balloon Selection - Balloons should be 10-20% larger than the adjacent normal (non-stenotic)
vessel diameter. A sneaky move would be to try and get you to measure a post-stenotic dilation.

A rough guessing guide (ifforced):

A \
\
C om m on Iliac E xternal Iliac C f a , P i-o x S f a
Aorta = 10-15mm D istal SFA = 5m m
= 8m m = 7m m = 6mm

fii

\i
**Popliteal would he 4 mm

As a general rule, larger balloons allow for more dilating force but the risk o f exploding the vessel or
creating a dissection is also increased.

Stent Selection - Stents should be 1-2 cm longer than the stenosis and 1-2 mm wider than the un-
stenosed vessel lumen

Special Situations -

(1) You have more than 30% residual stenosis (failed you have). The first thing to do (if possible)
is to measure a pressure gradient. If there is no gradient across the lesion, you can still stop and
claim victory. If there is a gradient you might be dealing with elastic recoil (the lesion
disappeared with inflation, but reappeared after deflation). The next step in this case is to place a
stent.

(2) You can’t make the waist go away with balloon inflation. Switch balloons to either a higher
pressure rated balloon, or a “cutting balloon.”

(3) You caused a distal embolization. First do an angiographic run. If the limb / distal vessels look
fine then you don’t need to intervene. If you threatened the limb, then obtain ipsilateral access
and go after the clot (“aspiration”).

(4) You exploded the vessel (“Extravasation”). This is why you always leave the balloon on the
wire after angioplasty. If you see extravasation get that balloon back in there quickly, and
perform a low pressure insufflation proximal to the rupture to create tamponade. You may need
to call vascular surgery (“the real doctors”).

(5) W hat if you are trying to cross a tight stenosis and you see som ething like this ?

T his is the classic " s p ir a l” o f a dissecting wire.

373
E1E@E1MEIE@ ^ S T E N T TG R A F T S ‘

“ EVAR” = Endo Vascular T H I S vs THAT: E n d o g r a fts VS O p e n R ep air:


abdominal aortic Aneurysm Repair.
These include the bifurcated iliac • 30 Day M ortality is LESS for E ndovascular R epair
systems and unilateral aortic + iliac (like 30% less)
systems.
• L ong Term A neurysm R elated M ortality (and total
m ortality) is the SA M E for open vs endovascular
“ TEV A R” = Thoracic
repair
Endo Vascular aortic Aneurysm
Repair. • G raft R elated C om plications and R e-interventions
are H IG H E R w ith E ndovascular R epair

Indications fo r EVAR:

(1) AAA larger than 5 cm (or more than 2x the size o f the normal aorta)

(2) AAA growing “rapidly” (more than 0.5 cm in 6 months)

Anatomy Criteria fo r EVAR:

• Proximal landing zone must be:

• 10 mm long,

• Non- aneurysmal (less than 3.2 cm),


10 mm Long
< 3.2 cm Wide
• Angled less than 60 degrees. < 60 Degree of Tortuosity

Device Deployment:

Tortuosity and Vessel Size are issues for device deploym ent. The general rules are that you have
problems if:

• Iliac vessels have an angulation > 90 degrees (especially if heavily calcified)

• Iliac artery diam eters < 7 mm (may need a cut down and the placem ent o f a tem porary conduit).

374
Absolute Contraindication to Infrarenal EVAR:

Landing sites that w o n ’t allow for aneurysm exclusion

C overing a critical artery (IM A in the setting o f know n SM A and C eliac occlusion.
A ccessory renals that are feeding a horseshoe kidney, dom inant lum bar arteries feeding the
cord).

Dealing with the Renals.

There are several anatom y vocab w ords that are w orth know ing for aneurysm s near the renals.

• “P a ra -R e n a l” - w hich is an um brella term for aneurysm s near the renals

• “J u x ta -R e n a l" - A neurysm that has a “short n eck ” (proxim al landing zone < 1 cm ) or one
that encroaches on the renals.

• “S u p ra -R e n a l” - A neurysm that involves the renals and extends into the m esenteries.

• “C raw ford Type 4 T horacoabdom inal A o rtic A neurysm ” - A neurysm that extends from the
12th intercostal space to the iliac bifurcation w ith involvem ent o f the origins o f the renal,
superior m esenteric, and celiac arteries.

T reating these types o f aneurysm s requires all kinds o f fancy stuff; snorkels, chim ney
technique, e tc ... A ll is beyond the scope o f the exam . Just know that it can be done, but it’s
not easy.

Complications:
Adam kiew icz T9-T12
The m ost feared/dreaded (testable)
com plication o f an aortic stent graft is
C eliac T12
paraplegia secondary to cord
ischem ia. You see this m ost
com m only w hen there is extensive SMA L1
coverage o f the aorta (specifically T9-
T12 A dam kiew icz territory), or a R en als L2
previous A A A repair. “ B ew are o f the
hair pinned turn” - fam ously refers to L3
the m orphology o f A dam kiew icz on IMA
angiogram .
L4
Sym ptom s o f possible / developing paraplegia post
procedure. N ext S tep = C S F d ra in a g e .

375
A A A p re / p o s t E n d o g r a ft

A fter an aneurysm has been treated w ith an endograft, things can still go south. T here are 5
described types o f endoleaks that lend them selves easily to m ultiple choice questions.

* T ype 1 : Leak at the top (A) or the bottom (B) o f the graft. T hey are typically high
pressure and require intervention (or the sac w ill keep grow ing).
* T ype 2: Filling o f the sac via a feeder artery. This is the M O S T C O M M O N type,
and is usually seen after repair o f an abdom inal aneurysm . The m ost likely culprits
are the IM A or a L um bar artery. T he m ajority spontaneously resolve, but som e m ay
require treatm ent. Typically, you follow the sac size and if it grow s you treat it.
* T ype 3 : This is a defect/fracture in the graft. It is usually the result o f pieces not
overlapping.
* T ype 4 : This is from porosity o f the graft. ( “4 is fr o m the P o r e ”). It’s o f historic
significance, and d o e sn ’t happen w ith m odem grafts.
* T ype 5 : This is endotension. It’s not a true leak and it m ay be due to pulsation o f the
graft w all. Som e people d o n ’t believe in these, but I ’ve seen them . They are real.

T ype 1 T ype 2 Type 3 T ype 4


(80%)

T re a tm e n t: The endoleaks that m ust he em ergently treated are the high flo w ones - Type 1
a n d Type 3. M ost IR guys / vascular surgeons (real doctors) w ill w atch a Type 2 for at least a
year (as long as it’s not enlarging). M ost Type 4s will resolve w ithin 48 hours o f device
im plantation.

376
EMBOLIZATION ^ ^

T h ere’s a bunch o f reasons you m ight w ant to do this. The big ones are probably stopping a
bleed and killing a tumor.

W h ich a g e n t do you w a n t? U nfortunately ju st like picking a catheter these types o f


questions tend to fall into the m ind reading category.

In general you are going to choose the agent based on the desired outcom e, and the need to
m inim ize risk. The m ost classic thinking goes som ething like this:

Size of the Vessel I Wish this Destruction to be Kill the Mother Fucker Examples /
I Wish to Destroy: Temporary or Permanent: (or just give him Sample
nightmares): Indications:
Permanent Coils (Lung AVM)

Gelfoam Pledget
Temporary (Trauma)
Liquid Agent
Permanent (R C C Ablation)

Small Particles
Temporary (Fibroid Embo)

Micro Sphere
(Chemo)

A nother w ay to think / groups the agent is the general class. I think this is the m ost helpful to
talk about them in an introductory sense. A fter I introduce them , w e w ill revisit w hat to pick
based on a m ultiple choose vignette.

M ech anical P articulate L iquid A gents

Coils PVA - Particles (p e rm a n e n t) Sclerosants

Vascular Plugs (A m p la tzer) G elfoam (tem porary) N on-S clerosants

A utologous Blood Clot


(tem porary)

377
M e c h a n ic a l A g en ts:

Coils:

These are typically used to perm anently occlude a THIS vs THAT:


large vessel. They com e in all kinds o f different sizes Coils vs Micro-Coils
and shapes. You can deploy them w ith a “pu sh ” via a
coaxial system , or if you d o n ’t need exact precision Coils: Deployed via standard
you can “ chase” them w ith a saline bolus.
4-7F catheter

It gets com plicated and beyond the scope o f the exam Micro: Deployed via Micro-
Catheter. If you try and
(probably), but there are a variety o f strategies for
deploy them through a
keeping these in place. Just know you can pack these
standard cath they can ball up
things behind an A m platzer, or you can use
inside the thing and clog it.
scaffolding techniques to hook sm all coils to a large
one

B uzzw ord "A ccurate D e p lo y m e n t’’ = D etachable Coil

Trivia: R em em ber never deploy these w ith a side-hole + end-hole catheter. You w ant end-
hole only for accurate deploym ent.

Trivia: N ever pack coils directly into an arterial pseudoaneurysm sac - m ore on this later in
the chapter.

A m platzer V ascular Plug (AVP)

This is a self expanding wire m esh that is m ade o f N itinol (therm al m em ory Jam es Bond
shit). You m ount this bom b on the end o f a delivery device/w ire. W hen deployed it
shrinks in length and expands in w idth.

B est Use - High Flow Situations, w hen you w ant to kill a single large vessel. If you are
thinking to yo u rself - I'm gonna need a bunch o f coils to take that beast dow n the answ er is
probably an am platzer plug.

378
P a rtic u la te A g en ts:
T H IS vs THAT:
These are grouped into: Gelfoam Powder vs
Gelfoam Pledgets/Sheets
• Tem porary: G elfoam , A utologous Blood C lot
Powder causes occlusion at the
• P erm anent: PVA Particles capillary level (tissue necrosis)

B est Use = Situations w here you w ant to block Pledgets/Sheets cause occlusion
m ultiple vessels. C lassic exam ples w ould be fibroids at the arteriole or larger level
and m alignant tum ors. (tissue infarct is uncommon)

You are D oing it Wrong / Avoiding Reflux = A n easy w ay to ask this w ould sim ply be “ W hen do
you stop deploying the agent?”

The classic teaching is to stop em bolization w hen the flow becom es “ to and fro.” If you
continue to pile the particulate agent in until you get total occlusion you risk refluxing the agent
into a place you d o n ’t w ant it to go.

THIS vs THAT: C o ils vs PVA P a r t ic le s

In m any cases if you can use coils, you can also use appropriately sized particles.

Size is one w ay to pick. C oils are good for m edium to sm all arteries. PVA is good for
m ultiple sm all arteries or capillaries.
Sm aller particles (less than 300 m icrons) are going to risk tissue necrosis in m any cases -
so if you w ant to preserve the tissue, th a t’s probably the w rong answ er.
A nother tip for picking betw een the tw o is the need for repeat A ccess. The classic exam ple
is the bronchial artery em bolization. T hese things tend to re-bleed. So you should N E V ER
ever use coils (this will block you from re-accessing).
Bronchial artery em bolization = Particles (> 325 m icrom eters).

N e x t S te p ?

Q : W hat do you do after placem ent o f an occlusion balloon in the setting o f particle
em bolization ?

A : Test injection to confirm adequate occlusion.

379
Liquid A g en ts:

These are grouped into:

• Sclerosants: A bsolute A lcohol (the one that hurts) and Sodium D odecyl Sulfate (SD S)

• N on-Sclerosants: O nyx (E thylene-V inyl A lcohol C opolym er) , E thiodol

S clerosants:

As w ould be expected, the sclerosant agents w ork by producing near im m ediate throm bosis /
irreversible endothelial destruction. As a result, non-targeted em bolization can be fairly
devastating. There are three m ain strategies for not causing a m ajor fuck up (i.e. burning a
hole in the d u d e ’s stom ach, infarcting his bow el, e tc ...).

(1) K now ing the anatom y really well through careful m apping

(2) Frequent interm ittent angiogram s during the em bolization procedure

(3) Use o f B alloon O cclusion to protect non-target sites.

N e x t S tep ?

Q: W hat do you do prior to deflating the occlusion balloon?

A : A ggressively aspirate (w ith a 60 cc syringe) to m ake sure all the poison is out o f there.

N o n -S c le ro sa n ts:

O n yx: T y p ic a lly u s e d fo r n e u ro p r o c e d u r e s , h y p e r v a s c u la r s p in e t u m o r s , shit like that.


It d ry s s lo w ly (o u ts id e in) a n d a llo w s f o r a s lo w e r, m o r e c o n tr o lle d d e liv e ry .

E th io d o l: T h is is a n oil th a t b lo c k s v e s s e ls at the a rte rio le level ( s a m e as th e re a lly


sm a ll PV A p a rtic le s). F o r s o m e r e a s o n , h e p a to m a s lo v e th is stuff, a n d it w ill
p r e f e re n tia lly flow to th e h e p a to m a . It is a ls o u n iq u e in th a t it is r a d i o - o p a q u e , w h ic h
h e lp s d e c re a s e n o n - ta r g e te d e m b o l iz a t io n a n d lets y o u tra c k t u m o r size o n f o llo w up.

380
Sin gle Best A n sw er Classic Scenario

• Autologous Blood Clot = Post-Traumatic High-Flow Priapism (or Priapism induced by the female
Brazilian Olympic volleyball team)

• Varicocele (Spermatic Vein) = Coils


• Uterine Fibroid embolization (Bilateral Uterine Artery)= PVA or microspheres 500-1000 pm

• Generic Trauma = Gel Foam in many cases.

• Diffuse Splenic Trauma (Proximal embolization) = Amplatzer plug in the splenic artery proximal
to the short gastric arteries. **Discussed in detail later in the chapter.

• Pulmonary AVM = Coils

• Hemoptysis (Bronchial artery embolization) = PVA Particles (> 325 pm).

• Hyper-vascular Spinal Tumor = Onyx

• Total Renal Embolization = Absolute ethanol

• Partial or Selective Renal Embolization = Glue (bucrylate-ethiodized oil)

• Segmental Renal Artery Aneurysm = Coils

• Main Renal Artery Aneurysm = Covered Stent (or coils after bare metal stent)

• Peripartum hemorrhage = Gel Foam

• Upper GI Bleed = Endoscopy First (if that fail then in most cases coils)

• Lower GI Bleed = Usually Microcoils

LARGE Vessel • small Vessel - Post E m b olization Syndrom e:


Permanent Permanent

Pain, nausea, vom iting, and low grade fever - is


Particles
Coils basically an expected finding. You d o n ’t need to
order blood cultures - w ithout other factors to
Liquid Sclerosants
Amplatz Occluder m ake you consider infection. T here is a ru le o f
Thrombin 3 days - it starts w ithin the first 3 days, and goes
aw ay w ithin 3 days o f starting.
LARGE Vessel - Ethiodol
Temporary
'-r ., T he vignette is m ost classic for a large fibroid
small Vessel - em bolization, but it’s actually com m on after a
Temporary solid organ (e.g. liver) - the tum or ju st needs to
Gelfoam Pledget /
Sheet be big. Som e texts suggest prophylactic use o f
Microspheres anti-pyrexial and antiem etic m eds prior to the
Autologous Clot procedure.
Gelfoam Powder

381
SE C T IO N 6:
b h m e @e @e T Ac ute L imb
♦ he
CU P.se OF IH € B X C O N X IO P . *

“T hreatened L im b” - A cute lim b ischem ia can be secondary to throm botic or em bolic events.
Frequent sites for em boli to lodge are the com m on fem oral bifurcation and the popliteal
trifurcation. You can also get m ore distal em boli resulting in the so called blue toe syndrom e.

As crazy as this m ay sound to a R adiologist, physical exam is actually used to separate patients
into 3 categories: viable, threatened, or irreversible. T his chart (or som ething sim ilar) is how
m ost people triage.

Capillary Muscle Sensory Arterial Venous


Category Return Paralysis Loss Doppler Doppler
Not
1 - Viable Intact None None + +
Threatened

2a -Threatened Salvageable Intact/Slow None Partial - +

Salvageable
Slow/
2b - Threatened if immediate Partial Partial - +
Absent
intervention

NOT
3 - Irreversible Salvageable Absent Complete Complete - -
*Amputation

K now w ho y o u can a n d c a n ’t treat

“C ritical L im b Is c h e m ia ” - T h i s is described as rest


pain for two w eeks (or ulceration, or gangrene). ACR Appropriate:
Embolism Above / Below
G eneral Id ea on Treatm ent: An im portant point to the Common Femoral
realize is that lysis o f a clot only re-establishes the Artery
baseline (w hich w as likely bad to start w ith). So
after you do lysis, consider additional therapy - Isolated suprainguinal
(angioplasty, surgery, stenting, e tc ...). I f there is em bolism probably should be
com bined inflow and outflow disease, you should rem oved surgically.
treat the inflow first (they ju st do better). - F ragm ented distal em boli
should have endovascular
S u rg e ry vs Throm bolysis: If it has been occluded
throm bolytic therapy
for less than 14 days, throm bolysis is superior, if
m ore than 14 days, (surgery is superior).

382
A n k le - B ra c h ia l In d e x (A B I)

The idea behind the ABI is that you can com pare the blood pressure in the upper arm , to that of
the ankle and infer a degree o f stenosis in the peripheral arteries based on that ratio. In a norm al
person, ratios are usually slightly greater than 1. In patients w ith occlusive disease, they will be
less than that - w ith a low er num ber correlating roughly w ith the extent o f disease.

ABI

1.0 Normal No Symptoms


0.75-0.95 Mild Mild Claudication
0.5-0.75 Moderate Claudication
0.3-0.5 Moderate - Severe Severe Claudication
< 0.3 Severe or “Critical” Rest Pain

H o w th ey do it: You take blood pressures in both arm s, and both ankles. You only use one o f
the arm m easurem ents (the higher one). For the actual ratios, opinions vary on this - m ost
people do it by dividing the higher o f either the dorsalis pedis or p osterior tibial systolic pressure
(at the ankle) by the higher o f either the right or left arm systolic pressure.

F alse N u m b e rs? A rterial calcifications (com m on in diabetics w ith calcific m edial sclerosis)
m ake com pression difficult and can lead to a false elevation o f the A BI. T his is w hen you
will see ratios around 1.3 — those are bullshit, m eans the exam is non-diagnostic.

Toe P ressures: As above, diabetics will have noncom pressible vessels - w hich m akes A B Is
w orthless. W hat you can do is look at the toe pressure. The reason this w orks is because the
digital arteries are not as affected by this disease process. A norm al systolic toe pressure is
greater than 50 m m H g, and the ratio (toe-brachial index) should be m ore than 0.6. The testable
trivia is that i f the toe pressures are less than 3 0 m m ulcers are less likely to heal.

S e g m e n ta l L im b P ressu res: A m odification to the standard ABI involves pressures at the thigh,
calf, and ankle — if there is a pressure drop o f m ore than 20-30 you can infer that this is the level
o f disease. T his allow s you to sorta sorta sorta guess w here the level o f disease is.

383
Spectral Waveform Analysis:

The norm al pulsatile w ave is the result o f the


pum ping action o f the left ventricle
transm itted to the aortic root and then to the
foot. As the LV contracts you have a jet o f
blood that dynam ically expands the aortic
root. As the bolus o f blood travels tow ards
the feet the vessels will continue to expand
along the path — like a cartoon snake that has
eaten a m ouse (or your neighbors cat). The
w ave falls as the cardiac cycle enters diastole.

T here is a secondary event w hich is the rebound off Normal

the high resistance tibial vascular tree. T his is why A Worsening Disease
the norm al w ave has an up-dow n-up look to it —
“triphasic” they call it. T his bounce back or
rebound effect dem onstrates norm al arterial
com pliance. As the vessel hardens you lose this.
W ith progressive disease there is less and less
com pliance to the point w here the prim ary w ave
Tri-Phasic Bi-Phasic Mono-Phasic
barely even stretches the vessel.

Ulcer Location Trivia (dinosaur IR g uys love this - it really g ets their dicks hard):

* M edial A nkle = Venous Stasis


* D orsum o f Foot = Ischem ic or Infected ulcer
* Plantar (Sole) Surface o f Foot = N eurotrophic U lcer

Who are Rutherford and Fontaine? These are “useful” categories and classifications o f
signs and sym ptom s o f peripheral arterial disease.

False Numbers? A rterial calcifications (com m on in diabetics) m ake com pression difficult
and can lead to a false elevation o f the AB1.

Post-Operative Bypass Vocabulary:

* Prim ary Patency - U ninterrupted patency o f the graft w ith no procedure done on the
graft itse lf (repair o f distal vessels, or vessels at either anastom osis does not count as loss
o f prim ary patency).
* A ssisted Prim ary Patency - Patency is never lost, but is m aintained by prophylactic
interventions (stricture angioplasty etc..).
* Secondary Patency - G raft patency is lost, but then restored w ith intervention
(throm bectom y, throm bolysis, etc..).

3 84
“ W h e re to A c c e s s ? ”

One sim ple w ay to ask a threatened leg treatm ent question is to ask for the best route o f
access per lesion. A gain, like m any IR questions, this falls into the “depends on w ho you
ask” , and/or “ read m y m ind” category. If forced to choose, this is how I w ould guess:

L e s io n A ccess
First Choice - Ipsilateral CFA. If that is down also
Iliac (which it often is). I’d pick the contralateral CFA

CFA Contralateral CFA

SFA Ipsilateral CFA

Fern-Pop G raft Ipsilateral CFA

First Choice - Direct Stick.


Fem -Fem C ross-O ver Second choice-inflow CFA

If you are presented w ith other scenarios, the rule m ost people use is “shortest, m ost direct
approach.”

When w o u ld y o u use the contralateral CFA ? T here are tw o general situations:

1. The Ipsilateral CFA is occluded.

2. The patient is very very fat. Even fatter than your norm al acute leg patient. These are
the guys/gals w ho got the m ilkshake (instead o f the diet coke) w ith the baconator. As a
point o f gam esm anship, if the question header specifically m entions that the patient is
obese they are likely leading you tow ards contralateral access.

A G a m esm a n sh ip : W atch out for “retrograde” vs “ antegrade” access term inology in


the distractors. The nom enclature for a dow nw ard (tow ards the toe) access is
“ antegrade.” The term inology is based on the directions o f the arterial flow.

• A ntegrade access = tow ards the toes.

•R etrograde access = tow ards the heart.

385
G en eral P roced u ral T rivia I Possible “N e x t S te p s ”

There are a whole bunch of ways to do this. In the most generic terms, you jam the catheter into the
proximal clot and infuse TPA directly into the mother fucker. Every 6-8 hours you check to see if you
are making progress. People call that “check angiography.”

What i f you can't cross the clot with a wire? If they spell that out in the vignette, they are trying to tell
you that this clot is organized and probably won’t clear with thrombolysis.

What i f there is no clearing o f the clot during a "check angiogram ” ? If they specifically state this,
they are describing "lytic stagnation, ” which for most reasonable people is an indication to stop the
procedure.

The patient develops "confusion ” ? Neuro symptoms in a patient getting TPA should make you think
head bleed. Next step would be non-con CT head.

The patient develops “tachycardia and hypotension ” ? This in the setting o f TPA means the patient is
bleeding out. Next step would be (1) go to the bedside and look at the site. Assuming he/she isn’t
floating in a lake of their own blood (2) CT abdomen/pelvis and probably stopping the TPA.

End Point ? Most people will continue treating till the clot clears. Although continuing past 48 hours
is typically bad form.

Venous T re a tm e n t

Varicose Vein Treatment: Just know that “tumescent anesthesia” (lots o f diluted subcutaneous
lidocaine) is provided for ablation of veins. Veins arc ablated using an endoluminal heat source. A
contraindication to catheter-based vein ablation is DVT (they need those superficial veins).

DVT: The primary complications of DVT are acute PE and chronic post thrombotic syndrome (PTS).
There are several clinical predictive models to keep everyone who comes in the ER from getting a CTPA -
“Wells Score” is probably the most famous. Recently described is this “Thrombus Density Ratio” as a
superior predictor of PE in patients with known DVT on CTV. The density of thrombus on CTV has been
shown to be higher in patients with both DVT and PE relative to just DVT. Thrombus Density Ratio of
46.5 (thrombus HU / normal vein HU) = probable PE.

Phlegmasia alba (painful white leg) and Phlegmasia cerulea dolens (painful blue leg) - archaic
physical diagnosis terms that are high yield for the exam of the future. Phlegmasia alba = massive DVT,
without ischemia and preserved collateral veins. Phlegmasia cerulea dolens = massive DVT, complete
thrombosis of the deep venous system, including the collateral circulation. These are described as
extreme sequella of May-Thurner - but can occur in any situation where you get a punch of DVT
(pregnancy, malignancy, trauma, clogged IVC filter, etc..)

Post Thrombotic Syndrome (PTS): This is basically pain and stuff (venous ulcers) after a DVT. Risk
factors include being old (>65), a more proximal DVT, recurrent or persistent DVT, and being fat.
PTS is usually diagnosed between 6 months and 2 years after DVT. VEINES-QQL is the scoring
system used to diagnose and classify severity of PTS. Catheter-directed intrathrombus lysis of
iliofemoral DVT is done to prevent post thrombotic syndrome. This is not needed as much with
femoropopliteal DVT as it will recanalize more frequently and have less severe post thrombotic
syndrome.

386
S E C T I O N 7:

t F ilters

An IVC filter is used in the follow ing situations:

• Proven PE w hile on adequate anticoagulation

• C ontraindication to anticoagulation w ith clot in the fem oral or iliac veins

• N eeding to com e o ff anticoagulation - com plications. T here are a few additional


indications that are less firm (basically, w e think he/she m ight get a D V T and we c a n ’t anti­
coagulate).

V ocab: Why N o t L e a v e Them In?

* P erm anent Filters: Do N ot C om e O ut Depending on who bought


you lunch (gave you a free
* R etrievable F ilters: C an C om e O ut, But Do N ot H ave To
pen), thrombosis rates vary.
* Temporary Filter: C om e out, and have a com ponent sticking
outside the body to aide in retrieval In general (for the purpose
o f multiple choice) about
P osition (before subm ission): 10% o f the permanent
filters thrombose within 5
years.
The device is usually placed infrarenal w ith a few exceptions
(see below chart).

W hy isn i it alw ays ju s t p o sitio n e d suprarenal? A supra-renal filter has a theoretic increased
risk o f renal vein throm bosis. T here is zero evidence behind this - like m ost things in
m edicine.

In d ic a tio n F ilt e r P la c e m e n t R a tio n a le

Pregnancy Supra-renal To avoid com pression

Clot in the Renals or


Supra-renal G et above the clot
G onadals

E ither bilateral iliac, or


D uplicated IVCs supra-renal (above the G otta block them both
bifurcation)

R isk o f clot by passing filter


C ircum aortic Left Renal Below the low est renal
via the renals

387
m t u A -Cava: I f the IVC is less than 28 m m , then any filter can be placed. If it’s bigger

than that, you m ight need to place a b ird ’s nest type o f filter w hich can be used up to 40 mm.
You can also ju st place bilateral iliac filters.

R andom T rivia:

* A “G unther Tulip” has a superior end hook for retrieval


* A “ S im on-N itinol” has a low profile (7F) and can be placed in sm aller veins (like an arm
vein).
* All filters are M RI com patible

P rio r to p la c in g th e F ilte r:

You n e e d to do an angiographic run. W here I trained, the classic pim ping question for
residents on service w as to “nam e the 4 reasons you do an angiogram prior to filter
placem ent!” The only answ er that w ould not result in “ additional training” (m ore w eekend
PICC w orkups) was:

1. C onfirm patency o f the IVC

2. M easure the size o f the IVC

3. C onfirm that you are dealing w ith 1 IVC

4. D ocum ent the position o f the renal veins

C o m p lic a tio n s /R is k s :

M alposition: The tip o f the filter should be positioned at the level o f the renal vein. If it’s
not, honestly it’s not a big deal
M igration: The filter can m igrate to another part o f the IVC, the heart, or even the
pulm onary outflow tract. If it goes to the heart, you need surgery. If it’s ju st superior, you
need to snare it out.
Throm bosis: A lthough the incidence o f PE is decreased, the ris k o f D V T is in cre ase d .
C aval throm bosis is also increased, and you should know that clot in the filter is a
contraindication to rem oval (you need to lyse it, before you rem ove it).
IV C Perforation: A strut going through the caval wall is com m on and d o e sn ’t m ean
anything. How ever, aortic penetration, ureteral perforation, duodenal perforation, or
lum bar vessel laceration can occur (rarely) from a strut hanging out o f the cava - this is a
bigger problem .
D evice Infection: A relative contraindication to IV C filter placem ent is bacterem ia.

388
P o sitio n in g th e F ilte r:

Renals on an IV C Gram: There are two w ays O it O


to show the renals on an IVC Gram. There is
the nice way w here they opacify norm ally and O bvious Sneaky
it’s obvious, and there is the sneaky way w here
you see the "steam ing effect ” o f unopacified
blood allow ing you to infer the position.

Obviously the sneaky w ay is m ore likely to


show up on the exam.

The Tip: For standard anatom y, the standard answ er for a cone shaped fdter is to put the apex
at the level o f the renals. Som e people think the high flow in this location helps any clot that
m ight get stuck in the filter dissolve.

What i f there is clot in the IV C ? The filter should be positioned above the m ost cranial
extension o f the clot. As m entioned in m y glorious IVC Filter position chart, if the clot extends
beyond the renals you need a suprarenal filter.

What i f you fu ck up the deployment (severe tilt, legs won V open, etc...) ? If it’s retrievable,
you m ay be able to snare it and restart. If it’s perm anent you are kind o f hosed. Som e people
will try and stick a second filter above the retarded one.

F ilte r R em oval:

The longer these things stay in, the m ore likely they will throm bose. Prior to rem oval you
should perform an angiogram o f the IVC. The m ain reason to do this is to evaluate for clot.

• M ore than 1 cm 3 o f clot = Filter Stays In

• Less than 1 cm-1 o f clot = Filter Com es Out

You snare the filte r but when you p u ll on it you meet resistance ? In the real w orld, people
will yank that m other fucker out o f there. The IVC is the Rodney D angerfield o f vessels - no
respect. For m ultiple choice? Stop and assum e that it c a n ’t be retrieved.

A ngiogram should also be done after rem oval o f the filter to m ake sure you d id n ’t rip a hole in
the IVC. I f you did rip a hole in it - N ext Step - A ngioplasty balloon with low pressure
insufflation to to create tam ponade. If that d oesn’t work, m ost people w ould try a covered stent
graft. If you created a wall injury/dissection ? A gain - answ ers will vary, but the classic
answ er is system ic anticoagulation.

389
S E C T IO N 8: JttF

t iiiiiiiiB i D ia l y s is

- THE FUCKING FISTULOGRAM -

G enerally speaking there are tw o types o f “perm anent” access options for dialysis;
(1) the arterio-venous fistula and (2) the arterio-venous graft.

A V F istula - This is a subcutaneous anastom osis betw een an artery and adjacent native vein
(for exam ple the radial artery to the cephalic vein). All things equal, the preferred access
(over the graft).

A V G raft - This is also a subcutaneous anastom osis betw een an artery and adjacent native
vein. Except this tim e the distance betw een the vessels is bridged w ith a synthetic tube graft.

Arteriovenous Fistula Arteriovenous Graft

W hat are the pros and cons o f each types?

Pros o f AV G raft: C ons o f AV G raft:

- Ready for use in 2 w eeks -Less overall longevity


- Easier to declot (clot is usually confined -Prom otes hyperplasia o f the venous intim a
to the synthetic graft) at or dow nstream from the graft vein
anastom osis, resulting in stenosis and
eventual obstruction
Pros o f AV Fistula:
-M ore infections (foreign graft m aterial)
- Lasts L onger & M ore D urable
- M uch less prone to developm ent o f
C ons o f AV Fistula:
venous neointim al hyperplasia at or
dow nstream from the artery-vein -N eeds 3-4 M onths to “ M ature” (vein to
anastom osis. enlarge enough for dialysis)
- Few er infections

390
Why do grafts/fistulas need treatment (politics and greed) ? T he prim ary reason is “slow
flow s.” It’s im portant to understand that nephrologists get paid per session o f dialysis. If
they can do a session in 1 hour or 4 hours they m ake the sam e am ount o f m oney. T herefore
they w ant them running fast. So, really “ slow -flow ” is referring to slow cash flow in the
direction o f the nep h ro lo g ist’s pocket.

F or the p u rp o se o f m ultiple choice I ’d go with:

< 600 cc/m in for graft = d iagnostic fistulogram

< 500 cc/m in for fistula = diagnostic fistulogram

H aving said that, you m ay find different num bers different places - the w hole issue is
controversial based on the real m otivation people have for treating these. Som e texts say a
fistula can m aintain patency w ith rates as low as 80 cc/m in, and grafts can m aintain patency
w ith rates as low as 450 cc/m in. A lso rem em ber m edicare w o n ’t pay for tw o treatm ents
w ithin 90 days, so m ake sure you treat on day 91.

Why do grafts/fistulas need treatment (actualpathophysiology)? Its a violation o f nature to


have a AF Fistula / G raft pulsating in your arm . Y our body w o n 't tolerate it forever.
N eointim al hyperpasia develops causing an ever-w orsening stenosis. If they d o n ’t get
treated, they will eventually throm bose. All fistulas/grafts m ust die.

391
“W o rk in g it U p”

The only thing w orse then actually doing a fistulogram is having to talk w ith and exam ine the
patient prior to the procedure. N early all the IR texts and any program w orth its sn u ff will
“w ork them up” starting w ith physical exam .

Patient arrives in the IR departm ent for “ slow flow s.” N ext Step = P hysical E xam

This is the buzzw ord orientated algorithm that I w ould suggest for dealing w ith physical
exam / history related fistula/graft questions:

“Arm Swelling”

LOOK “Chest Wall Collaterals” Central Venous Stenosis


“Breast Swelling”

“Discolored Hand”
Dialysis-Associated S teal
LOOK “Pale Colored Hand”
(DASS)
S y n d ro m e
“Pallor of the Hand”

“High-Pitched Bruit”

LISTEN “Bruit in Systole Only” Localized Stenosis


“Discontinuous Bruit”
“Water Hammer Pulse” Pre Stenosis
FEEL
“Diminished Pulse” Post Stenosis

W hat is n o rm a l? A norm al graft has an easily com pressible p u lse , a lo w -p itch ed bruit that is
present in both systole + d ia sto le, and a thrill that is palpable w ith com pression only at the
arterial anastom osis.

L o c a lizin g a S ten o sis - C lassic E xam ple - Straight F orearm Graft:

392
GRAFTS:

Where is the problem (usually) in grafts? T he m ost com m on site o f obstruction is venous
outflow (usually at or ju st distal to the graft-to-vein anastom osis). This is usually secondary
to intim al hyperplasia.

What about the norm al th r ill and b ru it in a g ra ft ? T here should be a thrill at the arterial
anastom osis, and a low pitched bruit should be audible throughout the graft.

What i f the b ru it is high pitched? H igh Pitch = Stenosis, Low Pitch = N orm al

What are you thinking i f I te ll you the dude has a swollen arm and chest ? This is classic
for central venous stenosis.

F IS T U L A S :

Where is the problem (usually) in fistulas? It’s m ore variable - you are less likely to be
asked this. If you are forced - I’d say venous outflow stenosis - typically junta-anastom otic
or ru n o ff vein (AV anastom osis stenosis is uncom m on).

I f you f i x a stenotic area - they are good to go rig h t ? N ope - they reoccur about 75% o f the
tim e w ithin 6 m onths.

What about the “th r ill” in the fistula, is this a helpful fin d in g ? Yes - there should be a
continuous thrill at the anastom osis. If it is present only w ith systole then you are dealing
w ith a stenosis. A lso, if you can localize a thrill som ew here else in the venous outflow - that
is probably a stenosis.

What i f the fistu la is very “pulsatile” ? This indicates a m ore central stenosis - the fistula
should be only slightly pulsatile.

Should there be a b ru it ? A low pitched bruit in the outflow vein is an expected finding.

“Steal Syndrome” - T he classic story is “cold p a in fu l fin g ers” during dialysis, relieved by
m anual com pression o f the fistula. Too m uch blood going to the fistula leaves the hand
ischem ic. The issue is usually a stenosis in the native artery distal to the fistula. F ixing this
is typically surgical (DR1L = D istal R evascularization and Interval L igation o f Extrem ity, or
Flow R eduction B anding).

393
ACCESS and TR EA TM EN T:

Contraindications ? Infection is the only absolute one. If you fuck with an infected fistula or graft
the patient could get endocarditis. If you don’t fuck with it, the patient will probably still get
endocarditis but infectious disease will have to blame it on someone else at the QA meeting.

What i f it's “ Fresh ” ? A “relative contraindication” is a new graft or fistula. “New” to most people
means less than 30 days. Significant stenosis prior to 30 days strongly suggests a surgical fuck up
(“technical problem” they call it). Not to mention that a new dilating anastomosis is high risk for
rupture. Those grafts are doomed to never reach long-term patency.

Access less than 30 days old with stenosis. Next Step = Send them back to the surgeon.

What about “long segments” ? You will read some places that stenotic segments longer than 7 cm
respond poorly to treatment. Some people even consider this a “ relative contraindication.” If the
question writer actually spells out the length o f the stenosis greater than 7 cm he/she probably wants
you to say send them back to surgery. In reality there are plenty of stubborn IR guys that will try and
treat multiple long lesions because there is no better way than to prove one’s manhood.

What about a contrast allergy? You can use CO 2 for runs.

What direction do you access the graft ? Access is typically directed towards the venous
anastomosis - unless you are thinking arterial is the problem (which is much less common).
Remember the lingo “antegrade” and “retrograde” refers to the direction o f blood flow. Antegrade is
the typical route fo r venous problems, and retrograde is the typical route for arterial inflow issues.

How do you typically look at the arterial anastomosis ? The move most places teach is to obstruct
the venous outflow (with a clamp, blood pressure cuff, angioplasty balloon, finger - or whatever)
which allows the contrast to reflux into the artery.

What are the moves fo r angioplasty o f a narrow spot ? Give them heparin (3000-5000 units).
Exchange your catheter for a 5 or 6 F sheath over a standard 0.035-inch guidewire. Dilate the
narrow spot with a 6-8 mm balloon with multiple prolonged inflations. Remember to never take that
balloon off the wire when you are doing diagnostic runs - as you might need to rapidly put it back if
you caused a tear.

When do you place a stent ? There are two main reasons (1) you arc getting bad elastic recoil, or
(2) you have recurrent stenosis within 3 months o f angioplasty.

Does Nitro have a role? You can use a vasodilator (like nitroglycerin) to distinguish between spasm
and stenosis. The spasm should improve. The stenosis will be fixed.

What is considered a Successful Treatment? (1) Improved Symptoms (arm swelling better, etc..),
or (2) less than 30% residual stenosis.

What about Aneurysms ? Small ones get monitored for size increase, but the classic teaching is that
these are managed surgically.

General Vascular Access Trivia: Remember that PICC lines should not be put in dialysis (or
possible dialysis - CKD 4 or 5) patients because they might need that arm for a fistula.

394
S E C T I O N 9: J fe

t IE IE 1E @ P IP JP @ a ~

T IP S & B R TO
>

M l
l i i i i i i i i i i i i i A

J|P

T IP S (T ra n s ju g u la r In tr a h e p a tic P o rto s y s te m ic S h u n t) -

What is this portal hypertension? The portal vein gives you 70-80% o f your blood flow to
the liver. The pressure difference betw een the portal vein and IVC (‘fPSG” , portosystem ic
gradient) is norm ally 3-6 m m Hg. Portal HTN is defined as pressure in the portal vein >
10mm Hg or PSG > 5 mm Hg. The m ost com m on cause is EtOH (in N orth A m erica).

What does portal hypertension look tike? On ultrasound we are talking about an enlarged
portal vein (>1.3-1.5 cm), and enlarged splenic vein ( > 1.2 cm), big spleen, ascites,
portosystem ic collaterals (um bilical vein patency), and reversed flow in the portal vein.

Who gets a TIPS ? A ccepted indications include:

• Variceal hem orrhage that is refractory to endoscopic treatm ent

• Refractory ascites.

• Budd Chiari (throm bosis o f the hepatic veins) ** m ost authors w ill include this

Preprocedural steps fo r TIPS? You need two things. (1) An ECHO to evaluate for heart
failure (right or left). (2) Cross sectional im aging to confirm patency o f the portal vein.

How is a TIPS done? The real answ er is do an 1R fellow ship.

First thing you do is m easure the right heart pressure. If it is elevated (10-12 m m H g) you stop
(absolute contraindication). A norm al right heart pressure is around 5 m mHg.

If it is norm al, you proceed w ith the procedure.


Access the jugular vein on the right, go dow n the
IVC to the hepatic veins, opacify the veins, do a
w edge pressure (don’t blow the capsule off), use
C O 2 to opacify the portal system . Then stick
“Crotch to C rotch” from the hepatic veins to the
portal vein (usually right to right). Then put a
covered stent in and balloon it up.

Lastly check pressures and m ake you sure you


d id n ’t over do it (usually w ant a gradient around
9-12 “ less th a n 12” ).

395
Which direction do you
turn the catheter when
you are moving fro m the
right hepatic vein, to the
right portal vein?

You w ant to turn


anterior.

K ee p in g S c o re

W hile you are busy pretending you are a surgeon, w hy not pretend that you are a
m edicine doctor also? For the purpose o f m ultiple choice, anything that resem bles “ Real
D octor” w ork is alw ays high yield. "Score C alculation ” is the poster child for the kind
o f fringe know ledge board exam iners have traditionally loved to ask (this w as definitely
true for the old oral boards). The two highest yield scores are the M ELD and the C hilds-
Pugh.

What is this “ M E L D ” Score ? This w as initially MELD Child Pugh


developed to predict three m onth m ortality in TIPS
patients. N ow it’s used to help prioritize w hich
drunk driving, Hep C infected. A lcoholic should Bilirubin Bilirubin
get a transplant first. M ELD is based on liver and
renal function - calculated from bilirubin, INR, INR PT
creatinine. M ELD scores greater than 18 are at
higher risk o f early death after an elective TIPS.
creatinine Albumin
W h a t a b o u t th is “ C h ild s -P u g h ” S core ? This is
the “ old one,” w hich was previously used to
-------- Ascities, Hepatic
determ ine transplant urgency prior to the M ELD . Encephalopathy
It w orks for TIPS outcom es, too, but is "less
accurate ” than a M ELD. This score assesses the
severity o f liver disease by looking at the bilirubin, Greater than 18 = Class B and C are
album in, PT, ascites, and hepatic encephalopathy. High Risk Death High Risk
The trivia to know is that class B & C are risk
factors o f variceal hem orrhage.

Trivia = “ Sim plest prognostic m easure” = Serum Bilirubin. > 3 m g/dL is associated w ith
an increase in 30-day m ortality after TIPS.

396
What are the contraindications f o r TIPS? Som e sources w ill say there is no “absolute”
contraindication. O thers (m ost) w ill say severe heart failure (right or left), - but especially
right. T hat the w hole reason you check the right heart pressure at the beginning o f the
procedure. If you are forced to pick a contraindication and right heart failure is not an
option, I w ould choose biliary sepsis, or isolated gastric varices w ith splenic vein
occlusion. A ccepted (by m ost) “relative” contraindications include cavernous
transform ation o f the portal vein, and severe hepatic encephalopathy.

The main acute post procedural complications o f TIPS include: C ardiac decom pensation
(elevated right heart filling pressures), accelerated liver failure, and w orsening hepatic
encephalopathy.

Evaluation o f a “ N orm al TIPS ”

B ecause the stent decom presses the portal system , you w ant to see flow directed into the
stent. Flow should reverse in the right and left portal vein and flow directly into the stent.
Flow in the stent is typically 90-190 cm /s.

Stenosis / M alfunction:

* Elevated m axim um velocities (> 200 cm /s) across a narrow ed segm ent.
* Low portal vein velocity (< 30 cm /s is abnorm al).
* A tem poral increase (or decrease) in shunt velocity by m ore than 50 cm /s is also
considered direct evidence.
* “ Flow C onversion” w ith a change o f flow in a portal vein branch from tow ards the
stent to aw ay from the stent.
* A n indirect sign o f m alfunction is new or increased ascites.

TIPS Follow-Up

These things tend to fail (50% prim ary patent w ithin 1 year for a bare m etal stent), so they
need tight follow up.

W orsening A scites, B leeding, Etc (things that m ake you think the TIPS isn ’t w orking)

N ext Step = Venogram with pressures

PSG >12 m m H g. N ext Step = Treat the stenosis (angioplasty’ + balloon)

Trivia: The stenosis usually occurs in the hepatic vein, or w ithin the TIPS tract.

397
Addressing Hepatic Encephalopathy - D ropping the gradient too low increases the risk o f
HE. If the TIPS is too open you m ay need to tighten it dow n w ith another stent.

What is an alternative to TIPS fo r treatment o f refractory ascites? T here is a rarely


indicated thing called a “peritoneovenous shunt.” This stupid thing has a high rate o f
infection and throm bosis, and can even lead to DIC. It’s designed to allow drainage o f the
ascites through a tunneled line all the w ay up to the system ic circulation (jugular).

BRTO (Balloon-Occluded Retrograde Transverse O bliteration).

TIPS and BRTO are brother and sister procedures. W here the TIPS takes blood and steers it
aw ay from the liver (to try and help the side effects o f portal hypertension), the BRTO does
the opposite - driving m ore blood into the liver (to try and help w ith the side effects o f extra
hepatic shunting). The inverted indications and consequences are highly testable:

TIP S BRTO
Treat Esophageal Varices Treat Gastric Varices
Place a shunt to divert blood around liver Embolize collaterals to drive blood into liver

Complication is worsening hepatic Complication is worsening esophageal varices


encephalopathy and worsening ascites
Improves esophageal varices and ascites Improves hepatic encephalopathy

“ The Moves” : The general idea is that you


access the portosystem ic gastrorenal shunt
from the left renal via a transjugular or
transfem oral approach. A balloon is used to
occlude the outlet o f either the gastrorenal or
gastro-caval shunt. Follow ing balloon
occlusion, a venogram is perform ed. A
sclerosing agent is used to take the vessels
out. A fter 30-50 m inutes you aspirate the
rem aining sclerosing agent and let dow n the
balloons.

Trivia: The m ost com m on side effect o f BRT


is gross hem aturia.

398
SJ < S E C T I O N 1 O:
1111111111
♦ HEPATIC & BILIARY INTERVENTION

B ilia ry D u c t A n a to m y T riv ia :

The ductal anatom y m im ics the segm ental anatom y. The sim ple version is at the hilum .
There are tw o m ain hepatic ducts (right and left) w hich jo in to m ake the com m on hepatic
duct. The right hepatic duct is m ade o f the horizontal right p osterior (segm ent 6 & 7) and
vertical right anterior (segm ent 5 & 8). T he left duct has a horizontal course and drains
segm ent 2 and 4.

For w hatever reason, IR guys love to grill residents about ductal variants ( o f w hich there are
m any). There was a dinosaur GI guy w here I trained w ho also obsessed over this stuff.
A pparently, obscure anatom ic trivia tickles a psychopathology com m on to A cadem ic
Radiologists. As a result, I w ould know the 2 m ost com m on variants. The right posterior
segm ent branch draining into the left hepatic duct is the m ost com m on. The second m ost
com m on is trifurcation o f the intrahepatic radicles.

Common Hepatic Duct

“Normal” Duct Anatomy (57%) (16%) (12%)

399
B ilia ry D ra in a g e :

The role o f P T C (P ercutaneous T ranshepatic C holangiogram ) and P T B D (P ercutaneous


Transhepatic B iliary D rainage) is centered around situations w hen ER CP and endoscopy
have failed or are not possible (R oux-en-Y ).

T hings to do before th e p ro ced u re:

• C heck the coags - correct them if necessary (vitam in K, FFP, e tc ...).

• M ost institutions give prophylactic antibiotics (ascending cholangitis is bad).

A p p ro a ch es:

T here are tw o approaches: right lateral m id axillary for the right system , or subxyphoid for
the left system . Realistically, diagnostic cholangiogram and PTB D is usually done from
the right. The left is m ore technically challenging (although better tolerated by the patient
because the tube isn ’t in-betw een ribs) and usually there is a hilar stricture that w o n ’t
allow the left and right system to com m unicate.

“The M o v e s ” - R ig h t-S id e d A p p ro a c h

Line up on the p a tie n t’s right flank / m id axillary line. When I say “Below the 10th
Find the 1Oth rib. D o n ’t go higher than the 10th rib - Rib,” I mean caudal to the 10th
rib, not actually under the rib.
alw ays below (avoiding the pleura can save you a ton
Always puncture at the TOP
o f headaches). Prior to jam m in g the needle in, m ost EDGE OF A RIB to avoid the
reasonable people put m etal forceps (or other m etal intercostal artery (which runs
tool) over the target and fluoro to confirm you are over under the rib).
the liver and below the pleural reflection.

N ow the fun begins. The basic idea is to pretend the patient is a voodoo doll o f the
A ttending (or childhood torm entor) that you hate the m ost. Proceed to blindly and
random ly jam a chiba needle in and inject slow ly under fluoro as you pull back (but not all
the w ay out). O bviously less sticks is better and it’s ideal to do in less than 5 (m ost places
will still consider less than 15 ok). O nce you get into a duct the system will opacify. You
then can pick your target (posterior is best for best drainage). You stick again, w ire in, and
place the catheter into the duodenum .

A non-dilated system can be very difficult and In the Duct?


there is an old school trick w here you stick the
• Ducts - Flow Towards the Hilum
gallbladder (on purpose) and retrograde fill the • Vein = Flow Cranially towards the
system . The problem w ith that is you have to Heart
keep a drain in the gallbladder as well. • Artery - Flow Towards the Periphery

400
The Moves” - L e ft Sided A p p ro ac h

This tim e you use a sub-sternal / subxyphoid approach w ith ultrasound. M ost people aim
for the anterior inferior peripheral ducts. O therw ise the m oves are pretty m uch the sam e.

Catheter/ Stent Choice - Bare Bones o f Trivia:

M ost stent placem ent is preceded by a period o f biliary decom pression w ith an internal-
external drain. Plastic stents are cheaper but have a short patency period. M etal stents will
stay patent longer but c a n ’t be rem oved. M etal stents are not usually used in benign disease
unless the patient has a long life expectancy.

Internal-E xternal drains are the standard for crossing lesions. They have superior stability
to a straight drain or pigtail. They offer the advantage o f possible conversion to an internal
only drain (save those bile-salts).

Som e testable
trivia is that m any
centers will
m anually punch
Side
som e additional
Holes
side holes in the
proxim al portion o f
the tube to m ake
sure that drainage
adequate.

The key is to N O T
position any side
holes outside the
Do NOT Put the
liver (proxim al to Side Holes Here
liver parenchym a).

401
Q “ N e x t S t e p ” - T e s t a b le S c e n a r io s I T r iv ia :

• T here is extensive ascites. N ext Step = Drain it prior to doing the PTC.

• T here is a sm all am ount o f ascites. N ext Step = O pinions are like assholes (everyone has
one), so y o u ’ll hear different things for this. I think m ost people w ould look to m ake sure
the liver still abuts the peritoneum at the puncture site. If it does, then they will do a right
sided approach. If it d o esn ’t then they will use ultrasound and go substem al on the left.

• R ight A p proach with no filling o f the left ducts. N ext Step = Slow ly and carefully roll
the patient on their side (right side up). The right ducts are dependent - so this is actually
fairly com m on. N ow obviously if there is a know n obstruction you d o n ’t need to roll
them . T he rolling is to prove it’s not a real obstruction.

• You do your contrast run and the patient instantly goes into a full rigor.
N ext Step = L ook around the room for the person you are going to blam e for injecting too
forcefully. “ R igor” is a m ultiple choice buzzw ord for cholangitis. Y es... full on biliary
sepsis can happen instantly w ith a forceful injection. This is actually pretty easy to do if
the patient has strictures or an obstructive neoplastic process. In those cases you will w ant
to have your Scapegoat / “N ot M e” (tech, m ed student, resident, fellow, drug rep, non-
english-speaking international observer) do the injection. For the purpose o f m ultiple
choice som e good next step options w ould be: aggressive resuscitation, place a drain, and
inform the prim ary team / ICU that the Scapegoat gave the patient biliary sepsis.

Forceful Injection = ICU Visit for Cholangitis

Buzzword = “Rigors after Injection”

• You encounter (or expect stones). N ext Step = D ilute contrast to 200-250 m g/m l to
avoid obscuring filling defects.

• You c a n ’t cross the obstruction with a wire. N ext Step = Place a pigtail drain and let the
system cool dow n for like 48 hours. Try again w hen there is less edem a.

402
C h o lec ys to s to m y:

This is done w hen you have a super sick patient you c a n ’t take to the O R, but the patient has
a toxic gallbladder. In cases o f acalculous cholecystitis (w ith no other source o f sepsis), 60%
o f the tim e cholecystostom y is very helpful. It’s a “tem porizing m easure.” You have to give
pre-procedure antibiotics. T here are tw o approaches:

* T ransperitoneal - T his is preferred by m any because it’s a direct approach, and avoids
hitting the liver. The m ajor draw back is the w ire / catheter often buckles and you lose
access (and spill bile everyw here). This is typically not the first choice. H ow ever in
patients w ith liver disease or coagulopathy it m ay be preferred (depending on w ho you
ask). I f the question w riter specifically states (or infers) that the p a tie n t has an increased
risk o f bleeding this is p ro b a b ly the right choice. O therw ise, if forced to choose, pick the
Transhepatic route

* T ranshepatic - The m ajor plus here is that w hen


you cross the liver it stabilizes the w ire and
m in im izes th e c h a n c e o f a bile leak. T his is the
route m ost people ch o o se.

o Trivia = Typically you go through segm ents 5


a n d 6 on y our w ay to the gallbladder

o Trivia = This route transverse the “bare area” /


upper one third o f the gallbladder
(hypothetically).

Im p o rta n t Trivia:

* Prior to the procedure, m ake sure the bow el isn ’t interposed in front o f the liver/
gallbladder. If a m ultiple choice w riter w anted to be sneaky he/she could tell you the
patient has "C hilaiditi S y n d ro m e ” - w hich ju st m eans that they have bow el in front o f
their liver. Som e sources will list this as a contraindication to PC.

* Even if the procedure instantly resolves all sym ptom s, you need to leave the tube in for
2-6 w eeks (until the tract m atures), otherw ise you are going to get a bile leak.
* A fter that ‘‘at least 2 w eek ” period you should p erfo rm a cholangiogram to confirm that
the cystic duct is patent before you pull the tube.

* M ost places will clam p the tube f o r 48 hours p r io r to rem oval. T his helps confirm
satisfactory internal drainage.

M a n a g in g B ile L e a k - Bile leak is bad as it can lead to m assive biliary ascites and chem ical
peritonitis. M ost people will try and place a tube w ithin the bile ducts to divert bile from the
location o f the leak (this usually w orks).

403
G e n e ra l B iopsy P e a rls

There are tw o prim ary techniques for sam pling tissue:

(1) Fine N eedle A spiration — C ytology

(2) C utting N eedle (“ C ore”) — B iopsy

Fine N eedle A spiration:

T his is for situations w hen you only need a few cells. It is typically perform ed through a
21 or 22G C hiba needle. Vacuum aspiration with a 20 cc syringe is applied as you pass
the needle back and forth through the target.

Trivia: A pply “gentle” suction as you rem ove the needle. If you suck too hard a tiny
sam ple could get lost in the syringe. If you forget to apply suction the sam ple will stay in
the patient.

The needle is sm all so the risks are sm all.

C utting / C ore N eedle

This is for situations when you need


a larger sam ple. T here are lots o f
devices but the m ost basic
m echanism involves a needle with
two parts; an outer shaft for cutting,
and an inner stylet.
O uter S ty le t C lo se s To Cut S am ple
Trivia: For the purpose o f m ultiple
choice, the target is “cut” w here the
outer shaft is advanced.

Trivia: The general rule is pick the shortest length needle that will reach the target.

Trivia: “A utom ated S ystem s” fire both the inner and outer com ponents to take the
sam ple. The key point is that w ith these system s the sam ple is taken from tissue 10-20
m m in front o f the needle.

404
C o n v e n tio n al L iv e r B iopsy -

You can do targeted approaches


(for a specific lesion) or you
This path might
can do non-targeted approaches be ok - if it’s thick
enough
(sam pling). G eneral pearls
include: trying to cross the
> 2 cm
would be
capsule only once, biopsy the ideal
subcapsular m asses through an
area o f uninvolved liver, and
avoid the diaphragm .

) This path might


If given the choice, you w ant to This direct path into be ok - if it’s thick
biopsy peripheral lesions lesion will ‘‘bleed like enough
stink” > 2 cm
through 2-3 cm o f norm al liver would be
ideal
prior to hitting the target.

This is done to avoid a blood bath.

N e x t S tep : There is ascites = D rain it prior to doing the biopsy

Trivia: M ild shoulder pain (referred pain) is com m on after liver biopsy.

Trivia: Prolonged S houlder Pain (> 5 m ins) = Possible Bleed "K ehr Sign ”.

N e x t Step: Prolonged S houlder Pain (> 5 m ins) = R e-evaluation w ith ultrasound. A lw ays
look behind the liver (M o rriso n ’s pouch) to see if blood is accum ulating. B leeding after
liver biopsy occurs m ore from biopsy o f m alignant lesions (com pared to diffuse disease).

C o n traindications: U ncorrectable coagulopathy, throm bocytopenia (< 50,000), infections in


the right upper quadrant - are contraindications for a conventional biopsy.

Trivia: B iopsy o f carcinoid m ets is controversial and death by carcinoid crisis has occurred
after biopsy.

N e x t S tep : M assive ascites or severe coagulopathy = Transjugular approach

405
T ra n s ju g u la r L iv er B iopsy

The rationale is that the liver


S p e c ific In d ic a tio n s :

capsule is never punctured, so • Severe C oagulopathy


bleeding is less o f a risk.
• Massive ascites
O bviously this is a n o n ­
targeted biopsy for the • Failure o f prior percutaneous liver biopsy
diagnosis o f infectious,
• Massive obesity ("Fat Even By West Virginia Standards ”)
m etabolic, and som etim es
• Patients on mechanical ventilation
neoplastic processes
(classic exam ple = grading • Need for additive vascular procedures like TIPS
chronic hep C).

P rocedural Trivia:

The general technique is to access the hepatic veins via the IVC (via the right ju g u la r vein).
M ost people will tell you to biopsy through the right hepatic vein w hile angling the sheath
anterior. The reason this is done is to get the biggest bite o f tissue, and avoid capsular
perforation (w hich w as the entire point o f this pain in the ass procedure).

Trivia: Right Sided Jugular Route


is the superior route (better than left
IJ, or fem oral)

Trivia: B iopsy via the Right


H epatic Vein by angling anterior.
N ever perform an anterior biopsy
from the m iddle hepatic vein.

Trivia: This procedure has the


added benefit o f allow ing you to
m easure hepatic venous pressures -
w hich can guide therapy or assess
varix bleeding risk.

406
H e p a tic I S p le n ic T ra u m a -

Em bolization is a potential m ethod for dealing


I n d ic a tio n s
w ith significant traum a to the hepatic or *agreed upon by most:
splenic arteries. O pinions on the exact role o f
• Continuous hemorrhage (active extrav)
angiography vary betw een institutions, so
in a patient who is borderline stable
“read m y m ind” questions are likely. post resuscitation

I think the m ost likely type o f indication • Early ongoing bleeding after a surgical
attempt to gain primary hemostasis
question m ight actually be w ho does N O T go
to angio? • Reblceding after successful initial
embolization
The m ost accepted contraindication in a
bleeding patient is probably a very busted-up • Post traumatic pseudo-aneurysm and
AVFs (even if they aren’t currently
unstable dude w ho needs to go straight to the
bleeding).
OR for em ergent laparotom y.

Tools and Strategy - Hepatic Considerations:

• G elfoam , pledgets, particles, and/or m icrocoils are typically used.

• M assive non-selective hepatic artery em bolization is usually avoided to reduce the risk o f
large volum e tissue necrosis.

• W hat's the m ain issue with tissue necrosis? H epatic abscess developm ent (w hich is fairly
com m on in a m ajor liver injury anyw ay).

• Trivia: C oils should N O T be placed in the


pseudoaneurysm sac. This can lead to a
late rupture. The strategy is to occlude
Distal End First
the distal and proxim al parent vessels.
Y ou'll w ant to perform “ com pletion
angiography” to prove the thing is
If 1 II) ^-1 1 i)
Sandwich Technique”\o exclude
arterial pseudo aneurysm Then Pull Back
occluded prior to catheter rem oval.

H epatic surface is bleeding from m ore


than one spot. N ext Step = G elfoam or particles.

H epatic Pseudoaneurysm s can be treated at the site o f injury (w ith the sandw ich technique)
because they are not end arteries (no collaterals). Plus the liver has a dual blood supply.

407
Tools and Strategy Continued - Spleen Considerations:

• Splenic laceration (w ithout active extravasation) is N O T considered an indication to angio


(by m ost people). R em em ber to use your m ind reading pow ers to confirm the question
w riter agrees.

• The spleen does not have a dual blood supply, and is considered an “end organ” unlike the
liver. So if you go nuts em bolizing it you can infarct the w hole fucking thing.

• Focal Splenic A bnorm ality. N ext Step = Selective E m bolization treatm ent

• M ultiple Bleeding Sites. N ext Step = Use a proxim al em bolization strategy, and drop an
A m platzer plug into the splenic artery proxim al to the short gastric arteries. The idea is to
m aintain perfusion but reduce the pressure to the spleen (slow er blood will clot), w ith the
benefit o f preserved collateral supply and less infarction risk.

• Trivia: Even w ith this proxim al em bolization strategy the patient usually does not require
vaccination post em bolization, as a lot o f functional tissue should rem ain.

Short
Gastric
Arteries

Proximal
Embolization Site

408
HCC T re a tm e n t:
A C R Appropriate: Liver
You will read in som e sources that transplant is the only way Transplant
to “cure” an HCC. O thers will say transplant, resection, or — Transplantation should be
ablation are “curative” if the tum or is small enough. A rterial considered ONLY in patients
em bolization (TACE) is typically used in situations w here < 65 years o f age with limited
tumor burden (1 tumor < 5
the tum or burden is advanced and the patient cannot undergo cm or up to 3 tumors < 3 cm).
surgery.

T ransarterial C h em oem b olization (TA C E) - M ost people will consider this first line for
palliative therapy in advanced cases. The m echanism relies on H C C ’s preference for arterial
blood. High concentration o f chem otherapy w ithin Lipiodol (iodized oil transport agent) is
directly delivered into the hepatic arterial system . The tum or will preferentially take up the oil
resulting in a prolonged targeted chem otherapy. The Lipiodol is usually follow ed up with
particle em bolization, with the goal o f slow ing dow n the w ashout o f the agent.

Absolute Contraindication = D ecom pensated (acute on chronic) Liver failure.

Trivia: Som e sources will list portal vein throm bosis as a contraindication (because o f the risk
o f liver infarct). O thers say portal vein throm bosis is fine as long as an adjustm ent is m ade to
lim it the degree o f em bolization and you can docum ent sufficient hepatic collateral flow.
Simply read the m ind o f the question w riter to know w hich cam p they are in.

Trivia = TACE in Patients with a biliary stent, prior sphinctertom y, or post W hipple are all
high risk for biliary abscess.

Trivia = “Sterile ch olecystitis” or “chem ical ch o lecystitis” are buzzw ords that w hen used in
the setting o f TACE should lead you to believe that the agent was injected into the right
hepatic artery’ prior to the ta ke o ff o f the cystic artery (artery to the gallbladder) .

Trivia = TACE will prolong survival better than system ic chem o

Trivia: Unfortunately, repeat TACEs can result in a ton o f angio tim e and therefore a ton o f
radiation. Patient do som etim es get skins burns (usually on their left back because o f the
RAO cam era angle).

RFA: Tum or is destroyed by heating the tissue to 60 degrees C (140 F). Any focal or nodular
peripheral enhancem ent in the ablation lesion should be considered residual / recurrent
disease. Som etim es, on the im m ediate post treatm ent study you can have som e reactive
peripheral hyperem ia - but this should decrease on residual studies. Im portant trivia is that
RF ablation is indicated in patients w ith HCC and colorectal m ets (w ho can ’t get surgery).

TACE + RFA: As a point o f trivia, it has been shown that TACE + RFA for HCC lesions
larger than 3cm, will im prove survival (m ore so than either treatm ent alone). This is still not
curative.

409
Y ttrium -90 R ad ioem bolization - An alternative to
TACE is using radioactive em bolic m aterials
(Y-90). T he prim ary testable trivia regarding Y-90 W h a t is th is Y t t r iu m ?
therapy is understanding the pre-therapy w ork up.
Yttrium-90 is a high-energy beta
T here are basically tw o things to know: emitter with a mean energy of
0.93MeV. It has no primary gamma
(1) L u n g Shunt F raction - You give Tc-99 emission. Yttrium-90 has a half-life of
64 hours. After administration, 94% o f
M A A to the hepatic artery to determ ine how
the radiation is delivered over 11 days
m uch pulm onary shunting occurs. A shunt (4 half-lives). The maximum range of
fraction that w ould give 30 Gy in a single irradiation from each bead is 1.1 cm.

treatm ent is too m uch (Y-90 is


contraindicated).

(2) The take o f f o f the right gastric. The fear is that you get non-targeted poisoning o f
the stom ach, leading to a non-healing gastric ulcer. To help prevent reflux o f the
Y-90 (poison) into places you d o n ’t w ant (basically anyw here th a t’s not liver)
prophylactic em bolization o f the right gastric and the G D A is perform ed. T he right
gastric origin is highly variable, and can com e o ff the proper hepatic or the left
hepatic.

Trivia Review:

• Shunt Fraction > 30 G y to the Lungs = N o Y-90

• B efore you give the poison, em bo the right gastric (w hich has a variable take off) and
G D A - so you d o n ’t put a hole in the stom ach.

LHA

If you inject the Y-90 here, you can


imagine it refluxing into the right
gastric or GDA

410
Generalized Tum or T reatm ent Trivia (Regardless of the Organ)

RFA

• Tum ors need to be less than 4 cm or you c a n ’t “ cure” them . You can • C ure = < 4 cm
still do RFA on tum ors bigger than 4 cm but the buzzw ord you w ant • D ebulk = > 4 cm
for this is “debulking".

• You alw ays need a bum m argin o f 0.5-1.0 cm around the tumor. So your target is the tum or
+ another 1 cm o f healthy organ.

• A key structure (som ething you d o n ’t w ant to bum up) that is w ithin 1 cm o f the lesion is
considered by m ost to be a contraindication to RFA. Som e people w o n ’t cook lesions near
the vascular h ilu m , or near the gallbladder. Be on the look out for bowel. It is possible to
cook bow el adjacent to a superficial lesion. If they are asking you if a lesion is appropriate
for RFA and it’s superficial look for adjacent bow el - that is probably the trick.

• RFA requires the application o f a “ G rounding pad” on the p a tie n t’s leg. B lankets should be
jam m ed betw een the arm s/body and betw een legs to prevent closed circuit arcs/bum s.

• “H ot W ithdraw al’’ supposedly can reduce the risk o f tu m o r seeding. B asically you leave
the cooker on as you rem ove the probe to burn the tract.

• “H eat S in k ” - this is a phenom enon described exclusively w ith RFA. Lesions that are near
blood vessels 3m m or larger m ay be difficult to treat (w ithout getting fancy) because the
m oving blood rem oves heat aw ay from the lesion.

• You can overcook the turkey. Tem peratures at 100 C or greater tend to carbonize the tissue
near the probe, reducing electrical conductance (resulting in suboptim al treatm ent). A round
60 C is the usual target.

A A A A
• “P ost A b la tio n S y n d r o m e ” - Just like a tum or em bolization you can get a low grade fever
and body aches. The larger the tum or, the m ore likely the syndrom e (just like
em bolization).

• Low G rade Fever and B ody A ches Post A blation. N ext Step = Supportive C are

• Persistent Fever x 2-3 w eeks post ablation. N ext Step = Infection w orkup.

411
M icrow ave

Sim ilar to RFA is that it cooks tum ors. The testable differences are that it can generate m ore
power, can cook a bigger lesion, requires less ablation tim e, it’s less susceptible to heat sink
effect, and it does N O T require a ground pad.

C ryoab lation

Instead o f burning the tum ors, this technique uses extrem e cold in cycles w ith thaw ing. The
freeze-thaw cycles fuck the cells up pretty good. The cold gun is generated by the
com pressing argon gas. I actually knew a guy w ho constructed a sim ilar device shortly after
an industrial accident left him unable to survive outside o f subzero environm ents.

Trivia = T haw ing is w hat actually kills the cancer cells

Trivia = I f you are planning on treating im m ediately after biopsy, m ost sources w ill advise
you to place the probes first, then biopsy, then treat. If you try and place the probes you
m ake a bloody m ess then you m ight not get accurate probe placem ent. Just d o n ’t biopsy the
probe. Seriously, if you crack the probe and the high pressure gas leaks out - shit is gonna
explode (better have your m edical student ready as a shield).

Trivia: It hurts less than RFA - so patients need less sedation

Trivia: The risk o f bleeding is higher than w ith RFA - because you a re n ’t ablating the sm all
vessels

412
Treatm ent Response

RFA T reatm ent R esponse:

Size:

• W eek 1-4: It’s ok for the lesion to get bigger. This is a reactive change related to edem a,
tissue evolution, e tc ...

• M onth 3: The lesion should be the sam e size (or sm aller) than the pre-treatm ent study.

• M onth 6 : The lesion should be sm aller than pre-treatm ent.

Contrast Enhancement:

• C entral or Peripheral E nhancem ent is N E V E R norm al in the lesion post treatm ent.

• You can have “benign peri-ablational enhancem ent” - around the periphery o f the ablation
zone. This should be sm ooth, uniform , and concentric. It should N O T be scattered,
nodular, or eccentric (those are all w ords that m ean residual tum or).

Time Interval

• M ultiphase C T (or M R) at 1 m onth. If residual disease is present at this tim e, N ext Step =
Repeat treatm ent (assum ing no contraindications)

• A dditional follow up is typically at 3-6 m onths intervals.

TA CE T reatm ent R esp on se

On follow up CT, you need to have pre and post contrast im aging including w ashout. The
iodized oil is going to be dense on the pre-contrast. The m ore dense oil is in the tum or the
better outcom e is likely to be. The necrotic tissue should not enhance. If there is enhancem ent
and/or w ashout in or around the tum or, then you have viable tum or that needs additional
treatm ent. Beam hardening from the iodized oil can cause a problem .

“Zone o f A blation " is the preferred nom enclature for the post-ablation region on im aging.

A lso, D ude, “C hinam an” is not the preferred nom enclature. A sian-A m erican

413
C ryoab lation T reatm en t R esp onse

Post therapy study is typically perform ed at 3 m onths, w ith additional follow ups at 6 m onths
and 12 m onths.

A good result should be low er in density relative to the adjacent kidney. On M R, a good result
is typically T2 dark and T1 iso or hyper.

Size: Just like RFA, ablated lesions can initially appear that they grew in size relative to the
pre-treatm ent study. W ith tim e they should progressively shrink (usually faster than w ith RFA).
An increase in size (after the baseline post treatm ent) should be considered recurrent tumor.

Enhancement: A ny nodular enhancem ent ( >10H U change from pre-contrast run) after
treatm ent should be considered cancer.

Vocab

“ResiduaI tum or ” or “Incom plete Treatm ent” = Vocab w ords used w hen you see focal
enhancem ent in the tum or ablation zone o f a patient for their first post therapy study.

“Recurrent tum or ” = W ord used w hen you see focal enhancem ent in the tum or ablation zone
that is new from the first post therapy study.

414
a , S E C T IO N 11: @ EIEJH EEIM E
L u m in a l G l

G Tubes:

A “G- Tube” is a gastric tube, placed directly into the stomach. They arc prim arily used as an
attempt to prolong the suffering o f stroked out A lzheim er Patients with stage 4 sacral decubitus
ulcers.

Traditional m ethod (Radiographically Inserted Gastrostomy - RIG): The basic idea is that you
put an NG tube down and pump air into the stomach until it smushes flat against the anterior
abdominal wall. Then you spear it and secure it with 4 “T-Tacks” to tack the stomach to the
abdominal wall in the gastric body. Then spear it again, wire in and dilate up to the size you
want. Typically, the T-Tacks are rem oved in 3-6 weeks. O ther things that you can do is give a
cup o f barium the night before to outline the colon.

If the patient has ascites. Next Step = Drain that first.

The Ideal Target:

Left o f M idline (lateral to


the rectus m uscle to a v o id
inferior epigastric)

M id to D istal Body

Equal distance from the


greater and less curves - to
a v o id arteries

Anatomy Trivia: The cardia o f the stomach is actually the most posterior portion.

There is another method often called a "PIG” because o f the Perioral route. In that version you
stab the stomach and tread a wire up the esophagus. Then you grab the wire, slip the tube over
it, and advance the tube over the wire into the stomach all the way out the stabbed hole.
Then it’s back to the nursing home for Grandma.

Honestly, it’s probably best to do it with a scope, but since we arc Radiologists my official
statement is that only a Radiologist can do this procedure well.

H ow long does G ranny n eed to wait before she can have her ensure via the G-Tube? D epends
on w ho you ask. Som e people w ill say 12-24 hours fasting post placem ent. O ther people will
say use it right away. It depends on the brand and practitioners bias. To know the correct
answ er for the exam - sim ply read the m ind o f the person w ho w rote the question.

415
E s o p h a g e a l S te n ts

Probably the m ost com m on indication for one o f these is esophageal cancer palliation. These
are usually placed by G l, but that d o e sn ’t m ean you w o n ’t get asked about them .

In the real w orld, m ost people d o n ’t even size these things. The overw helm ing m ajority o f
lesions can be covered by one stent. H aving said that, for the purpose o f m ultiple choice you
need a stent w ith a length at least 2 cm longer than the lesion on each side. You do the
procedure through the m outh. I im agine it w ould be great fun to try and place a stent through
the nose - if you really hated the person. You give them som e oral contrast to outline the
lesion. An am platz w ire is dropped dow n into the stom ach. The stent (usually s e lf expanding)
is deployed over the wire.

Post Angioplasty? M ost people d o n ’t angioplasty after deploym ent o f the stent. H ow ever, if
the tum or is bulky and near the carina, som e sources will suggest doing a pre-stent
angioplasty test up to 20 m m to see if this invokes coughing / stridor. The concern is that a
large tum or m ay get displaced against the carina and cause a respiratory em ergency. If the
patient d o e sn ’t cough from the test you are safe to deploy the stent (probably).

Upper 1/3 Cancer: M ost esophageal cancers are in the low er 1/3. If the question specifically
tells you it’s higher up (or show s you), they m ay be leading you tow ards a “t/o« ’/ cover the
larynx dum bass! ” question. T he w ay to avoid this is to have endoscopy do the case so they
can identify the cords. If that isn ’t an option then placing a sm aller device m ight be an
alternative.

Stent Drops into the Stomach: M ost people will ju st leave the m otherfucker alone.
How ever, if the patient is sym ptom atic, endoscopic rem oval is the textbook answer.

Stent Occludes. Next Step = E sophagram . The m ost com m on cause is food im paction - w hich
som etim es can be cleared w ith a soda. If that fails, the next step is endoscopy. If it’s not food
but instead tum or overgrow th, som etim es you can place a second stent. It depends on a lot o f
factors and asking that w ould be horse shit.

Gam esmanship:

• A cute obstruction is likely food

• W orsening sym ptom s over tim e is likely tumor.

416
Gl B leed

You can split GI bleeds into two categories upper (proxim al to ligam ent o f Treitz) and lower.

Upper GI:

Som e testable trivia is that 85% o f upper GI bleeds are from the left gastric, and often i f a
source cannot be identified, the left gastric is taken down prophylactically. If the source o f
bleeding is from a duodenal ulcer, embolization o f the GDA is often perform ed. A bout 10% o f
the tim e, an upper GI bleed can have bright red blood per rectum.

“Pseudo-Vein” Sign - This is a sign o f active GI bleeding, with the appearance o f a vein
created by contrast pooling in a gastric rugae or m ucosal intestinal fold. If you aren ’t sure if
it’s an actual vein, the “pseudo-vein” will persist beyond the venous phase o f injection.

Dieulafoy’s Lesion - This is a m onster artery in the subm ucosa o f the stom ach w hich pulsates
until it causes a teeny tiny tear (not a prim ary ulcer). These tears can bleed like stink. It’s
typically found in the lesser curvature. It’s not exactly an AVM, m ore like angiodysplasia.
Som etim es you can treat it w ith clips via endoscopy. Som etim es it needs endovascular
em bolization.

When I say pancreatic arcade bleeding aneurysm, you say celiac artery stenosis.

There is a know n association w ith celiac artery com pression (m edian arcuate
ligam ent) and the dilation o f pancreatic duodenal arcades w ith pseudoaneurysm
form ation.

A R e tro g ra d e fillin g o f th e
h e p a tic a rte ry s h o u ld
m a k e yo u th in k a b o u t
C e lia c s te n o s is (or
G am esm anship: It is o c c lu s io n )
classically show n w ith
an angiographic run
through the SM A ,
show ing a dilated
collateral system and
retrograde filling o f the In je c tio n in S M A , fills
a d ila te d p a n c re a tic
hepatic artery.
d u o d e n a l c o lla te ra l
s y s te m

417
Upper GI B leed “N ext S te p ” Algorithm

Treat with Failed or Not


Positive Possible
E ndoscopy

U pper GI Bleed E ndoscopy A ngio

Three Hopefully
Negative Phase CTA Targeted, Non-
Target if Needed

Low er GI:
The w ork-up for low er GI bleeds is different than upper GI bleeds. With the usual caveat that
algorithm s vary w ildly from center to center, this is a general way to try and answ er next step
type questions regarding the workup.

Stable “C on servative
E ndoscopy ^
'"c ^ M anagem ent 55
L ow er GI Bleed
'V Three
Unstable Phase CTA
*Tachycardia + nr / °fy.
Hypotension iin /^ p V
RBC Scan \ %
v A ngio

ACR Appropriateness specifically states that in a STABLE patient with low er GI bleeding that
endoscopy is first line.

ACR Appropriate: High Yield Trivia is that


Intermittent / Obscure GI Bleeding
nuclear scintigrap hy
— GI Bleeding that continues (or recurs) despite negative
upper endoscopy and colonoscopy is described as “obscure GI (R B C b leed ing scan) is
bleeding. "T he actual culprit is often from the small bowel m ore sensitive than
(arteriovenous malformation).
angiography.
— There is no clear consensus on the optimal study to
interrogate the small bowel. Bleed Scan = 0.1 m L/m in
CTA = 0.4 m l/m in
— ACR Appropriateness Criteria rank CT angiography and
capsule endoscopy as the most appropriate choices in this A ngiography = 1.0 m L /m in
situation. Tc-99m RBC scan is considered as a “reasonable
alternative” for localization - but only in the setting o f active
bleeding. Remember GI bleed scan only works if there is
active bleeding.

418
Causes with Angiographic Buzzwords:

£ 6 •A n giodysp lasia - Right Sided Finding. “Early Draining Vein. ” Em bolization o f


angiodysplasia rarely stops a re-bleed and these often need surgery.

m • D iverticulosis - Left Sided Finding (usually). M ore com m only venous. I f arterial,
“fillin g the diverticulum fir s t ” is classic.

c-y-) *M eckles - U sually show n on M eckles scan (99mTc-N a-pertechnetate). The feeding
d artery (vitelline) has a classic look with “extension beyond the m esenteric border, ”
“no side branches ” and “a corkscrew app ea ra n ce” o f the term inal portion.

Technical A spects / Trivia:

You will w ant runs o f all 3 vessels (SM A, Celiac, IM A). Som e old school guys will say to start
with the IMA because contrast in the bladder will obscure that territory as the procedure
continues. T hat’s not really an issue anym ore w ith m odem DSA and starting with the SM A
will typically be the highest yield. You have to sub select each vessel. Runs in the aorta are
not good enough and that w ould never be the right answer.

What i f you don't see bleeding? You can try “provocative angiography” -w hich is not nearly
as interesting as it sounds. This basically involves squirting som e vasodilator (nitro 100-200
m eg) or throm bolytic drug (tPA 4 m g) into the suspected artery to see if you can m ake it bleed
for you.

What i f you do see it bleeding? A dm inister som e street justice. A nyone who trained in the last
30 years is going to prefer m icrocoils and PVA particles. O ld guys m ight use gel-foam .
Alcohol should not be used for low er GI bleeds (causes bowel necrosis).

M icrocoils: Good because you can see them . Good because you can place them precisely.
Bad because they deploy right w here you drop them. So you need to go right up next to that
bleed to avoid a large bow el infarct.

Trivia = Inability to advance the m icro-catheter peripherally is the m ost com m on cause o f
m icrocoil em bo failure

I say “non-selective em bolization o f bowel with m icrocoils,” you say “bowel infarct”

PVA: Good because they are “ flow directed.” So you d o n ’t need to be as peripheral
com pared to the m icrocoils. Bad because you have less control.

Trivia: Particles m ust be 300-500 m icrons. Particles that are sm aller w ill/could cause
bow el infarct.

419
But Prometheus my Geriatric Attending says to use Vasopressin?
Betw een me and you this argum ent was settled in 1986 by a lady
nam ed Gom es. Her study show ed coils stopped GI bleeds 86% o f the
tim e, com pared to 52% for vasopressin and the shit we have today is
w ay better m aking the disparity even greater. H aving said that, some
Dinosaurs still do it.

For the purpose o f m ultiple choice, this is w hat I w ould know:

• Vasopressin works as a vasoconstrictor

• Vasopressin does not require superselection. You can squirt it right into the m ain trunk o f
the artery.

• Vasopressin sucks because the re-bleed rates are high (once the drug w ears off)

• Vasopressin can actually cause non-occlusive m esenteric ischem ia (N O M I)

• Vasopressin should N O T be used with large artery bleeding (i.e. splenic pseudoaneurysm ),
bleeding at sites w ith dual blood supply (classic exam ple is pyloroduodenal bleed), severe
coronary artery disease, severe hypertension, dysrhythm ias, and after an em bolotherapy
treatm ent (risk o f bow el infarct).

Post Embolization: You need to do angiography post


em bolization to look for collateral flow (if there is a dual
supply). The classic exam ple is: after perform ing an
em bolization o f the GDA (for duodenal ulcer), you need to
do a run o f the SM A to look at the inferior
pancreaticoduodenal (collateral to the GDA). You m ight
have to take that one out too, but obviously that w ould
increase the risk o f bowel infarct.

Trivia - Risk o f bowel infarct is w ay low er for upper GI


bleeds (because o f the extensive collateral supply), relative
to bleeds distal to the ligam ent o f Trietz.

the SMA

420
t — k
S E C T I O N 12:
A b s c e s s D r a in a g e *
!

G en eral Tactics:

In general, there are tw o m ethods, you can use a trocar or you can use the seldinger technique
(w ire guided).

* Trocar: You nail it w ith a spinal needle first. Then adjacent to the needle (in tandem )
you place a catheter.
* Seldinger: O ne stick w ith a needle, then w ire in, dilate up and place a catheter.

Drain Size: T he grosser and thicker stu ff will need a b igger tube. If forced I’d go with:

• 6-8 F for clear fluid

• 8-10 F for thin pus

• 10-12 F for thick pus

• 12F+ for collections w ith debris or in collections that sm ell like a Z om bie farted.

Drain Type: You pretty m uch alw ays use a pigtail. I w o u ld n ’t guess anything else.

Trivia / G am esm an sh ip

• Any “next step” question that offers to turn d o ppler on p rio r to sticking it w ith a needle is
alw ays the right answer. T rying to trick you into core needling a pseudoaneurysm is the
oldest trick in the book.

• D ecom pressing the urinary bladder prior to a pelvic abscess drainage is often a good idea.

• C ollection has pus. N ext step = aspirate all o f it (as m uch as possible) prior to leaving the
drain

• You c a n ’t advance into the cavity because it’s too fibrous/thick w alled. Next Step ? I ’d try
a hydrophilic coated

• Fam ily m edicine w ant you to put a 3 w ay on that 12 F drain. N ext step = d o n ’t do that.
You are reducing the functional lum en to 6F.

421
Trivia / G am esm an sh ip Continued

• Fam ily m edicine w ants you to hold o ff on antibiotics till after you drain this unstable septic
shock patien t's abscess. N ext step = d o n ’t do that. A ntim icrobial therapy should never be
w ithheld because som e knuckle head is w orried about sterilizing cultures. (1) C ultures
alm ost never change m anagem ent from the coverage they w ere on anyw ay, (2) the traum a
o f doing the drainage will seed the bloodstream with bacteria and m ake the sepsis w orse.

• Fam ily m edicine w ants to know how m any cc to flush this com plex (but sm all) abscess
w ith? R em em ber that “flushing" and “irrig a tio n ” are different. Flushing is done to keep
the tube from clogging w ith viscous poop. Irrigation is w hen you are w ashing out the
cavity (the solution to pollution is dilution) for com plete cavity drainage. G oing nuts with
the irrigation can actually cause a bacterem ia. The vignette could say som ething like
“ w axing and w aning fever corresponding to flush schedule.” The next step w ould be to
train the nurses / fam ily m edicine to lim it the volum e to less than the size o f the cav ity .

• You irrigate the abscess with 20 cc o f fluid but w hen you aspirate back you only get 5ccs.
N ext Step? Stop irrigating it! You have a big problem . The fluid (w hich is dirty) is being
w ashed into a location that is not able to be sucked back out by the tube. So you are
creating a new pocket o f infection that isn't being drained.

• C atheter started out draining but now is stopped. N ext Step = (1) confirm that it is in the
correct location and not kinked - m ight need im aging if not obvious at bedside, then (2) try
flushing it or clearing an obstruction w ith a guidew ire. If the catheter is clogged for real
then y o u ’ll need to exchange it - probably for a larger size. If the tract is m ature (older than
a w eek) you can probably get a hydrophilic guidew ire through the tract into the collection
to do an easy exchange.

• R em ove the catheter w hen: (1) drainage is less than lOcc / day, (2) the collection is
resolved by im aging (CT, U ltrasound, e tc ...), and (3) there is no fistula.

• Persistent Fever > 48 H ours post drainage. The patient should get better pretty quickly
after you drain the abscess. If they a re n ’t getting better it im plies one o f tw o things (1) you
did a shitty jo b draining it, or (2) they have another abscess som ew here else. E ither w ay
they need m ore im aging and probably another drain.

• The drainage am ount spikes. T his is a bad sign. In a norm al situation the drainage should
slow er taper to nothing and then once you confirm the abscess has resolved you pull the
drain. Spikes in volum e (especially on m ultiple choice exam s) suggest the form ation o f a
fistula. N ext step is going to be m ore im aging, possibly w ith fluoro to dem onstrate the
fistula (urine, bow el, pancreatic duct, bile duct, e tc ...).

422
Pelvic A b scess D rain age - tubo-ovarian abscess, diverticular abscess, or peri-appendiceal

G eneral Ideas for C h oosing the C orrect Route:

(1) All things equal, pick the shortest route

(2) Avoid bow el, solid organs, blood vessels (inferior epigastrics are classic) , nerves

(3) Try not to contam inate sterile areas

(4) C hoose the m ost dependent position possible (usually p o ste rio r or lateral) to facilitate
drainage

R outes

M ost abscesses in the pelvis are layering in a dependent position so anterior routes are
typically not easy. In general there are 4 routes; transabdom inal, transgluteal, transvaginal,
and transrectal. I ’m gonna try and cover the pros/cons and testable trivia for each route.

T ransabdom inal - The pull o f gravity tends to cause infection to layer in the m ore posterior
spaces. A s a result transabdom inal approaches tend to be long, and therefore violate one o f
the 4 general ideas. If you are show n an abscess w here this w ould be the best, shortest route
then rem em ber to w atch out for the inferior epigastrics. For sure there will be an option to
stick the trocar right through one o f them . M ake sure you ID them before you choose your
answer.

Transgluteal - The transgluteal


approach is done for a variety o f
posterior targets. The patient is
positioned prone for targeting.

Avoid the sciatic nerve and gluteal


arteries by:

• A ccess through the sacrospinous


ligam ent

• M edial as possible

• Inferior to the piriform is

D isadvantages: L egit risk o f artery/


nerve injury. Prone to catheter
kinking. G otta use CT (radiation).

423
E ndolum inal Routes: There is a subset o f perverts who prefer to biopsy and drain things through
the vagina (tuna purse) and/or the rectum .... Not that there’s anything wrong with that. Well
actually the prim ary disadvantage o f both o f these “endoluminal routes” is catheter stability.
M any catheters arc literally pooped out within 3-4 days. Although advocates for these routes will
argue that (a) they are more fun to do, and (b) most collections resolve within 3 days.

Transvaginal: Biopsy and/or drainage through the vagina (pink taco) has the advantage o f
providing a very short safe route that can be guided by transvaginal ultrasound, allowing for no
radiation and very accurate placement. This was the classic in office route for drainage o f
infected gynecologic fluid collections (P1D related). The procedure is done in the lithotomy
position. Catheter size is traditionally limited to 12F (or smaller). You should never do this to a
patient under the age o f 14 - not even Jared from Subway would try that.

Although controversial, it is possible (and well described in the literature) to drain / biopsy
adnexa cysts through the vagina (penis fly trap).

Vaginal prep / cleansing prior to the procedure is controversial and unlikely to be tested.

Transrectal: O f the three routes (gluteal, vaginal, rectal) transrectal is supposedly the least
painful - although in my literature review the psychological pain was not discussed (this kinda
thing would really fuck with my m achismo). Essentially this route offers all the advantages of
the transvaginal route (ultrasound guidance, very short / safe route) plus the added advantage of
pre-sacral access. Depending on what you read, people will argue this is first line (over trans-
gluteal) for pre-sacral collection but that is highly variable.

Choosing between transgluteal and trans-rectal for a pre-sacral collection would be the worst
“read my m ind” question ever. If forced into that scenario I would set aside the psychologic
trauma to the alpha male ego and use (1) the size o f the collection - do you think that will drain
before he/she poops the catheter out ?, and (2) is the transgluteal route safe - are the vessels
nerves obviously in the w ay?

Prep with a cleansing enema is not controversial and is endorsed pretty much everywhere.

P re -sa c ra l
R ectou terin e collection P re -sa c ra l
collection
R e c to v e sic a l
p o u ch
collection

R e ctu m
The B a b y R e ctu m (prison wallet)
Cannon (prison wallet)
not drawn to sc a le

424
D iverticu lar A b scess

There are a few pearls / special considerations that we should discuss regarding the
diverticular abscess.

Size: The typical threshold for a diverticular abscess to be drained is 2 cm. A nything sm aller
than that will be m ore trouble than it is worth.

Tube Choice: Rem em ber the grosser and thicker stu ff will need a bigger tube. D iverticular
abscesses form because o f a perforated diverticulum . Thus, you can com e to the logical
conclusion that you need a tube capable o f draining shit. For the purpose o f m ultiple choice,
anything sm aller than 10F is probable N O T the right answer.

Gas: If the abscess is gas producing (they w ould have to tell you the bulb suction fdls rapidly
with gas), the correct next step is to treat the collection like a pleural drain in a patient with an
air leak (i.e. put on w ater seal).

L iver A b scess

Lots o f etiologies for these, but d o n ’t forget to think about the appendix or diverticulitis. The
draining o f these things is som ew hat controversial with som e authors feeling the risk o f
peritoneal spread out w eighs the benefits and reserving the drainage for p atien t’s w ith a poor
prognosis. O ther authors say that everyone and their brother should get one, and consider it
first line treatm ent.

A pearl to draining these things is to not cross the pleura (you’ll give the dude an em pyem a).
If there is a biliary fistula, prolonged drainage will usually fix it (biliary drainage or surgery is
rarely needed).

Trivia: Biopsy / A spiration o f Echinococcal cysts can cause anaphylaxis. Surgical rem oval o f
the presum ed echinococcal cysts should be discussed with surgery before attem pting the
procedure in IR (you w ant to be able to blam e it on them , if shit goes bad).

R enal A b scess

Renal abscess is usually secondary to ascending infection or hem atogenous spread. The term
“perinephric a b sc e ss” is used when they perforate into the retroperitoneal space. W hen they
are small (< 3-5 cm) they will resolve on their ow n w ith the help o f IV antibiotics.

Indications for aspiration or drainage include a large (> 3-5 cm ), sym ptom atic focal fluid
collection that does not respond to antibiotic therapy alone.

The strategy is to use ultrasound and stick a pig tail catheter in the thing. A fter a few days if
the thing is not com pletely drained you can address that by upsizing the tube. If you create or
notice a urine leak, y o u ’ll need to place a PCN. There are really only relative contraindication
- bleeding risk e tc ..., and the procedure is generally well tolerated with a low com plication
rate.

425
P erirenal L ym phocele

This is seen in the setting o f a transplant. W hen they are sm all you typically ju st w atch them.
However, on occasion they get big enough to cause local m ass effect on the ureter leading to
hydronephrosis. You can totally aspirate them , but they tend to recur and repeated aspiration
runs the risk o f infecting the collection. For m ultiple choice 1 w ould say do this: A spirate the
fluid and check the creatinine. If it’s the sam e as serum it’s probably a lym phocele ( if it s
more then i t ’s a urinoma). Either way you are going to drain them with a catheter. However,
if it’s a lym phocele you m ight sclerose the cavity (alcohol, doxy, povidine-iodine).

*Urinomas (that are persistent) o f any size are drained.

P ancreas D rain age

R em em ber that necrotizing pancreatitis is bad, but infected necrotizing pancreatitis is a


death sentence. So, be careful draining som ething that is N O T infected already (otherw ise
you m ight m ake it infected). If you a re n ’t sure if it’s infected, consider aspirating som e for
culture (but not placing a tube).

Indications: G eneral indications include infected collections or collections causing m ass


effect (bow el or biliary obstruction).

Progression to surgery: I f you can get 75% reduction in 10 days, the drain is good enough.
If not, the surgeons can use the tract for a video-assisted retroperitoneal debridem ent (w hich
still avoids open debridem ent).

P ancreatic C u taneous F istu la : O ther than pancreatic pseudocysts, m ost pancreatic


collections are either brow n or grayish. W hen the fluid is clear, you should think about
pancreatic fluid, and send a sam ple for am ylase to confirm . I f this lasts m ore than 30 days
then you have y o u rse lf a “p ersisten t pan creatic fistula. ” N ice jo b idiot... you could have ju st
left it alone. T hat will teach you to let those m edicine docs pressure you into doing stu ff
th a t’s not indicated. It m ay be possible to treat that w ith octreotide (synthetic som atostatin)
to inhibit pancreatic fluid, although in these cases extended drainage is usually needed.

You’ve Been Talked into Draining the Pseudocyst — Which Route:

General Rule: If the pseudocyst communicates with the pancreatic duct drainage will be prolonged
(6-8 weeks in most cases). You can try and use somatostatin to slow it down.

Most Cases: Transperitoneal with CT guidance — avoid organs, avoid going through the stomach
twice.

Can’t Avoid the Stomach or Patient has a known Duct Communication (so they gonna have a tube
for a long time) - Transgastric Approach — so it drains into the stomach

426
U r i n a r y C| N T E R V E N T I O N

Percutaneous Nephrostomy (PCN) -

There are 3 m ain reasons you m ight subject som eone to this:

Stones
R elief o f U rinary
O bstruction C ancer

Urine Leak

U rinary
Urine Fistula (for p elvic CA
D iversion
o r inflam m atory process)

Severe Refractory’
H em orrhagic C ystitis

W hitaker Test (ifit's 1970)


A ccess for
A ccess for Stone R em oval
D iagnostic and
(PCNL)
T herapeutic
Procedures D ilation o r Stenting o f
Stricture

PCN C on traindications (A bsolute):


IN R S h o u ld b e le s s than 1.5
Severe C oagulopathy ►
P LT > 50K

T echnically N ot P o s s i b l e ► • A p p ro a ch w ould c ro s s colon,


sp le e n , o r liver

427
Technical Stuff:
• Prior to the procedure, it would be ideal if you normalized the potassium (dialysis). Certainly
anything about 7 should be corrected prior to the procedure.

• Hold anti-platelet drugs for at least 5 days prior to the procedure.

• The lower pole o f a posteriorly oriented calyx is ideal.


The reason you use a posterior lateral (30 degrees) D e 9 re e s off sagittal
, . . , , . f (tow ards the b ack)
approach is to attack along B ro d els Avascular Zone
(area between the arterial bifurcation).

• Skin entry site should be 10 cm lateral to the midline (not beyond the posterior axillary line).
You don’t want to go too medial unless you want to try and dilate through the paraspinal
muscles. You don’t want to go too lateral or you risk nailing the colon.

• Choosing a lower target m inimizes the chance o f pneumothorax. Additional benefit o f the
posterior calyces approach is that the guidewire takes a less angled approach (com pared to an
anterior calyces approach).

• Direct stick into the collecting system without passing through renal parenchyma is NOT a
good move (high risk o f urine leak).

• Dilated System = Single Stick: Ultrasound and stick your ideal target (low and posterior), then
use fluoro to wire in, dilate up, and then place the tube. Alternatively you can do the whole
thing under CT.

• Non-Dilated System = Get your partner to do it (these blow). If forced to do = Double Stick.
Ultrasound and stick anything you can. Opacify the system. Then stick a second time under
fluoro in an ideal position (low and posterior), then wire in, dilate up, and then place the tube.
Alternatively you can do the whole thing under CT.

• The posterior calyces (your target) will be seen “end on’’ if you use contrast. The anterior ones
should be m ore lateral. If you use air, you should ju st fill the posterior ones (which will be
non-dependent with the patient on their belly. Air is useful to confirm.

You place the drain and get frank pus back. Next Step = Aspirate the system

Abou.IOOTLa.ara/toMidW

E leva te the
targeted sid e

428
Special Situations:
Nephrostomy on Transplant - The test w riter w ill likely w rite the question in a w ay to
m ake you think it’s crazy to try one o f these. T ransplant is N O T a contraindication. In fact
it’s technically easier than a posterior / native kidney.

Testable Transplant Trivia:

• A nterolateral C alyx Should be Targeted (in stead o f p o sterio r)

• Entry site should be LA TERA L to the transplant to avoid entering the peritoneum

• M iddle to U pper Pole (in stead o f a low er p o le)

Percutaneous Nephrostolithotomy - This is done to rem ove stones in conjunction w ith


urology. The idea is very sim ilar w ith a few differences. T he m ost testable difference is
that the site is often the upper pole (instead o f low er pole) to m ake ston e access easier.
The tube / hole is bigger and there is m ore risk o f bleeding.

“ Tube Fell Out ” - The trick to handling these scenarios is the “ freshness” o f the tube. If the
tract is “fresh, ” w hich usually m eans less than 1 w eek old, then you have to start all over
w ith a fresh stick. If the tract is “m ature,” w hich usually m eans older than 1 w eek, you can
try and re-access it with a non-traum atic wire.

Catheter Maintenance: E xchange is required every 2-3 m onths because o f the


crystallization o f urine in the tube. Som e hospitals / departm ents will do exchanges m ore
frequently than 2 m onths and that is because o f how w ell this p a y s ... uh 1 m ean they do it
for excellent patient care.

“ Encrusted Tube ” - If this thing gets totally gross it can be very difficult to exchange in the
norm al fashion. T he m ost likely “next step” is to use a hydrophilic w ire along the side o f
the tube (sam e tract) to m aintain access.

Ureteral Occlusion - Som etim es urology w ill request that you ju st kill the ureters all
together. This m ight be done for fistula, urine leak, or intractable hem orrhagic cystitis.
There are a bunch o f w ays to do it. The m ost com m on is probably a sandw ich strategy
w ith coils. T he sandw ich is m ade by placing large coils in the proxim al and distal ends o f
the “ nest”, and sm all coils in the m iddle. Big C oils = Bread, Sm all C oils = Bacon.

429
N e p h ro u re te ra l S te n t (N U S )

This is used w hen the patient needs long-term drainage. It's w ay better than having a bag o f
piss strapped to your back.

Benign ureter strictures

M alignant ureteral obstruction (by f a r


LONG TERM
the m ost common indication)
> R elief o f U r in a r y ------- ► NUS
O bstruction
U reteric injury

U reteric calculus undergoing lithotripsy.

Technically they can be placed in a retrograde (bladder up) or an antegrade (kidney dow n)
fashion. You are going to use the antegrade strategy if (a) y o u ’ve got a nephrostom y tube,
or (b) retrograde failed.

Can you go straight from N ephrostom y to N U S ? Yes, as long as you d id n ’t fuck them up
too bad getting access. If they are bleeding everyw here or they are uroseptic you should
wait. Let them cool dow n, then bring them back to covert to the NU S.

Who should N O T g et a N U S ? A nyone w ho d o e sn ’t have a bladder that w orks (outlet


obstruction, neurogenic bladder, bladder tum ors, etc..). It m akes no sense to divert the urine
into a bladder that c a n ’t empty.

v\
\
\

Obstruction

V-....
Bag o

PCN NUS - with External and Internal Drainage. Double J


Allows urine to pass from the kidney to bladder The Ultimate Goal
through the obstruction (for those that can get one)

430
In te rn a l N U S - D o ub le J: T his is the ultim ate goal for the patient. T he testable
stipulation is that this will require the ability to do retrograde exchanges (via the bladder).

“The S afety ” - A safety PCN - is often left in place after the deploym ent o f a double J PCN.
The point is to m ake sure the stent is going to work.

The typical protocol:

1. Place the double J and the safety

2. Cap the safety - so that the internal N U S is draining the patient

3. Bring the patient back in 24-48 hour and “ squirt the tube” (antegrade nephrostogram ).
The system should be non-obstructed.

4. If it’s w orking you pull the safety.

5. If it’s N O T w orking you uncap the safety and ju st leave it as a PCN.

S u p ra p u b ic C y s to s to m y -

Done to either (a) acutely decom press the bladder or (b) decom press long-tenn outflow
obstruction (neurogenic bladder, obstructing prostate cancer, urethral destruction, etc..)

The best way to do it is with ultrasound in the fluoro suite. The target is m idline ju st above the
pubic sym physis at the junction o f the m id and low er thirds o f the anterior bladder wall. You
chose this target because:

• The low stick avoids bowel and the peritoneal cavity

• The low 1/3 and mid 1/3 junction avoids the trigone (which will cause spasm).

• The vertical m idline is chosen to avoid the inferior epigastric.

Use ultrasound and stick it, confirm position w ith contrast, w ire in and then dilate up. Use a
small tube for tem porary stuff and a larger tube for m ore long-term stuff. You can alw ays
upsize to a foley once the tract is m ature. A 16F foley is ideal for long-term drainage.

Contraindications:

• Buncha Pelvic Surgeries - Extensive scar


• Being a Big Fat Pig/C ow
• C oagulopathy
• Inability to distend bladder
• Inability to displace overlying sm all bow el

431
W eapo n o f C h o ic e

Let’s do a rapid review o f urinary diversion options.

Obstruction Obstruction

Obstruction

Obstruction

Scenario 1 Scenario 2. Scenario 3.


Dude with a ureteral stricture Dude with a ureteral TCC Dude with an outflow
PLUS a Bladder Mass obstruction (horrible
prostate cancer)

If you can cross the lesion, This guy has obstruction at This guy has obstruction
and the bladder works then the ureter and the bladder. at the bladder. He
internal Double J NUS is idea The only option is to divert at doesn’t need to cross
the level of the kidney. He the ureter. He gets a
gets a PCN. Cystostomy

432
R enal B iop sy - This can be done for tw o prim ary reasons:

(1) renal failure or (2) cancer biopsy.

Non-Focal: The renal failure w orkup “non-focal bio p sy ” is typically done w ith a 14 - 18
gauge cutting needle , w ith the patient either prone or on their side (target kidney up). The
m ost obvious testable fact is that you w ant tissue from cortex (low er pole if possible) to
m axim ize the yield o f glom eruli on the specim en and m inim ize com plications by avoiding
the renal sinus. The com plication rate is relatively low, although sm all AV fistulas and
pseudo-aneurysm s are relatively com m on (m ost spontaneously resolve). Som e hem aturia is
expected. In a high risk for bleeding situation a transjugular approach can be done but that
requires know ing w hat you are doing.

Focal: It used to be thought that focal biopsy should N E V E R be done because o f the
dreaded risk o f upstaging the lesion and seeding the track. T his has been show n to be very
rare (<0.01% ). H aving said that I think it’s still the teaching at least in the setting o f
pediatric renal m asses. This procedure is probably better done w ith CT. T he patient is
placed in w hatever position is best, but the lateral decubitus w ith the lesion side dow n is
“preferred” , as it stabilizes the kidney from respiratory m otion, and bow el interposition.
Just like w ith ultrasound, not crossing the renal sinus is the w ay to go. Just put the needle in
the tumor. If it’s cystic and solid m ake sure you hit the solid part. Som e texts recom m end
both fine needle and core biopsy. The core biopsy is going to give a higher yield. A testable
pearl is that if lym phom a is thought likely, a dedicated aspirate should be sent for flow
cytom etry. A s w ith any renal procedure hem aturia is expected (not gross - ju st a little).
Renal colic from blood clots is rare.

ACR Appropriate/SIR Practice Guidelines: Renal Biopsy

— Renal Bx is a procedure w ith “ significant bleeding risk, difficult to detect or control.”

— SIR guidelines recom m end holding aspirin for 5 days prior to the procedure.

— Why 5 D ays ? A spirin irreversibly inhibits platelet function and since platelet lifespan is
about 8-10 days, patients w ith norm al m arrow w ill replenish 30-50% o f their platelets
w ithin 5 days o f w ithholding the w illow bark (aspirin).

433
R enal RFA: Radiofrequency ablation (RFA) is an alternative to partial nephrectomy and
laparoscopic nephrectomy. It can be used for benign tumors like AMLs, renal AVMs, and even for
RCCs. Angiomyolipomas (AMLs) are treated at 4cm because o f the bleeding risk. Sort o f a general
rule is that things that are superficial you can bum with RFA. Things that are closer to the collecting
system it may be better to freeze (cryoablation) to avoid scaring the collecting system and making a
stricture. Pyeloperfusion techniques (cold D5W irrigating the ureter) can be done to protect it if you
really wanted to RFA. If anyone would ask, RFA has no effect on GFR (it won’t lower the GFR).

Things that make you think recurrent/residual disease after therapy.


( 1) Any increase in the size beyond the acute initial increase,
(2) Areas o f “nodular” or “crescentic” enhancement, or
(3) A new or enlarging bright T2 signal.

There is a paper in AJR (2009) that says that lesions that are < 3cm will appear larger in 1-2 months
and lesions >3cm do not grow larger - when successfully treated. So, smaller lesions may initially
get bigger but after that - any increase in size should be considered tumor recurrence.

R enal A rteriograp h y: You should always do a non-


selective aortogram first to sec how many arteries feed the
kidney, where they are , etc. Sometimes the aortogram will show
you an obvious ostial problem which you can then select down
on and address. Otherwise, you need to do selective angiography
and look at each vessel. LAO is the projection of choice for
L A O M inim izes A n giogra ph ic
looking at the renals. Sometimes the stenosis is further out, in
O verla p from the Aorta
fact branch artery stenosis is a cause of hypertension in kids.

Angioplasty o f Renal Arteries: Used to treat hypertension caused by atherosclerosis (usually ostial)
or FMD. Risks include thrombosis, and vessel spasm. Calcium channel blockers can be given to
decrease the risk o f spasm. Fleparin should be on board to reduce thrombosis risk. Most people take
daily aspirin the day before and every day after for 6 months, to reduce the risk of restenosis.

• Indications fo r angioplasty = Renal Vascular HTN or Azotemia


• Atherosclerosis at the Ostium = Angioplasty + Stent
• FMD - usually mid vessel = Angioplasty’ Alone

But Prometheus!?! - 1 was reading the New England Journal...

Don't read the NEJM. The NEJM is run by a bunch o f family medicine doctors who hate all
procedures. They published a thing called the CORAL trial in 2014, that showed no added
benefit from angio + stenting in the setting o f renal vascular stenosis compared to high quality
medical therapy.

This remains controversial and several prominent 1R guys still like to stent, especially if they can
measure a pressure gradient in the renal artery. For the purpose o f multiple choice, if “high
quality medical therapy” is a choice for treated RAS related hypertension, that is probably the
right answer — otherwise, pick angio + stent.

434
Renal H em orrhage:

Traum a to the kidney (usually iatrogenic from biopsy or diversion procedure) can typically be
em bolized. The renal arteries are “ end arteries,” w hich m eans that collaterals are not an
issue. It also m eans that infarction is a legit issue so if you w ant to salvage the kidney you
need to try and get super selective. H aving said that, d o n ’t be an idiot and fuck around trying
to get super selective w hile the patient is bleeding to death. R em em ber m ost people have two
o f these things, plus in a w orst case scenario there is alw ays dialysis. Bottom line: if you get
into trouble and the patient is crashing, ju st trash the w hole thing.

Next S tep : A rterial traum a from the nephrostom y tube placem ent. B leeding source is occult
on angio. N ext step ? R em over the nephrostom y tube (over a guidew ire), then look again.
O ften the catheter tam ponades the bleed, m aking it tougher to see.

Gam esmanship: Oral boards guys used to be sticklers for the phrase “over a wire. ” In other
w ords if you ju st said “ I ’d rem ove the PC N ” they w ould ding you. You have to say “ I ’d
rem ove the PCN over a wire." The only reason I bring this up is the use o f possible
distractors / fuckery.

M aybe som ething like this:

Q: The highly skilled Interventionalist grants the Fellow the great privilege o f perform ing a
fresh stick nephrostom y. The clum sy, good for nothing Fellow m anages to place the tube, but
now there is a large volum e o f bright red blood in the tube and the P atien t’s blood pressure is
dropping rapidly. You start fluids and perform an em ergent renal arteriography. T he source o f
bleeding is not seen. What is the best next step?

A: R em ove the PCN and repeat another angio run


B: Kick the fellow in the shin for using too m uch fluoro tim e J u st like a M id g et using
C: Call U rology and adm it you need help from a “ real doctor” a u rin a l... y o u g o tta
sta y on y o u r toes.
D: R em ove the PC N over a wire and repeat another angio run.
Read all the choices!
Renal A neurysm s

“Look, man. 1 only n eed to know one thing: where they are " - Private Vasquez

• Sm all Segm ental A rteries = coils

• Main R enal Artery’ = C overed Stent to exclude the aneurysm . A lternatively, you could place
a bare m etal stent across the aneurysm and then pum p detachable coils into the sac.

435
SECTION 14:
P u l m o n a r y In t e r v e n t io n

P le u ra l D ra in a g e - M ost everyone has done a


few thoracenteses as a resident. I ju st w ant to touch on a
few testable points.

• R em em ber that you go "above the rib ” to avoid the


neurovascular bundle.

• If you pull o ff too m uch fluid too fast you can possibly
get pulm onary edem a from re-expansion (this is
uncom m on).

• If it’s m alignant you m ight end up w ith a trapped lung (lung w o n ’t expand fully)- in other
w ords a thick pleural rind or fibrothorax, can prevent lung reexpansion - m akes
percutaneous drainage pointless in m any cases. A “ vacu-thorax” - in the setting o f a
trapped lung, does not m ean anything, and does not need im m ediate treatm ent even if it’s
big. If you really need to fix it, y o u ’ll need a surgical pleurectom y / decortication.
Pleurodesis (w hich can be done to patients w ith recurrent pleural effusions), does N O T
help in the setting o f trapped lung.

• Pneum othorax is rare but is probably the m ost com m on com plication (obviously it’s m ore
com m on w hen done blind).

A dditional Trivia related to Chest Tubes:

• C ontinuous air bubbles in the Pleur-evac cham ber represent an air leak, either from the
drainage tubing or from the lung itself. In the setting o f m ultiple choice - think about a
bronchopleural fistula.

• INR should generally be < 1.5 prior to placem ent o f a chest tube.

• In the paravertebral region, the intercostal vessels tend to course o ff o f the ribs and are
therefore m ore prone to injury if this route is chosen for chest tube placem ent

C hoice o f D rainage C atheter


P a ra p n e u m o n ic E ffu sio n / E m p y e m a M a lig n a n t E ffu sio n

In p a tie n t O u tp a tie n t In p a tie n t O u tp a tie n t

1 5.5 In d w e llin g
1 2 -1 4 Fr 10 Fr 14 F
(P leurX e tc ...)

436
Lung A b scess: Just rem em ber that you can drain an em pyem a (pus in the pleural
space), but you should N O T drain a lung abscess because you can create a bronchopleural
fistula (som e people still do it).

Lung RFA - R adiofrequency ablation o f lung tum ors can be perform ed on lesions
betw een 1.5cm and 5.2cm in diam eter. The m ost com m on com plication is pneum othorax
(m ore rare things like pneum onia, pseudoaneurysm , bronchopleural fistula, and nerve
injury have been reported). The effectiveness o f RFA is sim ilar to external beam radiation
w ith regard to prim ary lung cancer. The m ajor advantage o f lung RFA is that it has a
lim ited effect on pulm onary function, and can be perform ed w ithout concern to prior
therapy.

Im aging (CT and PET) should be perform ed as a follow up o f therapy. T hings that m ake
you think residual /recurrent disease: nodular peripheral enhancem ent m easuring m ore than
10 m m , central enhancem ent (any is b a d ) , grow th o f the RFA zone after 3 m onths (after 6
m onths is considered definite), increased m etabolic activity after 2 m onths, residual activity
centrally (at the burned tum or).

Lung B iopsy - The m ost com m on Reducing the Risk of Pneum othorax
com plication is pneum othorax, w hich occurs - Post Biopsy

about 25% o f the tim e (m ost either resolve Enter the lung at 90 degrees to pleural
spontaneously or can be aspirated), w ith surface
about 5% needing a chest tube. The second Avoid interlobar fissures
m ost com m on com plication (usually self- Put the patient puncture side DOW N after
the procedure
lim iting) is hem optysis.
No talking or deep breathing after the
procedure (at least 2 hours)
The testable p e a rls include:
If the patient is a cougher, consider
• The low er lung zones are m ore affected by postponing the procedure - or giving
empiric anti-tussive meds
respiratory m otion,

• The lingula is the m ost affected by cardiac


Nonspecific Thoracic Core Biopsy
m otion, Results - Next Step:

• Avoid vessels greater than 5 m m, Repeat the biopsy and / or close follow up.
Nonspecific biopsy results don’t m ean shit
• Try and avoid crossing a fissure (they — especially in the lung.
alm ost alw ays get a pneum othorax),
Biopsy is only helpful when you get an
• A reas lateral to and ju st distal to the tip o f a actual result (cancer, ham artom a, etc...).
Otherw ise - you could have ju st missed, or
biopsy gun will be affected by “ shock w ave
targeted the infection behind the cancer.
injury” , so realize vessels can still bleed
from that.

437
C h es ts T u b e I P ig ta il P la c e m e n t:

Potential algorithm to deal with Procedural Pearls


pneumothorax post biopsy cases: You can usually get away with a small-caliber,
(6-10 French) catheter. A 10 French Pigtail
Catheters would require an 18G needle / 0.035
Amplatz wire. You would need a larger tube if
there is fluid (otherwise it will get clogged). You
should use CT guidance since you obviously
have it available. Most people will tell you to
use the so called “ triangle o f safety,” located
above the 5th intercostal space, mid-axillary line.
This has the thinnest muscle, lets you avoid the
breast in females (and fat sloppy dudes) - plus
keeps you free o f the axillary vasculature,
diaphragm, liver, and spleen.

Always go along the superior aspect o f the rib to


avoid the neurovascular bundle along the inferior
border o f each rib.
Heimlich valves will let the patient remain
ambulatory, otherwise you can use a
conventional water seal device.
In most cases, the tube can be removed 1-2 days
Probable should place a tube after the procedure.

Obstruction Air Leak Subcutaneous


Emphysema
Detected by noticing (being told) Detected by persistent bubbling
that the water-seal chamber isn’t within the water seal chamber. Typically detected by
fluctuating with respiration or crepitus on physical exam, or
Air leak = Air within the pleural
coughing while the drainage system shown on chest x-ray.
space. This is expected after initial
is set to gravity.
insertion o f a chest tube, (with an Confirm the tube is in the
This means either (1) the lung is actively resolving pneumothorax). pleural space. Specially,
fully expanded or (2) the tube is It becomes a problem when it is make sure the side holes are
clogged — CXR will tell you the new or persistent. ALL within the pleural space.
difference. Look for those fucking side
Next Step: CXR confirm position
It is controversial to “milk” the o f the tube. Inspect the bandage - holes. Reposition if needed.
tubing - plenty o f people still do it. usually a vaseline bandage covers If the tube is appropriately
Some people put TPA in the tube - the insertion site. If everything positioned, subcutaneous
people do lots o f crazy shit beyond looks ok - you might be dealing emphysema is self-limited -
the scope o f the exam .... probably. with a bronchopulmonary fistula. do nothing.

438
Thoracic Angio
This section is going to focus on the two m ain flavors o f pulm onary angio; pulm onary artery’
(done for m assive PE and pulm onary AVM treatm ent) and bronchial artery (done for
hem optysis).

P u lm on ary A rtery

The prim ary indications for pulm onary arteriography is diagnosis and treatm ent o f m assive PE
or pulm onary AVM.

Technical Trivia:

The “G rollm an” catheter, which is a preshaped 7F, is the classic tool. You get it in the right
ventricle (usually from the fem oral vein) and then turn it 180 degrees so the pigtail is pointing
up, then advance it into the outflow tract. Som e people will say that a known LBBB is high
risk, and these patients should get prophylactic pacing (because the w ire can give you a
RBBB, and RBBB + LBBB = asystole). An im portant thing to know is that patients with
chronic PE often have pulm onary hypertension. Severe pulm onary hypertension needs to be
evaluated before you inject a bunch o f contrast. Pressures should alw ays be m easured
before injecting contrast because you m ay w ant to reduce your contrast burden. Oh, one last
thing about a n g io ... never ever let som eone talk you into injecting contrast through a swan-
ganz catheter. It’s a TER R IB LE idea and the stupid catheter will blow apart at the hub. I
would never ever do th at....

Next Step: Cardiac dysrhythm ias (v-tach) during


procedure. Next Step ? R e-position the catheter / P ulm onary A n giograp h y
wire Relative Contraindications

Pulm onary E m bolism - Patients with PE should Pulmonary HTN with elevated right
be treated with m edical therapy (anticoagulation
heart pressures (greater than 70
systolic and 20 end diastolic).
with Coum adin, Heparin, or various new er agents),
allow ing the emboli to spontaneously undergo If you need to proceed anyway - they
lysis. In patients who c a n ’t get anticoagulation (for get low osmolar contrast agents
w hatever reason), an IVC fdter should be placed. injected in the right or left PA (NOT the
The use o f transcatheter therapy is typically main PA).
reserved for unstable patients with m assive P E .
Left Bundle Branch Block - The
M assive PE? Just think lotta PE with hypotension. catheter in the right heart can cause a
right block, leading to a total block.
In those situations, catheter directed throm bolysis,
throm boaspiration, m echanical clot fragm entation, If you need to proceed anyway - they
and stent placem ent have all been used to address
get prophylactic pacing.
large clots.

4 39
Pulm onary AVM - T hey can occur sporadically. For the purpose o f m ultiple choice w hen
you see them think about H H T (H ereditary H em orrhagic T elangiectasia / O sier W eber
Rendu). Pulm onary AVM s are m ost com m only found in the low er lobes (m ore blood flow )
and can be a source o f right to left shunt (w orry about stroke and brain abscess). The rule
o f treating once the afferent (feeding) artery is 3m m is based on som e tiny little abstract
and not pow ered at all. H aving said that, it’s quoted all the tim e and a frequent source o f
trivia that is easily tested. The prim ary technical goal is to crush the feeding artery (usually
with coils) as close to the sac as possible. You d o n ’t w ant that think reperfusing from
adjacent branches. Pleurisy (se lf lim ited) after treatm ent seem s to pretty m uch alw ays
happen.

K ey Trivia:

• HHT A ssociation

• Brain A bscess / Stroke - via paradoxical em boli

• Treat once the afferent (feeding) artery is 3m m

• C oils in the feeding vessel, as close as possible to the sac

Special Situation - R asm ussen A neurysm

This is an aneurysm associated w ith chronic pulm onary infection, classically TB. The trick
on this is the history o f hem optysis (w hich norm ally m akes you think bronchial artery).

“ It’s a T rap!” - A dm iral G ial A ckbar

Next Step Strategy to a v o id the trap:

Patient blah blah blah hem o p ty sis Next Step? B ronchial A rtery A ngio

• B ronchial A rtery A ngio is negative, still bleeding. O h, and his PPD is positive. Next
Step ? Pulm onary A rtery angio to look for R asm ussen A neurysm s

• R asm ussen A neurysm identified. Next Step ? Coil em bolization (yes coils for hem optysis
- this is the exception to the rule).

440
B ro n ch ia l A rte r y

The prim ary indication for pulm onary arteriography is diagnosis and treatm ent o f m assive
hem optysis.

H em optysis - M assive hem optysis (> 300 cc) can equal death. B ronchial artery em bolization
is first line treatm ent (bronchial artery is the culprit 90% o f the tim e). U nique to the lung,
active extravasation is N O T typically seen w ith the active bleed. Instead you see tortuous,
enlarged bronchial arteries. The m ain thing to w orry about is cord infarct. For m ultiple
choice the m ost likely bad actor is the “h airp in -sh a p ed ” anterior m edullary artery
(A dam kiew icz). E m bolizing that thing or anyw here that can reflux into that thing is an
obvious contraindication. I f present, those bad boys typically arise from the right intercostal
bronchial trunk.

P articles (> 325 m icrom eters) are used (coils should be avoided - because if it re-bleeds you
ju st jaile d y o u rse lf out).

“Hairpin Turn”o\
The vast majority (90%) of th e A n te rio r
bronchial arteries are located M e d u lla ry A rte ry
within the lucency formed by
the left main bronchus. This is
right around the T5-T6 Level

There is a ton of vascular


variation but the pattern of an
intercostobronchial trunk on the
right and two bronchial arteries
on the left is most common
(about 40%)

In the low er thoracic / upper lum bar region the prim ary feeding artery o f the anterior spinal
cord is the legendary anterior radiculom edullary artery (artery o f A dam kiew icz). This
vessel m ost com m only originates from a left sided posterior intercostal artery (typically
betw een T 9 -T 1 2 ), w hich branches from the aorta. T he distal portion o f this artery, as it
m erges w ith the anterior spinal artery, creates the classic (and testable) "hairpin" turn.

It is w orth noting that A dam kiew icz can originate from the right bronchus (like 5% ).

441
O cclu sio n o f C e n tra l V e in s (SVC S y n d ro m e ) -

Yes - 1 know this really isn ’t a pulm onary thing, but it is in the chest so I ’m going to talk
about it here.

A cute vs C hronic SV C O cclusion:

• A cute = N o C ollaterals
• A cute = E m ergency

• C hronic = Has C ollaterals


• C hronic = N ot an Em ergency

There are a variety o f w ays to address occlusion o f the SV C. The goal is to return in line
flow from at least one ju g u la r vein dow n through the SV C. M ost com m only throm bolysis is
the initial step, although this is rarely definitive. The offending agent (often a catheter)
should be rem oved if possible. If the process is non-m alignant, often angioplasty alone is
enough to get the jo b done (post lysis).

Technical Trivia:

• M alignant causes: you should do lysis, then angioplasty, then stent.

• N on-m alignant causes: m ay still need a stent if the angioplasty d o e sn ’t rem ove the
gradient (if the collateral veins are still present).

• Self-expanding stents should N O T be used, as they tend to m igrate.

• The last pearl on this one is not to forget that the pericardium extends to the bottom part o f
the SVC and that if you tear that you are going to end up w ith hem opericardium and
possible tam ponade.

442
a SECTION 15:
R e p r o d u c t iv e In t e r v e n t io n

U te rin e A r te r y E m b o liza tio n (U A E):

Can be used for bleeding or the bulk symptoms o f fibroids. Procedure may or may not help with
infertility associated with fibroid. If you are paying cash.... it definitely helps.

Patient Selection (not all fibroids were created equal). To do this you need a pre-op MRI/MRA to
characterize the fibroids and look at the vasculature.

Subtypes:
• Degenerated leiomyoma are more likely to have a poor response
(these are the ones that don’t enhance).
• “Cellular” Fibroids - the ones with high T2 signal tend to respond well to embolization.
Most fibroids “Flyaline Subtype" are T2 dark.
• Smaller lesions do better than larger lesions.

Location:
• Submucosal does the best. Intramural does the second best.
• Serosal does the third best (it sucks). It speaks the third most Italian -
• Cervical fibroids do NOT respond well to UAE —they have a different blood supply.

• Intracavitary Fibroids - Less than 3 cm.


O Next Step = GYN referral for hysteroscopic resection
• Intracavitary Fibroids - Less than 3 cm , with failed hysteroscopic resection
O Next Step = IR Embo
• Large Serosal Fibroid, patient wants to be pregnant, no history o f prior myomectomy
O Next Step = GYN referral for myomectomy
• Pedunculated Serosal Fibroid
O Next Step = GYN referral for resection
• Broad Ligament Fibroid
O Next Step = Refer to voodoo priest (these don’t do well with UAE and are
technically challenging to operate on).

Pedunculated Subserosal
Subserosal

Intracavitary
Pedunculated
Submucosal

Intra-
Ligamentary

Submucosal

Intramural
Cervical

443
PreTreatment Considerations / Trivia

• Remember fibroids are hormone responsive. They grow with estrogen (and really grow during
pregnancy). Gonadotropin-releasing medications are often prescribed to control fibroids by
blocking all that fancy hormone axis stuff.
• The testable trivia is to delay embolization for 3 months if someone is on the drugs because
they actually shrink the uterine arteries which makes them a pain in the ass to catheterize.
• The EMMY trial showed that hospital stays with UAE are shorter than hysterectomy
• The incidence of premature menopause is around 5%
• D V T / PE is a known risk of the procedure (once pelvic vein compression from large fibroid
releases - sometimes the big PE flies up). The risk is about 5%.

Contraindications: Pregnancy. Uterine/Cervical Cancer. Active Pelvic Infection, Prior Pelvic


Radiation, Connective Tissue Disease, Prior Surgery with Adhesions (relative)

Treatment Trivia
• Occlusion of small feeding arteries cause fibroid infarction (and hopefully shrinkage). Embolic
material is typically PVA or embospheres for fibroids (targeting the pre-capillary level). If ask to
choose an agent - I'd say "particles" - don't pick coils, or glue. For postpartum hemorrhage /
vaginal bleeding, gel foam or glue is typically used.
• Most people will say either 500-700 micro or 700-900 micron particle sizes. As a point of trivia
smaller particle size docs not give you a better result for fibroids —but can help with
Adenomyosis.
• Treatment of adenomyosis with UAE is done exactly the same way, and is an effective treatment
for symptomatic relief (although symptoms recur in about 50% o f the cases around 2 years post
treatment). As above - slightly smaller particles are typically used for this (vs fibroids).
• Fibroids should reduce volume 40-60% after the procedure. If you are treating intracavitary
fibroids they should turn to mush and come out like a super gross chunky vegetable soup period
mix. You actually want that - if they stay (“retained”) inside they can get infected.

Anatomy Trivia:
• Remember the uterine artery is off the anterior division o f the internal iliac
• Regardless of the fibroid location, bilateral uterine artery embolization is necessary to prevent
recruitment of new vessels
• In most cases, branches of the ovarian artery feed the fibroids via collaterals with the main
uterine artery. Uterine artery can be identified by the characteristic "corkscrew" appearance of
its more disc branches —named the Helicine branches (twisty like a helix)

444
Uterine Artery Embolization Continued:
Post Embolization Syndrome: I mentioned this earlier but just wanted to remind you that it’s
classically described with fibroid embolization. Remember you don’t need to order blood cultures -
without other factors to make you consider infection. The low-grade fever should go away after 3
days. Some texts suggest prophylactic use o f anti-pyrexial and antiemctic meds prior to the
procedure.
• 3 Days or less with low grade fever = Do nothing
• More than 3 Days with fever = “Work it up” , cultures, antibiotics, etc...

H y s te r o s a lp in g o g r a m (H S G ):

I'm 100% certain no one went into radiology to do these things. You do it like a GYN exam. Prep
the personal area with betadine, drape the patient, put the speculum in and find the cervix. There
are various methods and tools for cannulating and maintaining cannulation o f the cervix (vacuum
cups, tenaculums, balloons). Insertion o f any of these devices is made easier with a catheter and
wire. Once the cervix and endometrial cavity have been accessed, the contrast is inserted and
pictures are obtained.
Contraindications: Pregnancy, Active Pelvic Infection. Recent Uterine or Tubal Pregnancy.

Trivia:
• The ideal time for the procedure is the proliferative phase (day 7-14), as this is the time the
endometrium is thinnest (improves visualization, minimizes pregnancy risk).
• It’s not uncommon for a previously closed tube to be open on repeat exam (sedative, narcotics,
tubal spasm - can make a false positive).
• Air bubbles can cause a false positive filling defect.
• Intravasation - The backflow o f injected contrast into the venous or lymphatic system, used to
be an issue during the Jurassic period (when oil based contrast could cause a fat embolus). Now
it means nothing other than you may be injecting too hard, or the intrauterine pressure is
increased because of obstruction.
• The reported risk o f peritonitis is 1%.

Fallopian Tube Recanalization (FTR):

Tubal factors (usually P1D / Chlamydia) are responsible for about 30% o f the cases in female
infertility ~ depending on what part of the country you are from sometimes much more (insert joke
about your hometown here). Tubal obstruction comes in two flavors; proximal / interstitial, or
distal. The distal ones get treated with surgery. The proximal ones can be treated with an
endoscope or by poking it with a wire under fluoro.

Things to know:
• You should schedule it in the follicular/proliferative phase (just like a HSP) - day 6-12ish.
• You repeat the HSG first to confirm the tube is still clogged. If clogged you try and unclog it
with a wire ( “selective salpingography ”).
• Hydrophilic 0.035 or 0.018 guidewire (plus / minus microcatheter) is the typical poking tool
• Repeat the HSG when you are done to prove you did something
• Contraindications are the same as HSG (active infection and pregnancy)

445
P e lv ic C o n g e stio n S y n d ro m e

- W omen have m ystery pelvic pain. This is a real (m aybe) cause o f it. They blam e dilated
ovarian and periuterine veins in this case, and give it a nam e ending in the word “ syndrom e” to
m ake it sound legit. The sym ptom s o f this “ syndrom e” include pelvic pain, dyspareunia,
m enstrual abnorm alities, vulvar varices, and low er extrem ity varicose veins. The sym ptom s
are most severe at the end o f the day, and with standing.

D iagnosis ? Clinical sym ptom s + a gonadal vein diam eter o f 10 m m (norm al is 5 mm).

Treatment ? GnRH agonists som etim es help these patients, since estrogen is a vasodilator. But
the best results for treatm ent o f this “ syndrom e” are sclerosing the parauterine venous plexus,
and coils/plugs in the ovarian and internal iliac veins (perform ed by your local Interventional
Radiologist). This is often staged, starting with ovarian veins plugged first, and then (if
unsuccessful) iliac veins plugged second.

Trivia: M ost optim al results occur when the entire length o f both gonadal veins are em bolized.

Complications ? Com plications are rare but the one you w orry about is throm bosis o f the
parent vein (iliac or renal), and possible throm bus m igration (pulm onary em bolism ).

Will it g et better on its own ? The sym ptom s will classically im prove after m enopause.

V a ric o c e le

- They are usually left-sided (90% ), or bilateral (10% ). Isolated right-sided varicoceles should
prom pt an evaluation for cancer (next step = CT Abd).

When do you treat them ? There are three indications: (1) infertility, (2) testicular atrophy in a
kid, (3) pain.

Anatomy Trivia (regarding varicoceles): R em em ber that m ultiple venous collaterals


“pam piniform plexus” or “sperm atic venous plexus” drain the testicles. Those things com e
together around the level o f the fem oral head, form ing the internal sperm atic vein. The left
internal sperm atic vein drains into the left renal vein, and the right internal sperm atic vein
drains into the IVC. C om m on variants include: m ultiple veins on the right term inating into the
IVC or renal vein, or one right-sided vein draining into the renal vein (instead o f the IVC).

Why Varicoceles Happen'. The “prim ary factor” is right angle entry o f the left sperm atic vein
into the high pressure left renal vein. N ut-cracker syndrom e (com pression o f the left renal vein
betw een the SM A and aorta) on the left is another cause (probably m ore likely asked).

Basic Idea: You get into the renal vein and look for reflux into the gonadal vein (internal
sperm atic) which is abnorm al but confirm s the problem . You then get deep into the gonadal
vein, and em bolize close to the varicocele (often with foam ), then drop coils on the way back,
and often an A m platzer or other occlusion device at the origin.

446
a S E C T IO N 16:
M IS C T O P IC S **

V e rte b ro p la s ty

There is a paper in the N EJM that says this doesn’t work. Having said that, NEJM doesn’t like
any procedures. T hey’re run by fam ily m edicine doctors. They are equally am oral to the
person that will do any non-indicated procedure. R egardless o f the actual legitim acy, it’s a big
cash cow and several prom inent Radiologists have m ade their nam es on it... so it will be
tested on as if it’s totally legit and w ithout controversy.

Trivia to Know:

• Indications = Acute to subacute fracture with pain refractory to m edical therapy or an


unstable fracture with associated risk if further collapse occurs.
• Contraindications = Fractures with associated spinal canal com pression or im proving
pain w ithout augm entation.
• There is a risk o f developing a new vertebral fracture in about 25% o f cases. The
literature says you should “counsel patients on the n eed for additional treatments
p rio r to undergoing vertebroplasty.
• The cem ent can em bolize to the lungs.
• Risk o f local neurologic com plications are about 5%.

L ym p h an g io g ram :

1950 called and they w ant to stage this cancer. Prior to CT, M RI, and US injecting dye into
the toes was actually a w ay to help stage m alignancy (m ets to lymph nodes, lym phom a, etc..).

A nother slightly m ore m odem application is to use this process as the first step in the
em bolization o f the thoracic duct. W hy w ould you take dow n the thoracic duct? If it’s
leaking chylous pleural effusions - status post get hacked to pieces by a good for nothing
Surgery Resident.

Technical Trivia:

This is done by first injecting about 0.5 cc m ethylene blue dye in betw een the toes bilaterally.
You then w ait h alf an hour until the blue lym phatic channels are visualized. You then cut
down over the lym phatic channels and cannulate with a 27 or 30 gauge lym phangiography
needle. An injection with lipiodol is done (m axim um 20 ml if no leak). I f you inject too
much there is a risk o f oil pneum onitis. You take spot film s in a serial fashion until the
cistem a chyli (the sac at the bottom o f the thoracic duct) is opacified. At that point you could
puncture it directly and superselect the thoracic duct to em bolize it, typically w ith coils.

447
S ta n d in g W aves:

Standing w aves are an angiographic phenom enon (usually) that results in a ringed layering
o f contrast that sorta looks like FM D . A com m on trick is to try and m ake you pick betw een
FM D and Standing W aves.

O bviously it’s bullshit because in real life standing w aves typically resolve prior to a
second run through the sam e vessel, and even if they stayed around they tend to shift
position betw een each run (up or dow n). FM D on the other hand is an actual physical
irregularity o f the vessel wall so it’s fixed betw een runs and d o e sn ’t go away.

M orphology sh ould he y o u r strategy’ for m ultiple


choice:

Standing w aves are very sym m etric and evenly


spaced.

FM D is m ore irregular and asym m etric.

Standing Waves FMD

“T he L ingo”

"Give me a 10 x 6 Balloon ” - This m eans a 10 m m diam eter x 6 cm length balloon

"Give me 20 f o r 3 0 ” - This m eans do an angio run at 20 cc/sec for a total o f 30 mL.

"Squirted” - A n A ngiogram — O h really? A splenic lac w ith active extrav? L et’s call IR
right aw ay and get him squirted.

“ Thrash ” - A difficult case

“Hot M ess ” - I have an adm it for you. This lady is a hot m ess.

“That p o o r lady ” - A w ay o f feigning sym pathy.

“Sick as Stink ” - also, “sick A N D stinks” be careful not to m ess this up.

448
A rte ry of In te re s t C A rm A n gu lation M isc

A ortic Arch 70 D egrees LA O “C andy C ane”

Innom inate (R ight Subclavian RA O In the LA O the right


& Right C om m on Carotid) subclavian and right
com m on carotid overlap

Left Subclavian LAO

M esenteric Vessels Lateral to Steep RA O

Left Renal LA O Sam e side as renal

Right Renal RA O or LA O - T his is controversial - a


depending on w ho you lot o f sources w ill say
ask. you can get aw ay w ith
LAO.

Left Iliac B ifurcation RAO O pposite side com m on

Right Iliac Bifurcation LAO O pposite side com m on

Left C om m on Fem oral B ifurcation LA O Ipsilateral O blique

Right C om m on Fem oral B ifurcation RA O Ipsilateral O blique

T h e C o n fu s in g O b liq u e V ie w s

Normally, views are defined by the direction of the x-ray beam.


However, in Angio it gets a little squirrely. The sidedness refers to the side of the 1.1.

RAO: The LAO : The


imaging imaging
intensifier is intensifier is
on the right on the left
side of the side of the
patient. patient.

A reasonable A reasonable
person might person might
call this LPO call this RPO
- but they - but they
would be would be
wrong. wrong.

449
S u p erficial or D eep? - U n d erstan d in g G eom etry

Som etim es it’s difficult to tell i f you are superficial or deep to the lesion you are trying to
put a needle in under fluoro. You can problem solve by tilting the 1.1, tow ards the p a tie n t’s
head or tow ards the p atien t’s feet.

If you tilt tow ards the head, a superficial needle will be shorter but a deep needle will look
longer.

If you tilt tow ards the feet, a superficial needle w ill be longer but a deep needle will look
shorter.

Arrow = Needle
Image Receptor

tilted towards
1.1, tilted towards
patients feet:
patients head:
• Superficial Needle
• Superficial Needle
looks longer
looks shorter
• Deep Needle looks
• Deep Needle looks
shorter
longer

► A 4Tube
7 - . .....

A ir Em bolus

C lassic C linical B uzzw ords: “ Sudden onset shortness o f breath’’ “ W hoosh sound” or
“ Sucking sound” during central catheter insertion.

N ext Step: “Durant's m an eu ver” = left-lateral decubitus + head-dow n positioning. O ther


verbiage = “right side up” or “ left side dow n”, “trendelenburg”

N ext N ext Step: 100% O xygen

45 0
M e d ic a tio n s :

Anti-Coagulation Issues:

* R em em ber that Platelets R eplace Platelets.


* C ryoprecipitate is used to correct deficiencies o f fibrinogen.
* Heparin: The h a lf life is around 1.5 hours. Protam ine Sulfate can be used as a m ore
rapid H eparin A ntidote.
* Protam ine can cause a sudden fall in BP, B radycardia, and flushing
* C oum adin: V itam in K can be given for C oum adin but that takes a w hile (25-50m g IM
4 hours p r io r to procedure) , m ore rapid reversal is done w ith factors
(cryoprecipitate).
* R em em ber that patients w ith “ H IT ” (H eparin Induced T hrom bocytopenia) are at
increased risk o f clotting - not bleeding. If they need to be anti-coagulated then they
should get a throm bin inhibitor instead (rem em ber those end in “ru d in ” and “gatran”).
* The Life Span o f a Platelet is 8-10 days
* IV D esm opressin can increase factor 8 - m ay be helpful o f hem ophilia

M ed icatio n M ech anism T rivia

Inhibits throm boxane A 2 Irreversible - w orks the life


A spirin from arachidonic acid by an o f the platelet (8-12 days).
irreversible acetylation

Binds antithrom bin 3 - and M onitored by PTT. Can be


H eparin increases its activity. reversed w ith protam ine
sulfate

Inhibits the binding o f A D P


Plavix (C lopidogrel) to its receptors - leads to
inhibition o f G P Ilb/IIIa

Inhibits vitam in K M onitored by INR. D elay


dependent factors (2,7,9,10) in onset o f activity (8-12
hours). A ction can be
C oum adin antagonized by vitam in K -
but this takes tim e (4 hours).
For im m ediate reversal give
factors (cryopercipitate)

A ct directly or indirectly to TPA has a very short


T hrom bolytic A gents
convert plasm inogen to biologic h a lf life - betw een
(TPA)
plasm in (cleaves fibrin) 2-10 m ins.

451
ACR Appropriate: / SIR Practice Guide: Pre-Procedure Hold

— For procedures w ith a M O D ERA TE risk o f bleeding (liver or lung biopsy, abscess
drain placem ent, vertebral augm entation, tunneled central line placem ent)

— 1NR should be corrected to < 1.5 prior to the procedure.


— A spirin need not be held,
— C lopidogrel (plavix) should be held for 5 days.
— Platelet count should be m ore than 50,000.

Sedation Related:

* “C onscious Sedation” is considered “ m oderate sedation” , and the patient should be


able to respond briskly to stim uli (verbal com m ands, or light touch). N o airw ay
intervention should be needed.
* Flum azenil is the antidote for Versed (M idazolam ).
* N arcan is the antidote for O pioids (M orphine, Fentanyl).

Local A nesthesia (L idocaine)

• M axim um D ose is 4-5 m g/kg


• A dirty trick w ould be to say - “ Lido w ith Epi” - in w hich case it is 7 m g/kg
• Som e basic scrub nurse m ath:
• 1% Plain Epi - 10 m g per 1 m L
• So 1 m g per 0.1 mL
• A nd we said M axim um D ose is 5 m g/kg, so it w ould be equal to 0.5 m L / kg
• R em em ber that sm all doses in the right spot can cause a serious reaction.
• 150 m g in the thecal sac can cause total spine anesthesia and the need for a
ventilator.
• D irect arterial injection can cause im m ediate seizures.
• Tinnitus and dizziness are the earliest signs o f toxicity.
• Local anesthesia agents have a low potential for allergy - although it can still occur, it’s
usually a bogus allergy once a real history is taken. M ost “ allergies” to lidocaine are
actually vaso-vagal, or other C V side effects from epinephrine m ixed with lidocaine
• T here are elaborate m echanism s for testing for a true allergy, or reaction to
m ethylparaben (a preservative).
• So w hat if the allergy is real? or you c a n ’t prove it’s false? - Som e texts describe using
an antihistam ine such as diphenydram ine (w hich can have anesthetic properties).

Green, Steven M , Steven G. Rothrock, and Julie Gorchvnski. "Validation o f diphenhydramine as a dermal
local anesthetic. "Annals o f emergency medicine 23.6 (1994): 1284-1289.

452
In d ic a tio n s C o n tra in d ic a tio n s

O nly one absolute w hich is an


unstable patient w ith m ultisystem
dysfunction (unless angio is life
N um erous; usually saving).
A ngiography diagnosis o f and treatm ent
o f vascular disease T here are num erous relatives
including inability to lay flat,
uncooperative patient, and connective
tissue diseases

D iagnosis o f DVT,
Ascending E valuate Venous
V en ography m alform ation or tum or
encasem ent.

E valuation o f post-
C ontrast R eaction
D escending throm botic syndrom e;
Venography v alvular incom petence and
P regnancy
dam age follow ing D V T

T horacic O utlet Severely com prom ised


Venography Syndrom e, Venous cardiopulm onary status
(N on-inclusive) A ccess, Pacer Placem ent,
Eval for fistula

C a n ’t get anticoagulation,
Failed anticoagulation Total throm bosis o f IVC
(clot progression),
M assive PE requiring IVC too big or too small
IVC F ilte r
lysis, C hronic PE treated
w ith *Sepsis is N O T a contraindication,
throm boendarterectom y. including septic throm bophlebitis
T raum a high risk DVT

A bsolute: R ight to left


cardiopulm onary shunt, U ncorrectable
M aking the nephrologist coagulopathy, fistula infection.
Fistulography m oney (“ slow flow s” they
call it). R elative is significant
cardiopulm onary disease (a declot
invariably causes PE)

453
In d ica tio n s C o n tra in d ica tio n s

A bsolute: Heart Failure (especially


Variceal bleeding
right heart failure). Severe
TIPS refractory to endoscopy.
encephalopathy. R apidly progressing
R efractory ascites.
liver failure.

Perform ed prior to A bsolute: U ncorrectable


P ercutaneo us percutaneous biliary C oagulopathy, Plavix or other anti­
T ra n s h e p a tic interventions, platelet agent
C holedochojejunostom y
C holangiography
patients (liver transplant) R elative: Large Volume A scites
(PTC)
w ith suspected (consider para and left sided
obstruction approach)

N o absolute contraindications
B asically C B D
P ercutaneo us obstruction (w ith failed
R elative: Large Volume A scites
B ilia ry D rain age ER C P), cholangitis, bile
(consider para and left sided
duct injury/leak.
approach), C oagulopathy

C holecystitis in patients
w ho are not surgical
candidates, U nexplained
No absolute contraindications
sepsis w hen other
P ercutaneo us sources excluded, A ccess
Relative: Large Volume A scites
C holecystosom y to biliary tree required
(consider para and left sided
and other m ethods failed
approach), C oagulopathy

O bstructive U ropathy
(N ot hydronephrosis),
P ercutaneo us U rinary diversion (leak, U ncorrectable coagulopathy,
N ephrostom y fistula), A ccess for C ontrast Reactions
percutaneous
intervention

454
Blank for Scribbles

455
456
14
frM][TglfTg]fTg]flmlfTM][Tmirrml[TglffMlfTmlfTm]fElfTg]frg]frgl

Mam mo

P r o m e t h e u s L i o n h a r t , M . D .

457
A B rief O verview - For th e com plete novice:

M any foreign graduates have had little or no experience with mammography. I ’ll try and give you a
very brief overview before I get into the testable trivia.

In America, wom en are told they need to have screening mam mograms done once a year. Because
breast cancer is common (and scary), most women will get this done, and will continue to get it
done into their 90s. They will literally take people from the ICU who are in cardiogenic shock
down to x-ray to keep them up to date on their screening m am mograms (usually at the patient’s
request).

I want you to think about mammograms in two categories: (a) the screener - these are the people
who are coming in once a year, and have no symptoms other than being female, and (b) the
diagnostic - these people have a symptom (palpable lump, nipple discharge, breast pain, etc..).
These two are handled totally different.

Nomenclature is rigidly controlled with regard to reporting. “ BI-RADS” is a book o f approved


vocabulary words for describing the findings, and a system for reporting results based on a
percentage o f risk. The basic idea is that you use scary words like “spiculated” to describe things
you think are cancer, and benign words like “fat density” to describe things you think are not cancer.

Here are some basic scenarios - I’ll explain all the stuff in the chapter and we can revisit some more
scenarios at the end o f the chapter. This is just for you to get a feel for how this works if you have
never been around it before. D on’t spaz if you are lost, it will be clear by the end o f the chapter and
then we will do them again with some quizzing.

(1 a) A 50 year old women comes in for a normal annual screening mammogram. She has no
symptoms, and is just following the recommendation. She gets two standard views - a cranial caudal
view “CC” and a medial lateral oblique view “M LO.” The Radiologist is in a hurry because he has
200 o f them to read, so he just calls it normal (Bi-Rads 1). The patient returns to the screening pool
- and will get imaged again in 1 year. She is relieved to know she doesn’t have cancer.

(lb ) The same 50 year old returns 1 month later after she feels a lump. This is no longer a screening
study but instead a “diagnostic” because she has a symptom. She gets another mammogram - this
time with a palpable m arker on the image. She also gets compression spot views (smaller paddle)
over the palpable area. In the area o f the palpable finding, there is a cluster o f calcifications. They
were present in retrospect on the mammogram performed 1 month prior, but are new from the year
before. Because they are calcifications you need magnification views to fully characterize them. On
the mag views you describe them as "coarse heterogeneous." Because they are new from the prior
year they must be biopsied, but you aren’t done yet. Diagnostic mammogram usually includes an
ultrasound - not because o f the calcifications but because there was a palpable finding. Just
remember no FEM ALE with a palpable gets out the door without an ultrasound. You ultrasound the
area and find no mass. The next step is to biopsy the calcs (which are BR-4, for intermediate
suspicion). You recommend stereo to biopsy them (NOT ultrasound) because you couldn’t see
them with ultrasound. The results yield fibrocystic change, which you agree is a possibility given
the BR-4. When you agree that the biopsy results make sense with the imaging / clinical scenario
you use the word “concordant. ” “The results were concordant” said the Resident to his Attending.
You inform the patient o f the results. She is relieved to know she doesn’t have cancer.

458
(2) A 60 year old wom en com es in for a normal annual screening m am m ogram . She has no
symptoms, and is just following the recom m endation. She gets two standard views - a cranial
caudal view “CC ” and a medial lateral oblique view “M LO .” The Radiologist is in a hurry
because he has 200 o f them to read, but still m anages to see a new mass in the m edial breast on
the CC view. The breasts are pretty dense and it’s hard to find it in on the M LO. So the
Radiologist calls it an “asym m etry” and BR-0 (BR-0 because more w ork up is needed). The
patient returns in a week for a diagnostic study - she is convinced that she’s gonna die o f breast
cancer. You can hear her crying in the waiting room. She gets a diagnostic study with spot
compression over the m ass on the CC view, as well as ML and M LO views. You do m anage to
see the mass on the CC and in the superior breast on the ML view. The mass is well
circumscribed, round, and equal density. Because she is getting a diagnostic w ork up, she gets an
ultrasound too. Before you go in the ultrasound room you guess about where the m ass is going to
be. Since it’s medial and superior on the right breast you guess 2 o ’clock. You locate it quickly,
and find it to be an anechoic cyst - com m on in w om en going into m enopause. You tell her it’s
consistent with a benign cyst. She cries with jo y and thanks you repeatedly for “saving her life.”
She sends you cookies and brownies every year at Christm as with very long letters about how you
saved her. She uses a lot o f religious references that m ake you uncomfortable. Som e years you
throw the food away without eating it, other years you give it to the techs. The simple cyst was
reported as a BR-2 (essentially 0% chance o f cancer).

(3) A 45 year old wom an com es in for a norm al annual screening m am m ogram . She has no
symptom s and is just following the recom m endation. She gets the two standard views - “C C ” and
“M LO.” The Radiologist is in a hurry because he has 200 o f them to read, but still m anages to see
a new m ass in both the M LO and CC views. Because this is a screener he still has to give it a
BR-0 (you never BR-4 or BR-5 o ff a screener). She returns for additional imaging as a diagnostic
patient with spot com pression views. A spiculated, irregular, high density m ass is seen. She is
placed in an ultrasound room for further characterization. The ultrasound shows an irregular,
circumscribed, anti-parallel, hypoechoic m ass, with posterior shadowing and an echogenic halo
(all the scary bad words). You are certain the m ass is a cancer based on these features so you scan
the rest o f her breast looking for other tumors, and you look in her axilla for abnormal nodes. You
find several enlarged lymph nodes in her axilla. You recom m end ultrasound guided biopsy o f
both the m ass and the m ost suspicious lymph node. You give her a BR-5 (>95% chance o f
cancer). The biopsy results com e back as fibrocystic change. This result is not “concordant” with
the findings you m ade on m am m ography and ultrasound so you instead use the word
“discordant” and recom m end surgical excision. “The results were discordant” said the Resident
to his Attending. Gross path shows a high grade invasive lesion.

459
SECTION 1: Jfc

t A n a to m y b e m e im e h e

N ipple: The nipple is a circular sm ooth m uscle that overlies the 4 lh intercostal space. There
are typically 5-10 ductal openings. Inversion is w hen the nipple invaginates into the breast.
Retraction is w hen the nipple is pulled back slightly. They can both be norm al if chronic. If
they are new, it should m ake you think about underlying cancers causing distortion. The
nipple is supposed to be in profile so you d o n ’t call it a m ass. T he areola will darken
norm ally w ith puberty and parity. N ipple enhancem ent on contrast enhanced breast MRI is
n o r m a l, don ’t call it Pagets!

Fibroglandular Tissue: The breast m ound is fibrous


tissue w ith fat, ducts, and glands laying on top o f the
anterior chest w all. The axillary extension is called the
“to// o f Spence.” The upper outer quadrant is m ore
densely populated w ith fibroglandular tissue, w hich is
w hy m ost breast cancers start there. T here is usually no
dense tissue in the m edial/ inferior breast and
retroglandular regions. These are considered “danger
zones” and are often w here the cancer hides.

Danger Zones - where there is


usually no dense fibroglandular
tissue

C o o p er’s L igam ents: T hese are thin sheets o f fascia


that hold the breasts up. They are the tiny w hite lines on m am m ography and the echogenic
lines on US. Straightening and tethering o f the ligam ents m anifests as “architectural
distortion” w hich occurs in the setting o f surgical scars, radial scars, and IDC.

Breast A sym m etry: This is com m on and norm al (usually), as long as there are no other
findings (lum ps, bum ps, skin thickening, etc..). F or m ultiple choice, an asym m etric breast
should make you think about the “shrinking breast ” o f invasive lobular breast cancer. I f the
size difference is new or the parenchym a looks asym m etrically dense, think cancer.

460
Lobules: T he lobules are the flow er shaped m ilk m akers o f the breast. The term inal duct
and lobule are referred to as a “term inal duct lobular unit” o r TD LU . T his is w here m ost
breast cancers start.

Ducts: The ductal system branches like the roots or branches o f a tree. T he branches overlap
w ide areas and are not cleanly segm ented like slices o f pie. T he calcifications that appear to
follow ducts (“ linear or segm ental” ) are the ones w here you should w orry about cancer.

L actiferous Sinus: M ilk from the lobules drains into the m ajor duct under the nipple. The
dilated portion o f the m ajor duct is som etim es called the lactiferous sinus. T his thing is
norm al (not a m ass).

Blood Supply / L ym phatic D rainage: The m ajority (60% ) o f blood flow to the breast is via
the internal m am m ary. T he rest is via the lateral thoracic and intercostal perforators. N early
all (97% ) o f lym ph drains to the axilla. T he rem aining 3% goes to the internal m am m ary
nodes.

461
A xillary N ode Levels: The axilla is sub-divided into three separate levels using the
pectoralis m inor m uscle as a landm ark. Supposedly drainage progresses in a step w ise
fashion - from level 1 -> level 2 -> level 3 and finally into the thorax.

R otter N odes: T hese are the nodes betw een the pec m inor and m ajor. They have a fancy
nam e w hich usually m akes them high yield. H ow ever, R otter w as G erm an and test w riters
tend to prefer French sounding trivia. The only exception to this is N azis. G erm an sounding
m edical vocab w ords nam ed after N azis are fair gam e. To save you the trouble o f looking it
up - R otter died before H itler took pow er so he w a sn ’t a Nazi (probably). Since they
probably a re n ’t gonna ask the vocab w ord, the only other conceivable piece o f trivia I can
im agine being asked w ould be that these are at the sam e level as level 2.

Axillary Lymph Node Levels


Level 1: Lateral to Pec Minor
Level 2: Deep to the Pec Minor
Level 3: Medial and Above Pec Minor
Rotter Node: Between the Pec Minor and Major

M etastasis to the Internal M am m ary N odes: I f you can see them on ultrasound they are
abnorm al. Isolated m ets to these nodes is not a com m on situation (m aybe 3% ). W hen you
do see it happen, it’s from a m edial cancer. M ore com m only, m ets in this location occur after
disease has already spread into the axilla (in other w ords - it’s spreading everyw here).

462
Sternalis M uscle: T his is an A unt M innie. It’s a non-functional m uscle next to the sternum
that can sim ulate a m ass. A bout 5% o f people have one and it’s usually unilateral.

f Bs.
\
t 8C t ™

m
r A
*
4

"M
Sternalis - Only on CC, Never on MLO
' ’

The m ain testable trivia is:

1. WTF is that ?- R ecognize the A unt M innie, and d o n ’t get tricked into doing a biopsy on
it, e tc ...
2. H ow You See It? It is O NLY SE EN O N T H E C C V IE W .

H andling this in real life is all about the old gold. Find that thing on the priors (even better is
a C T ) , CC only, n ever on the M LO .

463
B re a s t D ev e lo p m en t:

The "milk s tr e a k ’’ is the em bryologic buzzw ord to explain the location o f the norm al breast
and location o f ectopic breast tissue. Just know that the m ost com m on location for ectopic
breast tissue is in the axilla (second m ost com m on is the infram am m ary fold). Extra
nipples are m ost com m only in the sam e locations (but can be anyw here along the “m ilk
streak”). A t birth, both m ales and fem ales can have breast enlargem ent and produce m ilk
(m aternal horm ones). As girls enter puberty, their ducts elongate and branch (estrogen
effects), then their lobules proliferate (progesterone effects). I f you biopsy a breast bud
(w hy w ould you do that?) you could dam age it and potentially fuck up breast
developm ent.... and then get sued.

• F ollicular Phase (day 7-14): Estrogen D om inates. Best tim e to have both
m am m ogram and M RI.

• Luteal Phase (day 15-30): Progesterone D om inates. This is w hen you get som e
breast tenderness (m ax at day 28-30). B reast density increases slightly.

• Pregnancy: Tubes and D uct Proliferate. T he breast gets a lot denser (m ore
hypoechoic on US), and ultrasound m ay be your best bet if you have a m ass.

• Perim enopausal: Shortening o f the follicular phase m eans the breast gets m ore
progesterone exposure. M ore progesterone exposure m eans m ore breast pain, m ore
fibrocystic change, m ore breast cyst form ation.

• M enopause ("The F loppy S ta g e ”): Lobules go dow n. D ucts stay but m ay becom e
ectatic. Fibroadenom as w ill degenerate (they like estrogen), and get their “popcorn”
calcifications. Secretory calcifications will develop (*but not for 15-20 years post
m enopause).

• H orm one R eplacem ent Therapy: B reasts get m ore dense (especially estrogen-
progesterone com bos). Breast pain can occur, typically peaking in the first year.
Fibroadenom a (w ho like to drink estrogen) can grow.

464
High Y ield T rivia R eg a rd in g B reast A n a to m y / P h ysiology

The nipple can enhance w ith contrast on M RI. T his is norm al (not Pagets).
• M ost cancers occur in the upper outer quadrant.
• M ost cancers start in the term inal duct lobular unit (TD LU ).
M ajority (60% ) o f blood flow is via the internal m am m ary.
M ets to the Internal M am m ary N odes are uncom m on (3% ) - seen in m edial cancers.
• A xillary N ode Levels (1, 2, 3 - lateral to m edial)
• Stem alis is usually unilateral, and only on the CC , N E V E R on M LO.
B reast Tenderness is m ax around day 27-30.
M am m ography and M RI are best perform ed in the follicular phase (days 7-14).
D o n ’t B iopsy a prepubescent breast - you can affect breast developm ent
Perim enopause (5 0 ’s) is the peak tim e for breast pain, cyst form ation
Fibroadenom as will degenerate (buzzw ord popcorn calcification) in m enopause
• Secretory C alcifications (buzzw ord “ rod-like) w ill develop 10-20 years post
m enopause

W hatcha You Know A bout Lactation?


Loadedfo-fo on the low where the cheese at?

Density: A s m entioned above, the breast gets a lot denser in the 3rd trim ester.
M am m ogram s m ight be w orthless, and ultrasound could be your only hope. In other
w ords, ultrasound has greater sensitivity than m am m o in lactating patients.

D ensity Trick: Pituitary Prolactinom a, or m eds (classically antipsychotics) can create a


sim ilar bilateral increased density.

Biopsy: You can biopsy a breast that is getting ready to lactate / lactating - you ju st need
to know there is the risk o f creatin g a m ilk fistula. I f you m ake one, they will have to
stop breast feeding to stop the fistula. T he fistula can get infected, but th a t’s not very
com m on.

G alactocele: T his is one o f those “benign fat containing lesions” that you can BR-2.
This is typically seen on cessation o f lactation. T he location is typically sub-areolar. The
appearance is variable, but can have an A u nt M in nie look w ith a fat-fluid level. It’s
possible to breast abscess these things up.

L actating Adenom a: T hese things look like fibroadenom as, and m ay actually be a
charged up fibroadenom a (they like to drink estrogen). U sually these are m ultiple. If
you get pressed on follow up recom m endation for these I w ould say 4-6 m onths
postpartum , post delivery or after cessation o f lactation -via ultrasound. T hey usually
rapidly regress after you stop lactation.

4-65
lPlPMMlMMMM ^ _ S E C T IO N 2: leiMIMPMPIPIM

t .J H U T ech n iq u e & A r tifa c ts

Basics: A s I m entioned in the introduction, a screening m am m ogram starts w ith tw o standard


view s; a cranial caudal view and a m edial lateral oblique view.

T e c h n ic a lly A dequate?

The first step in reading a m am m ogram is verifying that the technique is satisfactory. For the
purpose o f m ultiple choice there are a few easy w ays to test this.

Cranial Caudal View “CC” Medial Lateral Oblique View “MLO”

The P osterior N ipple Line - this is draw n on the M LO from the nipple to the chest wall.
You need to touch pectoralis m uscle to be adequate.

Then on CC , you draw a line from the


nipple back tow ards the chest wall.
To be adequate you m ust be w ithin 1
cm o f the length o f the posterior
nipple line.

466
T e ch n ic a lly A dequate Cont.

So other poin ts an d trivia:

Ideally, the infram am m ary fold should be visualized


“C am el N o s e ” is the buzzw ord used to describe a breast on M LO that has not be pulled
“up and out” by the tech
* The nipple should be in profile in one o f tw o view s (to avoid m issing the subareolar
cancer).
R elaxed pectoralis m uscles are preferred (c o n v ex , instead o f concave) - show ing m ore
breast tissue.

Positioning Trivia:

When do you g e t a LM O view ? T he M LO is the standard, but som etim es you need a LM O.
The answ er is w om en w ith kyphosis or pectus excavatum . O r to avoid a m edial
pacem ak er / central line.

M LO View Trivia'. The M LO view contains the m ost breast tissue o f all the possible view s

When using Spot Com pression Views: A big point is the recom m endation to leave the
collim ator open, giving you a larger field o f view , and helping to ensure that you got w hat
you w anted to get. Sm all paddles give you better focal com pression. Large paddles allow
for good visualization o f land m arks.

When using M agnification Views: A CC and M L (true lateral) are obtained. You get a M L
(as opposed to a M L O ) to help catch m ilk o f calcium .

When using a True L ateral View ML vs LM: U sing a true lateral is useful for localizing things
seen on a single view only (the CC). A trick I use is w h atev er 1 said on the screener, is the
last letter I’d use on the call backs. In other w ords, if it’s L ateral on the screener you w ant an
M L on the diagnostic. If it’s M edial on the screener then you w ant a LM on the diagnostic.
The reason is that you are m oving it closer to the receptor. If you see the area o f interest on
the M LO only (not the C C ), you should pick M L - because m ost (7 0 % ) breast cancers
occur laterally. — This w ould m ake a good m ultiple choice question.

467
P rim a ry Im a g e P e c to ra l M o tio n A rtifa c ts P re d o m in a te s a t th e
V ie w M u s c le s h o u ld In fe rio r P a rt o f th e B re a s t (e s p e c ia lly in
be se e n to th e w rin k ly flo p p y s tin k y s a g g y ones)
M a x im iz e d Level o f th e s e c o n d a ry to a la c k o f c o m p re s s io n .
V is u a liz a tio n o f N ip p le
Mediolateral th e A x illa ry and
The “ sw eep
Oblique View P o s te rio r T is s u e P e c to ra l
u p a n d o u t”
(MLO) te c h n iq u e is
M u s c le s h o u ld
u se d b y te c h s
b e R e la x e d
to re d u c e
(c o n v e x
a rtifa c t in th is
a n te rio r
lo c a tio n .
b o rd e r)

P rim a ry Im a g e S h o u ld ha ve a s m a ll a m o u n t o f If y o u la c k a d e q u a te
V ie w s kin a t th e m o s t m e d ia l a s p e c t c o v e ra g e a t th e
to c o n firm a d e q u a te c o v e ra g e p o s te rio r la te ra l e d g e o r
Id e a lly a x illa ry ta il th e n e x t
Craniocaudal m a x im iz e s th e C h e s t w a ll to n ip p le s h o u ld be a p p ro p ria te s te p is an
View (CC) p o s te rio r m e d ia l w ith in 1 c m o f th e c h e s t w a ll to e x a g g e ra te d la te ra l C C
tis s u e (the s p o t p e c to ra l m u s c le o n th e M LO . v ie w (XCCL).
th a t ca n be
m is s e d on th e
M LO )

90 d e g re e v ie w 9 0 d e g re e v ie w

C an b e u s e d to
C an be u s e d to tria n g u la te (m e d ia l to
tria n g u la te (m e d ia l to
th e n ip p le le s io n s w ill rise o n th e tru e
th e n ip p le le s io n s w ill
lateral - “ m u ffin s ris e ” )
rise o n th e tru e la te ra l -
Mediolateral Lateromedial “ m u ffin s ris e ” )
S h o w s th e la te ra l b re a s t (the o n e
(ML) (LM)
c lo s e s t to th e d e te c to r) in b e tte r d e ta il S h o w s th e m e d ia l b re a s t
in b e tte r d e ta il.

R e m e m b e r th e p o s te rio r
m e d ia l b re a s t is th e
to u g h e s t is im a g e .

' jjp|l L r r‘ *j "

_______________
Q: Conan! What is Best in Life ? A : To crush you r enemies, and see them driven before you

Q: Conan! What is Best ... View Given the Following Circumstances ?

“Nodule” seen only in CC View Rolled CC


“Nodule” favored to be in the skin Tangential (TAN)
“Nodule" favored to be milk o f calcium True Lateral

“Nodule” in the far posterior medial breast Cleavage View (CV)

“Eklund Views” or
Breast Implants
Implant Displaced (MLOID, CCID)
Calcifications Magnification View
Yes, I’m using the term “nodule” deliberately to annoy academic breast imagers who hate that word.
Never pick the word "nodule ” on the exam !

468
Basic Artifacts:
Blur: C an be from breathing or inadequate com pression (typically along the inferior breast
on the M LO ). It can be tricky to pick up. The strategy 1 like to use is to look at C o o p e r’s
Ligam ents - they should be thin w hite lines in the fat. If they are thick or fuzzy - it is
probably blur (or edem a). If there is skin thickening, think edem a.

You see blur in 3 scenarios


( 1 ) patient m oved,
( 2 ) exposure w as too long,
( 3 ) exposure w as too short.

Blur: “Coopers are too thick”tor normal skin

Grid Lines: B asically m am m ogram s alw ays use a grid (unless it’s a m ag view ). That
w ould m ake a good m ultiple choice question actually. N o grid on m ag view s. So, the grid
w orks by m oving really fast, and only keeping x-rays that m ove straight in.

Grid Lines: 3 Examples - “Horizontal Lines”

469
a S E C T IO N 3: [iaingllTainaifiSltrglliaifrS]
Lo calizatio n

You are trying to find around 3-8 cancers


per 1000 m am m ogram s. A nother way to
ask this is to say that you are supposed to
have a Positive Predictive Value (PPV i)
o f around 4% (in other words anything
other than a BR1 or BR2 on a screener).
This is dem anded by the various
regulating bodies.

Be aware that certain areas can som etim es


only be seen on a single view. For
exam ple, the m edial breast on a C C may
not be seen on M LO , and the Inferior
Posterior Breast on M LO may be
excluded from the C C . That m akes these Medial Breast
areas “high risk” for m issing a cancer. - Can be excluded on Inferior Posterior
the MLO View Breast
- Can be excluded on
the CC View

It’s recom m ended to look at m am m ogram s from 2 years prior ( if available) for com parison.
M akes it a little easier to see early changes.

Localizing a lesion (only seen in the M LO view): This is a very basic skill, but if you had
absolutely no interest in m am m ography or ju st terrible training, a refresher m ight be useful as
this is applicable to m ultiple choice tests. A lesion that is seen in the M LO only will rise on the
true lateral (M L) if it is m edial on the CC film. A lesion that is seen on the M LO only will fall
on the true lateral (M L) if it is lateral on the CC film. The popular m nem onic is “L ead Sinks,
and Muffins Rise ” - L for lateral, and M for medial.

“M uffins

470
Localizing a lesion (only seen in the C C view ): S om etim es you can only see the finding in
the CC view. If you w ant to further characterize it w ith ultrasound, figuring out if it’s in the
superior or inferior breast could be very helpful. O ne m ethod for doing this is a “ rolled CC
view .”

R olled C C View: T his w orks by positioning the breast for a CC view, but prior to placing the
breast in com pression you rotate the breast either m edial or lateral along the axis o f the
nipple. Your reference point is the top o f the breast.

• If you roll the breast m edial; a superior tum or will m ove m edial, an inferior tum or will
m ove lateral.
• If you roll the breast lateral; a superior tum or w ill m ove lateral, an inferior tum or will
m ove m edial.

In other w ords, superior tum ors m ove in the direction you roll and in ferior tum ors m ove
in the opposite direction you roll. The “su perior ” vs “inferior ” is inferred based on how it
m oves w hen you (the tech) roll the boob.

~ \

C a n c e r in s u p e r io r b re a s t R o lle d M e d ia lly CC v ie w , R o lle d L a te r a lly CC v ie w ,


o n ly seen in CC v ie w m o v e s th e c a n c e r m e d ia l m o v e s th e c a n c e r la te r a lly

~ \

C a n ce r in R o lle d M e d ia lly CC v ie w , R o lle d L a te ra lly CC v ie w ,


o n ly seen m o v e s th e c a n c e r la te ra l m o v e s th e c a n c e r m e d ia lly

471
t SECTION 4
BI-RADS
liH M U H M M

B I-RA D S is an acronym for Breast Im aging-R eporting and D ata System . It w as developed
by the A C R to keep everyone on the sam e page, in a sim ilar w ay the DSM w as developed for
psych. You c a n ’t have people ju st calling stu ff “breast n o dules” .

* BI-RADS Assessm ent Categories'.


0: Incom plete
1: N egative
2: Benign finding!s)
3: Probably benign — < 2% C hance o f CA
4: Suspicious abnorm ality — 2 - 95% C h ance o f CA
* Som e people use 4a (low suspicion), 4b (interm ediate suspicion), and 4c
(m oderate suspicion).
5: H ighly suggestive o f m alignancy — > 95% C h ance o f CA
6: K now n biopsy - proven m alignancy

BI-RADS 0: This is your incom plete w orkup. They com e in for a screener, you find
som ething suspicious. You give it a B I-R A D S 0, and bring them back for spots, m ags, or
ultrasound. You w ould also BI-R A D S 0 anything that required a technical repeat (blur,
inadequate posterior nipple line, cam el nose, e tc ....).

BI-RADS 1: It’s norm al.

BI-RADS 2: Benign findings. E xam ples w ould be cysts, secretory calcifications, fat
containing lesions such as oil cysts, lipom as, galactoceles and m ixed-density ham artom as.

* M ultiple bilateral w ell circum scribed, sim ilar appearin g m asses - This is BR-2
unless one is grow ing or different than the rest. T he general rule is to not ultrasound
these things unless one is palpable.

* M ultiple F oci - This M RI finding is also a classic BR2.

472
BI-RADS 3: A key point is that BR-3 by definition m eans it has less than 2% chance o f
being cancer. This is often a confusing topic. You can only use BR3 on a baseline. You
c a n ’t call anything BR3 that is new. The typical BR3 scenario: 45 year old com es in for
screening and has a focal asym m etry. She gets called back for diagnostic w ork up w ith spots
and ultrasound. She is found to have m ass w ith im aging features classic for fibroadenom a.
This can get a BR-3, and be follow ed (som e places follow for 2 years, in 6 m onth intervals).
Any change over that tim e ups it to BR-4 and it gets a biopsy.

Things you can BR-3:


* Finding consistent with fibroaden om a
* F ocal asymmetry>that looks like breast tissue (becom es less dense on com pression).
* G rouped Round C alcifications

What i f i t ’s palp a b le? This is a controversial topic. C lassic teaching is that palpable lesions can
not be BR3. H ow ever, recent papers have show n that a palpable lesion consistent w ith a
fibroadenom a has less than 2% chance o f cancer. Som e people think the new B l-R A D S will
change this rule. I really doubt they w ill paint you into a c o m e r on this one - given the
controversy.

BI-RADS 4: This is defined as having a 2-95% chance o f m alignancy. Som e people will
subdivide this into 4A, 4B. 4 C depending on the level o f suspicion. U ltim ately you are going to
biopsy it, and be prepared to accept a benign result.

BI-RADS 5: T his is defined as > 95% chance o f m alignancy. W hen you give a BR -5, you are
saying to the pathologist “ if you give m e a benign result, I’ll have to recom m end surgical
biopsy.” In other w ords, you c a n ’t accept benign with a BR-5.

BI-RADS 6: T his is path proven cancer.

473
Basic Flow - These are essentially y our choices in a w ork up.

BR 0 - You made a suspicious finding


and they need a diagnostic workup

BR 0 - Technical Repeat for Blur,


inadequate positioning etc...

BR 1- Totally Normal
S creen in g M am m ogram
(no sym ptom s)
BR 2 - M ultiple bilateral, well circum scribed,
sim ilar appearing m asses

BR 2 - R edem onstration o f an unchanged


previously w orked up thing - a cyst e tc ...

BR 2 - You w ork it up and it’s a benign thing -


fat containing lesion, cyst, e tc ... This returns to
screening
D iagnostic M am m ogram
(either a call back, BR 3 - Very specific situation w here you
or som eone with are dealing w ith a baseline screener, now
a sym ptom ) called back. Findings m eet one o f the
three things described above;
(fibroadenom a, fat w ith breast tissue,
group o f round calcs). This gets 2 years
o f follow up.

BR 4 - Suspicious finding, but you a re n ’t


convinced it’s cancer. In other w ords,
you w ould accept a benign result. - This
gets a biopsy.

BR 5- S uspicious finding, that you are


convinced is cancer. In other w ords, you
w ould N O T accept a benign result. - This
gets a biopsy.

474
BI-RADSTerminology
In addition to the “0-6” babysitting, the various regulatory bodies have decided there are only
a few w ords they will trust you w ith, depending on w hat m odality you are using.

Plain M am m ography:

“ M ass” - This is a space occupying lesion seen in tw o different projections

D escribing the mass: You need to cover (1) Shape, (2) M argin, (3) D ensity
(1) Shape: R ound, O val, Irregular - “ R O I ”
(2) Margin: C ircum scribed, O bscured, M icrolobulated, Indistinct, Spiculated - “ C O M IS”
(3) D ensity (relative to breast parenchym a: Fat D ensity (radiolucent), Low D ensity, Equal
Density, High D ensity

Trivia: O f all the possible descriptors - m argin is the m ost reliable feature for determ ining
benign vs m alignant.

“ A sym m etry” - U nilateral deposition o f tissue that d o e sn ’t quite look like a m ass.
* A sym m etry - This is a density (only seen in one view ) that m ay or m ay not be a m ass,
and is often a term used in screeners for BR-0 prior to call back.
* G lobal Asym m etry - “ greater volum e o f breast tissue than the contralateral side” ,
around one quadrants w orth (or m ore). It’s gonna get a call back, and then B R -2 ’d on
a baseline.
* F ocal A sym m etry - T his is seen in tw o projections, m ight be a m ass - needs a spot
com pression.
* D eveloping A sym m etry - W asn’t there before, now i s ... or bigger than prior.

U ltrasound:

D escribing the m ass: You need to cover : (1) Shape, (2) O rientation, (3) M argin, (4) Echo
pattern, (5) Posterior acoustic features

(1) Shape: Round, O val, Irregular (not round or oval)


(2) Orientation: Parallel (w ider than tall), N ot-Parallel (taller than w ide)
(3) Margin: C ircum scribed, Indistinct, A ngular, M icrolobulated, Spiculated
(4) Echo P attern : A nechoic, H yperechoic, H ypoechoic, Isoechoic, or C om plex (cystic/
solid)
(5) P osterior Features: N one, E nhancem ent, S hadow ing

475
MRI:

T here has recently been a vocabulary change in the Lexicon, and I’m going to briefly cover
the changes.

Background Parenchym al Enhancement:


• This is a new ly added B 1-RADS “ feature.” In the literature, they specify that this
description is based o ff the first post contrast sequence (sounds testable to m e). The
• C ategories are : none, m inim al, m ild, m oderate, and m arked.

Lesion Analysis: There are 3 basic categories for this:


• Foci ( < 5 m m): You d o n ’t need to describe shape and m argin on these. They are too
sm all.
• M ass ( > 5 m m): This will have shape, m argin, internal enhancem ent characteristics, &
T2.
• N on -M ass E nhancem ent: D istribution, Internal E nhancem ent, T2

D escribing M asses:
• Shape: Round, O val, and Irregular. T he w ord “ lobulated” has been rem oved from the
lexicon, so expect that to be a distractor.
• M argin: C ircum scribed, Irregular, and Spiculated. T he w ord “ sm ooth” has been rem oved
from the lexicon, so expect that to be a distractor.
• Internal E n hancem ent Patterns: H om ogenous, H eterogenous, Rim , and D ark Internal
Septations. “ Enhancing Internal S eptations” and “ C entral E nhancem ent” are N O T term s in
the new vocab - and will likely be distractors.

T2 Signal - This is a new “ feature” o f the lexicon


• Hyperintense:
• G reater than parenchym a (on T2)
• G reater than or equal to fat (on T2)
• G reater than or equal to w ater (on T2 Fat Sat)

NM E - “D istribution ”
• Focal, Linear, Segm ental (triangle shaped pointing tow ards nipple - suggestive o f a duct),
Regional (large area - not a duct), M ultiple R egions (tw o or m ore regions) and Diffuse.

NME - Internal Enhancement


• H om ogenous, H eterogenous, C lum ped (looks like cobblestone), C lustered Ring (this is a
bu zzw o rd fo r DCIS o r IDC). “ R eticular” and “ D endritic” have been rem oved and will
likely be distractors.

4 76
MRI BIRADS C ont..

Kinetic Curves are also described. I’ll talk about this m ore in the B reast M RI Section.

A ssociated Findings'. You are allow ed to talk about nipple retraction, skin thickening, edem a,
invasion o f the pec m uscles, pre contrast signal, and artifacts.

Implants'. W hen you talk about im plants you have to describe the type (silicone vs saline),
location (retroglandular vs retropectoral), and lum inal features like radial folds, keyhole,
linguine, e tc ... I’ll cover this m ore in the B reast M RI section.

477
lMIMIMMMMMJM v S E C T IO N 5.

t C a lc i f i c a t i o n s W

C alcifications can be an early sign o f breast cancer. “T he earliest sign,” actually, according
to som e. C alcifications basically com e in three flavors: (1) artifact, (2) benign, and (3)
suspicious.

A rtifa c ts S im u la tin g C a lc ific a tio n s :

D eodorant: High density m aterial seen in the


axilla is the typical appearance. A nother trick
is to show a speck o f high density m aterial
that d o e sn ’t change position on different
view s (inferring that it’s on the im age
receptor).

Deodorant Artifact

Z in c O xide: This is in an ointm ent old ladies like to put on their floppy sw eaty breasts. It
can collect on m oles and m im ic calcifications. If it disappears on the follow up it w as
probably this (or another derm al artifact).

M etallic A rtifact: It’s possible for the electrocautery device to leave sm all m etallic
fragm ents in the breast. These will be very dense (m etal is d enser than calcium ). It will also
be adjacent to a scar.

478
Benign vs Suspicious:
The distinction betw een benign and suspicious is m ade based on m orphology and distribution
(those BI-R A D S descriptors). Since m ost breast cancers start in the ducts (a single duct in m ost
cases), a linear or segm ental distribution is the m ost concerning. The opposite o f this w ould be
bilateral scattered calcifications.

' ■jJf ^■tjrL


* m H r" '- j a r

Segmental Linear Grouped Regional Scattered/ Diffuse

More Evil than Ursula


(villainous sea witch) Benign
from the Little Mermaid

Benign:
Derm al C a lc ific a tio n s : These are found anyw here w om en sw eat (folds, cleavage,
axilla). Just think folds. T hey are often grouped like the paw o f a bear, or the foot o f a baby.
The trick here is that these stay in the sam e place on CC, and M L O view s. This is the so
called “tattoo sign. ” If you are asked to confirm these are derm al calcs, I ’d ask for a
“tangential view .”

479
Benign - Continued

V a sc u la r C a lc ifica tio n s: These are parallel linear calcifications. It’s usually obvious,
but not alw ays.

Popcorn C a lc ifica tio n s: This is an im m ediate buzzw ord for degenerating fibroadenom a.
The typical look is they begin around the periphery and slow ly coalesce over subsequent im ages.

S e c re to ry (Rod-Like) C a lc ifica tio n s: T hese are


big, easily seen, and point tow ard the nipple. They are
typically bilateral. The buzzw ord is “cigar sh aped with a
lucent center. ” A nother buzzw ord is “dashes but no dots. ”
The buzz age is “ 1 0 -2 0 y e a rs after menopause. ” D o n ’t be
an idiot and call these in a prem enop ausal patient, they
happen because the duct has involuted.

Secretory Calcifications

E g g sh e ll C a lc ifica tio n s: “ Fat necrosis” 1 call them . It can be from any kind o f traum a
(surgical, or accidental - play ground related). If they are really m assive you m ay see the w ord
“liponecrosis m acrocystica.” As I’ve m entioned m any tim es in this book, anything that sound
Latin or French is high yield for m ultiple choice. “Lucent C en tered ’’ is a buzzword.

D ystrophic C a lc ifica tio n s: T hese are also seen after radiation, traum a, or surgery. These
are usually big. The buzzw ord is “irregular in shape. ” T hey can also have a lucent center.

Round: The idea is that these things develop in lobules, are usually scattered, •e .
bilateral, and benign. W hen benign (w hich is m ost o f the tim e) they are goin g to *. •
be due to fib ro cystic change (m ost o f the tim e). The best w ay I ’ve heard to think .* •
about these is the sam e as a m ass.

W hen m asses are bilateral, m ultiple, and sim ilar they are considered benign (B R -2). W hen a
m ass is by itse lf or different it’s considered suspicious. R ound calcifications are the sam e
way. They are usually bilateral and sym m etric (and benign). If they are clustered together,
by them selves, or new, they m ay need w orked up (just like a m ass). R em em ber that if
grouped round calcs are on the first m am m ogram you can BR-3 them .

480
Benign - Continued
M ilk o f C alcium :
This has a very characteristic look, and because o f
that, questions can only be asked in one o f tw o
ways: (1) w hat is it? - show n as CC then M L,
(2) w hat is it due to ?

(1) On the CC view the calcifications look pow dery


and spread out, on the M LO view they m ay
layer. I suspect they will show you a M L view
because they should layer into a m ore linear
appearance, w ith a curved bottom “tea-cupped." “tea-cupped”
appearance on
For the pu rpose o f gam esm anship i f they show
true lateral view.
you a ML view on a calcs question - look hard
f o r anything that resem bles tea-cupping. Fluid-Fluid in
(2) It’s fluid-fluid in a lobule - due to fibrocystic
a lobule
change.

Milk of Calcium - Tea Cups on ML

No Calcifications on the Biopsy?


This is a com m on trick. A pparently M ilk o f C alciu m needs to be view ed w ith polarized
light to assess birefringence. O therw ise, you c a n ’t see it. I im agine there are several w ays to
get at that via m ultiple choice.

481
Suspicious

Am orphous - T hese things look like pow dered sugar, and you should
not be able to count each individual calcification.

D istribution is key w ith am orphous calcs (like m any other types before). If
the calcs are scattered and bilateral they are probably benign, if they are
segm ental they are probably concerning.

C o a rse H etero gen eo u s - T hese calcifications are countable, but


| their tips are dull. If you picked one up it w ould not be poke you.

^ ^ They are usually b igger than 0.5 m m . D istribution and com parison to
^ priors is alw ays im portant. They can be associated w ith a m ass
(fibroadenom a, or papillom a).

Fin e Pleom orphic - These calcifications are countable, and their tips
appear sharp. If you picked one up it w ould poke you. They are usually
sm aller than 0.5 m m . This pattern has the second highest likelihood o f
m alig n an cy . .. p robably — see discussion on the fo llo w in g page.

Fin e Lin e a r / Fine Lin e a r B ra n ch in g - T his is a distribution


that m akes fine pleom orphic calcifications even m ore suspicious. The
DDx narrow s to basically D C IS or an atypical look for secretory calcs or
vascular calcs. This pattern has the highest likelihood o f m alignancy.

DDx A m orphous C a+2 D D x Fine


DDx C oarse H eterogeneous C a+2
P leom orphic C a+2
Fibrocystic C hange
Fibroadenom a
(most likely) Fibroadenom a (less likely)
Papillom a
Sclerosing A denosis Papillom a (less likely)
Fibrocystic C hange
C olum nar Cell C hange Fibrocystic C hange
DCIS (low - interm ediate grade)
DCIS (low grade) DCIS (high grade)

482
Suspicious - Continued

C a lc ifica tio n s A sso c ia te d w ith Fo ca l Asym m etry/M ass:


W hen you see increased tissue density around suspicious calcifications, the chance o f an
actual cancer goes up. T his is som etim es called a “ p u ff o f sm ok e” sign , or a “w arning
shot.” This is a situation w here ultrasound is useful, for extent o f disease.

G am esm an sh ip - N ext Step:

U ltrasound is N O T typically used to evaluated pure calcification findings. E xceptions


w ould be (a) if the patient had a m ass associated w ith the calcifications, or (b) if the patient
had a palpable finding - then they w ould get additional evaluation w ith ultrasound.

G am esm anship “H ig h est Suspicion fo r M alig n an cy”

D epending on w hat you read and w ho you ask, Fine L inear B ranching and Fine Pleom orphic
C alcifications have the H ighest Suspicion for M alignancy. So w hich one is it?

For sure fine linear branching is the w orst. M orphologically it m im ics the ductal
proliferation o f suspicious calcifications (D C IS). T he confusion is that som e people use
fine pleom orphic as an um brella term under w hich linear and branching form s exist.

So how to handle this on m ultiple choice?

• If the answ er choices include fine linear branching then that is the correct answer.

• If the answ er choices do N O T include fine linear branching but instead have you pick fine
pleom orphic vs coarse heterogenous or som e other obviously benign calcs (egg shell,
e tc ...) then for sure pick fine pleom orphic.

483
j S E C T I O N 6:
B e n i g n P a t h

Mondor D ise a se : This is a throm bosed vein that presents as a tender palpable cord. It
looks exactly like y o u ’d expect it to w ith ultrasound. You d o n ’t anticoagulate for it (it’s not
a DVT). T reatm ent is ju st NSA1DS and w arm com presses.

Fat C on tain ing L e sio n s: T here are five classic fat containing lesions, all o f w hich
are benign: oil cyst / fat necrosis, ham artom a, galactocele, lym ph nodes, and lipom a. O f
these 5, only oil cyst/fat necrosis and lipom a are considered “pure fat containing” m asses.

* H am artom a - The buzzw ord is “breast w ithin a


breast.” They have an A unt M innie appearance on
m am m ography, although they are difficult to see on
ultrasound (they blend into the background).

* G alactocele - Seen in young lactating w om en.


This is typically seen on cessation o f lactation. The
location is typically sub-areolar. The appearance is
variable, but can have an A unt M innie look w ith a Hamartoma
fat-fluid level. It’s possible to breast abscess these - “Breast within a Breast’
things up.

* O il C yst / Fat N ecrosis - These are areas o f fat necrosis w alled o ff by fibrous tissue.
You see this (1) random ly, (2) post traum a, (3) post surgery. The peripheral
calcification pattern is typically “ egg shell.” I f yo u see a ton o f them yo u m ight think
about steatocystom a m ultiplex (som e zebra w ith ham artom as).

* Lipom a - These are typically radiolucent w ith no calcifications. E nlargem ent o f a


lipom a is criteria for a biopsy.

* Intram am m ary Lym ph node: T hese are norm al and typically located in the tissue
along the pectoral m uscle, often close to blood vessels. T hey are N O T seen in the
fibroglandular tissue.

P ractice Point: D oes she n eed an ultrasound i f i t ’s p a lp a b le? U sually a palpable finding
is going to get an ultrasound. If you are under 30, m ost people will skip the m am m o and
go straight to ultrasound. One o f the exceptions is a fat containing lesion definite benign
B R -2er on diagnostic m am m ography.

484
P seu doan gio m atou s Stro m al H yp e rp la sia (PA SH ): This is a benign
m yofibroblastic hyperplastic process (hopefully that clears things up). It’s usually big (4-6
cm), solid, oval shaped, w ith well defined borders. A ge range is w ide they can be seen
betw een 18-50 years old. Follow up in 12 m onths (annual) is the typical recom m endation.

Pseudoangiom atous Strom al H yperplasia = Benign thing with a sca ry sounding name

Fibroadenom a - T his is the m ost com m on palpable m ass in young w om en. The typical
appearance is an oval, circum scribed m ass w ith hom ogeneous hypoechoic echotexture, and a
central hyperechoic band. If it’s show n in an older patient, it’s m ore likely to have coarse
“popcorn” calcifications - w hich is a buzzw ord. O n M R I, i t ’s T2 bright w ith a type 1
enhancem ent (progressive enhancem ent).

Phyllodes: A lthough I clum ped this in benign disease, this thing has a m alignant
degeneration risk o f about 10%. They can m etastasize - usually hem atogenous to the lungs
and bone. T his is a fast grow ing breast m ass. T hey need w ide m argins on resection, as they
are associated w ith a higher recurrence rate if the m argin is < 2 cm. It occurs in an older age
group than the fibroadenom a (40s-50s). B iopsy o f the sentinel node is not needed, because
m ets via the lym phatics are so incredibly rare ( if it does m et - it’s hem atogenous).

D istinguishing Features o f Phyllodes Tumor


* R apid Growth
* H em atogenous Mets
* M iddle-A ge to O lder Women
* M imics a F ibroadenom a

485
S E C T I O N 7: 4 B
C an cer p^ i w .

IDC - Invasive Ductal Carcinoma


I DC - Invasive Ductal C arcinom a is by far the m ost com m on invasive breast cancer, m aking
up about 80-85% o f the cases. T his cancer is ductal in origin (duh), but unlike DCIS is not
confined to the duct. Instead it “ invades” through the duct and if not found by the heroic
actions o f M am m ographers it will progress to distal m ets and certain death. Clinically, the
m ost com m on story is a hard, non-m obile, painless m ass. On im aging, the m ost com m on
look is an irregular, high density m ass, w ith indistinct or spiculated m argins, associated
pleom orphic calcifications, and an anti-parallel shadow ing m ass w ith an echogenic halo on
ultrasound.

In vasive D uctal N O S - By far the m ost com m on type o f breast cancer is the one that is
undifferentiated and has no distinguishing histological features. “ N ot O therw ise S pecified”
or N O S they call it. These guys m ake up about 65% o f invasive breast cancer.

L e s s Com m on (but still te stab le ) ID C Su b typ e s

ID C T y p e s - (Other than NOS)

Sm all spiculated slow O ften conspicuous on ultrasound. A ssociated


Tubular grow ing m ass w ith a with a R adial Scar. C ontralateral breast will
favorab le prognosis. have cancer 10-15% o f the tim e.

Round (or lobulated) and


IVIucinous U ncom m on. B etter outcom es than ID C-N O S
circum scribed m ass

Round or Oval A xillary nodes can be large even in the


circum scribed m ass, absence o f m ets. Typically younger patient
M edullary
w ithout calcifications. (40s-50s). B etter outcom e than ID C-N O S
-25% have BR C A 1 m utation

A xillary nodes are N O T com m on. Typically


C om plex cystic and seen in elderly people, favors people w ho are
Papillary
solid. not w hite, and is the 2nd m ost com m on (behind
ID C-N O S).

486
M ultifocal B re a st C a n c e r M u lticen tric B re a st C a n c e r

M ultiple prim aries in the sam e quadrant M ultiple prim aries in different quadrants
(classically sam e duct system )
Think o f this like “m u lti-cen ter” clinical
Less than 4-5 cm apart from one another trial; m ultiple discrete tin-related sites.

Syn ch ro n ou s B ila te ra l B re a st C a n c e r - T h i s is seen in 2-3% o f w om en on


m am m ography, w ith another 3-6% found w ith M RI. The risk o f bilateral disease is increased
in infiltrating lobular types, and m ulti-centric disease.

D C IS - This is the “earliest form o f breast cancer.” In this situation the “ cancer” is confined
to the duct. H istologists grade it as low, interm ediate, or high. H istologists also use the term s
“com edo”, and “non-com edo” to subdivide the disease. If anyone w ould ask, the com edo
type is m ore aggressive than than the non-com edo types.

Testable Trivia:
10% o f DCIS on im aging m ay have an invasive com ponent at the tim e biopsy is done
• 25% o f DCIS on core biopsy m ay have an invasive com ponent on surgical excision.
8% o f DC IS will present as a m ass w ithout calcifications
M ost com m on ultrasound appearance = m icrolobulated m ildly hypoechoic m ass with
ductal extension, and norm al acoustic transm ission

If a test w riter w ants you to com e dow n on this they w ill show it in 1 o f 3 classic ways:
(1) suspicious calcifications (fine linear branching or fine pleom orphic - as discussed above),
(2) non m ass enhancem ent on M RI, or (3) m ultiple intraductal m asses on galactography.

P a ge ts - P aget’s disease o f the breast is a high yield topic. It is basically a carcinom a in


situ o f the nipple epiderm is. A bout 50% o f the tim e the patient will have a palpable finding
associated w ith the skin changes.

Things to know about Breast P agets:


• A ssociated w ith high grade D C IS (96 % )
• W edge biopsy should be done on any skin lesion that affect the nipple-areolar
com plex that d o e sn ’t resolve w ith topical therapy.
• Pagets is N O T considered T4. The skin involvem ent does not up the stage in this
setting.

487
Lobular - ILC
Lobular ( I L C ) : This is the second m ost com m on type o f breast cancer (ID C-N O S being the
m ost com m on). It m akes up about 5-10% o f the breast CA cases.

This pathophysiology lends itself well to m ultiple choice questions:

Cell decides to be cancer -> Cells lose “e-cadherin” -> Cells no longer stick to one another and
begin to infiltrate the breast “ like the web o f a spider” -> This infiltrative pattern does not cause
a desm oplastic reaction so it gets m issed on m ultiple m am m ogram s -> Finally som eone (you)
notices som e architectural distortion w ithout a central m ass, on the CC view only. You get
fancy and call it a “dark star."

On Ultrasound: The typical look is an


ill-defined area o f shadow ing w ithout a
mass.

ILC - S hadow ing w ithout discrete mass

“S h rin kin g B re a st” -


This is a buzzw ord for
ILC. The breast isn ’t
actually sm aller, it ju st
d o e sn ’t com press as m uch.
So w hen you com pare it to
a norm al breast, it appears
to be getting sm aller. On
physical exam , this breast
m ay actually look the
sam e size as the other one.

Shrinking Breast

488
Lobular - ILC - Continued

THIS vs THAT: ILC VS IDC: ILC is m ore often m ultifocal. ILC less often m ets to
the axilla. Instead, it likes to go to strange places like peritoneal surfaces. ILC m ore often has
positive m argins, and is m ore often treated w ith m astectom y although the prognosis is sim ilar to
IDC.
Things to know about ILC:

It presents later than IDC


Tends to occur in an older population
It often is only seen on one view (the CC - as it com presses better)
C alcifications are less com m on than w ith ductal cancers
M am m o B uzzw ord = D ark Star
M am m o B uzzw ord = Shrinking Breast
U ltrasound B uzzw ord = Shadow ing w ithout m ass
On M RI - w ashout is less com m on than w ith IDC
A xillary m ets are less com m on
Prognosis o f IDC and ILC is sim ilar (unless i t ’s a pleom orphic ILC - which is bad)
M ore often m ultifocal and bilateral (com pared to ID C) - up to 1/3 are bilateral

D a rk S ta r

*W '■ IB
,3 1
-

■■

“Dark Star” - Distortion without a central mass

A rchitectural distortion w ithout a central m ass.


The D D x includes: lobular carcinom a, radial scar, surgical scar, and ID C -N O S.

489
Inflammatory Breast Cancer (IBC1:
IBC an asshole with a notoriously terrible prognosis (at presentation -30% will have metastases).

Clinical Scenario: The classic clinical scenario is a hot swollen red breast that developed rapidly
over 1-3 months. They may even deploy the French sounding word “peau d’orange,” - which
basically means skin that looks like a delicious ripe grapefruit. Although there may be a mass on the
mammogram, in the most classic scenario there isn’t a focal palpable mass.

“Skin Thickening ” is a mammography buzzword (non-specific). Skin thickening is not (by itself)
specific and lots of stuff including CHF can also cause skin thickening. In the case of inflammatory
breast cancer the skin thickening is the result o f tumor emboli obstructing the lymphatics.

Probably Fuckery: It is likely the question writer will try to make you think mastitis - even though the
scenario isn’t really classic for that. Remember - mastitis is seen in breast feeding women — that is
the most common scenario. If it is just “random woman with a hot swollen breast” - you 100% should
think cancer first. Even if they put them on antibiotics, and she has a history of recurrent infections
or whatever — that is all probably bullshit.

Mind Map:
Swollen, Red Breast, Thick Confirming the diagnosis o f IBC requires: Both
Skin — looks like the peel (1) Tissue Diagnosis and (2) Clinical Evidence of
o f a Grapefruit - rapid inflammatory disease the diagnosis o f IBC
clinical presentation
Evaluate for (< 3 months) +/- fever
a focal lump MRI
or skin Best method for detecting the primary lesion in IBC
fluctuance to Most common finding = extensive or segmental
help target NML enhancement + diffuse skin thickening
your US scan

■ ■ ■ ■ ■ ■ ■ ■ ■ ■ a mp m mm m a
Consider
Hypoechoic Solid mass with Punch Biopsy MRI for
Skin thickening
Multiloculated suspicious (usually works, biopsy
but no focal mass
Collection features but not always) ^ ta rg e tin g

Drain it under Ultrasound. Confirm Biopsy the KAntibiotics ^ Incomplete response to


& Give Antibiotics Response / Mother Fucker to see if it antibiotic treatment within
(send culture & gram) Resolution gets better 1-2 weeks -Presum e IBC-
ll
The inflammation associated with
inflammatory breast cancer can actually
Trivia: improve with antibiotics, but does NOT
resolve. So, don't be fooled (in the real
• IDC is the most common subtype to result world or on a multiple choice test).
in IBC (although any subtype of primary
breast CA can)
This vs T hat = IBC vs LABC
• IBC is stage 4
Inflam m atory Breast Locally A dvanced
C ancer (IBC) Breast C ancer (LA BC )
Treatment: They will try and do
chemotherapy prior to surgery because the Rapid onset Prolonged onset
chance of a positive margin is so high. The
standard mastectomy is done for “local Younger (mid 50s) Older (mid 60s)
control” , which just sounds awful. 30% mets at presentation 10% mets at presentation

490
High Risk lesions:
There are 5 classic high risk lesions that m ust com e out after a biopsy; Radial Scar, A typical
Ductal Hyperplasia, A typical Lobular H yperplasia, LCIS, and Papillom a.

Radial S c a r: This is not actually a scar, but does look like one on histology. Instead you
have a bunch o f dense fibrosis around the ducts giving the appearance o f architectural
distortion (dark scar).

Things to know:
• This is high risk an d has to come out
• I t ’s associated with D C IS and/or ID C 10%- 30%
• I t ’s associated with Tubular Carcinom a*

A typical Ductal H yp erp lasia (ADH): This is basically DCIS but lacks the
quantitative definition by histology (< 2 ducts involved). It com es out (a) because it’s high risk
and (b) because DCIS burden is often underestim ated w hen this is present. In other words,
about 30% o f the tim e the surgical path w ill get upgraded to DCIS.

Lobular Carcinom a in Situ (L C IS ): This is classically occult on m am m ogram . “An


incidental finding” is som etim es a buzzw ord. The best w ay to think about LCIS is that it can
be a precursor to ILC, but isn ’t obligated to be. The risk o f conversion to an invasive cancer is
less when com paring DCIS to IDC. Just like pleom orphic ILC is w orse than regular ILC, a
pleom orphic LCIS is m o ’ badder than regular LCIS.

A typical Lobular H yp erp lasia (ALH ): This is very sim ilar to LCIS, but histologists
separate the two based on if the lobule is distended or not (no with A LH , yes w ith LCIS). It’s
considered m ilder than LCIS (risk o f subsequent breast CA is 4-6x higher w ith ALH, and 1 lx
higher with LCIS). For the CO RE, the answ er is excision. In the real w orld, som e people do
not cut these out, and it’s controversial.

Papillom a: A few m ost com m ons com e to m ind w ith this one. M ost com m on intraductal
m ass lesion. M ost com m on cause o f blood discharge. You typically see these in w om en in
their late reproductive years / early m enopausal years (average around 50). The classic
location is the subareolar region (1cm from the nipple in 90% o f cases).

— Mammogram: O ften norm al - occasionally ju st show ing calcifications.


—US: W ell-defined sm ooth w alled hypo-echoic m ass. M aybe cystic w ith solid com ponents.
Also, tends to have associated duct dilation.
— Galactographv: Solitary filling defect, w ith dilated duct.

M ultiple Papillom as: These tend to be m ore peripheral. On m am m ography it’s gonna be a
m ass(es) or a cluster o f calcifications w ithout a mass.

491
Phyllodes: Y e s... I m entioned this already under benign disease. 1ju st w anted to bring it
up again to m ake sure you rem em ber that this thing has a m alignant degeneration risk o f
about 10% (som e texts say up to 25% ). This is a fast grow ing breast m ass. It occurs in an
older age group than the fibroadenom a (40s-50s).

Multiple Masses: Sounds Bad But Actually BR-2

To call m ultiple m asses you need to have


m ultiple (at least 3) bilateral well
circum scribed m asses w ithout suspicious
features. This gives you a BR -2.

One com m on trick is to show m ultiple


unilateral m asses, that d o esn ’t fly - they
have to be bilateral.

Lym phom a:

B re a s t L y m p h o m a

Primary Breast Lymphoma Primary Lymphoma Secondary Lymphoma


L e ss C o m m o n M o re C o m m o n
A lw a y s , for the purpose of multiple choice, most common secondary
malignancy or mets to
N on H o d g k in (D iffu se L a rg e B -C e ll)
involve the breast

T yp ica l L o o k:
U su a lly a h y p e rd e n s e m a ss
• U s u a lly S o lita ry T yp ica l L o o k:
(,Architectural distortion is rare)
• U s u a lly L a rg e r • In fla m m a to ry
(compared to th ic k e n in g w ith o u t a
secondary) a n d m o s t m a s s (b u t c a n lo o k
“ IH C ” s ta in in g is n e e d to c o n firm ly m p h o m a
o fte n p a lp a b le lik e a n y th in g )
• C y s tic o n US

Do axillary nodes = lymphoma? Bro.... anything can give you axillary nodes

492
* IE @ SE IE M E @ U j K S E C_ T_ I_ O
_ _N_ _8 :
S y m p t o m a t ic D irty P i l l o w s

B re a st Pain: This is super com m on and typically cyclic (w orse during the luteal phase o f
the m enstrual cycle). Pain in both breasts that is cyclical does not need evaluation. Instead it
needs a fam ily m edicine referral for som e “ therapeutic com m unication.” Focal non-cyclic
breast pain m ay w arrant an evaluation.

Trivia: T he negative predictive value o f com bined m am m ogram and US for “ focal pain” is
right around 100%. W hen breast cancer is found it’s usually elsew here in the breast
(asym ptom atic).

Sym ptom s that are actu ally w orrisom e for can cer include: skin dim p ling, focal skin
thickening, and nipple retraction.

N on-Focal S k in T h ick e n in g I B re a st Edem a: This is usually the result o f


benign conditions (congestive heart failure, renal failure). F or m ultiple choice tests it will
alw ays be bilateral (in the real w orld you can sleep on one side and have asym m etric edem a).
As long as the breast isn ’t red, you can feel confident that it will be benign. On
m am m ography you w ill see trabecular thickening (diffuse, and favoring the dependent
portions o f the breast).

B re a st Inflam m ation: The sw ollen red breast. T his finding has a differential o f two
things: (1) m astitis / abscess, (2) inflam m atory breast cancer.

• M astitis / A bscess: This is a sw ollen red breast w hich is painful (Inflam m atory breast CA
is often painless). Patients are usually sick as a dog. O bviously it’s associated w ith
breast feeding, and is m ore com m on in sm okers and diabetics. A bscess can develop
(usually Staph A.).

* Inflam m atory Breast C ancer: As previously discussed, this has a terrible prognosis.
The general rule is that a breast that d o e sn ’t respond to antibiotics gets a skin biopsy to
exclude this. T he typical age is 40s-50s. You are going to have an enlarged, red breast
w ith a “peau d ’orange” appearance. The breast is often N O T painful, despite its
appearance. M am m ogram m ight show a m ass (or m asses), but the big finding is diffuse
skin and trabecular thickening. The treatm ent is fair gam e for m ultiple choice because it
is different than norm al breast cancer. Instead o f going to surgery first, inflam m atory
breast cancer gets “cooled dow n” w ith chem o and/or radiation - then surgery.

493
The leaky Tit
W om en present w ith nipple discharge all the tim e, it’s usually benign Multiple Ducts
(90% ). The highest yield inform ation on the subject is that: (Benign)
spontaneous, bloody, discharge from a single d uct is your m ost
Single Ducts
suspicious feature com bo. Serous disch arge is also suspicious.
(Maybe Malignant)
The risk o f discharge being cancer is directly related to age (very • Papilloma
uncom m on under 40, and m ore com m on over 60). . DCIS

* Discharge is Bad when i t ’s - Spontaneous, Bloody, and fro m a Single Duct

M ilky D isch a rge : M ilky discharge is N O T suspicious for breast cancer but can be
secondary to thyroid issues or a pituitary adenom a (prolactinom a). A ny m edication that m esses
w ith dopam ine can stim ulate prolactin production - (antidepressants, neuroleptics, reglan).

C a u s e s o f D is c h a r g e ( N o t M ilk y )

B e n ig n C a u s e s W o r r is o m e C a u s e s

Intraductal Papilloma (90%) - single intraductal


Pre-Menopausal Woman = Fibrocystic Change
mass near nipple

Post Menopausal Women = Ductal Ectasia DCIS (10%) - multiple intraductal masses

D uctal E c ta s ia - The m ost com m on benign cause o f nipple discharge in a post m enopausal
w om an. On galactography you will see dilated ducts near the subareolar region, w ith
progressive attenuation m ore posteriorly.

Papillom a - D iscussed previously- this is the m ost com m on cause o f bloody discharge.
A s before they can be single or m ultiple, and carry a sm all m alignant risk (5% ).

G a la cto gra p h y
• U g h ... you take a 27 or 30 gauge blunt tipped needle and attem pt to cannulate the duct
w hich is leaking. To determ ine w hich duct you w ant - y o u ’ll need to have the patient
squeeze the breast to dem onstrate w here it’s com ing from .

• If you m anage to cannulate the duct - gently inject 0.2 - 0.3 cc contrast (rare to need m ore
than 1 cc). You then do m am m ogram s (m agnification CC and M L). Filling defect(s) get
w ire localization.

• Contraindications: A ctive infection (m astitis), inability to express discharge at the tim e o f


galactogram , contrast allergy, or prior surgery to the nipple areola com plex.

494
PMMMIMMME A. R CH |TECTU
S E C RA
TIOL ND |S9T: O R T |O N ^ ——————— "W f
t

AD: We 're talking about unchecked aggression here, Dude. We are talking about distortion
o f the norm al architecture w ithout a visible m ass. T his m anifests in a few w ays, including
focal retraction, distortion o f the edge o f the parenchym a, or radiation o f the norm al thin
lines into a focal point.

A rchitectural D istortion vs Summation Artifact: T his is the prim ary differential


consideration, w ith sum m ation o f norm al vessels, ducts, and ligam ents being m uch m ore
com m on. The difference is sum m ation should N O T radiate to a central point (A D w ill).

AD - All lines radiate to a point Summation - Lines continue past each other

Surgical Scar vs Som ething Bad: Scars should progressively get lighter and harder to see.
Som e people say that in 5-10 years a benign surgical scar is often difficult to see.
Lum pectom y scars tend to stick around longer than a benign biopsy. Basically, look at the
priors; if it is a surgical scar, it better be getting less dense. If it’s increasing, you gotta stick
a needle in it.

Work Up o f AD: If you see it on a screener you will w ant to BR-0 it, and bring it back for
spot com pression view s. I f it persists ju st know you are either going to B R -4 or BR-5 it
(unless you know it’s a surgical scar). You should still ultrasound it for further
characterization (m ay help you decide betw een a 4 and a 5).

495
U ltrasound Trivia: The use o f harm onic tissue im aging can m ake it easier to see som e
lesions. Be aw are that com pound im aging can m ake you lose your posterior features,
especially w hen they are soft to start w ith - like the shadow ing o f an ILC. R em em ber, even
if you see nothing, this gets a biopsy. H arm onics can also m ake not so sim ple cysts look
sim ple by reducing superficial reverberation.

Things to K now f o r AD:

’ R adiating lines to a single point = AD


w ry *AD + C alcifications = IDC + DC1S
A *AD w ithout C alcifications = ILC
’ Even w ith no ultrasound or MRI correlate, A D gets a biopsy.
A ’N ever ever ever ever ever BR-3 an area o f AD.
’ Even if it has been there a w hile, it still needs to be w orked up.
^ ‘ R em em ber ILC can grow slowly.
’ Surgical scars should get less dense w ith tim e ... not m ore dense.

496
SECTION lO:
Ly m p h N o d e s

You found a breast cancer - now w hat? Before you m ake the patient cry, it’s tim e to stage the
disease. U ltrasound her arm pit. A bout 1 in 3 tim es you are going to find abnorm al nodes.

Unilateral vs Bilateral: This can help you if you are thinking this could be system ic. U nilateral
adenopathy should m ake you w orry about a cancer (especially if they have a cancer on that
side).

Biopsy It? Som e people will recom m end biopsy if you have the follow ing abnorm al features.

• Cortical Thickness greater than 2.3 m m (som e people say 3 m m)


• Loss o f Central Fatty Hilum - “m ost specific sign”
• Irregular O uter M argins.

Staging Trivia: Level 1 and Level 2 nodes are treated the same. Rotter nodes are treated as
Level 2. Level 3 and supraclavicular nodes are treated the same.

Special “Sneaky” Situations:

Gold Therapy: Long ago, w hen the pyram ids w ere still young, rheum atoid arthritis was
treated w ith “chrysotherapy.” W hat they can do is show you an “Aunt M innie” type picture
with very dense calcifications w ithin the node.

Snow Storm Nodes: A nother Aunt M innie look is the silicone infiltration o f a node from either
silicone leaking or rupture.

497
t S E C T I O N 1 1* ^
Ma le B r ea st a pP

T here is no m ore hum iliating w ay to die for a m an than breast cancer. T he good new s is m ale
breast cancer is uncom m on. The bad new s is that w hen it occurs it is often advanced and
invasive at the tim e o f diagnosis - Valar M orghulis.

The m ale breast does N O T have the elongated and branching ducts, or the proliferated lobules
that w om en have. This is key because m en do N O T get lobule associated pathology (lobular
carcinom a, fibroadenom a, or cysts).

G yn e co m astia: This is a non-neoplastic enlargem ent o f the epithelial and strom al elem ents
in a m an ’s breast. It occurs “physiolo gically” in adolescents, affecting about 50% o f adolescent
boys, and m en over 65. If you a re n ’t 13 or 65 it’s considered em barrassing and you should hit the
gym . If you are betw een 13-65 it’s considered pathology and associated w ith a variety o f
conditions (spironolactone, psych m eds, m arijuana, alcoholic cirrhosis, testicular cancer). There
are three patterns (nodular is the m ost com m on). Just think flam e shaped, behind nipple,
bilateral but asym m etric, and can be painful. T hings that m ake you w orry that it’s not
gynecom astia include not being behind the nipple, eccentric location, and calcification.

P attern s of G yn e co m a stia

“ Flam e S hap ed” centered behind


the nipple, radiating posterior as it
N odular
blends into the fat. B reast is often
(m ost com m on)
tender. U sually lasts less than 1
year.

R esem bles a branching tree. This


D endritic is a chronic fibrotic pattern.
U sually not tender.

M am m ographic pattern looks like


a w o m an ’s breast (diffuse
D iffuse
increase in density). You see this
G landular
in m en receiving estrogen
treatm ent.

Gynecom astia

498
P se u d o gyn e co m a stia "Bitch Tits ” - T his is an increase in the fat tissue o f the breast
(not glandular tissue). T here will N O T be a discrete palpable finding, and the m ound o f
tissue will not be concentric to the nipple.

Lipom a - A fter gynecom astia, lipom a is the second m ost com m on palpable m ass in a m an.

Male B re a st C a n c e r: It’s uncom m on in m en, and very uncom m on in younger m en


(average age is around 70). A bout 1 in 4 m ales w ith breast cancer have a BR C A m utation
(B R C A 2 is the m ore com m on). O ther risk factors include K linefelter Syndrom e, C irrhosis,
and chronic alcoholism . T he classic description is eccentric but near the nipple. It’s alm ost
alw ays an ID C -N O S type. DCIS can occur but is very rare in isolation. O n m am m ography it
looks like a breast cancer, if it w as a w o m a n ’s m am m ogram y o u ’d BR-5 it. On ultrasound
it’s the sam e thing, it looks like a BR-5. H aving said that, nodular gynecom astia can look
suspicious on ultrasound.

Things that make yo u think it's breast cancer:


* Eccentric to N ipple
* U nilateral
* A bnorm al Lymph nodes
* C alcifications
* Looks like breast cancer

Male Breast CA

Some trivia on calcifications: M icro-calcifications alone are uncom m on in m en. W hen you
see them they are less num erous, coarser, and associated w ith a m ass (25% o f m ale breast
cancers have calcifications).

Should men g e t screening m am m ogram s? H onestly, w om en sh o u ld n ’t even get them


(according to the N ew E ngland Journal o f M edicine). T his rem ains controversial, w ith the
bottom line being this: only K linefelter patients approach the screening range w ith regards to
risk.

As a point o f trivia: m ales with gyn ecom astia from gender reassignm en t on horm one
therapy are not high enough risk for screening m am m ogram s. O bviously, if they have a
palpable finding, they can get a diagnostic w ork up.

499
S E C T IO N 12: Jfe
iliiiiiiiiiiii ^ IM P L A N T S m jt

B a sic Overview : There are two types , saline and silicone. They both can rupture, but no
one really gives a shit if saline ruptures. Saline does not form a capsule, so you c a n ’t have
intracapsular rupture with saline. There is no additional im aging past m am m o for saline
rupture, and you ju st follow up with prim ary care / plastic surgeon. You can tell it’s saline
because you can see through it. For silicone you can have both intra and extra capsular rupture.
You can only see extra on a m am m ogram (can ’t see intra). Extra creates a dense “snow storm ”
appearance on US. Intra creates a “step la d d e r’’ appearance on US and a “linguine sign ” on
MRI. MRI is done with FS T2 to look at implants.

Big Points:
• You CAN have isolated intracapsular rupture.
• You CAN NO T have isolated extra (it’s alw ays with intra).
• If you see silicone in a lymph node you need to recom m end MRI to evaluate for
intracapsular rupture

Im plant Location: There are two subtypes:


• Subglandular (retromammary): Im plant behind breast tissue, anterior to pectoral
m uscle
• Subpectoral (retropectoral): Im plant betw een pectoralis m ajor and m inor m uscles

Silico n e Im plants

The body will form a shell around the foreign body (im plant), w hich allow s for both
intracapsular and extracapsular rupture (an im portant distinction from saline). A bout 25% o f
the tim e you will see calcifications around the fibrous capsule.

Things to know:
• Im plants are NOT a contraindication for a core needle biopsy
• Im plants do NO T increase the risk for cancer.

Salin e Im plants

There are also subglandular and subpectoral subtypes. You can tell the im plant is saline
because you can see through it. Im plant folds and valves can also be seen. If it ruptures no one
really cares (other than the cosm etic look). The saline is absorbed by the body, and you have a
collapsed implant. A practical point o f caution, be careful w hen perform ing a biopsy in these
patients - even a 25g FNA needle can burst a saline implant.

Trivia: Som e sources say that “physical exam ” is the test o f choice for diagnosing saline
implant rupture - this is variable depending on w hat you read / w ho you ask.

50 0
Im plant Com plications:
G enerally speaking, MR1 is the m ost accurate m odality for evaluating an im plant.

C a p su la r C o n tra ctu re : T his is the m ost com m on com p lication o f im plants. It


occurs secondary to contraction o f the fibrous capsule, and can result in a terrible cosm etic
deform ity. You see it in both silicone and saline im plants, but is m ost com m on in
subglandu lar silicone im plants. O n m am m o it looks like rounding or distortion o f the
im plant (com parisons will show progression).

Gel Bleed: Silicone m olecules can (and do) pass through the sem i-perm eable im plant
shell coating the exterior o f the surface. This does N O T m ean the im plant is ruptured. The
classic look is to show you silicone in the axillary lym ph nodes (rem em ber 1 sh o w ed a case
o f this under the lymph node section). Even w ith axillary lym ph nodes, this does N O T m ean
it has ruptured.

Rupture: As a point o f testable trivia, the num ber one risk factor for rupture is age o f the
im plant. R upture does not have to be post traum atic, it can occur spontaneously. R upture
w ith com pression m am m ography is actually rare.

* Saline: Saline rupture is usually very obvious (deflated boob). It d o e sn 't m atter all
that m uch (except cosm etically), as the saline is ju st absorbed. On m am m o, you will
see the “w added u p ” plastic w rapper. They cou ld easily w rite a question asking you
what m odality y o u n eed to see a salin e rupture. The answ er w ou ld be pla in mammo
(you don't n eed u ltrasound o r MRI).

* Silicone: This is a m ore com plicated m atter. You have tw o subtypes; isolated
intracapsular and intracapsular w ith extracapsular.

O Isolated Intracapsular: This w ill be occult on physical exam , m am m ography


and possibly ultrasound. You m ight see a stepladder on U ltrasound. M RI is
w ay m ore sensitive.

O Intracapsular with Extracapsular Rupture: This is usually obvious on


m am m ogram w ith dense silicone seen outside the capsule. The contour o f a
norm al intact im plant is sm ooth. Silicone outside the im plant can go to lym ph
nodes. On ultrasound you w ant to know the buzzw ord “snow storm ” pattern
- w hich is really ech ogen ic w ith no p osterior sh adow in g. A sneaky trick is
to show a lym ph node w ith a snow storm appearance on ultrasound. O n MRI
extracapsular silicon is T1 dark, and T2 bright. Lastly, a very im portant
concept is that you cannot have isolated extracapsular rupture. I f i t ’s
extracapsular, then it s also intracapsular.

501
R ad ia l Folds - T h e M im ic o f Rupture:

Radial folds are the norm al in-foldings o f the elastom er shell. They are the prim ary m im ic
for the linguine sign o f intracapsular rupture. To tell them apart ask y o u rse lf “do the fo ld s
connect with the periph ery o f the im plant? ” Radial folds should alw ays do this (linguine
does not).

Silicone Implant Rupture Summary

In tra c a p s u la r R u p tu re :
- R e m e m b e r th e “c a p s u le ” is n o t p a rt
o f th e im p la n t. It’s th e fib ro u s c o a t
y o u r b o d y m a k e s a ro u n d th e im p la n t
(th e o u te r b la c k line in m y d ia g ra m ).
- S ilic o n e ca n ru p tu re th ro u g h th e
sh e ll o f th e im p la n t, b u t s ta y s
c o n fin e d in s id e th e fib ro u s c o a t - th is
is in tra -c a p s u la r ru p tu re .
- T h e c la s s ic sig n is th e flo a tin g
“ lin g u in e ” - a s in th is c a se .

E x tra & In tra c a p s u la r:


- T h is is w h e n th e ru p tu re g o e s
th ro u g h th e “c a p s u le ” (th e th in g y o u r
b o d y m a d e ).
- You c a n N O T h a ve is o la te d e x tra
c a p s u la r s ilic o n e . It h a s to m a k e it
th ro u g h th e im p la n t s h e ll first.
- S ilic o n e o u ts id e th e c a p s u le ca n
c re a te a “s n o w s to rm ” lo o k on
u ltra s o u n d . It c a n a lso in filtra te lym p h
Snow Storm Node n o d e s and d o th e sa m e (s n o w sto rm
n o d e s ). R e m e m b e r g e l b le e d can
a ls o g iv e you a n o d e like th is.

R a d ia l F o ld s
' G u y s like s q u is h y b o o b s . T h e b ig g e r
a n d th e s q u is h ie r th e better.
" T h e re fo re , im p la n ts a re n o t b o u n d
tig h tly - so th e y ca n be sq u ish y.
" B e c a u s e th e y a re lo o s e ly b o u n d th e
s h e ll in -fo ld s c re a te s ra d ia l fo ld s
" T h e fo ld s a lw a y s a tta c h to th e
s h e ll*
' T h e fo ld s a re th ic k e r th a n a ru p tu re ,
b e c a u s e th e y re p re s e n t b o th la ye rs.

502
— * p o s t o ^ / P o s r t h e r a p y — f l

Reduction Mammoplasty and Mastopexy

R eduction M am m oplasty - Yes, there


is actually a subpopulation o f w om en
who w ant S M A L L E R breasts. I know,
it sounds im possible to believe , but
M am m oplasty is actually done to
reduce breast size. 1 can only pray that
the sadistic bastard w ho developed this
procedure has received appropriate
punishm ent (in this life or the next).

M astopexy - This is a “breast lift,”


Essentially, ju st a rem oval o f skin.
W omen get this done to address floppy,
saggy, pancake, or “pto tic” boobs. Typical Ch an ge s from Mammoplasty

N orm al F indings P ost M astopexy:


• S w irled A ppearance Affecting
Inferior Breast
• Fat Necrosis / OH Cysts
• Isolated Islands o f B reast Tissue

K eyhole Incision - T his is done for


M am m oplasty Keyhole Incision Mastopexy Keyhole Incision
both m am m oplasty and m astopexy,
creating a “ sw irled” appearance in the
inferior aspect o f the M LO.

503
Surgical Biopsy / Radiation
Term inology:
* L u m pectom y - Surgical R em oval o f C ancer (palpable or not)
* E xcisional Biopsy - Surgical R em oval o f Entire Lesion
* Incisional B iopsy - Surgical B iopsy o f a Portion o f the Lesion

Post B io p sy C h a n g e s:

The first post operative m am m ogram is usually obtained around 6-12 m onths after biopsy. The
key is that distortion and scarring are w orst on this film , and should progressively
im prove. On ultrasound, scars are supposed to be thin and linear. If they show you a focal
m ass like thickening in the scar - y o u ’ve gotta call that suspicious for local recurrence.

Fat necrosis and benign dystrophic calcifications m ay evolve over the first year or two, and are
the m ajor m im ics o f recurrence. Fat necrosis can be show n on M R (T1 / T2 bright, and then fat
sat drops it out).

R e cu rre n ce I R e sid u al D ise a se :

Numerical Trivia: Local recurrence occurs 6-8% o f the tim e when w om en have breast
conserving therapy. The peak tim e for recurrence is 4 years (m ost occur betw een 1-7).
W ithout radiation local recurrence is closer to 35% . Tum ors that recur early (< 3 years)
typically occur in the original tum or bed. Those that occur later are m ore likely to be in a
different location than the original primary.

What gets recurrent disease ? Risk o f recurrence is highest in the prem enopausal wom an
(think about them having an underlying genetic issue). O ther risks include: having an
extensive inarticulate com ponent, a tum or with vascular invasion, m ulti centric tum ors,
positive surgical m argins, or a tum or that was not adequately treated the first go around.

Residual Calcs: Residual calcifications are not good. Supposedly, residual calcifications near
or in the lum pectom y bed correlates with a local recurrence rate o f 60%.

New Calcs: W hen it does reoccur, som ething like 75% o f DCIS will com e back as
calcifications (no surprise). The testable pearl is the benign calcifications tend to occur early
(around 2 years), vs the cancer ones which com e back around 4 years.

Sentinel Node Failure: Sentinel node biopsy w orks about 95% o f the tim e (doesn’t work 5% o f
the time). So about 5 tim es in 100 you are going to have a negative node biopsy that presents
later with an abnorm al arm pit node.

Tissue Flap: The cancer is not going to start in the belly fat / m uscle. The cancer is going to
come from either the residual breast tissue or along the skin scar line. Screening o f the flaps is
controversial - with som e saying it’s not necessary. The need for screening o f tissue flaps is not
going to be asked. If you get asked anything it’s “w here the recurrence is com ing from / going
to be?”

504
S p e c im e n R a d io g ra p h y

If the path report says “close m argins” or “positive


m argins,” there is a very high chance you are
going to have cancer still in the breast. If you are
show n a specim en radiograph, there are tw o things
you need to look at in real life and on m ultiple
choice: (1) is the m ass / calcifications on the
sam ple, and (2) is the m ass / calcifications near
the edge or touching the edge. I f the m ass is at the
edge, the chance o f incom plete excision is goin g to
be near 80%. The "next step ” w ou ld be to call the
surgeon in the OR an d tell him /her that. Specimen Radiograph - Cancer at the Margin
-High chance of positive margin

Post R a d ia tio n C h an g es:

Practical Point (th e before p ictu re): The pre-radiation m am m ogram is very im portant. If
you can identify residual disease on it, the patient has m any m ore treatm ent options. If you
discover the residual disease after the radiation therapy has been given, y o u ’ve forced the
patient to undergo m astectom y.

R adiation C hanges: You are going to see skin thickening and trabecular thickening. This is
norm al post radiation, and should peak on the first post-R T m am m ogram .

This w ou ld be a classic testable scenario:


Film 1 Post RT: You see skin thickening / trabecular thickening
• Film 2: Skin thickening / trabecular thickening is better
Film 3: Skin thickening / trabecular thickening is w orse * - this is recurrent disease
(m aybe inflam m atory breast CA).

S e c o n d a ry A n g io s a rc o m a

The prim ary type is so rare 1 w o n ’t even m ention it. The secondary type is seen after
breast conservation therapy / radiation therapy. It takes around 6 years post radiation
therapy to develop one o f these things. C linically the classic presentation is “ red plaques
or skin nodules.” T he challenge w ith these is that the skin thickening due to the cancer is
often confused w ith post therapy skin thickening.

505
S ta g in g / S u rg ic a l P lan n in g

Breast C ancer Staging: The staging is based on size from T 1-T3, then invasion for T4.
T l = < 2 cm.
T2 = 2-5 cm
T3 = > 5 cm
T4 = “ A ny size” w ith chest wall fixation, skin involvem ent, or inflam m atory breast CA.
*Remember that Pagets is N O T T4.

Trivia: A xillary Status is the m ost im portant p red icto r o f o verall survival in breast cancer

Trivia: M elanom a is the m ost common tum or to m et to the breast.

The contraindications for breast conservation are high yield.

C o n tra in d ica tio n s for B re a st C o n se rvatio n

Inflam m atory Cancer,


Large C ancer Size R elative to Breast
M ulti-centric (m ultiple quadrants)
Prior R adiation Therapy, to the sam e breast
C ontraindication to R adiation T herapy (collagen-vascular disease).

506
* ih @ e @ e ie ie @
SECTION
B r ea st
14:
MRI

Breast M RI can be used for several reasons: High risk screening, extent o f disease (know n
cancer), axillary m ets w ith unknow n prim ary, diagnostic dilem m as, and possible silicone
im plant rupture. T he big reason is for high risk screening.

I'll ju st briefly go over how it’s done, and how it’s read.

You need a special breast coil and table set up to m ake it w ork. T he patient lies belly dow n
w ith her breasts hanging through holes in the table. You have to position them correctly
otherw ise they get artifact from their breasts rubbing on the coil. Basic sequences are going
to include a T2, and pre and post dynam ic (post contrast) fat saturated T l . R em em ber the
breast is a bag o f fat - so fat sat is very im portant. D ynam ic im aging is done to generate
w ash out curves (sim ilar to prostate M RI).

My basic algorithm fo r reading them is to:

(1) Look at the background uptake. I use this to set m y sensitivity w hen 1 com pare it to prior
studies. Ideally you used the sam e kind o f contrast, and im aged at the sam e tim e o f the
m onth. As I ’ll m ention below , horm one changes w ith fem ale cycles cause changes in
how m uch contrast gets taken up (less early, and m ore later).

(2) I look for m asses or little dots (foci). M IPS (m axim um intensity projections) are helpful
ju st like looking for a lung nodule. I f I see a m ass or dot I try and characterize it - first by
seeing if I can m ake it T2 bright. M ost T2 bright things are benign (lym ph nodes, cysts,
fibroadenom a). If it’s not T2 bright, I look at the features - is it a m ass? is it spiculated,
etc? These features are m ore im portant than anything else. Is it new ? N ipple
enhancem ent is ok - d o n ’t be a dum b ass and call it Pagets.

(3) Finally I’ll look at the w ash out curve, but honestly I ’ve m ade up m y m ind before I even
look at that. I will never let a benign curve b ack m e o ff suspicious m orphology.

(4) I deal w ith the findings sim ilar to m am m o. N ew m asses get B R -4 or BR -5. N M L E (non­
m ass enhancem ent) gets B R -4 ’d if new. T2 bright stu ff for the m ost part (there is one
exception o f m ucinous cancer) gets B R -2 ’d. A nything w ith a 4 or a 5 gets biopsy - via
M R guided stereo. I never pussy foot out and B R -0 som ething on M RI - unless it's a
technical problem (exam ple inadequate fat sat).

507
Who gets a screening M R I ?

• P eople with a lifetim e risk greater than 20-25%


• Includes p e o p le who g o t 20 G y o f radiation to the chest as a child

How do you estimate this risk, to decide who is 20-25%?

• You use one o f the risk m odels that includes fa m ily history (N O T the G ail m odel). I f the
question is which o f the fo llo w in g is N ot one to use ? The answ er is Gail. I f the
question is which o f the fo llo w in g do you pick? I 'd chose Tyrer-Cuzick, it s pro b a b ly the
best one out now.

P arenchym a Enhancem ent:

• Is it norm al ? - Yes

• Where is it m ost common ? - Posterior Breast in the upper outer quadrant, during the later
part o f the m enstrual cycle (luteal phase - day 14-28)
• H ow do you reduce it? - Do the MRI during the first part o f the m enstrual cycle (day 7-14).

• What does Tamoxifen do? - Tam oxifen will decrease background parenchym a uptake.
T h e n it c a u se s a re b o u n d .

Foci:

• H ow is it defined? R ound or oval, circum scribed, and less th a n 5 m m ,

• Are they high risk? U sually not. U sually they are benign (2-3% have a chance o f being
a bad boy).

• What w ou ld make yo u biopsy one? Seem ed different than the rest, ill-defined borders,
or su sp ic io u s e n h a n c e m e n t.

• Can you BI-RADs 3 one? If you have a solitary focus (< 5m m ) w ith persistent kinetics
on a baseline exam - you can B I-R A D S 3 it.

N M E (N o n -M a ss E n h a n c e m e n t):

• What is NM E ? It’s not a m ass - but m ore like a cloud or clum p o f tissue enhancem ent.

• What are the distributions ? Segm ental (triangular blob pointing at the nipple, indicates a
single branch), Regional (a bigger triangle), and D iffuse (sorta all over the place).

• Which one is more suspicious - hom ogenous or heterogeneous enhancem ent o f N M E ?


H eterogeneous is m uch m ore suspicious.

508
M asses:

• These are defined as being 5 m m or larger. T hey have definable vocabulary for their
features (round, oval, indistinct, e tc ...)
• When are these bad? T hey are bad w hen you call them bad w ords. Irregular shape,
speculated m argins, heterogeneous enhancem ent, or rim enhancem ent. O nce you say
those w ords you are going to have to biopsy them , because m orp h ology trum ps
kinetics. It d o e sn ’t m atter w hat the kinetics show s, you m ust biopsy suspicious
m orphology.

• When is kinetics helpful? W hen you are on the fence. If you have benign m orphology
and you have suspicious kinetics - you probably are going to need to biopsy that also.

K in e tics:

• Breast kinetics are perform ed in tw o portions:

o (1) Initial upslope phase that occurs over the first 2 m inutes. T his is graded as
slow, m edium , or rapid (fast),

o (2) T he w ashout portion w hich is recorded som etim e betw een 2 m inutes and 6
m inutes (around about). T hese are graded as either continued rise “type 1” ,
plateau “type 2 ” , or rapid w ashout “ type 3” .

Persistent “1”

509
C la s s ic Lo o k s:

Fibroadenom a: These things are classically T2 bright, round, w ith “ non-enhancing


septa” , and a type 1 curve.

D C IS: C lum ped, ductal, linear, or segm ental non-m ass enhancem ent. K inetics are
typically not helpful for DCIS.

I DC: Spiculated, irregular shaped m asses, w ith heterogeneous enhancem ent and a
type 3 curve.

ILC: D o esn ’t alw ays show enhancem ent.


%
V
1 .,
m ]

1
Fibroadenoma DCIS
-Non-enhancing Septations -Segmental NME

T 2 Bright T h in g s:

* U sually T2 Bright = Benign.


Things that are T2 B right include: Cysts, Lymph nodes, fat necrosis, Fibroadenom a.
* The exceptions (anytim e I sa y the w ord "except ” y o u n eed to think high yield!):
C olloid C ancer, and M ucinous C ancer can be T2 bright.

510
Pure Trivia:

* I f you have a patient with known breast CA, how often do yo u fin d a contralateral breast
CA? - A nsw er is 0.1-2% by m am m ogram , and 3-5% by M RI.

* Never BR-0 an M RI case. This is as m uch w orkup as you are going to get, so ju st call it
benign or biopsy it. You can actually B R -0 som ething if you really w ant to prevent a
biopsy - possible lym ph node - US and m am m o to confirm benign sorta situation. This
is still kinda w eak. For the purpose o f m ultiple choice, think tw ice before you B R -0 a
M RI case.

* Spiculated margins = 80% m alignancy. T his is the sin gle m ost predictive feature o f
m alignancy.

511
Estrogen: The m ore exposure to estrogen, the higher your risk
E strogen R elated Risks
o f breast cancer. A nything that prolongs this exposure is said
to increase risk. For exam ple, an early age to begin Early Menstruation
m enstruating or a late age to have m enopause. Horm one Late Menopause
replacem ent therapy with estrogen alone obviously increases Late age of first
exposure. Early m aturation o f lobules, which can be achieved pregnancy / or no kids.
Being Fat
by getting pregnant young, reduces your risk. Being fat Being a Drunk
increases estrogen exposure (m ore arom atase = more Hormone Replacement
estrogen). Being a drunk increases estrogen exposure - via (with estrogen)
m essing with its norm al breakdow n in the liver.

High Risk Lesions: Any o f the high risk lesions (ADH, ALH, LCIS, Radial Scar, Papillom a)
are associated with an increased risk. These are discussed m ore in detail later in the chapter.

Density: Density is considered a “m edium risk,” and is “dose dependent” with the denser you
are the m ore risk you have.

Chest Wall Radiation: Chest wall radiation (usually seen in lym phom a patients) is a big risk
factor, especially at a young age. The risk is supposed to peak around 15 years post treatm ent.
If the child had m ore than 20 Gy to the chest she is going to qualify for an annual
screening MRI - at age 25 or 8 years post exposure (whichever is later).

Relatives with Cancer: A first degree relative with breast cancer increases your lifetim e risk
from 8% to 13%. Two first degree relatives increases your risk to 21 %.

A ctu al M utations:

C hrom osom e 17. M ore com m on than type 2. Increased risk


BR CA 1
for breast, ovary, and various GI cancers.
C hrom osom e 13. M ale carriers have a higher risk w ith 2.
BR C A 2
Increased risk for breast, ovary, and various GI cancers.
T heir p53 does N O T w ork, and they are high risk for all
Li Fraum eni
kinds o f rare cancers.
Risk for breast cancer, follicular thyroid cancer, endom etrial
C ow den Syndrom e
cancer, and L h erm itte-D u clos (a brain ham artom a).
B annayan-R iley R uvalcaba A ssociated w ith developm ental disorders at a young age.
NF-1 “M oderate R isk” o f breast cancer

512
F e m a le C a n c e r R is k S yn d ro m es:

H e r e d it a r y S y n d ro m e s

T rip le N e g a tiv e (e s tro g e n ,


B re a s t C a n c e r (risk
H e re d ita ry B re a s t an d p ro g e s te ro n e , H ER 2
72% )
O v a ria n C a n c e r n e g a tiv e ) - ID C M e d u lla ry
O v a ria n C a n c e r (risk F a llo p ia n T ube,
S y n d ro m e -B R C A 1 S u b ty p e is th e m o s t
44% ) P a n c re a s , C o lo n
c o m m o n b re a s t C A
C a n c e rs A ls o a t
B re a s t C a n c e r (risk in c re a s e d ris k
H e re d ita ry B re a s t an d
69% )
O v a ria n C a n c e r
O v a ria n C a n c e r (risk
S y n d ro m e -B R C A 2
17% )

T h y ro id C a n c e r
(u s u a lly p a p illa ry).
A ls o in c re a s e d ris k
B re a s t C A is th e m o s t
o f v a rio u s b e n ig n
H a m a rto m a s in c o m m o n m a lig n a n c y (risk
th y ro id d is e a s e .
m u ltip le o rg a n s a n d 7 7 % ).
A n n u a l th y ro id
g ro s s fa c ia l and
C o w d e n S y n d ro m e s c re e n in g is
m o u th b u m p s In c re a s e d ris k o f o th e r
ty p ic a lly a d v is e d .
p la g u e th e s e b re a s t c o n d itio n s
u n fo rtu n a te s o u ls (fib ro a d e n o m a s , A D H ,
L h e rm itte -
fib ro c y s tic c h a n g e s )
D u c lo s (d y s p la s tic
g a n g lio c y to m a o f
th e c e re b e llu m )

H e re d ita ry D iffu se P ro p h y la c tic


D iffu s e G a s tric L o b u la r B re a s t C a n c e r
G a s tric C a n c e r G a s tre c to m y is
C a n c e r R isk ~ 7 0 % R isk - 4 0 %
S y n d ro m e re c o m m e n d e d .

B re a s t C a n c e rs are
L i-F ra u m e n i S y n d ro m e C a n c e rs lite ra lly
u s u a lly se e n in 3 0 s -4 0 s
(b a d p53) e v e ry w h e re .
w ith h ig h g ra d e .

513
B re a s t C a n c e r R isk M odels:

T here are several risk m odels, w hich have pros/cons and differences. I apologize in advance
for even suggesting you learn about th e s e .... but it ju st seem s testable to m e. I ’m so rry ...

Focuses on personal Doesn’t use genetics


Oldest and most
risk factors, biopsy o f (it’s too old school).
Gail Model validated breast cancer
ADH, and family Only validated in
risk model
history African Americans.
Do NOT include
Claus, BODICEA, and
Focus on genetics personal risk or breast
BRCApro
related risk factors.
Focus on personal
risk, biopsy with Does NOT include
Tyrer-Cuzick "Most Comprehensive ”
ADH or LC1S, family breast density.
history

High Yield Take H o m e Points R eg ard in g Risk:

* A nything that gets you m ore estrogen increases your risk

* BR CA 1 is m ore com m on than BRCA 2 (in w om en).

* M en w ith BR CA 2 get m ore cancer than m en w ith BR CA 1.

* Breast D ensity is an independent risk factor (denser the breast, the m ore the risk)

* 20 Gy o f R adiation to your chest as a kid buys you a screening M RI - at 25 or 8


years after exposure (*whichever is later)

C ow den Syndrom e - Bow el H am artom a, Follicular T hyroid Cancer, Lherm itte-


Duclos, and Breast C ancer

* All current risk m odels underestim ate life tim e risk.

* Tyrer C uzick is the m ost com prehensive risk m odel, but does not include breast
density.

* Exercise (probably more like not being fat) reduces the risk o f breast cancer

Tam oxifen and R aloxifene (SER M s) reduce incidence o f E R /PR positive cancers.
M ortality m ay not actual be reduced {sound fam iliar?).

514
S c re e n in g C o n tro v e rs y

A red glow burst suddenly across the enchanted sky as the dark lord o f statistics G ilbert
W elch published his now infam ous and devious w ork - "Effect o f three decades o f
screening m amm ography on breast-cancer incidence, ” in the unscrupulous N ew England
Journal o f M edicine.

“/ can make things m ove without touching them. I can make animals do what I want
without training them. 1 can make bad things happen to people who are mean to me. I can
make them hurt, i f I want... ” - G ilbert W elch w hen asked about his thoughts on screening
m am m ography.

This loathsom e, m aleficent, and repugnant study (along w ith several other large heavily
pow ered studies) have brought into question the practice o f screening m am m ogram s. I
highly recom m end you read these “ despicable ” papers, but please w ait till after the exam ,
because the people w ho w rite m ultiple choice questions about m am m ography are
definitely not the sam e people w ho w rote these papers.

For the purpose o f m ultiple choice tests, screening m am m ography saves lots o f lives, you
should buy pink ribbons, and low grade D C IS in a 95 year old needs a surgical consult.

Bleyer, Archie, and H. Gilbert Welch. "Effect o f three decades o f screening


m amm ography on breast-cancer incidence. " N ew England Journal o f M edicine 367.21
(2012): 1998-2005.

Miller, Anthony B., et al. "Twenty fiv e yea r follo w -u p fo r breast cancer incidence and
mortality o f the Canadian N ational Breast Screening Study: random ised screening tria l."
BMJ: British M edical Journal 348 (2014).

515
S E C T I O N 16:
A C R A p p r o p r i a t e n e s s

These all m ake great “next step questions.”

R em em ber scoring is 1-9, w ith 9 being the m ost appropriate and 1 being the least.

Breast Cancer Screening:

Variant 1: High Risk W om en. BRCA (plus untested first degree relatives), History o f Chest
Radiation, Risk Model Showing 20% or greater lifetime risk

• M am m o (H ighly A p propriate “9”):

• Beginning at age 25-30 or 10 years before age o f first-degree relative with breast cancer

• 8 years after radiation therapy, but not before age of 25.

• Mammography + MRI ? They are complementary examinations, both should be performed.

• T om osynthesis (H ighly A p propriate “9 ”):

• Beginning at age 25-30 or 10 years before age o f first-degree relative with breast cancer

• 8 years after radiation therapy, but not before age o f 25.

• Mammography + MRI ? They are complementary examinations, both should be performed.

• M RI (H ighly A ppropriate “9”):

• Mammography + MRI ? They are complementary examinations, both should be performed.

Variant 2: M ed iu m R isk W om en. Women with person history o f Breast CA, lobular
hyperplasia, Atypical Ductal Hyperplasia, or Risk Model Showing 15-20% life time risk

• Mammo and Tomo are “9s”

• MRI is a 7. Mammography + MRI ? They are complementary examinations, MRI should NOT replace
mammography.

Variant 3: A verage Risk W om en. Women with < 15 % Lifetime Risk

• Mammo and Tomo are “9s”

• MRI is a “3” which means it is NOT appropriate.

516
Screening for Transgender Women
Scre e n in g Annual M am m ogram IF:
• Past or C urrent H orm ones (E strogen & Progestin for > 5 years)
• > 50 years old

Trivia: BM I > 35 = increases Risk

Screening for Transgender Men


S cre e n in g Annual M am m ogram IF:
• They still have breast tissue (even if they had a reduction m am m oplasty)

Breast Pain:

Variant 1: C yclical, Unilateral or Bilateral. Age < 40.

• No imaging is appropriate.

• Ultrasound is the least inappropriate and it’s rated at a “2”

Variant 2: C yclical, Unilateral or Bilateral. Age > 40.

• No imaging is appropriate.

• Ultrasound, Mammo, and Tomo are the least inappropriate and all rated at a “2”

Variant 3: N O N -C yclical, U nilateral or Bilateral. Age < 40.

• Ultrasound Might Be Appropriate and is rated as a “5.”

Stage 1 Breast CA - Initial W orkup and Surveillance (No Symptoms)

Newly Diagnosed - rule out mets to the bones, chest, liver, and/or brain

• No imaging is appropriate. (CT, MR1, PET etc... not indicated with initial stage 1)

Surveillance / Rule O ut Local R ecurrence

• Diagnostic Mammo or Tomo is Appropriate and is rated as a “9.”

• Ultrasound might be appropriate and is rated as a “5.”

• MRI might be appropriate and is rated as a “5.”

517
Symptom atic M ale Breast:

Variant 1: A n y A ge with Physical E xam and H istory C on sisten t with


G y n ecom astia or P seu d og yn eco m a stia (bitch tits).

• No imaging is appropriate.

Variant 2: Younger than 25 years old w ith in d eterm in ate palpable.

• Ultrasound is Appropriate and is rated as an “8.”

• Mammo is in the “May Be Appropriate” category as a “5.” 1 would only do this if the ultrasound
doesn’t answer your question. ** Page 441, Scenario 4A - has suggested multiple choice strategy.

Variant 3: O ld er than 25 years old with in d eterm in a te palpable.

• Mammo is Appropriate and is rated as an “8”

• Ultrasound is in the “May Be Appropriate” category as a “5.”

Variant 4: O ld er than 25 years old with in d eterm in a te palpable. The


m a m m o g ra m w as in d eterm in ate or suspicious.

• Ultrasound is Appropriate and is rated as a “9.”

Variant 5: Physical exam is highly con cern in g for cancer. D ude has an
ulcerative m ass, axillary nodes, nipple retraction, e tc ...

• Mammo is Appropriate and is rated as an “9”

• Ultrasound is Appropriate and is rated as an “8” - to stage the breast and axilla just like a female breast
CA workup.

518
Random Situations - First/Next Step:

W om en > 40 w ith Palpable

• Mammo is Appropriate and is rated as a “9.”

W om en > 40 with M a m m o S uspicious for CA

• Ultrasound is Appropriate and is rated as a “9.”

W om en > 40 with M a m m o findings o f a L ip o m a at the Site o f a Palpable.

• No additional imaging is appropriate.

W om en > 40 w ith Palpable Findings and a N eg ativ e M a m m o

• Ultrasound is Appropriate and is rated as a “9.”

W om en < 30 Initial Evaluation

• Ultrasound is Appropriate and is rated as a “9.”

W om en 30-39 Initial E valuation

• Ultrasound is Appropriate and is rated as a “8.”

• Mammo is Appropriate and is rated as a “8.”

W om en < 30 U ltrasou n d is S u sp iciou s for C A

• Core Biopsy is Appropriate and is rated as a “9.”

• Mammo is Appropriate and is rated as a “8.”

W om en < 30 U ltrasound is N egative

• No imaging is appropriate.

W om en < 30 U ltrasou n d has a B9 finding (like a cyst)

• No imaging is appropriate.

W om en < 30 U ltrasound is BR3 able - ex a m p le fib ro ad en o m a

• Short interval follow up - usually Q 6 m onths x 2 years.

519
a SECTION 17. [MlMIMIMIMTMlMfM
P ro ce d u re s jr

The m ost com m on procedures are going to be ultrasound guided and stereotactic biopsy o f
m asses and calcification. I'll try and touch on the testable points.

U ltrasound:

• O verall ultrasound is faster and easier than stereo. If you can see the m ass under US - you
should do the biopsy under US.

• U sually a 14 gauge autom atic spring loaded device is used for m asses

• You should put the m ass on the far side o f the US screen - lets you see the length o f the
needle better

• Ideally 4 things should line up during the biopsy: the lesion, the transducer, the skin nick,
and the biopsy needle

• The needle angle should be parallel to the chest w all (pneum othorax is an em barrassing
com plication o f a breast biopsy)

• A nesthetic should be placed right up to but not into the lesion (especially w hen the lesions
is sm all).

• You should try and biopsy the deeper part o f a lesion first. If you obscure it from bleeding
at least you can still get the superficial part.

• If you have tw o lesions to biopsy, try and hit the sm aller one first. If the bigger one bleeds
it m ay obscure the sm aller one — it’s less likely the other w ay around.

• If you have a solid and cystic lesion - you should biopsy the solid part.

• A bout 90% o f the tim e you can m ake a diagnosis o ff 1 or 2 passes (though m ost texts still
recom m end doing 5).

Next Step Scenario - Like an idiot you injected a bunch o f air around the m ass, w hile you w ere
trying to give lidocaine. N ow you c a n ’t see the m ass. W hat do you do? You have to reschedule.
D o n ’t try to biopsy it blind.

520
A xilla:

• W hen you biopsy an axillary lym ph node you should target the n o d e ’s cortex.

• Core biopsy is preferred over FN A if you have no clue w hat it is. If you have know n breast
cancer and you are nearly certain you are dealing w ith a m et - FN A w orks fine.

S p e c ia l S ce n a rio - T h e C y st A sp iratio n

• Indications - A nxiety, pain, uncertain diagnosis.

• Size is N O T an indication for aspiration

• Cysts recur about 70% o f the tim e (this drops to around 15% if you inject air after you
aspirate).

The Hypoechoic Mass vs Dirty Cyst Scenario


-Classic “Next Steps” -

Next Step Scenario # / - You suspect a hypoechoic m ass is a debris filled cyst rather than
a solid m a s s ... but you a re n ’t totally sure. What should yo u do fir s t ? A spirate it.

Next Step Scenario #2 - Sam e hypoechoic m ass vs cyst - you aspirate it and you get non-
bloody fluid. You also notice the lesion disappeared. What do yo u do ? You should pitch
it, no need for cytology. You are done.

Next Step Scenario #3 - Sam e hypoechoic m ass vs cyst - you aspirate it and you get
bloody fluid. You also notice the lesion disappeared. What do yo u do ? Send it to
cytology and then place a clip.

Next Step Scenario #4 - Sam e hypoechoic m ass vs cyst - you aspirate it and you get
purulent “poop like” fluid. T he fluid sm ells like a zom bie farted. You also notice the
lesion disappeared. What do yo u do ? Send it to the m icrobiology lab for culture and
sensitivity.

N ext Step Scenario #5 - Sam e hypoechoic m ass vs cyst - you aspirate it and you get fluid.
You also notice the lesion does N O T disappear. What do yo u do ? Proceed to core biopsy
o f the residual solid m ass.

521
S te re o ta c tic B io p sy (using a mammogram to localize and target the lesion).

• This is the preferred m ove for calcifications. Typically the specim en


is x-rayed after the sam ple to confirm there are calcifications w ithin Gauge Size vs
Samples:
the biopsied tissue.
- 10-11 Gauge
• Vacuum assisted devices are typically used for calcifications. Needle = 12
Samples
• The biopsy is perform ed in com pression - w ith slightly less pressure
than a norm al m am m ogram . C om pressibility o f the breast tissue can - 7-9 Gauge
Needle = 4
N O T be less than 2-3cm (som e texts say 28 m m ). O therw ise you risk
Samples
throw ing the needle through the other side o f the breast into the
digital receptor. This is called a “negative stroke m argin.”

• Next Step Scenario: What i f the breast compresses too sm all ( < 2 0 mm) ? You should do a
w ire localization for excisional biopsy.

• A m arker (tiny piece o f m etal) should be placed after each biopsy. Clip m igration can occur
(accordion effect). You will need a m am m ogram in the orthogonal view to evaluate for this
post placem ent.

• QC “ Localization and A ccuracy Test” to verify system alignm ent and perform ance is
perform ed D aily before patient exams.

522
M Q S A

The U.S. Food and Drug A dm inistration M am m ography Q uality Standards A ct (M Q SA ) - yes that
is a real thing - dem ands a m edical audit and outcom e analysis be perform ed once a year. You are
forced to follow up patients with positive m am m os, and correlate w ith biopsy pathology results (so
you can see how m uch benign disease you biopsy and how m uch fear / anxiety you generate). You
have to grade the biopsy with the risk category (you c a n ’t accept benign results with a BR-5).

MQSA and Other Crap they could ask:

• 3 m onths o f m am m ography is required during residency training


• The recall rate should be less than 10%
M am m ography facilities are required to provide patients with w ritten results o f their
m am m ogram s in language that is easy to understand. A lso know n as a "lay report,” and
m ust be given w ithin 30 days o f the study.
A consum er com plaint m echanism is required to be established in m am m ography
facilities to provide patients with a process for addressing their concerns.
Patients can obtain their original m am m ogram s, not copies, w hen they are needed.
• For cases in w hich a facility’s m am m ogram s are determ ined to be substandard and a
risk to public health, facilities will notify the patients and their doctors and suggest an
appropriate plan o f action.
The “Interpreting Physician” is ultim ately responsible for the Q uality Control program .
• The required resolution o f line pairs is 13 lp/m m in the anode to cathode direction and
11 line pair / m m in the left right direction
• To m ake it pass im age quality; m ust show 4 fibers, 3 m icrocalcification clusters, and 3
m asses, plus “acceptable artifacts” .
• The dose phantom is 50% glandularity, 4.2 cm thick, and is supposed to have a dose
less than 3 m Gy per image (+ grid).
D on’t get it tw isted; there are no patient dose lim its in m am m ography, only a phantom
dose. A dense breast can result in a higher patient dose, w hich could easily exceed 3
mGy/view.
Typical patient and phantom doses are about 2 m G y per view, or 8 m G y for a bilateral
two view (Left CC + M LO, Right CC + M LO ) screening exam ination.
• The typical (average) com pressed breast is 6 cm, glandularity o f 15 to 20% .
Digital system s generally uses higher beam qualities w hich results in low er doses;
Digital m am m ography does not use fixed dose (screen-film ); can use as m uch (or little)
radiation as deem ed appropriate.
M ale Residents m ust urinate in the sitting position while on the m am m ography service
(standing urination is not allow ed per M QSA ).

523
... , g 1
S p e c ific Q A T a s k s
' ^ ::::::: ■ . — I .:.::./ ^ ..... ' ' .. . ‘ " ” . .

Processor QC D aily

D arkroom C leanliness D aily

V iew box C onditions W eekly

Phantom E valuation W eekly

Repeat A nalysis Q uarterly

C om pression Test Sem i-A nnually

D arkroom Fog Sem i-A nnually

Screen-Film C ontrast Sem i-A nnually

Evil O verlord behind M Q SA ? FDA

A ppropriate T a rg e t R an ge for M edical Audit

Recall Rate 5-7%

C ancers/ 1000 Screened 3-8


1 ^ ... -J ■ :
T h e P riv ile g e to Read a M am m ogram

D uring the last tw o years o f training you have to read 240


Form al T raining R equirem ent 3 m onths

D ocum ented H ours o f Education 60

M ale Resident Mam m ography Specific Trivia


©■■Tore©*li©iii m a y v a r y p©r i n s titu tio n

M ale residents m ay urinate in the follow ing


Seated only. Standing urination is prohibited.
position(s) w hile on the m am m o service ?

The penis o f the m ale resident should be in


Tucked posterior and secured w ith tape*
w hat orientation w hile on the m am m o
(Glue - i f perform ing a 3 month fo cu s time)
service ?

524
S E C T I O N 19:
S c e n a r i o / R e v i e w

As I prom ised in the first pages o f the chapter, I w ant to finish by rolling through som e
scenarios. This is m ainly to dem onstrate the w ork flow process and how you handle “next
step” type questions.

Scenario 1: A 40 year old woman presents for her baseline screening mammogram. You have the
great pleasure o f reading it. While conducting your normal reading pattern you notice the posterior
nipple line is 9cm on the MLO, but only 6 cm on the CC.

What do you do ? - The would be a technical call back

What BR ? - This is a BR-0

Scenario 2a: A 50 year old woman presents for her annual screening mammogram. You have the
great pleasure o f reading it. You notice a mass on two views in the lateral left breast.

What BR ? - This is also BR-0.

Next Step ? Return for diagnostic mammogram, including spot compression views and likely an
ultrasound.

Scenario 2b: Same patient returns for the diagnostic mammogram. You can clearly see the mass in
two views.

Next Step ? Ultrasound to further characterize

Scenario 2c: You put her in ultrasound and see an obvious shadowing angry, pissed off, mass that is
ulcerating through the skin e tc .... It 100% for sure cancer.

Next Step? You need to stage. Scan the rest of the breast for multi-focal masses, AND scan the axilla
for pathologic nodes.

Scenario 3a: A 50 year old woman presents for her annual screening mammogram. You have the
great pleasure o f reading it. You notice what looks like a mass in the CC view only (can’t find it in
the MLO). You call it an “asymmetry” because you can only see it in one view.

Next Step ? BR-0, and bring it back with spot compression views

525
Scenario 3b: Same patient returns for the diagnostic mammogram. After the paddle is applied there
does not appear to be any mass. It just looks like normal breast tissue. You look at prior imaging and
it looks pretty similar to the priors now.

Next Step? BR-1, and return to screening. This is the classic scenario of a “does not persist” callback.

B u t P r o m e t h e u s ! ? A C R C r i t e r i a s a y s ....

A common source of confusion is the distinction is between variant 1 and variant 2 ACR criteria for
male breasts. The overwhelming majority of male breast path is gynecomastia which will look like
a BR-5 mass on ultrasound. The ACR actually says you need no imaging to work it up.

This is how I would handle multiple choice on this:

(Scenario A) If the question specifically says “ACR criteria” and describes a palpable lesion in a
male, less than 25, with no other information in the question header to make you believe its gyno:

You need to pick Ultrasound.

(Scenario B) if the question does NOT say “ACR criteria” and describes a palpable lesion in
someone with risk factors for gyno (anabolic steroid use, pot smoking, etc....)

Then you should either do no imaging, (if physical exam is a choice pick that), or start with a plain
film (mammogram).

(Scenario C) The third possible scenario, which would be the sneakiest way to do this, would be to
show you a study obtained at another hospital o f a breast ultrasound showing a suspicious lesion in
a male around this age and ask you what to do next.

The answer here is always going to be x-ray (mammogram). A work up for cancer on a male
breast is NEVER EVER EVER complete without a mammogram (“man” o gram) - with the
teaching point being that gynecomastia looks like cancer on ultrasound, but is easily identified as
benign on a mammogram

526
Scenario 4a: A 24 year old MALE presents as with a palpable mass in his left breast.

Next Step ? Mammogram (never ultrasound a male breast before you get a mammogram).

Scenario 4b: A mammogram is obtained, and shows a flame shaped density under the nipple,
correlating with the palpable marker.

Next Step ? Interview the patient to see if you can come up for a reason for his gynecomastia (psych
rneds, marijuana use, etc...). You don’t want to miss a pituitary tumor. He tells you he smokes pot
every day. You tell him that he is a very bad boy - even though there is no evidence that marijuana
causes real hann and it's criminalization was based on false propaganda from the hemp industry.

Scenario 4c: After you tell him he is a bad boy for smoking the sticky icky, he still seems worried.
He tells you that he got an ultrasound at the outside hospital and they told him he had breast cancer.
He pulls a CD out of his pocket and asks you to look at it. You look at the outside images and sure
enough there is a shadowing mass in the area o f the palpable finding.

Next Step ? BR-2 Gynecomastia. This is the oldest trick in the book - gynecomastia looks like a scary
mass on US - that’s why you always start with the mammogram.

Scenario 5a: Screening m am m ogram is perform ed on a 70 year old w om an, w ho has a history o f
prior lum pectom y 4 years ago. You read in her chart that she refused radiation therapy. She
heard on the new s that radiation w as bad, so she decided on a m ore holistic approach (bananas)
— she also gets yearly therm ogram s. The area o f scarring in the resection bed looks m ore dense.

Next Step ? BR -0, and recall for spot com pressions.

Scenario 5b: T he spot com pressions show sm all calcifications in the area o f the lum pectom y
bed, and the scar is definitely m ore dense.

Next Step ? M ag view s to further characterize the calcifications. You decide they look pointy so
you call them fine pleom orphic.

Scenario 5c: You stick her in ultrasound to be com plete - it looks like a sm all m ass. You stage
the rem ainder o f the breast and axilla - and it looks pretty clean.

D iagnosis ? BR-5 - local recurrence.

527
528
15
rr m i r r m i r f m i r r m i rrm ] n s i i T m ] rrm ] r r m r r ^ i f T s i r r m i rrm ] n s i f r s i r r ^ j

RmllTmiriMlfrmirTalfimlfimiriBingirimlfimirrBlfMiriMlfrmirim]

P r o m e t h e u s L i o i m h a r t , M . D .

529
S E C T I O N 1:
P l a y e r vs E n v i r o n m e n t
iml i l i l l l i i l l i E

T h e 3 K inds o f Q u e s tio n s

1. The ones you know - you w ant to get 100% o f these right
2. The ones you d o n ’t know - you w ant to get 25% o f these right (sam e as a m onkey
guessing)
3. T he ones you can figure out w ith som e deep thought - you w ant to get 60-70% o f
these right.

If you can do that you will pass the test, especially if y o u ’ve read m y books.

My recom m endations:
■ For the ones you know, ju st get them right.
* For the ones you d o n ’t know - ju st say to y o u rse lf “ this is one I don't know,
Prometheus says ju s t try and narrow it down and guess. ”
■ For the ones you think you can figure out, m ark them , and go through the entire exam .
If you follow m y suggestion on the first tw o types o f questions you will have am ple
tim e left over for head scratching. O ther reasons to go ahead and do the w hole exam
before trying to figure them out is (a) you d o n ’t w ant to rush on the questions you can
get right, and (b) som etim es you will see a case that rem inds you o f w hat the answ er
is. In fact it’s not im possible that the stem o f another question flat out tells you the
answ er to a previous question.

Let your plans he dark, and impenetrable as night, and when you move, fall like a thunderbolt.
-Sun Tzu

530
S tu dying fo r a C-

For m any o f you this is the first tim e you truly do not need an A on the exam . 1 can
rem em ber in undergrad and m edical school feeling like 1 needed to get every question
right on the exam to m aintain m y total and com plete dom inance.

I felt like if I m issed a single question that I w o u ld n 't honor the class, I w o u ld n ’t m atch
radiology, and I’d end up in rural W est V irginia checking diabetic feet for ulcers in m y
fam ily m edicine clinic. The very thought o f a career in fam ily m edicine w as so horrible
that I'd begin to panic.

Panic d o e sn ’t help!

Truly this exam is not like that. You can m iss questions. You w ill m iss questions. You
can m iss a lot o f questions. You ju st need to m iss less than about 10% o f the room . No
m atter w hat they tell you, no m atter w hat you read all standardized exam s are cu rv ed . If
they passed 100% - the exam w ould be called a jo k e. If they failed 50% the program
directors w ould riot (after first punishing the residents w ith extra call). The exam will
m aintain a failure rate around 10-15%. W hat that m eans is that you only need to beat
10-15% o f the room . You d o n ’t need 99th percentile. T here is no rew ard for that. You
need 16th percentile. 16th percentile is a C-, that is the goal.

The reason I’m perseverating on this is that you need to avoid panic. If you m ark
20-30% o f the questions as “not sure” - or P rom ethean category 2 o r 3 - you m ight begin
to freak out. E specially if the inner gunner m edical student in you thinks you w o n ’t get
honors. Chill Out! It’s ok to m iss questions. Look around the room and know that you
studied harder and are sm arter than 15% o f the room .

Do not fle e the exam in tears !

Fate rarely calls upon us at a moment o f our choosing.

-Optimus Prime

531
E x p lo itin g th e “G en iu s N e u ro n ”

Have you ever heard someone in case conference take a case and lead with “It’s NOT this,” when
clearly “this” is what the case was? It happens all the time. Often the first thing out o f people’s
m ouths is actually the right answer, but m any tim es you hear people say “ it’s not” first. Ever
wondered why?

I have this idea o f a “Genius N euron.” You have one neuron that is superior to the rest. This guy
fires faster and is more reliable than his peers and because o f this he is hated by them. He is the
guy in the front row waving his hand shouting “I know the answ er!” You know that guy, that guy
is a notorious asshole. So, in your mind he shouts out the answer first, and then the rest o f the
neurons gang up on him and try and talk him out o f it. So the end product is “ It’s NOT this.”

For the purpose o f taking cases in conference, this is w hy you should always lead with “this
comes to m ind,” instead o f “it’s not.” Now, the practical piece o f advice I want to give you is to
trust your genius neuron. Seriously, there is a lot o f material on this test. But if you read this
book, there will be enough knowledge to pass the test existing somewhere between your ears. You
just have to trust that genius neuron.

How?? - Do it like this:

(1) Read the entire question. Look at all the pictures.

(2) Read ALL the answer choices. N ever stop at A thinking that is the answer.

(3) Look again at ALL the pictures - now that you see the choices.

(4) Choose the first answer your mind tells you is correct - the one your genius neuron thinks
it correct.

(5) After you have finished the test, and you are re-reviewing your answers, NEVER change
the genius neuron’s answ er except for two criteria. (A) You read the question wrong. (B)
You arc 100% sure that it is another choice, and you can give a reason why. N ever change
based on your gut feelings. Those secondary gut feelings are the stupid neurons trying to
gang up on the smart one. Just like in the real world, the stupid people significantly
outnum ber the smart ones.

I know this sounds silly, but I really believe in this. This is a real thing. I encourage you to try it
with some practice questions.

You either believe in y o u rself or you don't

-Captain James T. Kirk.

532
D ea lin g w ith th e L in ked Q u e s tio n

It is a m odern trend for m ultiple choice tests to have “ linked” questions. You m ay rem em ber
that U SM L E Step 3 had them , and it is rum ored that the C O R E Exam has them as w ell.

These are the questions that prom pt you w ith “this is your final answ er, you c a n ’t change
your answ er.” W hen you see this STOP!

If you are 100% sure you are right, then go on. I f you had it narrow ed dow n to tw o choices,
think about w hich one w ould be easier to w rite a follow up question about. T his m ight seem
obvious, but in the heat o f the battle you m ight get too aggressive. Slow dow n and think
tw ice on these.

The second point I w ant to m ake about these questions is finding som e Zen if you m iss it.
There are a lot o f questions on this test, it’s ok to m iss som e. You w ill still pass (probably).
People like you have alw ays studied for the A+, not the C-. So w hen you m iss a question it
m akes you freak out because you think you blew it. C alm the fuck dow n. You d o n ’t need an
A + this tim e. You d o n ’t need a B. You ju st need to pass so they d o n ’t get any m ore m oney
from you. Believe m e they have taken enough from you already. I ju st w ant you to
understand that you will m iss questions and it’s ok. If the second part reveals that you
dropped one, d o n ’t let it phase you. Just do y our best. T he m ost im portant fight is alw ays
your next one.

It isn't the mountains ahead to climb that wears you out; it s the pebble in your shoe

-Muhammad Ali

533
It ’s P o ssib le to K n o w Too M uch

If you were to begin studying and begin taking m ultiple choice practice questions and you plotted
your progress as you gained m ore knowledge you would notice something funny. At first you
would begin to get more and m ore questions rig h t... and then you would start to miss them.

Well how can that be? I will tell you that once you know enough all choices on the exam become
correct. Which o f the following can occur?... well actually they can all occur - I’ve read case
reports o f blah blah blah. That is what happens.

The trick is to not over think things. Once you’ve achieved a certain level o f knowledge, if they
give you a gift — take it. It’s usually not a trick (usually). D on’t look for obscure situations
when things arc true. Y es... it’s possible for you to know m ore than the person writing the
questions. Y es... I said it and it’s fucking true. These people don’t know everything. You can out
knowledge them if you study enough - and that is when you get yourself into trouble.

Take home point - once you’ve reached the peak (arrow on chart) - be careful over thinking
questions past that point.

"Always remember: Your focus determines you r reality’’

-Jedi M aster Qui-Gon Jinn

534
nn 8 Promethean laws For Multiple Choice nn
#1 - I f you have a gut feeling - go for it ! (trust the genius neuron)

#2 - D o n ’t over think to the extent that you veer from a reflexive answ er -
especially i f choices seem equally plausible (you can know too much).

#3 - Read A L L the choices carefully

#4 - If it seem s too obvious to be true (trickery), re-read it and then go with it


(even if it seem s too easy). Let it happen - it’s usually not a tr ic k ... usually.

#5 - A dd up w hat you know you know, and com pare with w hat you think you
know. W eight y o ur answ ers by w hat you K N O W you KNOW .

#6 - If you are torn betw een tw o choices, ask y o u rse lf “w hich o f these is
N O T correct ?” - Som etim es m aking your brain w ork backw ards will
elucidate the solution.

#7 - Do N O T change your answ ers ! (trust the genius neuron)

#8 - M ost Im portantly - D o n ’t Panic

M aybe I can't win, maybe the only thing I can do is ju s t take everything he s got. But to beat me,
he s gonna have to kill me, and to kill me, he ,’v gonna have to have the heart to stan d in fron t o f
me, and to do that, he has to be willing to die him self

- Rocky Balboa

535
S E C T IO N 2: A

t it K n it t h e S o c k s o f D e a th
AS G IFT S FO R ALU W H O DARE O P P O S E YOU
7F

P ro b le m S olvin g T h ro u g h M RI

D ifferent program s have variable volum e w ith M RI. Som e o f you will be excellent at it.
Som e o f you will suck at it. An im portant skill to have is to understand how to problem solve
w ith different sequences. The best w ay to do this is to have a list o f T1 bright things, T2
bright things, dark things, and things that restrict diffusion.

T l Bright T2 Bright T l and T2 Restricts Diffusion


DARK

Fat Fat Flow Void Stroke


--------- ------------------------ -
Melanin (Melanoma) Water Fibrosis / Scar Hypercellular Tumor
j
Blood (Subacute) Blood (Extracellular Metal Epidermoid
Mcthemoglobin)

Protein Rich Fluid Most Tumors Air Abscess (Bacterial)

Calcification Acute Demyelination


(Hyalinized)

Slow Moving Blood CJD

Laminar Necrosis T2 Shine Through

Be able to m ove through sequences and problem solve.

T hink about a Lipom a for exam ple. T his will be T1 bright, T2 bright, and fat sat out.
A nother exam ple m ight be som ething w ith layers in it. W hat can layer? Fat could layer,
w ater could layer, blood could layer, pus could layer. Fat w ould be bright/ bright. W ater
w ould be dark on T l. Pus w ould be dark on T2. Blood could do different things depending
on it’s age. Fat w ould sat out. Pus m ay restrict diffusion (like a subdural em pyem a). You get
the idea. Run through som e scenarios in your m ind. T he key point is to know your
differentials for this.

536
B attle T a c tic s : P e d s N e ck

This is m y suggested strategy. I typically start w ith cyst vs solid. T hen I consider location,
m orphology, and choice o f m odality (attem pted m ind reading o f the question w riter).

*MIDLINE
Ultrasound in Axial Planes
Thyro glossal Duct C yst C T / MRI in Sagital Planes
Posterior to Tongue
Anterior to Hyoid

‘ LA T E R A L
Brachial Cleft C yst Axial Plane Most Likely
Anterior to the
Sternocleidomastoid (type 2)

Looks Just like BC Cyst - but


Necrotic Level 2 Node the kid is too old (late teens /
“young adult”)
Thyroid CA, or Nasopharyngeal
HPV related CA
C T axial plane - most likely

T2 Bright
Looks C ystic Enhances
Hemangioma of Infancy
Will have flow in it on doppler
If you saw it you’d think to
yourself “if that doesn’t involute
that kid is never getting a date
to prom”

• OB Ultrasound - most likely


modality
C y stic Hygroma . J2 Brjght
• D O E S NOT Enhance
• Will NOT have flow in it on
doppler
• Turners, Downs.

• Kinda looks like a big dilated


Phlebectasia jugular vein (because it is a big
dilated jugular vein)
• No stenosis or collaterals

537
• Jugular Vein with a clot - they
will have to prove that has a
Septic Throm bophlebitis c 'ot 'n " Pr°bably with
Doppler US
• Lemierre’s Syndrome
• Septic Emboli to the lungs
• Recent EN T procedure, or
Infection
• Fusobacterium Necrophorum

Back of the tongue or in front


Ectopic Thyroid of the hyoid
Tc-MIBI, or 1-123

Ultrasound
Looks Solid Fibrom atosis Coli “Two Heads” of the
Sternocleidomastoid

• MRI o r C T
Rhabdom yosarcom a * Seriously pissed off looking
mass (probably in the orbit -
maybe in the masticator
mass)
• Enhances heterogenous,

MRI o r C T
Soft Tissue Mass,
Calcifications,
Metastatic Neuroblastoma
Restricted Diffusion
Classic is the orbit

538
B attle T a c tic s : C o n ge n ital H e art on C X R

Right
(1) What side is the arch on ? TOF or Truncus

£/ \
<g i Pulmonary \
^ / Vasculature L

Left
Truncus TOF
Types 1-3

Massive
Cyanotic (2) Heart Size Ebsteins or
Pulm onary A tresia w ithout V SD
*They have to tell you this,
N on-C ardiac (w o n ’t be cyanotic)
if they want a single answer
-Infantile H em angioendotheliom a
-Vein o f G alen M alform ation
1

NOT-Cyanotic (3) Pulmonary Blood Flow


o
Q. %

//
-T A PV R (especially type 3)
\
-TOF
AV
&

-D -T ransposition -E bsteins
-T runcus (look for R A rch) -T ricuspid A tresia
-“ T ingle V entricle”

C yanotic Not C yanotic


W alking through this outline. First ask TOF ASD
y o u rself is it cyanotic or not? T hey will TAPVR VSD
have to tell you this in the stem . Look Transposition PDA
Truncus PAPVR
for this in the stem every tim e, then cross
Tricuspid Atresia Aortic Coarctation (adult type -
out answ ers that are not cyanotic.
post ductal)

Example: Patient “X ” is a newborn cyanotic, what is the m ost likely Dx?


A -V S D
B -A S D
C- D em onic Possession
D -T O F

W ithout even looking at a picture (w hich they will probably show ), you know the answ er is
D, because that is the only cyanotic one listed. If you w ere w ondering about C - I did a
google scholar search for "Demonic Possession causing cyanosis ”, and although there w ere
a few case reports none com e dow n hard on cyanosis.

539
B attle T a c tic s : N eonatal C h e st

Lung Volume

HIGH

Symmetric Pattern
(looks like Pleural Effusions
pulm onary L e s s Sym m etric

\
Peri hilar / S treaky Not Usually
ed em a )

Transient Meconium
Surfactant-
Tachypnea Aspiration Beta Deficient
or Hemolytic
“Pulmonary E dem a Disease (SDD)
+ Effusions” Non G B Pneumonia
“Diffuse Granular”
Neonatal “Diffuse Granular”
Pneumonia

T H IS vs THAT: M eco n iu m A s p ir a tio n o r N on G ro u p B S trep P n eu m o n ia

This is s u p e r tou gh w ith o u t a n y h isto ry , a n d b e c a u s e o f th a t I f e e l like the te s t w r ite r h as


tw o o p tio n s : (J ) S to p b e in g an a s s h o le a n d g iv e y o u so m e h isto ry, (2 ) n o t in clu d e b oth a s
a n s w e r c h o ic e s - a ssu m in g o n ly o n e is c o r r e c t. N ow , a lo n g th o se lin es i f y o u s a w b o th a s
c h o ic e s a n d the q u e stio n h e a d e r g iv e s y o u n o h is to r y y o u c o u ld e lim in a te th em b oth a s
d is tr a c to r s - b e c a u s e th e y b o th c a n ’t b e c o r re c t.

E x a m p le: W h at c o lo r is th is b o x ^

A - Blue ^ I Don’t See a BOX!


How can I answer the question?!?
B - Looks R ed
There can be situations where the picture is not
C - It h a s a R e d A p p e a r a n c e necessary to get the question right. They could
show you a picture of a grilled cheese sandwich
and call it a chest x-ray. It could still be possible
to get the question right by eliminating all but one
possible choice. The question header can help
you disqualify (as we discussed with cyanotic
heart disease) or you can try and find choices that
cannot be distinguished from each other. Both
answers can’t be right - so they must both be
wrong.

540
B attle T a c tic s : P e d s C h e st & M isc

Prem aturity — G u e s s in g th a t th e k id is p re m a tu re c a n b e h e lp fu l fo r e lim in a tin g


c h o ic e s (M e c o n iu m A s p ira tio n is m o re o f a p o s t te rm th in g ), a n d ra is in g y o u r p re te s t
p ro b a b ility (S D D , o r N E C in a b e lly film ). T h e re a re tw o m a in c lu e s:

(1 ) H u m e ra l H e a d O ssific a tio n - T h is te n d s to o c c u r c lo s e r to te rm . I f th e h u m e ra l
h e a d is N O T o ssifie d y o u c a n a s s u m e (in th e w o rld o f m u ltip le c h o ic e ) th a t th e k id
is lik e ly p re m a tu re .

(2 ) L ack o f S u b c u ta n e o u s F a t - P re m a tu re k id s te n d to b e v e ry s k in n y , a lth o u g h I th in k
o f th is m o re o f a so ft sig n th a t is u s e fu l w h e n a b s e n t m o re th a n p re s e n t. I ’ll ju s t
say th a t if th e k id a p p e a rs c h u b b y h e is p ro b a b ly N O T p re m a tu re .

The Thing in the Lung:

Left Upper Lobe:

Think Congenital Lobar Emphysema (CLE) first. But,


remember CCAM has no lobar prevalence, so it can be
anywhere

Left Lower Lobe:

Think Sequestration First. Congenital Diaphragmatic


Hernia (CD H s) favors this side too

Case 1. Newborn with Case 2. 10 year old with *Intralobar is seen


congenital heart disease recurrent pneumonia old er kids,
**Extralobar is seen
A. Intralobar Sequestration A . Intralobar in infants with co-
B. E xtralobar Sequestration m orbids
Sequestration B. E xtralobar Sequestration ** CLE is in the
C. C ongenital Lobar C. C ongenital L obar upper lobe
E m physem a E m physem a

541
N G T u b e T r ic k s : The presence o f an N G
tube (especially if not placed correctly) should
alert you to som e form o f trickery.

The N G tube stops in the upper thoracic


esophagus: Think esophageal atresia (probably in
the setting o f VA CTERL).

The NG tube curling into the chest - it’s either (1)


in the lung, or (2) it’s in a congenital
diaphragm atic hernia. If 1 had to pick betw een the NG Tube Curls in the upper thoracic
tw o (and it w a sn ’t obvious), I’d say left side esophagus. First think Esophageal
hernia, right side lung - ju st because those are the Atresia. Then think V A C T E R L
m ore com m on sides.

T h e C la s s ic C o n g e n ita l L o b a r E m p h y s e m a T r ic k : T hey can show you a


series o f CX Rs. The first one has an opacity in the lung (the affected lung is fluid filled). The
next x-ray will show the opacity resolved. The follow ing x-ray w ill show it getting m ore
lucent, and m ore lucent. Until it’s actually pushing the heart over. This is the classic w ay to
show it in case conference, or case books

T h e S c h o o l A g e d C X R : Things to look fo r:

• Big H eart - Probably show ing you a sickle cell case. Look for bone infarcts in shoulders.
• L ucent L ung - Think foreign B ody (air trapping). R em em ber you put the affected side
dow n (if it rem ains lucent- that confirm s it).

THIS vs THAT: Cystic Fibrosis vs Primary Ciliary Dyskinesia


CF PCD
A bnorm al M ucus - C ilia c a n ’t clear it N orm al M ucus - C ilia d o n ’t w ork
B ronchiectasis (upper lobes) B ronchiectasis (low er lobes)
N orm al sperm , obliterated vas deferens N orm al vas deferens, sperm cannot swim
norm ally

542
B attle T a c tic s : P e d s M isc

T h e M andible: T here are only a few things that a m andible w ill be show n for w ith
regards to Peds. Think C affeys first - especially if the picture looks blurry and old (there
h asn ’t been a case o f this in 50 years). If it’s osteonecrosis think about O.I. on
bisphosphonates. If it’s a d w a rf case, think w ide angled m andible w ith Pycnodysostosis. A
“ floating tooth” could be EG .

T h e A bdom inal Plain Film - on a n ew b o rn - Problem Solving Bubbles:

Pattern Path N ext Step

Single Bubble: In a new born this is G astric


(antral or pyloric atresia).
In an older child think
gastric volvulus

D ouble Bubble D uodenal A tresia

Triple Bubble Jejunal A tresia

Single B ubble + Distal Gas C oncern for M id G ut N ext Step = U pper GI


+ “ Bilious Vom iting Volvulus

M ultiple D ilated Loops C oncern for low er N ext Step = C ontrast Enem a
obstruction

THIS vs THAT: Duodenal Atresia vs Jejunal Atresia:

D uodenal A tresia Jeju n al A tresia

D ouble B ubble Triple Bubble

Failure to C analize (often isolated atresia) V ascular Insult * M ore likely associated w ith
other atresias

A ssociated w ith D ow ns

543
T H IS v s THAT: I n tr a lo b a r v s E x tr a lo b a r S e q u e s tr a tio n

In tralobar E xtralobar

N o pleural covering Has its ow n pleural covering

M ore C om m on Less C om m on

Presents later w ith recurrent infection Presents early w ith other bad congenital things
(heart, etc...)

H e te ro ta x ia : This can be inferred or asked several w ays.

H eterotaxia Synd rom es


R ight Sided L eft Sided
Two Fissures in Left Lung O ne Fissure in Right Lung
A splenia Polysplenia
Increased C ardiac M alform ations Less C ardiac M alform ations
R eversed Aorta/I VC A zygous C ontinuation o f the IVC

O rbital C a lcificatio n s: L ess than 3 O lder than 3

R etinoblastom a Toxo

CM V Retinal A strocytom a

C olobom atous C yst

544
B attle T a c tic s : A bdom inal M a sse s I D iffu se Pathology

Peds Liver Masses

Associated:

Endothelial Progressively High Output CHF


Infantile Hepatic
Age 0-3 growth factor Calcify - as they
H em angiom a
is elevated involute Skin Hemangiomas

Risk Factor =
Prematurity
may cause
AFP is Calcifications Many
H epatoblastom a Age 0-3 precocious
elevated are Common Association:
puberty
Wilms,
Beckwith-
Weidemann

Calcification are
RARE
M esenchym al AFP is
Age 0-3 CYSTIC MASS “Developmental anomaly’’
H am artom a negative
Favors Right
Lobe

Kids with cirrhosis (biliary


AFP is
HCC Age > 5 atresia, Fanconi syndrome,
elevated
glycogen storage disease)

Calcifies more No Cirrhosis


Fibrolam ellar Subtvpe AFP is often than
Age > 5 Central Scar (scar does NOT
HCC negative conventional
HCC enhance, and is T2 dark)

Cystic /
Undifferentiated AFP is
Age > 5 Heterogeneously Known to rupture
Em bryonal Sarcom a negative
Solid Mass

Fetus - 4
Mets
Multiple Masses in the Setting o f Known Primary
(N euroblastom a, W ilm s) 6 - Early
Teens

545
■ ,
A d u lt B enign L iver M asses

Ultrasound CT MR Trivia

Hemangioma Hyperechoic Peripheral T2 Bright Rare in


Nodular Cirrhotics
Discontinuous
Enhancement

FNH Spoke Wheel Homogenous “Stealth Central Scar Bright on


Arterial Lesion - Iso Delayed Eovist
Enhancement on T l and (Gd-EOB-
T2” DTPA)

Hepatic Adenoma Variable Variable Fat OCP use, Can explode


Containing Glycogen and bleed
on In/Out Storage
Phase Disease

Hepatic Hyperechoic Gross Fat T1/T2 Bright Unlike renal Tuberous


Angiomyolipoma AML, 50% Sclerosis
don’t have fat

Su lfu r C olloid H O T or C O L D

Hepatic Adenoma COLD

40% HOT,
FNH 30% COLD,
30% Warm

Cavernous Hemangioma COLD RBC Scan HOT

HCC COLD Gallium HOT

Cholangiocarcinoma COLD

Mets COLD

Abscess COLD Gallium HOT

Focal Fat COLD Xenon HOT

R egenerative N odules D ysp lastic N od ules H CC

C ontains Iron C ontains Fat, G lycoprotein

T l Dark, T2 Dark T l Bright, T2 D ark T2 B right

Does N O T Enhance U sually D oes N O T Enhance D oes E nhance

546
This vs That: HCC vs This vs That:
Fibrolamellar Subtype HCC Central Scars of FIN H and Fibrolamellar HCC
HCC FL HCC
FNH FL HCC
Cirrhosis No Cirrhosis
T2 Bright T2 Dark (usually)
Older (50s-60s) Young (30s)
Enhances on Delays Does NOT enhance
Rarely Calcifies Calcifies
Sometimes Mass is Sulfur Colloid Mass is Gallium Avid
Avid (sometimes)
Elevated AFP Normal AFP

M ultiple Low D ensity (N O T C ystic) L iver Lesions

Think Colon First


M ets
- unless they have a known primary

HCC Does the Liver look Cirrhotic?

R egeneratice N odules Does the Liver look Cirrhotic ?

Low Density Nodes - Think Mycobacterium


Infections
Hyperenhancing Nodes - Think Bartonella

Spleen Should be Involved Also


Sarcoid Gamesmanship - Probably gets some hints in the form of a CXR, or
Labs (elevated ACE)

M ultiple C ystic L iver L esions In fection B uzzw ord s

Different Size Cysts Viral H epatitis Starry Sky (U S)


AD Polycystic
(small and big)
Kidney
Renal Cystic Disease
Pyogenic A bscess D ouble Target (C T)
Von
Small ( < 1.5 cm)
M eyenburg
Will NOT connect with Ducts
C om plex
Uniform distribution
(H am atrom as) C andida B u ll’s Eye (U S)

C holedochal WILL communicate with duct A m oebic A bscess “ Extra H epatic


Cysts (Caroli) Central “dot” Sign E xtension”

Bacterial and Fungal =


Multiple, R > L H ydatid D isease W ater Lily, Sand Storm
Amoebic = Single and Sub-
A bscess diaphragmatic CT( g ) @ > us
Peripheral Enhancement
Necrotic center will not S chistosom iasis Tortoise Shell ^
enhance

547
P rim ary H em ochrom atosis S econdary H em ochrom atosis

G enetic - increased absorption A cquired - chronic illness, and m ultiple


transfusions

Liver, P a n c re a s Liver, S p le e n

Heart, T hyroid, Pituitary

T his vs T hat: A ID S C h olangiopathy vs Prim ary Sclerosing C holangitis

A ID S PSC

Focal Strictures o f the extrahepatic duct > 2cm E xtrahepatic strictures rarely > 5m m

A bsent saccular deform ities o f the ducts H as saccular deform ities o f the ducts

A ssociated P apillary Stenosis

dm* rw-i *•
Biliary' D ilation

Intrahepatic E xtrahepatic ONLY

Prim ary Sclerosing C holangitis Post C holecystectom y

Infectious C holangitis Sphincter o f Oddi D ysfunction

Pancreatic H ead M ass Type 1 C holedochocyst

CB D Stone — Late C B D Stone - Early

B iliary Stricture

THIS vs THAT: C hronic P ancreatitis D uct D ilation


vs P ancreatic M alignancy D uct D ilation
.............
CP C ancer

D ilation is Irregular D ilation is uniform (usually)

D uct is < 50% o f the AP gland diam eter D uct is > 50% o f the A P gland diam eter
(obstructive atrophy)

548
Prim ary H em ochrom atosis S econdary H em och rom atosis

G enetic - increased absorption A cquired - chronic illness, and m ultiple


transfusions

Liver, P a n c re a s Liver, S p le e n

Heart, T hyroid, Pituitary

T his vs T hat: A ID S C h olan giop ath y vs P rim ary Sclerosin g C h olan gitis

A ID S PSC

Focal Strictures o f the extrahepatic duct > 2cm E xtrahepatic strictures rarely > 5m m

A bsent saccular deform ities o f the ducts H as saccular deform ities o f the ducts

A ssociated P apillary Stenosis

B iliary D ilation

Intrahepatic E xtrahepatic O NLY

Prim ary Sclerosing C holangitis Post C holecystectom y

Infectious C holangitis Sphincter o f Oddi D ysfunction

Pancreatic H ead M ass Type 1 C holedochocyst

CBD Stone — Late C B D Stone - Early

B iliary Stricture

T H IS vs THAT: C h ron ic P ancreatitis D u ct D ilation


vs P ancreatic M alignancy D u ct D ilation

CP C an cer

D ilation is Irregular D ilation is uniform (usually)

D uct is < 50% o f the A P gland diam eter Duct is > 50% o f the AP gland diam eter
(obstructive atrophy)

549
U ncom m on Types and C au ses o f Pancreatitis

A utoim m une Associated with Absence o f Attack Responds to Sausage Shaped


Pancreatitis elevated IgG4 Symptoms steroids Pancreas, capsule
like delayed rim
enhancement around
gland (like a scar).
No duct dilation.
No calcifications.

G roove Looks like a Less likely to cause Duodenal stenosis Soft tissue within the
Pancreatitis pancreatic head obstructive and /or strictures of pancreaticoduodenal
Cancer - but with jaundice (relative the CBD in 50% of groove, with or
little or no biliary to pancreatic CA) the cases without delayed
obstruction. enhancement

Tropic Young Age at onset, Increased risk o f Multiple large calculi


Pancreatitis associated with adenocarcinoma within a dilated
malnutrition pancreatic duct

H ereditary Young Age at Onset Increased risk of SPIN K -1 gene Similar to Tropic
Pancreatitis adenocarcinoma Pancreatitis

A scaris Most commonly Worm may be seen


Induced implicated parasite within the bile ducts
in pancreatitis

W hen I Say - A u toim m u n e P ancreatitis

I Say A utoim m une Pancreatitis You Say IgG4

A utoim m une Pancreatitis


R etroperitoneal Fibrosis
I Say IgG4 Sclerosing C holangitis
Inflam m atory Pseudotum or
R iedel’s T hyroiditis
____
THIS vs THAT:
A u toim m une P ancreatitis vs C h ron ic P ancreatitis

A u toim m une Pancreatitis C hronic Pancreatitis

No ductal dilation D uctal D ilation

No calcifications Ductal C alcifications

550
. .........
C ystic P ancreatic L esions
..... ,
.........................................

Main Branch
40s - 50s Main Duct High Malignant Potential (60%)
IPMN

Typically Benign
Side Branch
50s - 60s Favor Mead, Uncinate (maybe 5% will develop malignancy)
IPMN
Communicates with duct

Does NOT Communicate with Main Duct


Central Calcifications
Grandma
“Micro-Cystic” - “Honeycomb”
Serous Cystic F> M Favor Head
Benign
>60
Glycogen Rich
Associated with von Hippel Lindau

Does NOT Communicate with Main Duct


Mother
M ucinous Peripheral Calcifications
F> M Favor Body / Tail
Cystic Larger Cysts (sometimes uni-locular)
40s
Premalignant

Large (5 -1 0 cms)
Daughter Solid with Cystic Parts
Solid Pseudo-
F>M Favor Tail Enhances like a Hemangioma
Papillarv
20 s Has a Capsule
Asian or Black Female

Malignant Ulcer Benign Ulcer Direct Hernia Indirect Hernia

Width > Depth Depth > Width Less common More Common

Project behind the Medial to inferior Lateral to inferior


Located within Lumen
expected lumen Epigastric epigastric

Nodular, Irregular Edges Sharp Contour Defect in Hesselbach Failure o f processus


triangle vaginalis to close
Folds adjacent to ulcer Folds radiate to ulcer
NOT covered by
Covered by internal
Aunt Minnie: Carmen Aunt Minnie: internal spermatic
spermatic fascia
Meniscus Sign Hampton’s Line fascia

Sigm oid V olvulus C ecal V olvulus

Old Person (C onstipated) Y ounger Person (m ass, prior surgery, or 3rd


T rim ester Pregnancy)

Points to the RUQ Points to the LU Q

551
M ore C om m on In :
Crohns vs U lcerative Colitis
Crohns vs UC
C rohns UC
M ore C om m on
Slightly less com m on in Slightly m ore com m on in Path
W ith
the USA the USA

D iscontinuous “ Skips” C ontinuous


G allstones C rohns
Term inal Ileum - String
Rectum
Sign
Prim ary
Ileocecal Valve Sclerosing U lcerative C oliits
Ileocecal Valve “ O pen”
“ S tenosed” C holangitis
M esenteric Fat Increased
Perirectal fat Increased
"creeping fa t ” H epatic
C rohns
A bscess
Lym ph nodes are usually Lym ph nodes are N O T
enlarged usually enlarged

D oesn’t U sually M ake Pancreatitis C rohns


M akes Fistulae
1 Fistulae
1 .....

% This Could Be Useful

C holangio: CEA^ CA 19-9

Pancreatic CA:

Colon CA:
C EA

C EA |
CA 19-9

CA 19-9
t

552
Peds Cystic Renal Mass

Multicystic D ysplastic Kidney A R- P o lycystic Kidney D isease


- Neonate, No Renal Function (MAG 3) - Enlarged, Hyperechoic
- Associated with congenital UPJ obstruction - Microcystic
- Associated with reflux (VUR) -

Multilocular C ystic Nephroma


- “Micheal Jackson - Young Boy, Older Woman"
- Multiple Cysts - Herniates into the Renal Pelvis”

Cystic Wilms

Peds Tum or / M ass R ap id R eview Trivia

Mesoblastic
“Solid Tumor o f Infancy” (you can be born with it)
Nephroma

“Nephrogenic Rests” - left over embryologic crap that didn’t go away


Might turn into wilms (bilateral wilms especially)
Nephroblastomatosis
“Next Step” - f/u ultrasound till 7-8 years old
Variable appearance

90% + Renal Tumors


“Solid Tumor o f Childhood" - Never born with it
Wilms
Grows like a solid ball (will invade rather than incase)
Met to the lung (most common)

Clear Cell - Wilms Met to Bone

Brain Tumors
Rhabdoid - Wilms
It fucks you up, it takes the money (it believes in nothing Lcbowski)

Micheal Jackson Tumor (Young Boys, Middle Age Women)


Multi-Cystic Big cysts that don’t communicate
Nephroma Septal Enhancement
Can’t Tell it is not Cystic Wilms (next step = resection)

“Solid Tumor o f Adolescent ”


RCC
Syndromes - VHL, TS

Non-Hodgkin
Renal Lymphoma
Multifocal

553
N euroblastom a W ilm s

A ge: U sually around age 4


Age: usually less than 2 (can occur in utero)
(never before 2 m onths)

C alcifies 90% C alcifies R arely (<10% )

Encases Vessels (d o esn ’t invade) Invades Vessels (d o esn ’t encase)

Poorly M arginated Well C ircum scribed

D o esn ’t usually m et to bones (unless clear cell


M ets to Bones
W ilm s variant). Prefers lung.

N euroblastom a A drenal H em orrhage


H eterogenous and vascular C entrally H ypoechoic and A vascular
High on T2 , Iso-L ow on T1 High on T1 (7 days - 7 w eeks)
Will grow on follow up Should shrink on follow up

A d ult R C C A ssociations B ladder C ancer


Subtype Syndrom e / A ssociation
T ransitional Cell The “norm al” kind
C lear Cell Von H ippel-L indau CA B ladder CA » > U reter CA
Papillary H ereditary papillary renal Squam ous Cell C alcifications
.
carcinom a CA C hronic C atheter
Schistosom iasis (w orm )
C hrom ophobe Birt H ogg Dube
A deno- M idline
C arcinom a U rachus A ssociation
M edullary Sickle C ell Trait
B ladder Exstrophy

R enal C yst A ssociations Urethra


C ysts in K idneys are
A D PC K D Prostatic U rethra Transitional Cell
Liver BIG

C ysts in B ulbar / Penile Squam ous Cell


VHL
Pancreas U rethra

A cquired K idneys are U rethral A denocarcinom a


(urem ic) sm all D iverticulum

554
You See T h at T h in k T his

Big Kidney with Lots o f Cysts


AD Polycystic Kidney
Liver Cysts

Norm al Sized K idneys w ith Lots o f Cysts


Solid Renal M asses (RCCs)
Pancreatic Cysts (sim ple and serous cystic) von Hippel - Lindau
Pancreatic M asses
Adrenal M asses (paragangliom as)

Renal Cysts
M ultiple Fat C ontaining Renal M asses
Tuberous Sclerosis
(AM Ls) - m aybe bleeding
Lungs Cysts (LAM )

Small / Calcified Spleen


Gallstones (or absent GB) Sickle Cell
Bone Infarcts

Severe Pancreatic Fatty A trophy


Small bowel stool Cystic Fibrosis
Fatty Liver

Big Liver, Big Spleen


G aucher
Bone Infarcts
Extram edullary H em atopoiesis

Bilateral Adrenal M asses


(Pheochrom ocytom a - not adenom a) M EN 2
Thyroid C ancer

Islet Cell Tum ors


M EN 1
Pituitary A denom a

Renal M asses (W ilms, AM L)


Adrenal M asses (Pheochrom ocytom a)
NF-1
Skin N odules
Scoliosis

Vascular M alform ation in the Liver


Bowel A ngiodysplasia
O sier W eber Rendu
Enlarged Hepatic A rtery
(H ereditary H em orrhagic Telangiectasia)
Pulm onary AVM
Brain Abscess

C yst M orphology Trivia:

• ADPCKD - Round and Distributed Throughout Kidney

• ARPCKD - Tubular Cysts which Spare Cortex

555
B attle T a c t ic s : S c h e m a tic T h o ra c ic Pathology

Normal Thymus Lymphoma


- Age < 10 - Age > 1 0
- Homogenous - Mass Fffftnt ' Lymph Nodes
- No Mass Effect - S V C Compression > NHL

Teratoma
NS - Germ Cell Tumor
- Fat
- Cystic Big - Aggressive
- Hemorrhage - Klinefelter
- Calcifications
- Necrosis

Seminoma
- Straddle Midline
- Bulky
- Lobulated

556
DIP NSIP UIP
- A p ica l E m p h y s e m a B a s ila r G ro u n d G la s s - H o n e y c o m b in g - b a s ila r
(sm o kin g re la te d ) (s u b -p le u ra l s p a rin g ) p re d o m in a n t
- B a s ila r G ro u n d G la s s T ra c tio n B ro n c h ie c ta s is
- P e rip h era l B a s ila r S c le ro d e rm a A s s o c ia tio n
R e ticu la tio n (d ila te d e s o p h a g u s )
- S m o k e r - S e v e re e n d of
R B -ILD

LCH
* T h in w a lle d c y s ts - ■ N o d u le s w ith c a v ita tio n ■ T h in w a lle d c y s ts (le s s th a n
d is trib u te d e v e n ly (e a rly) LAM )
' T u b e ro u s s c le ro s is * A p ic a l - “ B iz a rre ” C y s ts (la te ) " G ro u n d G la s s - c le a rs w ith
' S m o k e r -2 0 s -3 0 s tre a tm e n t
' S p a rin g o f th e c o s to p h re n ic " S jo g re n s , R A, H IV
re c e s s e s

- A to ll / R e v e rs e H a lo S ig n -
Pneumonia ■ H a lo S ig n - G ro u n d G la s s
C o n s o lid a tio n a ro u n d
R e ve rse P u lm o n a ry a ro u n d c o n s o lid a tio n
G ro u n d G la s s
E d e m a P a ttern * A ir C re s c e n t - “in v a s iv e ”
- P atchy, P e rip h e ra l
(p e rip h e ra l)
C o n s o lid a tio n
G ro u n d g la s s and
c o n s o lid a tio n

557
U pper L obe P redom inant L ow er Lobe Predom inan t

M ost inhaled stu ff (not asbestosis). Coal A sbestosis


W orkers, and Silicosis. This includes
progressive m assive fibrosis.

CF Prim ary C iliary D sykinesia

RB -ILD M ost Interstitial Lung D iseases


(UIP, NSIP, DIP)

C entrilobular E m physem a Panlobular E m physem a (A lpha 1)

A nkylosing Spondylitis R heum atoid Lung

Sarcoid S cleroderm a (associated w ith N SIP)

C ollagen V ascular D isease P ulm on ary M anifestations

More pleural effusions and Fibrosis is uncommon. Can


pericardial effusions than get a “shrinking lung.”
Lupus
with other connective tissue
disease

Looks like UIP and COP. Reticulations with or without


Lower lobes are favored. honeycombing, and
Rheumatoid Arthritis
consolidative opacities which
are organizing pneumonia

N SIP> UIP; lower lobe Look for the dilated fluid filled
Scleroderma
predominant findings. esophagus.

LIP Extensive ground glass


Sjogrens attenuation with scattered thin
walled cysts.

Upper lobe fibrobullous Usually unilateral first, then


Ankylosing Spondylitis
disease progresses to bilateral.

Infections in A ID S by C D 4 A C R A ppropriateness Criteria:

First Line for Suspected M etastatic D isease


> 200 Bacterial Infections, TB
= CXR
R ecom m endation for patients on
< 200 PCP, Atypical Mycobacterial m echanical ventilation = D aily C X R
First Line for C hest Pain and High
CMV, Disseminated Fungal, S uspicion for A ortic D issection = C X R
< 100
Mycobacterial

558
C a rd ia c T riv ia :

P athology W hich Sequ en ce(s) m ost useful?

C ardiac M yxom a Low T l , High T2 (high m yxoid content)

A cute vs C hronic MI L ook at T2 - B right on A cute ; D ark on C hronic


(fibrous scar)

A rrhythm ogenic Right V entricular T l B right


D ysplasia (ARV D)

M icrovascular O bstruction First Pass Perfusion (25 seconds post G ad)

Infarct D elayed E nhancem ent (10-12 m ins post G ad)

C a rd ia c M R I E n h a n c e m e n t P a tte rn s :

Subendocardial: Infarct Transmural: Infarct Subendocardial Circumferential:


Amyloidosis *can also be transmural

DQDQ3
co
Midwall: Midwall: Midwall:
HCM Myocarditis, Idiopathic Dilated CM Myocarditis, Sarcoidosis

Epicardial:
Myocarditis, Sarcoidosis

559
C a r d ia c S u r g e r y T y p e s / In d ic a tio n s

Pulm onary A rtery B anding C H F in Infancy, Single Ventricle

A rterial Sw itch (Senning and M ustard) T ransposition o f the G reat A rteries

Rastelli (RV Baffle) T ransposition, Pulm onary O utflow O bstruction

Jatene (a type o f arterial sw itch) T ransposition

Ross D iseased A ortic Valves in C hildren

Bentall A ortic R oot / Valve R eplacem ent in M arfan

Additional Cardiac Surgery Pearls


G le n n B la lo c k T a u s s ig F o n ta n

A rtery to A rtery It's com plicated with multiple


Vein to A rtery
(Subclavian Artery to Pulmonary ve rsio n s - ste p s are unlikely to
(S V C to Pulmonary Artery)
Artery) b e tested

P rim a ry P u rp o s e : Take
P rim a ry P u rp o s e : B y p a s s the
s y s te m ic b lo o d d ire c tly to the P rim a ry P u rp o s e : In c re a s e
rig h t v e n tric le / d ire c t s y s te m ic
p u lm o n a ry c irc u la tio n (it p u lm o n a ry b lo o d flo w
c irc u la tio n in to th e PAs.
b y p a s s e s th e rig h t h eart).

M o s t T e sta b le C o m p lic a tio n s :


M o s t T e sta b le C o m p lic a tio n s : M o s t T e sta b le C o m p lic a tio n s :
-E n la rg e d R ig h t A rtiu m ca u s in g
-S V C S y n d ro m e -S te n o s is at th e s h u n t’s
a rrh y th m ia
-PA Aneurysms p u lm o n a ry in s e rtio n site
-Plastic Bronchitis

560
V a s c u litis :

L arge Vessel

Takayasu Young A sian Fem ale - thickened aneurysm al aorta

O ld Person w ith involvem ent o f the “crutches” / arm pit region


G iant Cell
(Subclavian, axillary, brachial).

C ogan Syndrom e Kid w ith eye and ear sym ptom s + A ortitis

M edium Vessel

PAN is more com m on in a M AN (M > F). Renal M icroaneurysm


PAN
(sim ilar to speed kidney). A ssociated w ith Hep B.

K awasaki C oronary A rtery A neurysm

Sm all Vessel (A N C A +)
.

W egeners N asal Septum E rosions, C avitary Lung Lesions

C hurg Strauss T ransient peripheral lung consolidations.

M icroscopic Polyangiitis D iffuse pulm onary hem orrhage

Sm all Vessel (A N C A -)
.............._ .......

HSP Kids. Intussusception. M assive scrotal edem a.

Behcets Pulm onary artery aneurysm

Buergers M ale sm oker. H and angiogram show s finger occlusions.

561
N u k e s T riv ia :

Tracer A nalog E nergy Physical H alf Life

“ Low ” - 140 6 hours


Tc - 99m

Iodine “ L ow ” - 159 13 hours


Iodine -123

“ L ow ” - 81 125 hours (biologic t l / 2


X enon - 133 30 seconds)

Potassium “ L ow ” - 135 (2% ), 167 73 hours


(8% ), use 71 ^01Hg
T hallium - 201 daughter x-rays

“ M edium ” - 173 (89% ), 67 hours


Indium -111 247 (94% )

Iron M ultiple; 93 (40% ), 184 78 hours


(20% ), 300 (20% ), 393
G allium - 67 (5% )

Iodine -131 Iodine “ H igh” - 365 8 days

Sugar “ H igh” - 511 110 m ins


Fluorine -18

50.5 DAYS
Strontium 89
(14 days in bone)
Sam arium 153 46 Hours

R adium 11 Days

Y ttrium 90 64 H ours

Rubidium 82 75 seconds

N itrogen 13 10 m ins

562
Probable C ritical O rgan (depending on who von ask)

Tc - MDP Bladder (some sources say bone)

Tc - Sulfur Colloid (IV) Liver

Tc - Sulfur Colloid (Oral) Proximal Colon

Tc - Pertechnetate Stomach > Thyroid (some sources say colon)

Tc - Sestamibi Proximal Colon

Tc - Heat Treated RBC Spleen > Heart

Tagged RBC - MUGA Heart

Tc - MAA Lung

Tc - DMSA Renal Cortex

Tc - MAG 3 Bladder

DTPA Bladder

1-123 M1BG Bladder (some sources say adrenal medulla)

1-131 M1BG Liver (some sources say adrenal medulla)

1-131,1-123 Thyroid

In-111 WBC Spleen

In-111 ProstaScint Liver

In-111 Octreoscan Spleen

Thallium 201 Renal Cortex

F I 8 FDG Bladder

Gallium Distal Colon

HIDA Gallbladder Wall

563
M ech anism o f L ocalization

Cross the cell membrane via lipophilic


Tc - Sestamibi Passive Diffusion
diffusion

Cross the cell membrane via lipophilic


Tc - Tetrofosmin Passive Diffusion
diffusion

Delivery is flow related - then diffuse into


Tc - HMPAO Passive Diffusion
brain

Delivery is flow related - then diffuse into


Tc - ECD Passive Diffusion
brain

DTPA Filtration

Carrier mediated transport across membrane


F 1 8 -F D G Facilitated Diffusion
via GLUT

Use ATP to move AGAINST concentration


1-123,1-131 Active Transport
gradient

Thallium Active Transport (Na/K Pump)

Rubidium Active Transport (Na/K Pump)

Active Transport (Na facilitated


M1BG
norepinephrine uptake system)

DMSA Active Transport

Secretion
Pertechnatate Active transport OUT o f a gland or tissue

Secretion
M A G -3 Secreted by peritubular capillaries

Secretion
Tc-99m IDA Secreted by hepatocytes

Sulfur Colloid Phagocytosis RES eats the colloid particles

Fleal Treated RBCs Sequestration

MAA Capillary Blockade Lung Perfusion

MDP Chemisorption Chemical Covalent + Hydrogen Bonding

SM -153 Chemisorption

Indium WBC Cellular Migration Cells migrate to the response o f stimuli

Octreotide Receptor Binding

DAT Scan
Receptor Binding
(1-123 Isoflupane)

564
Tum ors that are PE T C O L D N ot C an cer but PE T H O T

BAC (A deno In Situ) - Lung C ancer Infection

C arcinoid Inflam m ation

RCC O varies in F ollicular Phase

Peritoneal B ow el/L iver Im plants M uscles

A nything M ucinous B row n Fat

Prostate T hym us

F D G P E T - B rain

A lzheim er Low posterior Identical to Parkinson


tem poroparietal cortical D em entia
activity
M ulti Infarct Scattered areas o f decreased
activity
D em entia w ith Lew y B odies L ow in lateral occipital P reservation o f the m id
cortex posterior cingulate gyrus
(C ingulate Island Sign)
Picks / Frontotem poral Low frontal lobe

H untingtons Low activity in caudate


nucleus and putam en

Tc DTPA Tc MAG 3 Tc GH

Filtered (GFR) Secreted (ERPF) Filtered

Good For Native Kidneys with Concentrated better by kidneys Good for dynamic and cortical
Normal Renal Function with poor renal function imaging.

Critical Organ Bladder Critical Organ Bladder Critical Organ Bladder

Im m ediate Post OP
ATN Perfusion N orm al Excretion D elayed
(3-4 days post op)

C yclosporin Toxicity Long S tanding Perfusion N orm al E xcretion D elayed

A cute R ejection Im m ediate Post OP Poor P erfusion E xcretion D elayed

565
A gent L ocalization
Tc W BC In W B C Tc W BC Tc W B C
Indium W BC Spleen 4 H ours 24 H ours

Indium W BC Liver and B one


Renal N O Renal
D am aged M arrow Lung C lear

RBC Tagged Heart Lung


Bowel
GI NOGI Starting
RBC D am aged Spleen

Indium is B E T T E R than G allium G allium is B E T T E R than Indium


for E valuating for E valuating

• Suspected abdom inal-pelvic abscess due • Spine


to the lack o f a norm al bow el excretory • D iffuse Pulm onary Processes: G allium is
pathw ay probably the agent o f choice for the
evaluation o f pulm onary inflam m atory
abnorm alities.
• Lym phocyte m ediated infection

Tc H M PA O is B E T T E R than In W BC In W B C is B E T T E R than Tc H M PAO


for E valuatin g for E valuating

• C hildren (low er D ose) • Fever o f U nknow n O rigin


• Inflam m atory Bow el - *but you have to
im age early - like around an hour {or 30
mins - depending on who yo u ask).
• O steom yelitis in Extrem ity

566
N e u ro T riv ia :

Toxo Lym phom a


Ring Enhancing Ring E nhancing

H em orrhage m ore com m on after treatm ent H em orrhage less com m on after treatm ent

Thallium Cold T h alliu m H O T

PET Cold PET H ot

M R Perfusion: D ecreased CBV M R Perfusion: Increased (or D ecreased) CB V

A ID S
PM L Toxo C ryp tococcu s
E ncephalitis

Sym m etric T2 A sym m etric T2 Periventricular Ring D ilated


Bright Bright T2 B right E nhancem ent Perivascular
Spaces

T l dark E pendym al T hallium C old B asilar


E nhancem ent M eningitis

NF-1 O ptic N erve G liom as

NF-2 M SM E; M ultiple S chw annom as, M eningiom as, E pendym om as

VHL H em angioblastom a (brain and retina)

TS S ubependym al G iant C ell A strocytom a, C ortical Tubers

N evoid B asal Cell M edulloblastom a


Syndrom e (G orlin)

Turcot G B M , M edulloblastom a

C ow dens L herm itte-D ulcos (D ysplastic cerebellar gangliocytom a)

M axim um B le e d in g -A n e u r y s m Location

ACOM Interhem ispheric Fissure

PCO M Ipsilateral Basal C istern

M C A Trifurcation Sylvian Fissure

B asilar Tip Interpeduncular C istern, or Intraventricular

PICA Posterior Fossa or Intraventricular

567
............................................. ................ ' .........
Im aging
Path D em ographics Typical Location Trivia
• C haracteristics

40% show
Lateral nasal wall "entrapped bone"
centered at the middle Cerebriform Cerebriform Pattern
40-70
Inverting Papillom a meatus, with Pattern May have focal
M>F (4:1)
occasional extension 10% H arbor a hyperostosis on CT
into the antrum Squam ous Cell
CA

Dumbbell shaped with


Bimodal 20s & AVID hom ogeneous
E sthesioneuroblastom a waist at the cribiform
60s enhancem ent
plate

F ungating and
Ethmoid origin more Large, typically > Poorly defined
Broad Range
SNUC common than 4cm on Heterogeneous
(30s-90s)
maxillary presentation enhancement with
necrosis

Aggressive Antral Soft


Tissue Mass, with
destruction o f sinus
M ost C om m on walls Low signal
95% > 40 years Maxillary Antrum is
Squam ous Cell CA M alignancy o f on T2 (highly
old involved in 80%
Sino-N asal track cellular)
Enhances less than
some other sinus
malignancies

Enhancing mass
N early
JNA arising from the SPF
Exclusively Origin in the
(Juvenile Radiation alone in adolescent m ale
M ale S penopalantine
Nasopharyngeal cures in 80% Dark Flow Voids on
Rare < 8 or > Foram en (SPF)
Angiofibroma) Tl
25
Avidly Enhances

Homogeneous mass in
Usually older, Nasal Cavity > nasal cavity with bony
Sinonasal Lym phom a
Highly variable
peak is 60s Sinuses destruction
appearance
Low Signal on T2
(highly cellular)

-----------------------

568
S E C T IO N 3:
B ecome S u pr em e C o m m a n d e r
O f th e B a th ro o m
‘ H ig h Y ie l d T r i v i a , F o r m a t t e d f o r O p t i m a l r e v i e w w h i l e P o o p i n g

• When say ‘Subglottic Hem angiom a,” You Say PHACES Syndrome
• When say ‘PH ACES Syndrome,” You say Cutaneous Hemangioma
When say ‘Ropy A ppearance,” You say M econium Aspiration
When say ‘Post Term Delivery,” You Say M econium Aspiration
When say ‘Fluid in the Fissures,” You say Transient Tachypnea
When say ‘History o f c-scction”, You say Transient Tachypnea
• When say ‘Maternal sedation”, You say Transient Tachypnea
When say ‘Granular Opacities + Premature”, You say RDS
When say ‘Granular Opacities + Term + High Lung Volume,” You say Pneumonia
When say “Granular Opacities + Term + Low Lung Volume,” You say B-Hemolytic Strep
When say ‘Band Like Opacities”, You say Chronic Lung Disease (BPD)
When say ‘Linear Lucencies” , You say Pulmonary Interstitial Emphysema
• When say ‘Pulmonary Hypoplasia,” You say diaphragmatic hernia
• When say ‘Lung Cysts and Nodules,” You Say LCH or Papillomatosis
• When say ‘Lower lobe bronchiectasis,” You Say Primary Ciliary Dyskinesia
• When say ‘Upper lobe bronchiectasis,” You Say CF
• When say ‘Posterior mediastinal mass (under 2),” You Say Neuroblastoma
• When say ‘No air in the stom ach”, You say Esophageal Atresia
• When say ‘Excessive air in the stom ach”, You say “H ” Type TE fistula
• When say ‘Anterior Esophageal Impression,” You say pulmonary sling
• When say ‘Pulmonary Sling,” You say tracheal stenosis.
• When say ‘Single Bubble,” You say Gastric (antral or pyloric) atresia
• When say ‘Double Bubble,” You say duodenal atresia
• When say ‘Duodenal Atresia”, You say Downs
• When say ‘Single Bubble with Distal Gas,” You say maybe Mid Gut Volvulus
• When say ‘Non-bilious vom iting”, You say Hypertrophic Pyloric Stenosis
• When say ‘Paradoxial aciduria” You say Hypertrophic Pyloric Stenosis
• When say ‘Bilious vomiting - in an infant”, You say Mid Gut Volvulus
• When say ‘Corkscrew Duodenum” You say Mid Gut Volvulus
• When say ‘Reversed SMA and SMV” You say Malrotation
• When say ‘Absent G allbladder” You say biliary atresia
• When say ‘Triangle Cord Sign” You say biliary atresia
• When say ‘Asplenia” , You say “cyanotic heart disease”
• When say ‘Infarcted Spleen,” You say Sickle Cell
• When say ‘Gall Stones,” You say Sickle Cell
• When say ‘Short M icrocolon,” You say Colonic Atresia
• When say ‘Long M icrocolon,” You say M econium ileus or distal ileal atresia
• When say ‘Saw tooth colon,” You say Hirschsprung
• When say ‘Calcified mass in the mid abdomen o f a newborn”, you say Meconium Peritonitis
• When say ‘M econium ileus equivalent,” you say Distal Intestinal Obstruction Syndrome (CF)
• When say ‘Abrupt caliber change o f the aorta below the celiac axis” , You say Hepatic
Hemangioendothelioma,
• When 1 say “Cystic mass in the liver o f a newborn,” you say M esenchymal Hamartoma

569
• When say Elevated AFP, with mass in the liver o f a newborn,” you say Flepatoblastoma
• When say Common Bile Duct m easures more than 10 mm ”, You say Choledochal Cyst
• When say Lipomatous pseudohypertrophy o f the pancreas,” You say CF
• When say Unilateral Renal A genesis” You say unicom uate uterus
• When say Neonatal Renal Vein Thrombosis,” You say maternal diabetes
• When say Neonatal Renal Artery Thrombosis,” You say M isplaced Umbilical Artery Catheter
• When say Flydro on Fetal MRI,” You say Posterior Urethral Valve
• When say Urachus,” You say bladder Adenocarcinoma
• When say Nephroblastomatosis with Necrosis,” you say Wilms
• When say Solid Renal Tumor o f Infancy,” you say M esoblastic Nephroma
• When say Solid Renal Tumor o f Childhood,” you say Wilms
• When say Midline pelvic mass, in a female,” you say Hydrometrocolpos
• When say Right sided varicocele,” you say abdominal pathology
• When say Blue Dot Sign,” you say Torsion o f the Testicular Appendage
• When say Hand or Foot Pain / Swelling in an Infant”, You say - sickle cell with hand foot syndrome.
• When say ixtratesticular scrotal mass, you say embryonal rhabdomyosarcoma
• When say Narrowing o f the interpedicular distance,” you say Achondroplasia
• When say Platyspondyly (flat vertebral bodies),” you say Thanatophoric
• When say Absent Tonsils after 6 m onths” You say “Immune Deficiency”
• When say Enlarged Tonsils well after childhood (like 12-15)” You say “Cancer” ... probably
lymphatic
• When say Mystery Liver Abscess in Kid, ”You say “Chronic Granulomatous Disease”
• When say narrowed B Ring,” You say Schatzki (Schat”B ”ki Ring)
When say esophageal concentric rings,” You say Eosinophilic Esophagitis
When say shaggy” or “plaque like” esophagus, You say Candidiasis
When say looks like Candida, but an asymptomatic old lady,” you say Glycogen Acanthosis
When say reticular mucosal pattern,” you say Barretts
When say high stricture with an associated hiatal hernia,” you say Barretts
When say abrupt shoulders,” you say cancer
When say Killian Dehiscence,” you say Zenker Diverticulum
When say transient, fine transverse folds across the esophagus,” you say Feline Esophagus.
When say bird’s beak,” you say Achalasia
When say solitary esophageal ulcer,” you say CMV or AIDS
When say ulcers at the level o f the arch or distal esophagus,” you say M edication induced
• When say Breast Cancer + Bowel Hamartomas,” you say Cowdens
• When say Desmoid Tumors + Bowel Polyps,” you say Gardners
When say Brain Tumors + Bowel Polyps,” you say Turcots
When say enlarged left supraclavicular node,” you say Virchow Node (GI Cancer)
When say crosses the pylorus,” you say Gastric Lymphoma
• When say isolated gastric varices,” you say splenic vein thrombus
When say multiple gastric ulcers,” you say Chronic Aspirin Therapy.
When say multiple duodenal (or jejunal) ulcers,” you say Zollinger-Ellsion
When say pancreatitis after Billroth 2,” you say Afferent Loop Syndrome
When say Weight gain years after Roux-en-Y,” you say Gastro-Gastro Fistula
When say Clover L eaf Sign - Duodenum,” you say healed peptic ulcer.
When say ‘Sand Like Nodules in the Jejunum ,” you say W hipples

570
When say Sand Like Nodules in the Jejunum + CD4 <100,” you say MAI
When say Ribbon-like bow el,” you say Graft vs Host
• When say Ribbon like Jejunum ,” you say Long Standing Celiac
• When say Moulage Pattern,” you say Celiac (moulage = loss o f jejunal folds)
When say Fold Reversal - o f jejunum and ileum,” you say Celiac
When say Cavitary (low density) Lymph nodes,” you say Celiac
When say hide bound” or “Stack or coins,” you say Scleroderma
When say M egaduodenum,” you say Scleroderma
• When say Duodenal obstruction, with recent weight loss,” you say SMA Syndrome
When say Coned shaped cecum ,” you say Amebiasis
• When say Lead Pipe,” you say Ulcerative Colitis
When say String Sign,” you say Crohns
• When say Massive circum ferential thickening, without obstruction,” you say Lymphoma
When say Multiple small bowel target signs,” you say M elanoma
• When say Obstructing Old Lady H ernia,” you say Femoral Hernia
When say sac o f bow el,” you say Paraduodenal hernia.
When say scalloped appearance o f the liver,” you say Pseudomyxoma Peritonei
• When say HCC without cirrhosis,” you say Hepatitis B (or Fibrolamellar HCC)
When say Capsular retraction,” you say Cholangiocarcinoma
• When say Periportal hypoechoic infiltration + AIDS,” you say Kaposi’s
When say sparing o f the caudate lobe,” you say Budd Chiari
When say large T2 bright nodes + Budd Chiari,” you say Hyperplastic nodules
When say liver high signal in phase, low signal out phase,” you say fatty liver
When say liver low signal in phase, and high signal out phase,” you say hemochromatosis
When say multifocal intrahepatic and extrahepatic biliary stricture,” you say PSC
When say multifocal intrahepatic and extrahepatic biliary strictures + papillary stenosis,” you say AIDS
Cholangiopathy.
When say bile ducts full o f stones,” you say Recurrent Pyogenic Cholangitis
When say Gallbladder Comet Tail Artifact,” you say Adcnomyomatosis
• When say lipomatous pseudohypertrophy o f the pancreas,” you say CF
• When say sausage shaped pancreas,” you say autoimmune pancreatitis
When say autoimmune pancreatitis,” you say IgG4
When say lgG 4” you say RP Fibrosis, Sclerosing Cholangitis, Fibrosing M ediastinitis, Inflammatory
Pseudo tumor
When say Wide duodenal sweep,” you say Pancreatic Cancer
• When say Grandm other Pancreatic Cyst” you say Serous Cystadenoma
• When say M other Pancreatic Cyst” you say Mucinous
When say Daughter Pancreatic Cyst,” you say Solid Pseudopapillary
When say bladder stones,” you say neurogenic bladder
When say pine cone appearance,” you say neurogenic bladder
• When say urethra cancer,” you say squamous cell CA
• When say urethra cancer - prostatic portion,” you say transitional cell CA
When say urethra cancer - in a diverticulum ,” you say adenocarcinoma
When say long term supra-pubic catheter,” you say squamous Bladder CA
When say e-coli infection,” you say M alakoplakia
When say vas deferens calcifications,” you say diabetes

571
When I say “calcifications in a fatty renal mass,” you say RCC
When 1 say “protrude into the renal pelvis,” you say Multilocular cystic nephroma
When I say “no functional renal tissue,” you say M ulticystic Dysplastic Kidney
When 1 say “Multicystic Dysplastic Kidney,” you say contralateral renal issues (50%)
When I say “Emphysematous Pyelonephritis,” you say diabetic
When I say “Xanthogranulomatous Pyelonephritis,” you say staghom stone
When I say “Papillary Necrosis,” you say diabetes
When I say “shrunken calcified kidney,” you say TB (“putty kidney”)
When 1 say “bilateral medulla nephrocalcinosis,” you say Medullary Sponge Kidney
When I say “big bright kidney with decreased renal function,” you say HIV
When I say “history o f lithotripsy,” you say Page Kidney
When I say “cortical rim sign,” you say subacute renal infarct
When I say “history o f renal biopsy,” you say AVF
When 1 say “reversed diastolic flow,” you say renal vein thrombosis
When 1 say “sickle cell trait,” you say medullary RCC
When 1 say “Young Adult, Renal Mass, + Severe HTN,” you say Juxtaglomerular Cell Tumor
When 1 say “squamous cell bladder CA,” you say Schistosomiasis
When 1 say “entire bladder calcified,” you say Schistosomiasis
When 1 say “urachus,” you say adenocarcinoma o f the bladder
When I say “long stricture in urethra,” you say Gonococcal
When 1 say “short stricture in urethra,” you say Straddle Injury
When 1 say “Unicomuate Uterus,” you say Look at the kidneys
When 1 say “T-Shaped Uterus,” you say DES related or Vaginal Clear Cell CA
When I say “M arked enlargement o f the uterus,” you say Adenomyosis
When 1 say “Adenom yosis,” you say thickening o f the junctional zone (> 12 mm)
When 1 say “Wolffian duct rem nant,” you say Gartner Duct Cyst
When I say “Theca Lutein Cysts,” you say moles and multiple gestations
When I say “Theca Lutein Cysts + Pleural Effusions,” you say - Hyperstimulation Syndrome (patient on
fertility meds).
When I say “Low level internal echoes,” you say Endometrioma
W hen I say “T2 Shortening,” you say - Endometrioma - “Shading Sign”
When 1 say “Fishnet appearance,” you say Hemorrhagic Cyst
W hen I say “Ovarian Fibroma + Pleural Effusion,” you say M eigs Syndrome
W hen I say “Snow Storm Uterus, ” you say Complete Mole - 1st Trimester
W hen I say “Serum (3-hCG levels that rise in the 8 to 10 weeks following evacuation o f molar pregnancy,’'
you say Choriocarcinoma
When I say “midline cystic structure near the back o f the bladder o f a m an,” you say Prostatic Utricle
When 1 say “lateral cystic structure near the back o f the bladder o f a m an,” you say Seminal Vesicle Cyst
When I say “isolated orchitis,” you say mumps
When 1 say “onion skin appearance,” you say epidermoid cyst
When I say “multiple hypoechoic masses in the testicle,” you say lymphoma
When I say “cystic elements and macro-calcifications in the testicle,” you say M ixed Germ Cell Tumor
When 1 say “homogenous and microcalcifications,” you say seminoma
W hen 1 say “gynecomastia + testicular tumor,” you say Sertoli Leydig
When 1 say “fetal macrosomia,” you say Maternal Diabetes
When I say “one artery adjacent to the bladder,” you say two vessel cord

572
When 1 say ‘painless vaginal bleeding in the third trimester,” you say placenta previa
When I say ‘mom doing cocaine,” you say placenta abruption
When I say ‘thinning o f the myometrium - with turbulent doppler,” you say placenta creta
When I say ‘mass near the cord insertion, with flow pulsating at the fetal heart rate,” you say placenta
chorioangio ma
When I say ‘Cystic mass in the posterior neck -antenatal period,” you say cystic hygroma.
When say ‘Pleural effusions, and Ascites on prenatal US,” you say hydrops.
When say ‘M assively enlarged bilateral kidneys,” you say ARPKD
When I say ‘Twin peak sign,” you say dichorionic diamniotic
When I say ‘obliteration o f R aider’s Triangle,” you say aberrant right subclavian
When I say ‘flat waist sign,” you say left lower lobe collapse
When I say ‘terrorist + mediastinal w idening,” you say Anthrax
When I say ‘bulging fissure,” you say Klebsiella
When I say ‘dental procedure gone bad, now with jaw osteo and pneum onia,” you say Actinomycosis.
When I say ‘culture negative pleural effusion, 3 months later with airspace opacity,” you say TB
When I say ‘hot-tub,” you say Hypersensitivity Pneumonitis
When 1 say ‘halo sign,” you say Fungal Pneumonia - Invasive Aspergillus
When I say ‘reverse halo or atoll sign,” you say COP
When I say ‘finger in glove,” you say ABPA
When I say ‘ABPA,” you say Asthma
When I say ‘septic emboli + jugular vein thrombus,” you say Lemierre
When I say ‘Lemierre,” you say Fusobacterium Necrophorum
When I say ‘Paraneoplatic syndrome with SIADH,” you say Small Cell Lung CA
When I say Paraneoplatic syndrome with PTH,” you say Squamous Cell CA
When I say Small Cell Lung CA + Proximal W eakness,” you say Lambert Eaton
When I say Cavity fills with air, post pneum onectomy,” you say Bronchopleural Fistula
When I say malignant bronchial tumor,” you say carcinoid
When I say malignant tracheal tumor,” you say Adenoid Cystic
When I say AIDS patient with lung nodules, pleural effusion, and lymphadenopathy,” you say
Lymphoma
When I say Gallium Negative,” you say Kaposi
When I say Thallium Negative,” you say PCP
When I say Macroscopic fat and popcorn calcifications,” you say Hamartoma
When I say ‘Bizarre shaped cysts,” you say LCH
When I say Lung Cysts in a TS patient,” you say LAM
When I say ‘Panlobular Emphysema - NOT Alpha 1,” you say Ritalin Lung
When 1 say Honeycombing,” you say U1P
When I say ‘The histology was heterogeneous,” you say UIP
When I say ‘Ground Glass with Sub pleural Sparing,” you say NS1P
When I say ‘UIP Lungs + Parietal Pleural Thickening,” you say Asbestosis
When I say ‘Cavitation in the setting o f silicosis,” you say TB
When 1 say ‘Air trapping seen 6 months after lung transplant,” you say Chronic Rejection / Bronchiolitis
Obliterans Syndrome
When I say ‘Crazy Paving,” you say PAP
When I say ‘History o f constipation,” you say Lipoid Pneumonia - inferring mineral oil use / aspiration.
When 1 say ‘UIP + A ir trapping,” you say Chronic Hypersensitivity Pneumonitis
When 1 say ‘Dilated Esophagus + ILD,” = Scleroderma (with NSIP)
When 1 say ‘Shortness o f breath when sitting up,” you say Hepatopulmonary syndrome
When 1 say ‘Episodic hypoglycem ia,” you say solitary fibrous tumor o f the pleura
W hen I say ‘Pulmonary HTN with Normal Wedge Pressure,” you say Pulmonary Veno-occlusive disease.
When 1 say ‘Yellow Nails” you say Edema and Chylous Pleural Effusions (Yellow Nail Syndrome).
When I say ‘persistent fluid collection after pleural drain/tube placem ent,” you say Extrapleural
Hematoma.
W hen I say ‘Displaced extrapleural fat,” you say Extrapleural Hematoma.
When I say ‘M assive air leak, in the setting o f traum a,” you say bronchial or tracheal injury
When I say ‘Hot on PET - around the periphery,” you say pulmonary infarct

573
When I say “M ulti-lobar collapse,” you say sarcoid
When 1 say “Classic bronchial infection,” you say TB
When 1 say “ Panbronchiolitis,” you say tree in bud (not centrilobular or random nodules)
When 1 say “Bronchorrhea,” you say Mucinous BAC
When I say “ALCAPA,” you say Steal Syndrome
When I say “Supra-valvular Aortic Stenosis” you say Williams Syndrome
When 1 say “Bicuspid Aortic Valve and Coarctation” you say Turners Syndrome
W hen 1 say “Isolated right upper lobe edema,” you say M itral Regurgitation
W hen 1 say “Peripheral pulmonary stenosis,” you say Alagille Syndrome
W hen I say “Box shaped heart”, you say Ebsteins
When I say “Right Arch with Mirror Branching,” you say congenital heart.
W hen I say “hand/thumb defects + ASD,” you say Holt Oram
W hen I say “ostium primum ASD (or endocardial cushion defect),” you say Downs
When I say “Right Sided PAPVR,” you say Sinus Venosus ASD
When 1 say “Calcification in the left atrium wall,” you say Rheumatic Heart Disease
When I say “difficult to suppress m yocardium,” you say Amyloid
When I say “blood pool suppression on delayed enhancement,” you say Amyloid
When I say “septal bounce,” you say constrictive pericarditis
When I say “ventricular interdependence,” you say constrictive pericarditis
When I say “focal thickening o f the septum - but not Hypertrophic Cardiomyopathy,” you say Sarcoid.
When I say “ballooning o f the left ventricular apex,” you say Tako-Tsubo
When I say “fat in the wall o f a dilated right ventricle,” you say Arrhythmogenic Right Ventricular
Cardiomyopathy (ARVC)
When I say “kid with dilated heart and mid wall enhancement,” you say M uscular Dystrophy
When I say “Cardiac Rhabdomyoma,” you say Tuberous Sclerosis
When I say “Bilateral Ventricular Thrombus,” you say Eosinophilic Cardiomyopathy
When I say “Diffuse LV Subendocardial enhancement not restricted to a vascular distribution,” you say
Cardiac Amyloid.
When say “Glenn Procedure,” you say acquired pulmonary AVMs
When say “Pulmonary Vein Stenosis,” you say Ablation for A-Fib
When say “M ultiple Cardiac M yxomas,” you say C arney’s Complex
When say “vessel in the fissure o f the ligamcntum venosum ,” you say replaced left hepatic artery.
When say “vessel coursing on the pelvic brim ,” you say Corona Mortis
When say “ascending aorta calcifications,” you say Syphilis and Takayasu
When say “tulip bulb aorta,” you say Marfans
When say “really shitty M arfan’s variant,” you say Loeys-Dietz
When say “tortuous vessels,” you say Loeys-Dietz
When say “renal artery stenosis with HTN in a child,” you say N F -1
When say “nasty looking saccular aneurysm, without intimal calcifications” you say Mycotic,
When say “tree bark intimal calcification,” you say Syphilitic (Luetic) aneurysm
When say “painful aneurysm in smoker, sparing the posterior wall,” you say Inflammatory aneurysm,
When say “Turkish guy with pulm onary artery aneurysm,” you say Behcets
When say “GI bleed with early opacification o f a dilated draining vein,” you say Colonic Angiodysplasia
When say “spider web appearance o f hepatic veins on angiogram ,” you say Budd Chiari
When say “non-decompressible varicocele,” you say look in the belly for badness
When say “right sided varicocele,” you say look in the belly for badness
When say “swollen left leg,” you say May Thum er
When say “popliteal aneurysm ,” you say look for the AAA (and the other leg)
When say “most dreaded complication o f popliteal aneurysm ,” you say distal emboli
When say “Great saphenous vein on the wrong side o f the calf - lateral side,” you say Marginal Vein o f
Servel i - which is supposedly pathognomonic for Klippel-Trenaunay Syndrome
When say “A sian,” you say Takayasu
When say “Involves the aorta,” you say Takayasu
When say “Kids with vertigo and aortitis,” you say Cogan Syndrome
When say “Nasal perforation + Cavitary Lung Lesions,” you say Wegeners
When say “diffuse pulmonary hemorrhage,” you say M icroscopic Polyangitis
When say “Smoker + Hand Angiogram,” you say Buergers
When say “Construction worker + Hand Angiogram,” you say Hypothcnar Hammer

574
When I say “Unilateral tardus parvus in the carotid,” you say stenosis o f the innominate
When I say “Bilateral tardus parvus in the carotids,” you say aortic stenosis
When I say “Bilateral reversal o f flow in carotids,” you say aortic regurg
When 1 say “Lack o f diastolic flow on carotid US,” you say Brain Death
When I say IVC greater than 28 mm, you sat M ega Cava
When 1 say M ega Cava, you say Birds Nest Filter
When I say “Flairpin turn - during bronchial angiography,” you say anterior medullary (spinal cord) artery
When 1 say “ Fever, WBC, Nausea, and Vomiting after Uterine Artery Embolization,” you say Post
Embolization Syndrome (obviously could also be infection)
When I say “M ost medial vessel in the leg,” you say posterior tibial artery
When I say “the source o f 85% o f upper GI bleeds,” you say left gastric artery
When I say “the source o f bleeding from a duodenal ulcer,” you say GDA
When 1 say “ Pulmonary AVM,” you say HHT
When 1 say “most feared complication o f bronchial artery em bolization,” you say spinal cord infarct
When I say “high risk o f bleeding for liver transplant,” you say transjugular approach
When I say “m ost feared complication o f brachial arterial access,” you say compartment syndrome
When I say “cold painful fingers during dialysis,” you say “Steal syndrome”
When I say “ulcer on medial ankle,” you say venous stasis
When I say “ulcer on dorsum o f foot,” you say ischemia or infected ulcer
When I say “ulcer on plantar surface o f foot,” you say neutropenic ulcer\
When I say “pulsatile lower limb venous doppler,” you say right heart failure.
When 1 say “hot clumps o f signal in the lungs on Liver Spleen sulfur colloid,” you say too much A1 in the
Tc.
When 1 say “ HOT spleen,” you say WBC scan or Octreotide (sulfur colloid will be a warm spleen.
When 1 say “ Bone Scan with Hot Skull Sutures,” you say renal osteodystrophy
When I say “Bone Scan with Focal Breast Uptake,” you say breast CA
When I say “Bone Scan with Renal Cortex Activity,” you say hemochromatosis
When 1 say “Bone Scan with Liver Activity,” you say either too much Al, Amyloid, Hepatoma, or Liver
Necrosis
When 1 say “Bone Scan with Sternal Lesion,” you say breast CA.
When 1 say “ Bone Scan with Diffusely Decreased Bone Uptake,” you say (1) Free Tc, or (2)
Bisphosphonate Therapy.
When I say “Tramline along periosteum o f long bones,” you say lung CA
When 1 say “Super Hot Mandible in A dult,” you say Fibrous Dysplasia
When I say “ Super Hot Mandible in Child,” you say Caffeys
When I say “Periarticular uptake on delayed scan,” you say RSD
When I say “Focal uptake along the lesser trochanter,” you say Prosthesis loosening
When I say “Tracer in the brain on a VQ study,” you say Shunt
When I say “Tracer over the liver on Ventilation with Xenon,” you say Fatty Liver
When I say “Gallium Negative, Thallium Positive,” you say Kaposi
When I say “High T3, High T4, low TSH, - low thyroid uptake,” you say Quervains (Granulomatous
thyroiditis).
When I say “persistent tracer in the lateral ventricles > 24 hours,” you say NPH
When I say “Renal uptake on sulfur colloid,” you say CHF
When 1 say “Renal transplant uptake on sulfur colloid”, you say Rejection
When 1 say “Filtered Renal Agent,” you say DTPA (or GH)
When I say “Secreted Renal Agent,” you say MAG-3
When I say “PET with increased muscle uptake,” you say insulin
When I say “Diffuse FDG uptake in the thyroid on PET,” you say Hashimoto
When 1 say “1 see the skeleton on M1BG,” you say diffuse neuroblastoma bone mets
When 1 say “Cardiac tissue taking up FDG more intense than normal myocaridum,” you say hibernating
myocardium

575
1 say "made with a generator” , you say Tc99 and Rubidium
When say cervical kyphosis” , you say NF-1
When say lateral thoracic m eningocele,” you say NF-1
When say si lateral optic nerve gliomas,” you say NF-1
When say bilateral vestibular schwannoma,” you say NF-2
When say retinal hamartom a,” you say TS
When say retinal angioma,” you say VHL
When say 3rain tumor with restricted diffusion,” you say lymphoma
When say brain tumor crossing the m idline,” you say GBM (or lymphoma)
When say Cyst and Nodule in Child," you say Pilocystic Astrocytoma
When say Cyst and Nodule in Adult,” you say Hemangioblastoma
When say multiple hemangioblastoma,” you say Von Hippel Lindau
When say Swiss cheese tum or in ventricle,” you say central neurocytoma
When say CN3 Palsy,” you say posterior com municating artery aneurysm
When say CN 6 Palsy,” you say increased ICP
When say Ventricles out o f size to atrophy,” you say NPH
When say Hemorrhagic putam en,” you say Methanol
When say Decreased FDG uptake in the lateral occipital cortex,” you say Lewy Body Dementia
When say TORCH with Periventricular Calcification,” you say CMV
When say TORCH with hydrocephalus,” you say Toxoplasmosis
When say TORCH with hemorrhagic infarction,” you say HSV
When say Neonatal infection with frontal lobe atrophy,” you say HIV
When say Rapidly progressing dementia + Rapidly progressing atrophy,” you say CJD
When say Expanding the cortex,” Oligodendroglioma
When say Tumor acquired after trauma (LP),” you say Epidermoid
When say The Palate Separated from the Maxilla / Floating Palate,” you say LeFort 1
When say The M axilla Separated from the Face” or “Pyram idal” you say LeFort 2
When say The Face Separated from the Cranium,” you say LeFort 3
When say Airless expanded sinus,” you say mucocele
When say DVA,” you say cavernous malformation nearby
When say Single vascular lesion in the pons,” you say Capillary Telangiectasia
When say Elevated NAA peak,” you say Canavans
When say Tigroid appearance,” you say M etachromatic Leukodystrophy
When say Endolymphatic Sac Tumor,” you say VHL
When say T 1 Bright in the petrous apex,” you say Cholesterol Granuloma
When say Restricted diffusion in the petrous apex,” you say Cholesteatoma
When say Lateral rectus palsy + otomastoiditis,” you say Grandenigo Syndrome
When say Cochlear and semicircular canal enhancement,” you say Labyrinthitis
When say Conductive hearing loss in an adult,” you say Otosclerosis
When say Noise induced vertigo,” you say Superior Semicircular Canal dehiscence
When say Widening o f the maxillary ostium,” you say Antrochonal Polyp
When say Inverting papilloma,” you say squamous cell CA ( 10%)
When say Adenoid cystic,” you say perineural spread
When say Left sided vocal cord paralysis,” you say look in the AP window
When say Bilateral coloboma,” you say CHARGE syndrome
When say Retinal Detachment + Small Eye” you say PHPV
When say Bilateral Small Eye,” you say Retinopathy o f Prematurity
When say Calcification in the globe o f a child,” you say Retinoblastoma
When say Fluid-Fluid levels in the orbit,” you say Lymphangioma
When say Orbital lesion, worse with Valsalva,” you say Varix
When say Pulsatile Exophthalmos,” you say NF-1 and CC Fistula
When say Sphenoid wing dysplasia,” you say NF-1
When say Scim itar Sacrum,” you say Currarino Triad
When say bilateral symmetrically increased T2 signal in dorsal colum ns,” you say B12 (or HIV)
When say Owl eye appearance o f spinal cord,” you say spinal cord infarct
When say Enhancement o f the nerve roots o f the cauda equina," you say Guillain Barre
When say Subligamentous spread o f infection,” you say TB
When say, ‘Posterior elbow dislocation,” you say Capitellum fracture

576
When say “Chondroblastom a in an adult”, you say “Clear Cell Chondrosarcom a”
When say “M alignant epiphyseal lesion”, you say “Clear Cell Chondrosarcom a”
When say “Perm eative lesion in the diaphysis o f a child” , you say “Ewings”
When say “T2 bright lesion in the sacrum” , you say “Chordom a”
When say “Lytic T2 DARK lesion” , you say “ Fibrosarcoma”
When say “Sarcomatous transformation o f an infarct”, you say “M FH”
When say, “Epiphyseal Lesion that is N O T T 2 B right” , You say Chondroblastoma
When say, “short 4th metacarpal,” You say pseudopseudohypoparathyroidism and Turner Syndrome
When say, “band like acro-osteolysis,” You say Hajdu-Cheney
When say “fat containing tum or in the retroperitoneum ,” you say liposarcoma
When say “sarcoma in the foot” you say synovial sarcoma.
When say “avulsion o f the lesser trochanter,” you say pathologic fracture
When say “cross over sign,” you say pincher type Fem oroacetabular Impingement
When say “Segond Fracture,” you say ACL tear
When say “Reverse Segond Fracture,” you say PCL
When say “Arcuate Sign,” you say fibular head avulsion or PCL tear
When say “Deep Intercondylar Notch,” you say ACL tear
When say “Bilateral Patellar Tendon Ruptures,” you say chronic steroids
When say “Wide ankle mortise,” you say show me the proximal fibula (Maisonneuve).
When say “Bilateral calcaneal fractures,” you say associated spinal compression fx (“ lover’s leap”)
When say "D ancer with lateral foot pain,” you say avulsion o f 5th MT
When say “Old lady with sudden knee pain with standing,” you say SONK
When say “Looser’s Zones,” you say osteom alacia or rickets (vitamin D)
When say “Unilateral RA with preserved joint spaces,” you say RSD
When say “T2 bright tum or in finger,” you say Glomus
When say “Blooming in tumor in finger,” you say Giant Cell Tumor o f Tendon Sheath (PVNS)
When say “Atrophy o f teres minor,” you say Quadrilateral Space syndrome
When say “Subluxation o f the Biceps Tendon,” you say Subscapularis tear
When say “Too many bow ties,” you say Discoid Meniscus
When say “Celery Stalk ACL - T2” you say Mucoid Degeneration
When say “Drum stick ACL - T l ” you say M ucoid Degeneration
When say “Acute Flat foot,” you say Posterior Tibial Tendon Tear
When say “Boomerang shaped peroneus brevis,” you say tear - or split tear
When say “M eniscoid mass in the lateral ankle,” you say Anteriolateral Impingement Syndrome
When say “Scar between 3rd and 4th m etatarsals,” you say M orton’s neuroma
When say “Osteomyelitis in the spine,” you say IV drug user
When say “Osteomyelitis in the spine with Kyphosis,” you say TB (Gibbus Deformity)
When say “Unilateral SI joint lysis,” you say IV Drug User
When say “Psoas muscle abscess,” you say TB
When say “Rice bodies in joint,” you say TB - sloughed synovium
When say “Calcification along the periphery,” you say myositis ossificans
When say “Calcifications more dense in the center,” you say Osteosarcoma - reverse zoning
When say “Permeative lesion in the diaphysis o f a child,” you say Ewings
When say “Long lesion in a long bone,” you say Fibrous Dysplasia
When say “Large amount o f edema for the size o f the lesion,” you say Osteoid Osteoma
When say “Cystic bone lesion, that is NOT T2 bright,” you say Chondroblastoma
When say “Lesion in the finger o f a kid,” you say Periosteal chondroma
When say “looks like NOF in the anterior tibia with anterior bowing,” you say Osteofibrous Dysplasia,
When say “ RA + Pneum oconiosis,” you say Caplan Syndrome

577
When I say “ RA + Big Spleen + Neutropenia,” you say Felty Syndrome
When 1 say “Epiphyseal Overgrowth,” you say JRA (or hemophilia).
When I say “Reducible deformity o f joints - in hand,” you say Lupus.
When I say “destructive mass in a bone o f a leukemia patient,” you say Chloroma
When I say “shrinking breast,” you say ILC
When I say “thick Coopers ligam ents,” you say edema (CHF)
When 1 say “thick fuzzy coopers ligaments - with normal skin,” you say blur
When 1 say “dashes but no dots,” you say Secretory Calcifications
When I say “cigar shaped calcifications,” you say Secretory Calcifications
When 1 say “popcorn calcifications,” you say degenerated fibroadenoma
When I say “breast within a breast,” you say hamartoma
When I say “fat-fluid level,” you say galactocele
When 1 say “rapid growing fibroadenoma,” you say Phyllodcs
When I say “swollen red breast, not responding to antibiotics,” you say Inflammatory breast CA
When I say “lines radiating to a single point,” you say Architectural distortion.
When I say “Architectural distortion + Calcifications,” you say IDC + DCIS
When I say “Architectural distortion without Calcifications,” you say ILC
When I say “Stepladder Sign,” you say Intracapsular rupture on US
When I say “Linguine Sign,” you say Intracapsular rupture on MRI
When I say “Residual Calcs in the Lumpectomy Bed,” you say local recurrence
When I say “No Calcs in the core,” you say milk o f calcium (requires polarized light to be seen).

578
S E C T IO N 4:
T he S ledgeham m e r
B r u t e F o r c e T a c tic s

• Pulm onary Interstitial E m physem a (PIE) - put the bad side dow n
• B ronchial Foreign Body - put the lucency side dow n (if it stays that way, it’s positive)
• Papillom atosis has a sm all (2% ) risk o f squam ous cell CA
• Pulm onary sling is the only variant that goes betw een the esophagus and the trachea. This is associated
with trachea stenosis.
• Thym ic Rebound - Seen after stress (chem otherapy) - Can be PET-Avid
• Lym phom a - M ost com m on m ediastinal m ass in child (over 10)
• A nterior M ediastinal M ass w ith C alcification - E ither treated lym phom a, o r Thym ic Lesion (lym phom a
do esn ’t calcify unless treated).
• N euroblastom a is the m ost com m on posterior m ediastinal m ass in child under 2 (prim ary thoracic does
better than abd).
• H ypertrophic Pyloric S tenosis - N O T at birth, N O T after 3 m onths (3 w eeks to 3 m onths )
• Criteria for HPS - 4 m m and 14 m m (4m m single w all, 14mm length).
• A nnular Pancreas presents as duodenal obstruction in children and pancreatitis in adults.
• M ost com m on cause o f bow el obstruction in child over 4 = A ppendicitis
• Intussusception - 3 m onths to 3 years is ok, earlier or younger think lead point
• G astroschisis is ALWAYS on the right side
• O m phalocele has associated anom alies (gastroschisis does not).
• Physiologic G ut H ernia norm al at 6-8 w eeks
• AFP is elevated w ith H epatoblastom a
• Endothelial grow th factor is elevated w ith H em angioendotheliom a
• M ost C om m on cause o f pancreatitis in a kid = T raum a (seatbelt)
• W eigert M eyer Rule - D uplicated ureter on top inserts inferior and m edial
• M ost com m on tum or o f the fetus or infant - S acrococcygeal Teratom a
• M ost com m on cause o f idiopathic scrotal edem a - HSP
• M ost com m on cause o f acute scrotal pain age 7-14 - Torsion o f Testicular A ppendages
• Bell C lapper D eform ity is the etiology for testicular torsion.
• SCFE is a Salter H arris Type 1
• Physiologic Periostitis o f the N ew born d o e sn ’t occur in a new born - seen around 3 m onths
• A cetabular A ngle should be < 30, and A lpha angle should be m ore than 60.
• M ost C om m on benign m ucosal lesion o f the esophagus = Papillom a
• Esophageal W ebs have increased risk for cancer, and Plum m er-V inson Syndrom e (anem ia T w eb)
• D ysphagia Lusoria is from com pression by a right subclavian artery (m ost patients w ith aberrant rights
d o n ’t have sym ptom s).
• A chalasia has an increased risk o f squam ous cell cancer (20 years later)
• M ost com m on m esenchym al tum or o f the G1 tract = G IST
• M ost com m on location for G IST = Stom ach
• A bscesses are alm ost exclusively seen in C rohns (rather than UC)
• N odes + UC = C om m on in the setting o f active disease
• N odes (larger than 1cm ) + C rohns = C ancer

579
• D iverticulosis + N odes = C ancer (m aybe) -> next step endoscopy.
• K rukenberg T um or = Stom ach (G I) m et to the ovary
• M en etrier’s involves fundus and spares the antrum
• The stom ach is the m ost com m on location for sarcoid (in the G l tract)
• G astric R em nants have an increased risk o f cancer years after Billroth
• M ost com m on internal hernia = Left sided paraduodenal
• M ost com m on site o f peritoneal carcinom atosis = retrovesical space
• An injury to the bare area o f the liver can cause a retroperitoneal bleed
• Prim ary Sclerosing C holangitis associated with U lcerative C olitis
• E xtrahepatic ducts are norm al w ith Prim ary Biliary C irrhosis
• A nti-m itochondrial A ntibodies - positive w ith prim ary biliary cirrhosis
• M irizzi Syndrom e - the stone in the cystic duct obstructs the C BD
• M irizzi has a 5x increased risk o f G B cancer.
• D orsal pancreatic agenesis - associated with diabetes and polysplenia
• H ereditary and Tropical Pancreatitis - early age o f onset, increased risk o f cancer
• F e lty ’s S yndrom e - Big Spleen, RA, and N eutropenia
• Splenic A rtery A neurysm - m ore com m on in w om en, and m ore likely to rupture inpregnant w om en.
• Insulinom a is the m ost com m on islet cell tum or
• G astrinom a is the m ost com m on islet cell tum or w ith MEN
• U lcerative C olitis has an increased risk o f colon cancer (if itinvolves colon past the splenic flexure).
UC involving the rectum only does not increase risk o f CA.
• C alcifications in a renal C A - are associated w ith an im proved survival
• RCC bone m ets are “alw ays” lytic
• T here is an increased risk o f m alignancy w ith dialysis
• H orseshoe kidneys are m ore susceptible to traum a
• M ost com m on location for TC C is the bladder
• Second m ost com m on location for TC C is the upper urinary tract
• U pper Tract TCC is m ore com m only m ultifocal (12% ) - as opposed to bladder (4% )
• The cysts in acquired renal cystic disease im prove after renal transplant, although the risk o f renal CA in
the native kidney rem ains elevated. In fact, the cancers tend to be m ore aggressive because o f the
im m unosuppressive therapy needed to not reject a transplant.
• W eigert M eyer Rule - U pper Pole inserts m edial and inferior
• Ectopic U reters are associated w ith incontinence in w om en (not m en)
• Leukoplakia is pre-m alignant; M alakoplakia is not pre-m alignant
• E xtraperitoneal bladder rupture is m ore com m on, and m anaged m edically
• Intraperitoneal bladder rupture is less com m on, and m anaged surgically
• Indinavir (H IV m edication) stones are the only ones not seen on CT.
• Uric A cid stones are not seen on plain film
• E ndom etrial tissue in a rudim entary horn (even one that does N O T com m unicate) increases the risk o f
m iscarriage
• A rcuate U terus does N O T have an increased risk o f infertility (it’s a norm al variant)
• Fibroids w ith higher T2 signal respond better to U AE
• H yaline Fibroid D egeneration is the m ost com m on subtype
• A denom yosis - favors the posterior w all, spares the cervix

580
• H ereditary N on-Polyposis C olon C ancer (H N P C C ) - have a 30-50x increased risk o f endom etrial
cancer
• Tam oxifen increases the risk o f endom etrial cancer, and endom etrial polyps
• C ervical C ancer that has param etrial involvem ent (2B ) - is treated w ith chem o/radiation. C ervical
C ancer w ithout param etrial involvem ent (2A ) - is treated w ith surgery
• Vaginal cancer in adults is usually squam ous cell
• Vaginal R habdom yosarcom a occurs in children / teenagers
• Prem enopausal ovaries can be hot on PET (depending on the phase o f cycle). Post m enopausal ovaries
should N ever be hot on PET.
• T ransform ation subtypes: E ndom etriom a = C lear Cell, D erm oid = Squam ous
• Postpartum fever can be from ovarian vein throm bophlebitis
• Fractured penis = rupture o f the corpus cavem osum and the surrounding tunica albuginea.
• Prostate C ancer is m ost com m only in the peripheral zone, - A D C dark
• BPH nodules are in the central zone
• H ypospadias is the m ost com m on association w ith prostatic utricle
• Sem inal Vesicle cysts are associated w ith renal agenesis, and ectopic ureters
• C ryptorchidism increases the risk o f cancer (in both testicles), and the risk is not reduced by
orchiopexy
• Im m unosuppressed patients can get testicular lym phom a -hiding behind blood testes barrier
• M ost com m on cause o f correctable infertility in a m an is a varicocele.
• U ndescended testicles are m ore com m on in prem ature kids.
• M em branes disrupted before 10 w eeks, increased risk for am niotic bands
• The earliest visualization o f the em bryo is the “double bleb sign”
• H em atom a greater than 2/3 the circum ference o f the chorion has a 2x increased risk o f abortion.
• B iparietal D iam eter - R ecorded at the level o f the thalam us from the outerm ost edge o f the near skull
to the inner table o f the far skull.
• A bdom inal C ircum ference - does not include the subcutaneous soft tissues
• A bdom inal C ircum ference is recorded at the the level o f the ju n c tio n o f the um bilical vein and left
portal vein
• A bdom inal C ircum ference is the p aram eter classically involved w ith asym m etric IU G R
• Fem ur Length does N O T include the epiphysis
• U m bilical A rtery Systolic / D iastolic Ratio should N O T exceed 3 at 34 w eeks - m akes you think pre­
eclam psia and IU G R
• A full bladder can m im ic a placenta previa
• Nuchal lucency is m easured betw een 9-12 w eeks, and should be < 3 m m . M ore than 3m m is associated
with D ow ns.
• Lem on sign will disappear after 24 w eeks
• A quaductal Stenosis is the m ost com m on cause o f non-com m unicating hydrocephalus in a neonate
• The tricuspid valve is the m ost anterior
• The pulm onic valve is the m ost superior
• There are 10 lung segm ents on the right, and 8 lung segm ents on the left
• If it goes above the clavicles, it’s in the posterior m ediastinum (cervicothoracic sign)
• A zygos Lobe has 4 layers o f pleura
• M ost com m on pulm onary vein variant is a separate vein draining the right m iddle lobe
• M ost com m on cause o f pneum onia in A ID S patient is Strep P neum onia
• M ost com m on opportunistic infection in A ID S = PCP.

581
• A spergillom a is seen in a norm al im m une patient
• Invasive A spergillus is seen in an im m une com prom ised patient
• Fleischner Society R ecom m endations do N O T apply to p atie n t’s w ith know n cancers
• Eccentric calcifications in a solitary pulm onary nodule pattern is considered the m ost suspicious.
• A part solid nodule w ith a ground glass com ponent is the m ost suspicious m orphology you can have
• M ost com m on early presentation o f lung CA is a solitary nodule (right upper lobe)
• Lung Fibrosis patients (UIP, e tc ...) m ore com m only have low er lobe CA
• Stage 3B lung CA is unresectable (contralateral nodal in v o lv e m e n t; ipsilateral or contralateral
scalene or supraclavicular nodal involvem ent, tum or in different lobes).
• T he m ost com m on cause o f unilateral lym phangitic carcinom atosis is bronchogenic carcinom a lung
cancer invading the lym phatics
• T here is a 20 y ear latency betw een initial exposure and developm ent o f lung cancer or pleural
m esotheliom a
• Pleural effusion is the earliest and m ost com m on finding w ith asbestosis exposure.
• Silicosis actually raises your risk o f TB by about 3 fold.
• N itrogen D ioxide exposure is “Silo F iller’s D isease,” gives you a pulm onary edem a pattern.
• R eticular pattern in the posterior costophrenic angle is supposedly the first finding o f UIP on CXR
• S arcoidosis is the m ost com m on recurrent prim ary disease after lung transplant
• Pleural plaque o f asbestosis typically spares the costophrenic angles.
• Pleural effusion is the m ost com m on m anifestation o f m ets to the pleura.
• T here is an association w ith m ature teratom as and K linefelter Syndrom e.
• Injury close to the carina is going to cause a pneum om ediastinum rather than a pneum othorax
• H odgkin Lym phom a spreads in a contiguous fashion from the m ediastinum and is m ost often
unilateral.
• N on-H odgkin Lym phom a is typically bilateral with associated abdom inal lym phadenopathy
• MRI is superior for assessing superior sulcus tum ors because you need to look at the brachial plexus.
• L eiom yom a is the m ost com m on benign esophageal tum or (m ost com m on in the distal third).
• Esophageal L eiom yom atosis m ay be associated w ith A lp o rt’s Syndrom e
• Bronchial / Tracheal injury m ust be evaluated w ith bronchoscopy
• If you say C O P also say E osinophilic Pneum onia
• If you say BAC also say lym phom a
• Bronchial A tresia is classically in the LUL
• Pericardial cysts M U ST be sim ple, B ronchogenic cysts d o n ’t have to be sim ple
• PAP follow s a rule o f 1/3s post treatm ent; 1/3 gets better, 1/3 d o esn ’t, 1/3 progresses to fibrosis
• D ysphagia L usoria presents later in life as atherosclerosis develops
• C arcinoid is C O L D on PET
• W egener’s is now called G ranulom atosis with Polyangiitis - W egener w as a Nazi. A pparently he was
not ju s t a N azi, he w as a real asshole. I heard the guy used to take up tw o parking spots at the grocery
store on Sunday afternoons.
• T he right atrium is defined by the 1VC.
• T he right ventricle is defined by the m oderator band.
• The tricuspid papillary m uscles insert on the septum (m itral ones do not).
• Lipom atous H ypertrophy o f the Intra-A trial Septum - can be PET Avid (it’s brow n fat)
• LA D gives o ff diagonals
• RCA gives o ff acute m arginals
• LC X gives o ff obtuse m arginals
• RCA perfuses SA and AV nodes (m ost o f the tim e)
• D om inance is decided by w hich vessel gives o ff the posterior descending - it’s the right 85%
• LCA from the Right C oronary Cusp - alw ays gets repaired
• RCA from the Left C oronary C usp - repaired if sym ptom s
• M ost com m on location o f m yocardial bridging is in the mid portion o f the LAD.
• C oronary A rtery A neurysm - m ost com m on cause in adult = A therosclerosis
• C oronary A rtery A neurysm - m ost com m on cause in child = K aw asaki
• Left Sided SV C em pties into the coronary sinus

582
• Rheum atic heart disease is the m ost com m on cause o f m itral stenosis
• Pulm onary A rterial H ypertension is the m ost com m on cause o f tricuspid atresia.
• M ost com m on vascular ring is the double aortic arch
• M ost com m on congenital heart disease is a V SD
• M ost com m on A SD is the Secundum
• Infracardiac TA PV R classically show n w ith pulm onary edem a in a new born
• “L” T ransposition type is congenitally corrected (they are “ L”ucky).
• “ D” T ransposition type is doom ed.
• Truncus is associated w ith C A TC H -22 (D iG eorge)
• Rib N otching from coarctation spares the 1st and 2nd Ribs
• Infarct w ith > 50% involvem ent is unlikely to recover function
• M icrovascular O bstruction is N O T seen in chronic infarct
• A m yloid is the m ost com m on cause o f restricted cardiom yopathy
• Prim ary am yloid can be seen in m ultiple m yelom a
• M ost com m on neoplasm to involve the cardiac valves = Fibroelastom a
• M ost com m only the congenital absence o f the pericardium is partial and involves the pericardium
over the left atrium and adjacent pulm onary artery (the left atria l appendage is the m ost at risk to
becom e strangulated).
• G lenn shunt - SV C to pulm onary artery (vein to artery)
• B lalock-Taussig Shunt - Subclavian A rtery to Pulm onary A rtery (artery - artery)
• Ross Procedure - R eplaces aortic valve w ith pulm onic, and pulm onic w ith a graft (done for kids).
• A liasing is com m on w ith C ardiac M RI. You can fix it by: (1) opening your FOV, (2) oversam pling
the frequency encoding direction, or (3) sw itching phase and frequency encoding directions.
• G iant C oronary A rtery A neurysm s (> 8 m m ) d o n ’t regress, and are associated w ith M is.
• Wet Beriberi (thiam ine def) can cause a dilated cardiom yopathy.
• M ost com m on prim ary cardiac tum or in children = R habdom yom a.
• 2nd m ost com m on prim ary cardiac tum or in children = Fibrom a
• M ost com m on com plication o f MI is m yocardial rem odeling.
• U nroofed coronary sinus is associated w ith Persistent left SVC.
• M ost com m on source o f cardiac m ets = Lung C ancer (lym phom a #2).
• A -Fib is m ost com m only associated w ith left atrial enlargem ent
• M ost com m on cause o f tricuspid insufficiency is RVH (usually from pulm onary HTN / cor
pulm onale).
• A rtery o f A dam kiew icz com es o ff on the left side (70% ) betw een T8-L1 (90% )
• Arch o f Riolan - m iddle colic branch o f the SM A w ith the left colic o f the IMA.
• M ost com m on hepatic vascular variant = right hepatic artery replaced o ff the SM A
• The proper right hepatic artery is anterior the right portal vein, w hereas the replaced right hepatic artery is
posterior to the m ain portal vein.
• A ccessory right inferior hepatic vein - m ost com m on hepatic venous variant.
• A nterior tibialis is the first branch o ff the popliteal
• Com m on Fem oral A rtery (CFA): Begins at the level o f inguinal ligam ent
• Superficial Fem oral A rtery (SFA): Begins once the CFA gives o ff the profunda fem oris
• Popliteal A rtery: B egins as the SFA exits the adductor canal
• Popliteal A rtery term inates as the anterior tibial artery and the tibioperoneal trunk
• A xillary A rtery: Begins at the first rib
• Brachial A rtery: Begins as it crosses the teres m ajor
• Brachial Artery: B ifurcates to the ulnar and radial artery
• lntraosseous Branch: Typically arises from the ulnar artery
• Superficial A rch = From the U lna, D eep A rch = From the Radius
• The “coronary vein,” is the left gastric vein
• Enlarged splenorenal shunts are associated w ith hepatic encephalopathy.
• Aortic D issection, and intram ural hem atom a are caused by HTN (70% )
• Penetrating U lcer is from atherosclerosis.

583
• Strongest predictor o f progression o f dissection in intram ural hem atom a = M axim um aortic diam eter >
5cm.
• Leriche Syndrom e Triad: C laudication, A bsent/ D ecreased fem oral pulses, Im potence.
• M ost com m on associated defect w ith aortic coarctation = bicuspid aorta (80% )
• N eurogenic com pression is the m ost com m on subtype o f thoracic outlet syndrom e
• Splenic artery aneurysm - M ore com m on in pregnancy, m ore likely to rupture in pregnancy.
• M edian A rcuate C om pression - w orse w ith expiration
• Colonic A ngiodysplasia is associated with aortic stenosis
• Popliteal A neurysm ; 30-50% have A A A , 10% o f patient w ith A A A have popliteal aneurysm , 50-70% o f
popliteal aneurysm s are bilateral.
• M edial deviation o f the popliteal artery by the m edial head o f the gastrocnem ius = Popliteal Entrapm ent
• Type 3 Takayasu is the m ost com m on (arch + abdom inal aorta).
• M ost com m on vasculitis in a kid = H SP (H enoch-Schonlein Purpura)
• Tardus Parvus infers stenosis proxim al to that vessel
• 1CA Peak Systolic Velocity < 125 = “No Significant Stenosis” o r < 50%
• 1CA Peak Systolic Velocity 125-230 = 50-69% Stenosis or “M oderate”
• ICA Peak Systolic Velocity > 230 = >70% Stenosis or “ Severe”
• 18G needle will accept a 0.038 inch guidew ire,
• 19G needle will allow a 0.035 inch guidew ire.
• N otice that 0.039, 0.035, 0.018 w ires are in INCHES
• 3 French = 1 mm
• French size is the O U T SID E o f a catheter and the IN SID E o f a sheath
• E nd Hole O nly C atheters = H and Injection O nly
• Side H ole + End H ole = Pow er Injection O K , C oils N O T ok
• D ouble Flush Technique = For N euro IR — no bubbles ever
• “ S ignificant lesion” = A systolic pressure gradient > 10 m m Hg at rest
• T hings to N O T stick a drain in: T um ors, A cute H em atom a, and those associated w ith acute bow el
rupture and peritonitis
• Renal A rtery Stenting for renal failure - tends to not w ork if the C r is > 3.
• Persistent sciatic artery is prone to aneurysm
• Even if the cholecystostom y tube instantly resolves all sym ptom s, you need to leave the tube in for 2-6
w eeks (until the tract m atures), otherw ise you are going to get a bile leak.
• M ELD scores greater than 24 are at risk o f early death w ith TIPS
• The target gradient post TIPS (for esophageal bleeding) is betw een 9 and 11.
• A bsolute contraindication for TIPS - H eart Failure, Severe H epatic Failure
• M ost com m on side effect o f BRTO is gross hem aturia
• Sensitivity = GI B leed Scan = 0. lm L /m in , A ngiography = 1.0 m L/m in
• For G l B leed - after perform ing an em bolization o f the G D A (for duodenal ulcer), you need to do a run
o f the SM A to look at the inferior pancreaticoduodenal
• M ost com m on cause o f low er G l bleed is diverticulosis
• TACE will prolong survival better than system ic chem o
• TACE: Portal Vein T hrom bosis is considered a contraindication (som etim es) because o f the risk o f
infarcting the liver.
• Go above the rib for Thora
• Left B undle B ranch Block needs a pacer before a Thoracic A ngiogram
• N ever inject contrast through a Sw an G anz catheter for a thoracic angiogram
• You treat pulm onary AVMs at 3mm
• H em optysis - A ctive extravasation is N OT typically seen w ith the active bleed.
• UAE - G onadotropin-releasing m edications (often prescribed for fibroids) should be stopped for 3
m onths prior to the case
• The general rule for transgluteal is to avoid the sciatic nerves and gluteal arteries by access through the
sacrospinous ligam ent m edially (close to the sacrum , inferior to the piriform is).
• W hen to pull an abscess catheter; As a general rule - w hen the patient is better (no fever, W BC
norm al), and output is < 20 cc over 24 hours.

584
• If the thyroid biopsy is non-diagnostic, you have to w ait 3 m onths before you re-biopsy.
• Posterior lateral approach is the m ove for percutaneous nephrostom y
• You can typically pull a sheath w ith an A C T < 150-180
• A rtery calcifications (com m on in diabetics) m ake com pression difficult, and can lead to a false
elevation o f the AB1.
• Type 2 endoleaks are the m ost com m on
• Type 1 and Type 3 endoleaks are high pressure and need to be fixed stat
• Venous rupture during a fistula intervention can ofter be treated w ith prolonged angioplasty (alw ays leave
the balloon on the wire).
• Phlegm asia alba = m assive DVT, w ithout ischem ia and preserved collateral veins.
• Phlegm asia cerulea dolens = m assive DVT, com plete throm bosis o f the deep venous system , including the
collateral circulation.
• You are m ore likely to develop Venous T hrom boem bolism if you are paraplegic vs tetraplegic.
• C ircum aortic left renal vein: the anterior one is superior, the posterior one is inferior, and the filter should
be below the low est one.
• Risk o f D V T is increased w ith IVC filters
• Filter w ith clot > 1cm 3 o f clot = Filter Stays In
• A cute Budd Chiari w ith fulm inant liver failure = N eeds a TIPS
• Pseudoaneurysm o f the pancreaticoduodenal artery = “ Sandw ich technique” - distal and proxim al
segm ents o f the artery feeding o ff the artery m ust be em bolized
• M edian A rcuate L igam ent S yndrom e - First line is surgical release o f the ligam ent
• M assive H em optysis = B ronchial artery - Particles bigger than 325 m icrom eters
• A calculous C holecystitis = P ercutaneous C holecystostom y
• H epatic encephalopathy after T IPS = You can either (1) place a new covered stent constricted in the
m iddle by a loop o f suture - deployed in the p re-existing T IPS, (2) place tw o new stents - parallel to each
other (one covered s e lf expandable, one uncovered balloon expandable).
• R ecurrent variceal bleeding after placem ent o f a constricted stent - balloon dilation o f the constricted stent
• A ppendiceal A bscess - D rain placem ent * ju st rem em ber that a drain should be used for a m ature (w alled
off) abscess and no frank pertioneal sym ptom s
• Inadvertent catheterization o f the colon (after trying to place a drain in an abscess) - w ait 4 w eeks for the
tract to m ature - verify by o v er the w ire tractogram , and then rem ove tube.
• D V T w ith severe sym ptom s and no response to system ic anticoagulation = C atheter Directed
T hrom bolysis
• G eiger M ueller - m axim um dose it can handle is about lOOmR/h
• A ctivity level g reater than 100 m C i o f Tc-99m is considered a m ajor spill.
• A ctivity level greater than 100 m C i o f Tl-201 is considered a m ajor spill.
• A ctivity level greater than 10 m C i o f In -1 11, is considered to represent a m ajor spill.
• A ctivity level greater than 10 m C i o f G a-67, is considered to represent a m ajor spill.
• An activity level g reater than 1 m Ci o f 1-131 is considered to constitute a m ajor spill.
• A nnual D ose lim it o f 100 m rem to the public
• N ot greater than 2 m rem per hour - in an “unrestricted area"
• Total Body Dose per Year = 5 rem
• Total equivalent organ dose (skin is also an organ) per year = 50 rem
• Total equivalent extrem ity dose p er year = 50 rem (500m Sv)
• Total D ose to E m bryo/fetus o v er entire 9 m onths - 0.5rem
• N RC allow s no m ore than 0.15 m icro Ci o f M o per 1 m ili Ci o f Tc, at the tim e o f adm inistration.
• C hem ical purity (Al in Tc) is done w ith pH paper
• T he allow able am ount o f Al is < 10 m icrogram s
• Radiochem ical purity (looking for Free Tc) is done w ith thin layer chrom atography
• Free Tc occurs from - lack o f stannous ions or accidental air injection (w hich oxidizes)

585
• Prostate C ancer bone m ets are uncom m on with a PSA less than 10 m g/m l
• Flair Phenom enon occurs 2 w eeks - 3 m onths after therapy
• Skeletal Survey is superior (m ore sensitive) for lytic m ets
• AVN - Early and Late is C O L D , M iddle (repairing) is Hot.
• Particle size for VQ scan is 10-100 m icrom eters
• X enon is done first during the VQ scan
• A m iodarone - classic thyroid uptake blocker
• H ashim otos increases risk for lym phom a
• Hot nodule on Tc, sh o u ld n ’t be considered benign until you show that it’s also hot on I123. T his is the
concept o f the discordant nodule.
• H istory o f m ethim azole treatm ent (even years prior) m akes 1-131 treatm ent m ore difficult
• M ethim azole side effect is neutropenia
• In pregnancy PTU is the blocker o f choice
• Sestam ibi in the parathyroid depends on blood flow and m itochondria
• You w ant to im age w ith PET - follow ing therapy at interval o f 2-3 w eeks for chem otherapy, and 8-12
w eeks for radiation is the w ay to go. T his avoids “ stunning” - false negatives, and inflam m atory
induced false positive.
• 111In Pentetreotide is the m ost com m only used agent for som atostatin receptor im aging. The classic
use is for carcinoid tum ors
• M eningiom as take up octreotide
• In 111 binds to neutrophils, lym phocytes, m onocytes and even RBCs and platelets
• Tc99m HM PAO binds to neutrophils
• W B C s m ay accum ulate at post op surgical sites for 2-3 w eeks
• Prior to M IBG you should block the thyroid w ith Lugols Iodine or Perchlorate
• Scrotal Scintigraphy: The typical agent is Tc-99m Pertechnetate. T his agent is used as both a flow
agent and a pool agent.
• Left bundle branch block can cause a false positive defect in the ventricular septum (spares the apex)
• Pulm onary uptake o f T hallium is an indication o f LV dysfunction
• M IBG m echanism is that o f an A nalog o f N orepinephrine - actively transported and stored in the
neurosecretory granules
• M D P m echanism is that o f a Phosphate analog - w hich w orks via C hem isorption
• Sulfur C olloid m echanism = P articles are Phagocytized by RES
• T he order o f tum or prevalence in N F2 is the sam e as the m nem onic M SM E (schw annom a >
m eningiom a > ependym om a).
• M aldeveloped draining veins is the etiology o f Sturge W eber
• All phakom atosis (N F 1, NF -2, TS, and V H L) E X C EPT Sturge W eber are autosom al dom inant -
fam ily screening is a good idea.
• M ost C om m on Prim ary Brain T um or in A dult = A strocytom a
• “C alcifies 90% o f the tim e” = O ligodendrogliom a
• R estricted D iffusion in Ventricle = W atch out for C horoid Plexus X anthogranulom a (not a brain
tum or, a benign norm al variant)
• Pituitary - T1 Big and B right = Pituitary A poplexy
• Pituitary - N orm al T1 Bright = Posterior Part (because o f storage o f Vasopressin , and other storage
proteins)
• Pituitary - T2 B right = R athke C left C yst
• Pituitary - C alcified = C raniopharyngiom a
• C P A ngle - Invades Internal A uditory C anal = Schw annom a
• CP A ngle - Invades Both Internal A uditory C anals = Schw annom a with NF2
• CP A ngle - R estricts on D iffusion = Epiderm oid
• Peds - A rising from Vermis = M edulloblastom a
• Peds - “tooth paste” out o f 4th ventricle = E pendym om a
• A dult m yelination pattern: T 1 at 1 year, T2 at 2 years
• B rainstem and posterior lim b o f the internal capsule are m yelinated at birth.
• CN 2 and CN V 3 are not in the cavernous sinus

586
• Persistent trigem inal artery (basilar to carotid) increases the risk o f aneurysm
• Subfalcine herniation can lead to AC A infarct
• A D EM lesions w ill N O T involve the calloso-septal interface.
• M archiafava-B ignam i progresses from body -> genu -> splenium
• Post R adiation changes d o n ’t start for 2 m onths (there is a latent period).
• H ippocam pal atrophy is first w ith A lzheim er D em entia
• Beaked Tectum = C hiari 2
• Beaker A nterior Inferior LI = H urlers
• Som etim es B eaked Pons = M ulti-System A trophy
• M ost com m on T O R C H is CM V
• Toxo abscess does N O T restrict diffusion
• Small cortical tum ors can be occult w ithout IV contrast
• JPA and G angliogliom a can enhance and are low grade
• Nasal Bone is the m ost com m on fracture
• Z ygom aticom axillary C om plex Fracture (Tripod) is the m ost com m on fracture pattern and involves
the zygom a, inferior orbit, and lateral orbit.
• Supplem ental oxygen can m im ic SAH on FLA IR
• Putam en is the m ost com m on location for hypertensive hem orrhage
• Restricted diffusion w ithout bright signal on FLA IR should m ake you think hyperacute (< 6 hours)
stroke.
• E nhancem ent o f a stroke: Rule o f 3s - starts at day 3, peaks at 3 w eeks, gone at 3 m onths
• PAN is the M ost C om m on system ic vasculitis to involve the CN S
• Scaphocephaly is the m ost com m on type o f craniosynostosis
• Piriform aperture stenosis is associated w ith hypothalam ic pituitary adrenal axis issues.
• C holesterol G ranulom a is the m ost com m on prim ary petrous apex lesion
• Large vestibular aqueduct syndrom e has absence o f the bony m odiolus in 90% o f cases
• O ctreotide scan w ill be positive for esthesioneuroblastom a
• The m ain vascular supply to the posterior nose is the sphenopalatine artery (term inal internal
m axillary artery).
• W arthins tum ors take up pertechnetate
• Sjogrens gets salivary gland lym phom a
• M ost com m on intra-occular lesion in an adult = M elanom a
• Enhancem ent o f nerve roots for 6 w eeks after spine surgery is norm al. A fter that it’s arachnoiditis
• H em orrhage in the cord is the m ost im portant factor for outcom e in a traum atic cord injury.
• C urrarino Triad: A nterior Sacral M eningocele, A norectal m alform ation, Sarcococcygeal osseous
defect
• Type 1 Spinal AVF (dural AVF) is by far the m ore com m on.
• Herpes spares the basal ganglia (M C A infarcts do not)
• M ost com m on m alignant lacrim al gland tum or = adenoid cystic adenocarcinom a
• A rthritis at the radioscaphoid com partm ent is the first sign o f a SN A C or SLA C w rist
• SLA C w rist has a D ISI deform ity
• Pull o f the A bductor pollucis longus tendon is w hat causes the dorsolateral dislocation in the Bennett Fx
• Carpal tunnel syndrom e has an association w ith dialysis
• D egree o f fem oral head d isplacem ent predicts risk o f AVN
• Proxim al pole o f the scaphoid is at risk for AVN w ith fracture
• M ost com m on cause o f sacral insufficiency fracture is osteoporosis in old lady
• Patella dislocation is nearly alw ays lateral
• Tibial plateau fracture is w ay m ore com m on laterally
• SO N K favors the m edial knee (area o f m axim um w eight bearing)
• N orm al SI jo in ts excludes A nk Spon
• Looser Z ones are a type o f insufficiency fracture

587
• T score o f -2.5 m arks osteoporosis
• First extensor com partm ent = de Q uervains
• First and Second com partm ent = intersection syndrom e
• Sixth extensor com partm ent = early RA
• Flexor pollicis longus goes through the carpal tunnel, flexor pollicis brevis does not
• The pisiform recess and radiocarpal jo in t norm ally com m unicate
• T he periosteum is intact w ith both Perthes and A LPSA lesions. In a true bankart it isdisrupted.
• A bsent anterior/superior labrum , + thickened m iddle glenohum eral ligam ent is a Buford com plex.
• M edial m eniscus is thicker posteriorly.
• A nterior talofibular ligam ent is the m ost com m only tom ankle ligam ent
• TB in the spine - spares the disc space (so can brucellosis).
• Scoliosis curvature points aw ay from the osteoid osteom a
• O steochondrom a is the only benign skeletal tum or associated w ith radiation.
• M ixed C onnective Tissue D isease requires serology (R ibonucleoprotein) for Dx
• M edullary Bone Infarct w ill have fat in the m iddle
• B ucket H andle M eniscal tears are longitudinal tears
• A nterior D raw er Sign = ACL
• Posterior D raw er Sign = PCL
• “ M cM urray” = M CL
• N o grid on m ag view s.
• BR-3 = < 2% chance o f cancer
• BR-5 = > 95% chance o f cancer
• N ipple enhancem ent can be norm al on post contrast MRI - d o n ’t call it Pagets.
• U pper outer quadrant has the highest density o f breast tissue, and therefore the m ost breast cancers.
• M ajority o f blood (60% ) is via the internal m am m ary
• M ajority o f lym ph (97% ) is to the axilla
• The stem alis m uscle can only be seen on CC view
• M ost com m on location for ectopic breast tissue is in the axilla
• The follicular phase (day 7-14) is the best tim e to have a m am m ogram (and M RI).
• B reast Tenderness is m ax around day 27-30.
• Tyrer C uzick is the m ost com prehensive risk m odel, but does not include breast density.
• I f you had m ore than 20G y o f chest radiation as a child, you can get a screening MRI
• BR CA 2 (m ore than 1) is seen w ith m ale breast cancer
• BRC A 1 is m ore in younger patients, BRCA 2 is m ore in post m enopausal
• BRC A 1 is m ore often a triple negative CA
• Use the LM O for kyphosis, pectus excavatum , and to avoid a pacem aker / line
• Use the ML to help catch m ilk o f calcium layering
• Fine pleom orphic m orphology to calcification has the highest suspicion for m alignancy
• Intram am m ary lym ph nodes are N O T in the fibroglandular tissue
• Surgical scars should get lighter, if they get denser - think about recurrent cancer.
• You CA N have isolated intracapsular rupture.
• You CA N N O T have isolated extra (it’s alw ays w ith intra).
• If you see silicone in a lym ph node, you need to recom m end MRI to evaluate for extracapsular rupture
• The num ber one risk factor for im plant rupture is the age o f the im plant
• Tam oxifen causes a decrease in parenchym al uptake, then a rebound.
• T2 B right things - these are usually benign. D o n 't forget colloid cancer is T2 bright.

588
M ade in the USA
C oppell, TX
13 D ecem ber 2019

12908457R00324

12908457R00324

You might also like